You are on page 1of 241

Sumon K.

Sarkar
Bank Job Solution (BIBM) 1

Created by Sumon K. Sarkar  FB Group : BD job Info. (BCS & Bank)  Created by Sumon K. Sarkar 
Created by Sumon K. Sarkar  FB Group : BD job Info. (BCS & Bank)  Created by Sumon K. Sarkar 

Question Collection & Solution by


Sumon K. Sarkar

Last Questions Collection Date:


24 April, 2023
Written Math Solution (BIBM) FB Group : BD job Info. (BCS & Bank)
2

 Created by Sumon K. Sarkar  FB Group : BD job Info. (BCS & Bank)  Created by Sumon K. Sarkar 
Bank Job Solution (BIBM)
 Created by Sumon K. Sarkar  FB Group : BD job Info. (BCS & Bank)  Created by Sumon K. Sarkar 

Table of Content
Sl Name of the Exam Page
1. Combined 9 bank officer (2020 Year Based) 
2. Combined 9 bank Senior officer (2020 Year Based) 
3. Bangladesh Bank Assistant Maintenance Engineer – 2023 
4. Bangladesh Bank -Assistant programmer – 2023 
5. Bangladesh Bank -Assistant Director (Research) – 2023 
6. Bangladesh Bank - Officer (Electrical) – 2023 
7. Bangladesh Bank – Officer – 2022 
8. Bangladesh Bank -Assistant Director – 2022 
9. Bank Asia Ltd. (BO)-2022 
10. Standrad Bank Ltd. (TAO)-2022 
11. Modhumoti Bank Ltd. (PO) 2022 
12. Premier Bank Ltd. (TJO) Cash 2021 
13. UCB Bank Ltd. (PO) 2021 
14. Premier Bank Ltd. (TJO) General 2020 
15. Bank Asia Ltd (PO) 2020 
16. Premier Bank Ltd. (TJO) 2019 
17. NRB Bank Ltd. (MTO) 2019 
18. Modhumoti Bank Ltd. (PO) 2018 
19. Standard Bank Ltd.(TAO) Cash 2018 
20. Standard Bank Ltd.(TAO) 2018 
21. Premier Bank Ltd. (TAO) 2018 
22. NRBC Bank Ltd. (MTO) 2018 
23. Shahjalal Islami Bank Ltd. (TO) Cash 2018 
24. Shahjalal Islami Bank Ltd. (TO) 2018 
25. One Bank Ltd (SCO) 2018 
26. Langka Bangla Finance (MTO)2017 
27. National Bank Ltd. (PO) 2017 
28. Bank Asia Ltd. (MTO) 2017 
29. Modhumoti Bank Ltd (PO) 2017 
30. Social Islami Bank Ltd. (PO) 2017 
31. One Bank Ltd (SCO) 2017 
32. Modhumoti Bank Ltd (PO) 2016 
33. Modhumoti Bank Ltd (PO) 2016 
34. Standard Bank Ltd.(TAO) Cash 2016 

Written Math Solution (BIBM) FB Group : BD job Info. (BCS & Bank)
Bank Job Solution (BIBM) 3

Created by Sumon K. Sarkar  FB Group : BD job Info. (BCS & Bank)  Created by Sumon K. Sarkar 
35. Standard Bank Ltd.(TAO) 2016 
Created by Sumon K. Sarkar  FB Group : BD job Info. (BCS & Bank)  Created by Sumon K. Sarkar 

36. Shahjalal Islami Bank Ltd. (TO)Cash 2016 


37. Shahjalal Islami Bank Ltd. (TSO) 2016 
38. Shahjalal Islami Bank Ltd. (MTO) 2016 
39. Al-Arafah Bank Ltd (MTO)-2016 
40. Bank Asia (trainee officer) 2016 
41. Bangladesh Bank Officer (General)-2015 
42. Bangladesh Bank -Assistant Director (FF)-2015 
43. Midland bank (TO) 2015 
44. Midland bank (MTO) 2015 
45. National Bank Ltd. (PO) 2015 
46. National Bank Ltd. (PO) 2014 
47. Premier Bank Ltd (MTO)-2015
48. Modhumoti Bank Ltd(PO) 2015
49. Basic Bank Assistant Officer (cash) 2014 
50. Rakub Officer 2014 
51. Rakub Senior Officer 2014 
52. United Commercial Bank ltd. (senior Officer) 2014 
53. Union Bank Ltd. (senior Officer) 2014
54. UCB ltd. (MTO) 2013 
55. National Bank PO 2013 
56. Shahjalal Islami Bank Ltd. (TO) cash 2013 
57. Shahjalal Islami Bank Ltd. (TO) 2013 
58. Shahjalal Islami Bank Ltd. (TSO) 2013 
59. Shahjalal Islami Bank Ltd. (MTO) 2013 
60. Social Islami Bank Ltd. (PO) 2013 
61. Al arafah Bank ltd (MTO )2013 
62. Premiar Bank ltd (TJO) 2013 
63. Mutual trust Bank (MTO) 2013
64. Jamuna Bank (MTO) 2013
65. Social Islami Bank Ltd (PO) 2013
66. Premiar Bank ltd (MTO) 2012 
67. One Bank ltd (SCO) 2012 
68. Shahjalal Islami Bank Ltd. (TO) 2011 
69. Shahjalal Islami Bank Ltd. (TSO) 2011 
70. Shahjalal Islami Bank Ltd. (MTO) 2011 
71. Standard Bank Ltd.(TAO) Cash 2011 

Written Math Solution (BIBM) FB Group : BD job Info. (BCS & Bank)
4

 Created by Sumon K. Sarkar  FB Group : BD job Info. (BCS & Bank)  Created by Sumon K. Sarkar 
Bank Job Solution (BIBM)
 Created by Sumon K. Sarkar  FB Group : BD job Info. (BCS & Bank)  Created by Sumon K. Sarkar 

72. Standard Bank Ltd.(TAO) Cash 2011 


73. NCC Bank Ltd. (MTO) 2011 
74. UCB Bank Ltd. (MTO) 2011 
75. UCB Bank Ltd. (Officer) 2011 
76. IFIC (MTO) 2011 
77. UCB Bank Ltd. (Officer) 2010 
78. Shahjalal Islami Bank Ltd. (MTO) 2010 
79. Shahjalal Islami Bank Ltd. (MTO) 2010
80. Social Islami Bank Limited – (TO) 2010 
81. One Bank ltd (PO) 2010 
82. Bangladesh Bank Assistant Director 2010 
83. Bangladesh Bank Assistant Director 2009 
84. Basic Bank (Assistant Officer) 2009 
85. IFIC (PO) - 2009
86. National Bank Limited – (MTO) 2009
87. ICB - (Assistant Programmer) 2008 
88. Bangladesh Bank Assistant Director 2008 
89. Bangladesh Commerce Bank (Junior Officer) 2008 
90. Sonali, Janata, Agrani & Rupali Bank (BRC) – Senior Officer 2008 
91. Sonali, Janata, Agrani & Rupali Bank (BRC) – Officer 2008 
92. Sonali, Janata, Agrani & Rupali Bank (BRC) – Officer (cash) 2008 
93. Shahjalal Islami Bank Ltd. (TSO) 2007 
94. One Bank ltd (PO) 2007 
95. Premiar Bank ltd 2005
96. AB Bank (MTO) 2003
97. National Bank Limited – (MTO) 2004
98. IFIC (PO) 2002
99. Bangladesh Bank Assistant Director 2001 
100. Bangladesh Bank Officer (GENERAL) 2001 

Written Math Solution (BIBM) FB Group : BD job Info. (BCS & Bank)
Bank Job Solution (BIBM) 5

Created by Sumon K. Sarkar  FB Group : BD job Info. (BCS & Bank)  Created by Sumon K. Sarkar 
BIBM Question Analysis (Last 15years)
Created by Sumon K. Sarkar  FB Group : BD job Info. (BCS & Bank)  Created by Sumon K. Sarkar 

Speed Time and Distance


Ratio & Percentage

Simple and Comp.

Pipes & Cistern

Measurement
Particulars

Time & work


Profit & loss

Partnership
Simplicity

Geometry
Equation

Average

interest

Mixer
Age

Set
Combined 9 bank Com. officer (2020 Based) 1 1 1 1 1
Combined 9 bank Com. S. officer (2020 based) 2 1 1 1
Bangladesh Bank AME - 2023 1 1 1 1
Bangladesh Bank – AD (Research) 2023 1 1 1 1
Bangladesh Bank – Assistant prog. 2023 1
Bangladesh Bank – Officer (Electrical) 2023 1 2 1
Bangladesh Bank -Officer 2023 1 1 1 1 1
Bangladesh Bank -Assistant Director 2023 1 1 1 1 1
Bank Asia Ltd. (BO)-2022 1 1
Bank Asia Ltd. (TAO)-2022 1 1
Standrad Bank Ltd. (TAO)-2022 1 1
Modhumoti Bank Ltd. (PO) 2022 1 1 1 1
Premier Bank Ltd. (TJO) Cash 2021 1 1
UCB Bank Ltd. (PO) 2021 1
Premier Bank Ltd. (TJO) Gen 2020 1 1 1
Bank Asia Ltd (PO) 2020 1 1 1
Mutual Trust Bank Ltd(MTO)2019 2
Premier Bank Ltd (TJO) 2019 1 1 1 1
NRB Bank Ltd. (MTO) 2019 1 1 1 1
Modhumoti Bank Ltd (PO) 2018 1 1 1 1
Standard Bank Ltd.(TAO) Cash 2018 2
Standard Bank Ltd.(TAO) 2018 1 1
Premier Bank Ltd. (TAO) 2018 1 1 1 1
NRBC Bank Ltd. (MTO) 2018 1 1 1 1
Shahjalal Islami Bank Ltd. (TO) Cash 2018 1 1 1 1
Shahjalal Islami Bank Ltd. (TO) 2018 1 1 1 1
One Bank Ltd (SCO) 2018 2 1 1
Langka Bangla Finance (MTO)2017 1 1 1 1
National Bank Ltd. (PO) 2017 1 1 1 1
Bank Asia Ltd (MTO) 2017 1 1 1 1
Modhumoti Bank Ltd (PO) 2017 1 1 1 1
Social Islami Bank Ltd. (PO) 2017 1 1 1 1
One Bank Ltd (SCO) 2017 1 1 1 1
Modhumoti Bank Ltd (PO) 2016 1 1 1 1 1

Written Math Solution (BIBM) FB Group : BD job Info. (BCS & Bank)
6

 Created by Sumon K. Sarkar  FB Group : BD job Info. (BCS & Bank)  Created by Sumon K. Sarkar 
Bank Job Solution (BIBM)
 Created by Sumon K. Sarkar  FB Group : BD job Info. (BCS & Bank)  Created by Sumon K. Sarkar 

Modhumoti Bank Ltd (PO) 2016 1 1 1


Standard Bank Ltd.(TAO) Cash 2016 1 1 1 1 1
Standard Bank Ltd.(TAO) 2016 1 1 1 1 1
Shahjalal Islami Bank Ltd. (TO)Cash 2016 1 1 1 1
Shahjalal Islami Bank Ltd. (TO) 2016 1 1 1 1
Shahjalal Islami Bank Ltd. (MTO) 2016 1 1 1 1
Al-Arafah Bank Ltd (MTO)-2016 1 1 1 1 1
Bangladesh Bank Officer (General) 3 1 2 1 1
Bangladesh Bank -Assistant Director (FF) 1 1 1 1 1 1
Midland bank (TO) 2015 1 1
Midland bank (MTO) 2015 1 1
National Bank Ltd. (PO) 2015 1 1 1
National Bank Ltd. (PO) 2015 1 1 1
Basic Bank Assistant Off. 2014 1 1 1
Rakub Officer 2014 1 1 1 1 1
Rakub Senior Officer 2014 1 1 1 1 1
UCB ltd. 2013 1 1 1 1
National Bank PO 2013 1 1 1 1
Shahjalal Islami Bank Ltd. (TO) cash 2013 1 1 1 1
Shahjalal Islami Bank Ltd. (TO) 2013 1 2 1
Shahjalal Islami Bank Ltd. (TSO) 2013 1 1 1 1
Shahjalal Islami Bank Ltd. (MTO) 2013 1 1 1 1
Shahjalal Islami Bank Ltd. (PO) 2013 1 1 1
Al arafah Bank ltd (MTO )2013 1 1 1 1
Premiar Bank ltd (TJO) 2013 1 1 1 1
27
19
40

20
22
20
12

14
23
Total ( 15 years )
4
7
3
4

4
2
Simple and Comp. interest

Speed Time and Distance


Ratio & Percentage

Pipes & Cistern

Measurement
Time & work
Profit & loss

Partnership
Simplicity

Geometry
Equation

Average

Mixer
Age

Set

এ রকম গুরুত্বপূর্ণ ফাইল পপতে আমাতের গ্রুতপ প াখ রাখুন


এবং অবশ্যই ননতের বন্ধুতের ইনভাইট করুন

Written Math Solution (BIBM) FB Group : BD job Info. (BCS & Bank)
Bank Job Solution (BIBM) 7

Created by Sumon K. Sarkar  FB Group : BD job Info. (BCS & Bank)  Created by Sumon K. Sarkar 
Coming Soon……..
Created by Sumon K. Sarkar  FB Group : BD job Info. (BCS & Bank)  Created by Sumon K. Sarkar 

• BCS preliminary Previous Question Solve ( 10th -44th )


• Primary Previous Question Solve (2000 – 2022)
• BIBM Update preliminary Previous Question

আমাতের Group এ যা পাতবনঃ


• FBS preliminary Previous Question
• FBS Written Previous Question
• FBS Written Math
• CTI preliminary Previous Question
• CTI Written Math
• Art Faculty preliminary Previous Question
• Art Faculty Written Math
• Social Science preliminary Previous Question
• Social Science Written Previous Question
• 10 years DU Admission Test Question
• And Many more…

আমাতের গ্রুতপ েতেন করুন

Written Math Solution (BIBM) FB Group : BD job Info. (BCS & Bank)
8

 Created by Sumon K. Sarkar  FB Group : BD job Info. (BCS & Bank)  Created by Sumon K. Sarkar 
Bank Job Solution (BIBM)
 Created by Sumon K. Sarkar  FB Group : BD job Info. (BCS & Bank)  Created by Sumon K. Sarkar 

Combined Officer General (2020 year Based)


Post: officer (General) Exam Date: 31– 03 – 2023
1. A man engaged a worker on the condition that he would pay him Tk. 30000 and one uniform after
one year of service. The worker served only for 9 months and got Tk. 22000 and a uniform. What is
the price of the uniform? Answer: Tk. 20,000
অনুবােঃ এক বছতরর াকনরর পতর এক বযনি এক শ্রনমকতক 30000 টাকা এবং একনট ইউননফমণ প্রোতনর শ্তেণ ননতযাগ পেন। শ্রনমক মাত্র ৯
মাস াকনর কতরতছন এবং 22000 টাকা ও একনট ইউননফমণ পপতযতছন । ইউননফতমণর োম কে?
2. The cost of 3 horses is same as the cost of 5 cows. If total cost of 4 horses and 6 cows is Tk. 1900,
find the cost of one horse. Answer: Tk. 250
অনুবােঃ ৩নট প াডার োম ৫নট গরুর োতমর সমান। 4নট প াডা ও 6নট গরুর পমাট খর হতল 1900 টাকা, একনট প াডার োম কে?
3. A man won a lottery of Tk. 2000000. He gave 2/5 of the amount to his wife and 1/4 of the balance to
his son. He donated 2/9 of the balance amount. How much money was donated? Answer: Tk. 2,00,000
অনুবােঃ এক বযনি 2000000 টাকার একনট লটানর নেতেতছ। নেনন োর স্ত্রীতক 2/5 অংশ্ এবং বানক টাকার ¼ অংশ্ োর পছতলতক নেতযতছন।
নেনন অবনশ্ষ্ট টাকার 2/9 অংশ্ োন কতরতছন। কে টাকা োন করা হতযনছল?
4. Reena took loan of Tk. 1200 at simple interest rate for as many years as the rate of interest. If she paid
Tk. 432 as interest at the end of the loan period, what is the rate of interest? Answer: 6%
অনুবােঃ রীনা সহে সুতের হাতর 1200 টাকা ঋর্ পনন পযখাতন ঋতর্র সুতের হার ও ঋতর্র পমোতের সমে একই। যনে নেনন ঋতর্র পমযাে পশ্তে
432 টাকা সুে প্রোন কতরন, োহতল সুতের হার কে?
5. A square and an equilateral triangle have equal perimeters. If the diagonal of the 12√2 cm, then what
is the area of the triangle? Answer: 64√3 square cm
অনুবােঃ একনট বগণতেত্র এবং একনট সমবাহু নত্রভুতের সমান পনরসীমা রতযতছ। কর্ণ 12√2 পসনম হতল, নত্রভুতের পেত্রফল কে?

Combined Senior officer (2020 year Based)


Post: Senior officer Exam Date: 24– 02 – 2023
1. Certain amount of money is divided among A, B, and C in such a way that A gets 3 times as much as
B and B receives 2 times as much as C. If A receives Tk. 1500 more than C, what is the total amount
of money? Answer: Tk. 2700
অনুবােঃ নননেণষ্ট পনরমার্ অর্ণ A, B, এবং C এর মতযয এমনভাতব ভাগ করা হতযতছ যাতে A ,B এর 3 গুর্ পায এবং B , C এর 2 গুর্ পবনশ্ পায
। যনে A, C এর প তয 1500 পবনশ্ টাকা পায, েতব পমাট টাকার পনরমার্ কে?
2. The population of a village is 10,000. In one year, male population increase by 6% and female
population by 4%. If the population at the end of the year is 10520, find the original size of the male
population in that village. Answer: 6000
অনুবােঃ একনট গ্রাতমর েনসংখযা 10,000। এক বছতর পুরুে েনসংখযা 6% এবং মনহলা েনসংখযা 4% বৃনি পায। বছতরর পশ্তে েনসংখযা
10,520 হতল, পসই গ্রাতমর পুরুে েনসংখযার কে?
3. A bag contains coins of Tk.1, Tk.2, and Tk.5 denomination in the ratio 2:3:4. If the total amount is
Tk.280, find the number of coins of Tk.5 denomination in the bag. Answer: 40
অনুবােঃ একনট বযাতগ 2:3:4 অনুপাতে 1 টাকা, 2 টাকা এবং 5 টাকা মূতলযর কতযন রতযতছ। যনে পমাট 280 টাকা হয, োহতল বযাতগ 5 টাকার
কতযতনর সংখযা পবর করুন।
4. 3 years ago, father's age was twice the age of his 4 sons. In 3 years time, the father's will be equal to
the sum of the ages of the sons. Find the present age of the father. Answer: 39 years
অনুবােঃ 3 বছর পূতবণ, বাবার বযস োর 4 পছতলর বযতসর নিগুর্ নছল। ৩ বছতর নপোর বযস পুত্রতের বযতসর সমনষ্টর সমান হতব। নপোর বেণমান
বযস ননর্ণয করুন।

Written Math Solution (BIBM) FB Group : BD job Info. (BCS & Bank)
Bank Job Solution (BIBM) 9

Created by Sumon K. Sarkar  FB Group : BD job Info. (BCS & Bank)  Created by Sumon K. Sarkar 
5. If the perimeter of an isosceles right triangle is (6+3√2) meter, then what is the area of the triangle?
Created by Sumon K. Sarkar  FB Group : BD job Info. (BCS & Bank)  Created by Sumon K. Sarkar 

অনুবােঃ একনট সমনিবাহু সমতকার্ী নত্রভুতের পনরনয (6+3√2) নমটার হতল, নত্রভুতের পেত্রফল কে? Answer: 4.5 square meter

Bangladesh Bank
Post: Assistant Maintenance Engineer Exam Date: 04– 02 – 2023
4(√6+√2) 2+√3
1. Solve the problem: − Answer: 1
(√6−√2) 2−√3
2. A father divides his property between his two sons A and B. A invests the amount at compound interest
of 8% p.a. B invests the amount at 10% p.a simple interest. At the end of 2 years, the interest received
by B is Taka 1336 more than the interest received by A. Find A 's share in the father's father's property
of Taka 25,000. [Midland Bank MTO 15, SIBL PO 13, Bangladesh Bank-AME-2023] Answer: Tk 10,000 and Tk. 15,000
Abyev`t †Kvb wcZv Zvi m¤úwË Zvi `yB cyÎ A I B Gi g‡a¨ fvM K‡i †`q| A Zvi UvKv 8% Pµe„w× gybvdvq Ges B Zvi UvKv
10% mij gybvdvq wewb‡qvM K‡i| 2 eQi c‡i A Gi †P‡q B 1,336 UvKv †ewk gybvdv AR©b K‡i| Df‡qi UvKvi cwigvb wbY©q
Ki| Zv‡`i wcZvi Kv‡Q 25,000 UvKv wQj|
3. The percentage profit earned by selling an article for Taka 1920 is equal to the percentage loss incurred
by selling the same article for Taka 1280. At what price should the article be sold to make a 25% profit?
[South East Bank MTO 2013, BB AD(freedom fighter) 2015, SJIB MTO 2013, IBBL PO 2019, Sadharan Bima JO 2019, Sadharan Bima AM
2019, BB AD -AME - 2023]
Abyev`t GKwU cY¨ 1,920 UvKvq weµq Ki‡j kZKiv hZ jvf nq, cY¨wU 1,280 UvKvq weµq Ki‡j kZKiv ZZ UvKv ¶wZ nq|
25% jvf Ki‡Z n‡j cY¨wU‡K KZ `v‡g weµq Ki‡Z n‡e? Answer: Tk. 2000
4. AD is the longest side of the right triangle ABD shown in the figure. What is the length of the longest
side of ∆ABC? Answer: √41
A

B 5 C D

Bangladesh Bank
Post: Assistant Programmar Exam Date: 03– 02 – 2023
1 17 1
1. If x is an integer and 𝑥 + = , then value of 𝑥 − =? Answer: 15/4
𝑥 4 𝑥
2
2. A basket ball team has won 15 games and lost 9. If these games represent 16 percent of the games to
3
be played, then how many more games must the team win to average 75 percent for the season?
Answer: 93 more games
[ONE Bank PO 10, NCC Bank MTO 11, Premier Bank MTO 12, Midland Bank TO 15, Bangladesh bank AP 2023]
Abyev`t GKwU ev‡¯‹Uej wUg 15wU †Ljvq Rqjvf K‡i Ges 9wU †Ljvq civwRZ nq| hw` GB †Ljv¸‡jv Øviv Zv‡`i †gvU ‡Ljvi
2
16 % †Ljv‡K †evSv‡bv nq Zvn‡j H wmR‡b M‡o 75% †Ljvq †RZvi Rb¨ Zv‡`i‡K KZwU †Ljvq wRZ‡Z n‡e?
3
3. Students of a class are made to stand in rows. If 4 students are extra in each row, then there would be
2 rows less. If four students are less in each row, then there would be 4 more rows. What is the number
of students in the class? Answer: 96

Written Math Solution (BIBM) FB Group : BD job Info. (BCS & Bank)
10

 Created by Sumon K. Sarkar  FB Group : BD job Info. (BCS & Bank)  Created by Sumon K. Sarkar 
Bank Job Solution (BIBM)
 Created by Sumon K. Sarkar  FB Group : BD job Info. (BCS & Bank)  Created by Sumon K. Sarkar 

অনুবােঃ একনট ক্লাতসর নশ্োর্ণীতের সানরবিভাতব োাঁড করাতনা হয। প্রনে সানরতে ৪ েন নশ্োর্ণী অনেনরি হতল ২ সানর কম হতব। যনে প্রনে
সানরতে ারেন নশ্োর্ণী কম র্াতক, োহতল আরও 4নট সানর হতব। ক্লাতস নশ্োর্ণীর সংখযা কে?
4. In the given figure, PQT is an right triangle, then what is the area of square QRST. Answer: 50
R

Q S

P 5 T

Bangladesh Bank
Post: Assistant Director (Research) Exam Date: 03– 02 – 2023
1. Monthly incomes of two persons are in the ratio of 5: 4 and their monthly expenditures are in the ratio
of 9:7. If each person saves Tk. 500 per month, then what are their monthly incomes?
অনুবােঃ দুই বযনির মানসক আতের অনুপাে 5:4 এবং োতের মানসক বযতের অনুপাে 9:7। যনে প্রতেযক বযনি মাতস ৫০০ টাকা সঞ্চে
কতর, োহতল োতের মানসক আয কে? Answer: 5000 and 4000 respectively.
2. One man wanted to fence one side of his garden. If he uses posts 6 feet apart, the ends and start of the
side ends with posts. If he buys certain no of posts from market they are 5 less than required. If the
post are 8 feet apart the no of posts brought satisfied the requirements. What is the length of the garden?
How many posts were brought? Answer: Length 120 feet and Posts bought 16
অনুবােঃ একেন পলাক একনট বাগাতনর একপাতশ্ পবড়া লাগাতে ান। নেনন বাগাতনর ৬ ফুট অন্তর এক প্রান্ত পর্তক পশ্ে প্রান্ত পযণন্ত পপাস্ট
লাগাতবন। নেনন যেগুতলা পপাস্ট নকতনতছন ো প্রতোেতনর প তে ৫ নট পপাস্ট কম হে। নকন্তু যনে ৮ ফুট অন্তর পপাস্ট লাগান েতব পপাস্ট সংখযা
কম-পবনশ্ নকছুই হে না। বাগাতনর দে ণয কে ফুট? কেগুতলা পপাস্ট ক্রে করা হতেনছল?
3. An executive goes on a business trip. His daily schedule has a definite pattern. If he is busy with a
meeting in the morning, he is free in the afternoon. When he returns, he realises that he attended 15
meetings altogether. There were 12 free mornings and 13 free afternoons. What was the duration of
his trip? Answer: 20 days
অনুবােঃ একেন ননবণাহী একনট বযবসানযক সফতর যায। োর প্রনেনেতনর সমযসূ ীর একনট নননেণষ্ট পযাটানণ রতযতছ। যনে পস সকাতল নমনটং
ননতয বযস্ত র্াতক েতব পস নবতকতল নি র্াতকন। নেনন যখন নফতর আতসন, েখন নেনন বুঝতে পাতরন পয নেনন পমাট 15নট নমনটংতয অংশ্
ননতযতছন। 12নট মুি সকাল এবং 13নট মুি নবকাল নছল। োর সফতরর সমযকাল কে নছল?
4. A team of 2 men and 5 women complete 1/4th of a job in 3 days. After that another man joins them
and they all complete the next 1/4th of the job in 2 days. How many men can complete the whole job
in 4 days? [BB AD 14, PKSF AM 09, NCC Bank Officer 02, One Bank SCO 18, NRB Bank MTO 19] Answer: 6 men.
Abyev`t 2 Rb cyiæl Ges 5 Rb gwnjvi GKwU `j 3 w`‡b GKwU Kv‡Ri 1/4 Ask m¤c~Y© K‡i| 3 w`b ci GKRb AwZwi³ cyiæl
Kv‡R †hvM w`‡j Zviv 2 w`‡b AviI 1/4 Ask m¤cbœ K‡ib| KZRb cyiæl wg‡j evwK KvR 4 w`‡b m¤cbœ Ki‡Z cvi‡eb?

Written Math Solution (BIBM) FB Group : BD job Info. (BCS & Bank)
Bank Job Solution (BIBM) 11

Created by Sumon K. Sarkar  FB Group : BD job Info. (BCS & Bank)  Created by Sumon K. Sarkar 
Bangladesh Bank
Created by Sumon K. Sarkar  FB Group : BD job Info. (BCS & Bank)  Created by Sumon K. Sarkar 

Post: Officer (Electrical) Exam Date: 13– 01 – 2023


1. Taxi fare is described by the following relationship:
Total taxi Fare = Tk. A upto 2 km+Tk. B per km (exceeding 2 km)+Tk.60 for per hour waiting
A person paid Tk. 432 for running 52 km and 2 hours of waiting charge. The same person paid Tk.732
for running 102 km and 2 hours of waiting charge. Find the value of 'A' and 'B'. Answer: A = 12 & B = 6
[Shahjalal Islami Bank Ltd. (MTO) 2010 ]
Abyev`t wb‡Pi m¤c‡K©i mvnvh¨ †Uw· fvov †`Lv‡bv hvq:
†gvU †Uw· fvov = ¯’vqx PvR© A UvKv 2wKwg ch©šÍ + B UvKv 2 wKwg c‡i +60 UvKv cÖwZ N›Uv A‡c¶v PvR©|
GKRb e¨w³ 52 wKwg Ges 2 N›Uv A‡c¶vi Rb¨ 432 UvKv w`j| GKB e¨w³ 102 wKwg I 2 N›Uv A‡c¶vi Rb¨ 732 UvKv cÖ`vb
K‡i| A Ges B wbb©q Ki|
2. Three friends invest equal shares in a business. After careful calculation, they realized that each of
them would have invested Tk. 3,000 less if they had found 2 more friends with equal shares. What is
the amount of total investment in the business? Answer: Tk. 22,500
অনুবােঃ নেন বন্ধু একনট বযবসায সমান পশ্যার নবননতযাগ কতর। সেকণোর সনহে নহসাব করার পর োরা বুঝতে পারতলন পয, োতের প্রতেযতক
৫০ হাোর টাকা নবননতযাগ কতরতছ। 3,000 টাকা কম যনে োরা সমান পশ্যার সহ আরও 2 বন্ধু খুাঁতে পপে। বযবসায পমাট নবননতযাতগর
পনরমার্ কে?
3. The figure shows a circular flowerbed, with its center at O having radius of 8 feet. The flowerbed is
surrounded by a circular path, which is 3 feet wide. What is the area of the path in square feet?
অনুবােঃ ন ত্রনট একনট বৃত্তাকার ফুতলর নবছানা পেখায, যার পকন্দ্র O-পে রতযতছ 8 ফুট বযাসাযণ। ফুতলর নবছানা একনট বৃত্তাকার পর্ িারা
পবনষ্টে, যা 3 ফুট ওডা। পতর্র পেত্রফল কে বগণফুট? Answer: 57π

Bangladesh Bank
Post Name: Officer Exam Date: 18– 11 – 2022
1. A labour is paid Tk 100 per hour for working 8 hours in a day as regular pay. His payment is 1½ times
than the rate of regular pay in excess of 8 hours in a single day as overtime. It the labour received a
total of Tk 1000 for a single day, how long did he work on that day? Answer: 9 hours 20 minuties
Same As [ One Bank Ltd. Probationary Officer Exam Date : 2010]
অনুবােঃ ননযনমে পবেন নহতসতব নেতন ৮ ণ্টা কাে করার েনয একেন শ্রনমকতক ণ্টায ১০০ টাকা পেওযা হয। এক নেতন ৮ ন্টার পবনশ্
কাে করতল োর ওভারটাইম পপতমন্ট ননযনমে পবেতনর হাতরর প তয ১.৫ গুর্। এই শ্রনমক এক নেতনর েনয সবণতমাট ১০০০ টাকা পপল, পস
নেতন কেনেন কাে করল?
2. In a certain office, 1/3 of workers are women, 1/2 of the women are married and 1/3 of the married
women have children. If 3/4 of the men are married and 2/3 of the married men have children, what
part of the workers are without children? Answer: 11/18
অনুবােঃ একনট অনফতস, 1/3 অংশ্ কমণী মনহলা, 1/2 অংশ্ মনহলা নববানহে এবং 1/3 অংশ্ নববানহে মনহলাতের সন্তান রতযতছ। যনে 3/4
অংশ্ পুরুে নববানহে এবং 2/3 অংশ্ নববানহে পুরুতের সন্তান র্াতক, োহতল শ্রনমকতের পকান অংশ্ সন্তানহীন?

Written Math Solution (BIBM) FB Group : BD job Info. (BCS & Bank)
12

 Created by Sumon K. Sarkar  FB Group : BD job Info. (BCS & Bank)  Created by Sumon K. Sarkar 
Bank Job Solution (BIBM)
 Created by Sumon K. Sarkar  FB Group : BD job Info. (BCS & Bank)  Created by Sumon K. Sarkar 

3. Mr. A deposited a certain amount of money for a fixed period of time. On maturity, he received a total
of Tk. 60,000 and the ratio of interest and investment was 3:6. If the interest rate was 6.25% (simple).
calculate the time period for which the money was invested? Answer:8
Same as [BASIC Bank PO 1999, ONE Bank PO 07]
Abyev`t Mr. A GKwU wbw`©ó mg‡qi Rb¨ wKQz UvKv e¨vs‡K Rgv iv‡Lb| H mgq ci wZwb e¨vsK †_‡K †gvU 60,000 UvKv jvf
K‡ib G‡Z gybvdv Ges Avm‡ji AbycvZ nq 3 t 6| hw` gybvdvi nvi 6.25% nq Zvn‡j mgq wbY©q Ki|
4. 200 children came to the park last Sunday. All of the older children rode bicycles into the park and all
of the younger children came on tricycles. 480 Wheels rode into the park that day, all of them
functioning on the children's bicycles of tricycles. How many younger children came to the park last
Sunday? [ One Bank Ltd. Probationary Officer Exam Date : 2010] Answer: 80
অনুবােঃ এক অনফতস ১/৩ অংশ্ নারী কমণী , ১/২ অংশ্ নারী নববানহে এবং ১/৩ অংশ্ নববানহে নারীর সন্তান আতছ। যনে ৩/৪ অংশ্ পুরুে
নববানহে হে এবং ২/৩ অংশ্ নববানহে পুরুতের সন্তান র্াতক েতব কে অংশ্ কমণীর সন্তান নছল না?
5. The length and width of a garden is 60 meter and 20 meter respectively. Inside the garden, there is a
5- meter-wide path around it. What is the area of the path in square meter ?
অনুবােঃ একনট বাগাতনর দে ণয ৬০ নমটার এবং প্রস্থ ২০ নমটার। বাগাতনর পভেতরর ারনেতক ৫ নমটার প্রস্থ নবনশ্ষ্ট একনট রাস্তা আতছ। েতব
রাস্তার পেত্রফল কে? [Premier Bank TJO 18, SJIB TSO 18] [Standard Bank TAO Cash 16] Answer: 700 square meter
Bangladesh Bank
Post Name: Assistant Director Exam Date: 18– 11 – 2022
4 √𝑝 ×16𝑝𝑞 ×2−2√𝑝
1. × 4−𝑝𝑞 Answer: 1
8√𝑞 ×4 𝑝𝑞 ×2−3√𝑞
2. Every year Mr. Kalam saves Tk 5400 which is 15 percent of his annual income. Mr. Rahim spends Tk
2800 a month which is 87.5 percent of his monthly income. Who is earning more?
অনুবােঃ প্রনে বছর নমঃ কালাম ৫৪০০ টাকা সঞ্চয কতরন যা োর বানেণক আতযর ১৫ শ্োংশ্। েনাব রনহম মাতস ২৮০০ টাকা খর কতরন
যা োর মানসক আতযর ৮৭.৫ শ্োংশ্। পক পবনশ্ আয করতছ? Anwser: Mr. Rahim is earning more
3. The average weight of A, B and C is 40 kg. Weight of C is 24 kg more than A's weight and 3 kg less
than B's weight. What will be the average weight of A, B, C and D, if D weights 15 kg less than C?
অনুবােঃ A, B এবং C এর গড ওেন 40 পকনে। C এর ওেন A এর ওেতনর প তয 24 পকনে পবনশ্ এবং B এর ওেতনর পর্তক 3 পকনে
কম। যনে D-এর ওেন C পর্তক 15 পকনে কম হয, A, B, C এবং D-এর গড ওেন কে হতব? Answer: 38
4. Matin visited his cousin Raju during the summer vacation. In the morning, they both would go for
swimming. In the evening, they would play tennis. They would engage in at most one activity per day.
There were days when they took rest and stayed home all day long. There were 32 mornings when they
did nothing. 18 evenings when they stayed at home, and a total of 28 days when they swam or played
tennis. What duration of the summer vacation did Matin stay with Raju? Answer: 39 days
অনুবােঃ মানটণন গ্রীষ্মকালীন ছুনটর সময োর খালাতো ভাই রােুর বাসায পবডাতে যায। সকাতল োরা একসাতর্ সাোর কাটতে পযে, নবকাতল
পটননস পখলতে পযে। োরা প্রনেনেন বডতোর একনট কাে করে। এমন নকছু নেন নছল যখন োরা পুতরানেন বাসায বতস নবশ্রাম ননে। ৩২
নেন সকালতবলা োরা নকছু কতরনন, ১৮ নেন নবকালতবলা োরা বাসায নছল এবং পমাট ২৮ নেন োরা হয পটননস পখতলনছল অর্বা সাোর
পকতটনছল। অবকাশ্ যাপতনর কেনেন মানটণন রােুর সাতর্ নছল?
5. In the figure below, AB is perpendicular to BC and BD=DC, If AD= √10 cm and AC=4cm, then what
is the valu of BC? Answer: 2√2

Written Math Solution (BIBM) FB Group : BD job Info. (BCS & Bank)
Bank Job Solution (BIBM) 13

Created by Sumon K. Sarkar  FB Group : BD job Info. (BCS & Bank)  Created by Sumon K. Sarkar 
Bank Asia Ltd.
Created by Sumon K. Sarkar  FB Group : BD job Info. (BCS & Bank)  Created by Sumon K. Sarkar 

Post: Business Officer Exam Date: 25 – 03 – 2022


1. The price of 20 articles is the same as the selling price of X articles. If the profit is 25%, Then what is
the value of X ? Answer: 16
অনুবােঃ 20নট ননবতন্ধর মূলয X নট ননবতন্ধর নবক্রয মূতলযর সমান। যনে লাভ হয 25%, োহতল X এর মান কে?
2. A widow took 1/3rd of the estate. Each son of the twos took the remaining estate. If the widow and one
son took 3,60,000 Tk, then what is the value of the estate? Answer: tk. 540,000
অনুবােঃ একেন নবযবা সম্পনত্তর 1/3 অংশ্ ননতযনছতলন। দুেতনর প্রতেযক পছতল অবনশ্ষ্ট সম্পনত্ত ননতযনছল। যনে নবযবা ও এক পছতল
3,60,000 টাকা পনন, োহতল সম্পনত্তর মূলয কে?

Standrad Bank Ltd.


Post Name: Trainee Assistant Officer Exam Date: 11 – 03 – 2022
1. The ratio, by volume, of bleach to detergent to water in a certain solution is 2 : 40 : 100. The solution
will be altered so that the ratio of bleach to detergent is tripled while the ratio of detergent to water is
halved. If the altered solution will contain 300 liters of water, how many liters of detergent will it
contain? Answer: 60 litres
অনুবােঃ একনট নননেণষ্ট দ্রবতর্ নডটারতেন্ট পর্তক নিত র েতলর অনুপাে হল 2 : 40 : 100৷ দ্রবর্নট পনরবেণন করা হতব যাতে নি পর্তক
নডটারতেতন্টর অনুপাে নেনগুর্ হয এবং নডটারতেতন্টর েতলর অনুপাে অতযণক হয৷ পনরবনেণে দ্রবতর্ 300 নলটার েল র্াকতল, কে নলটার
নডটারতেন্ট র্াকতব?
2. Alif bought a bike, but he paid 15000 taka directly & remaining money with 8% interest in 2 years. In
total he paid 28,920 taka. What was the actual cost of the bike? Answer: Tk. 27,000
অনুবােঃ আনলফ একনট বাইক নকতননছল, নকন্তু পস 15000 টাকা সরাসনর পনরতশ্ায কতরতছ এবং বানক টাকা 8% সুতে 2 বছতর। সব
নমনলতয নেনন ২৮,৯২০ টাকা পনরতশ্ায কতরন। বাইকনটর প্রকৃে োম কে নছল?

Modhumoti Bank Ltd.


Post Name:Probationary Officer Exam Date : 11 – 03 – 2022
1. A, B, and C can do a work on 10, 30 and 60 days respectively. A is assisted by B and C on every
alternative days. How many days required to complete the work? Answer: 8 days
অনুবােঃ A, B, এবং C যর্াক্রতম 10, 30 এবং 60 নেতন একনট কাে করতে পাতর। প্রনে নবকল্প নেতন A-পক B এবং C সাহাযয কতর।
কাে পশ্ে করতে কে নেন লাগতব?
2. X can travel from village to school at 3km/h and return at 2 km/h. Total time taken 5 hours. What is
the distance from Village to School? Answer: 6 km.
অনুবােঃ X গ্রাম পর্তক স্কুতল পযতে পাতর 3 নকনম/ ন্টা পবতগ এবং নফরতে পাতর 2 নকনম/ ন্টা পবতগ। পমাট সময পলতগতছ 5 ন্টা। গ্রাম
পর্তক স্কুতলর েূরত্ব কে?
3. A rectangular Field with length 112 m and width 78 m. Inside the garden there is 2.5 m path. Find the
area of path and cost for constructing it at 4 tk per sqm. Answer: Tk. 3700
অনুবােঃ একনট আযেতেত্রাকার পেত্র যার দে ণয 112 নমটার এবং প্রস্থ 78 নমটার। বাগাতনর নভেতর 2.5 নমটার পর্ রতযতছ। পতর্র
পেত্রফল ননর্ণয করুন এবং প্রনে বগণনমটাতর 4 টাকা এনট ননমণাতর্র েনয খর কে হতব?
4. An article when sells at Tk 75 something loss. If it sells at Tk.96. gain would be double the former
loss. find the Cost price of that article. Answer: Tk. 82
অনুবােঃ একটা article ৭৫ টাকায নবনক্র করতল নকছু েনে হয।যনে ৯৬ টাকায নবনক্র হে েতব লাভ আতগর েনের নিগুর্ হতব। article
এর ক্রে মূলয কে?

Written Math Solution (BIBM) FB Group : BD job Info. (BCS & Bank)
14

 Created by Sumon K. Sarkar  FB Group : BD job Info. (BCS & Bank)  Created by Sumon K. Sarkar 
Bank Job Solution (BIBM)
 Created by Sumon K. Sarkar  FB Group : BD job Info. (BCS & Bank)  Created by Sumon K. Sarkar 

Premier Bank Ltd.


Post name:Trainee Junior Officer (Cash) Exam Date : 19 – 03 – 2021
1. The average age of a committee of eight members is 40 years. A member aged 55 years retired and his
place was taken by another member aged 39 years. Find the average age of the present committee.
অনুবােঃ আট সেতসযর একনট কনমনটর গড বযস ৪০ বছর। ৫৫ বছর বযসী একেন সেসয অবসর ননতযতছন এবং োর স্থতল ৩৯ বছর বযসী
অনয সেসয গ্রহর্ কতরতছন। বেণমান কনমনটর গড বযস ননর্ণয কর। Answer: 38 years
2. The ratio of the salary of A, B & C is 7:5:3. If B gets Tk 222 more than what C gets, then what is the
salary of A? Answer: Tk. 777
অনুবােঃ A, B এবং C এর পবেতনর অনুপাে 7:5:3। B যনে C এর প তে 222 টাকা পবনশ্ পায, োহতল A-এর পবেন কে?

UCB Bank Ltd.


Post name:Probationary Officer Exam Date : 19 – 11 – 2021
1. Mr. Salam has an open terrace adjacent to his apartment where his children play football. To protect
the children from filing, Mr. Salam wants to put up railing around three sides of his rectangular terrace
and leave a side of 20 feet unfenced as that side is covered by his apartment and needs no railing. If
the terrace has an area of 680 square feet, how many feet of railing does he need?
অনুবােঃ েনাব সালাতমর োর অযাপাটণতমতন্টর পাতশ্ একনট পখালা বারান্দা রতযতছ পযখাতন োর সন্তানরা ফুটবল পখতল। বাচ্চাতের পখলার
সমে যাতে না পতড় যাে পসেনয আযেতেত্রাকার বারান্দার ারপাতশ্ নেননেতক পরনলং লাগাতে ান এবং 20 ফুতটর একনট পাশ্ পবডনবহীন
রাখতে ান কারর্ পসই নেকনট োর অযাপাটণতমন্ট িারা আচ্ছানেে এবং পকান পরনলংতযর প্রতযােন পনই। বারান্দার পেত্রফল 680 বগণফুট
হতল, োর কে ফুট পরনলং লাগতব?

Premier Bank Ltd.


Post Name: Trainee Junior Officer (General) Exam Date : 21 – 10 – 2020
1. A man has Tk.100,000 invest. He invests Tk. 40,000 at 5% and Tk. 35,000 at 4% interest rate. In order
to have a yearly income of Tk. 5000, at what rate of interest he must invest the reminder amount.
অনুবােঃ একেন মানুতের 100,000 টাকা নবননতযাগ আতছ। নেনন টাকা 5% সুে হাতর 40,000 টাকা এবং 4% সুে হাতর 35,000
নবননতযাগ কতরন । যাতে বাৎসনরক আয 5000 টাকা। নক সুতের হাতর োর বানক টাকার উপর নবননতযাগ করতে হতব। Answer: 6.4%
2. M can do a piece of work in 40 days. He works at it for 5 days and then N alone finishes the remaining
work in 42 days. The two together could complete the work in how many days? Answer: 21.82 days
অনুবােঃ M 40 নেতনর মতযয একনট কাে করতে পাতর। নেনন এনটতে 5 নেন কাে কতরন এবং োরপর N একাই 42 নেতন বানক কাে পশ্ে
কতরন। দুেন নমতল কে নেতন কাে পশ্ে করতে পারতেন?
3. In a shop, the cost of 4 shirts, 4 pairs of trousers and 2 hats is Tk. 560. The cost of 9 shirts, 9 pairs of
trousers and 6 hats is Tk. 1290. What is the total cost of 1 shirt, 1 pair of trousers and 1 hat?
[Janata AEO (RC) 17] Answer: Tk. 150
অনুবােঃ একনট পোকাতন 4 নট শ্াটণ, 4 পোডা ট্রাউোর এবং 2 টুনপর োম 560 টাকা। 9 নট শ্াটণ, 9 পোডা ট্রাউোর এবং 6 নট টুনপর োম
1290। ১নট শ্াটণ, ১ পোডা পযান্ট ও ১নট টুনপর পমাট োম কে?

Bank Asia Ltd.


Post Name: Probationary Officer Exam Date : 2020
1. Raju can finish a work in 15 hours and Saju can finish the same work in 9 hours. They work together
firsty at 6 am but Saju stopped at 9 am. What the time required to finish the work by Raju?

Written Math Solution (BIBM) FB Group : BD job Info. (BCS & Bank)
Bank Job Solution (BIBM) 15

Created by Sumon K. Sarkar  FB Group : BD job Info. (BCS & Bank)  Created by Sumon K. Sarkar 
অনুবােঃ রােু একনট কাে 15 ন্টায পশ্ে করতে পাতর এবং সােু একই কাে 9 ন্টায পশ্ে করতে পাতর। সকাল ৬টায একসতে কাে
Created by Sumon K. Sarkar  FB Group : BD job Info. (BCS & Bank)  Created by Sumon K. Sarkar 

করতলও সােু সকাল ৯টায পর্তম যায। রােুর কাে পশ্ে করতে কে সময লাগতব? Answer: 4.00 PM
2. Mr. Zahir received Eid bonus of Tk. 7875. He distributed the money among his three daughters Dame
Lipi, Rini and Shahnaz. Lipi got 3/5 of what Shahnaz got. Shahnaz got double of what Rimi got.
Determine the actual amount of money that each of the sisters got. Answer: tk. 1875 , tk. 3750, tk. 2250
[AB Bank AO 14, BISIC Officer 14, IFIC Bank MTO 13, DBBL MTO(2) 09, Bank Asia PO 20]
Abyev`t Rwni mv‡ne 7,875 UvKv C‡`i †evbvm cvb| GB UvKv Zvi wZb Kb¨v wjwc, wiwg I kvnbv‡Ri g‡a¨ fvM K‡i †`b| wjwc,
kvnbv†Ri Ask cvq I kvnbvR, wiwgi wظY UvKv cvq| cÖ‡Z¨‡K KZ UvKv K‡i cvb, wbY©q Ki|
3. Mr. Arif have been invested two schemes, first amount Tk. 5000 in 2 years and second amount Tk.
3000 in 4 years at the same rate of interest. Total interest he received Tk. 2200 at the end of a year.
Calculate the rate of interest. Answer: 10%
4. Tanim bought some oranges. He gave ½ of them to his sister, 1/4 th of the remainder to his neighbor,
3/5th of those left to his children and had 6 left in the end. How many oranges did Tanim buy?
Abyev`t Zvwbg wKQz Kgjv‡jey wKbj, hvi A‡a©K †m Zvi †evb‡K w`j| Aewkó Kgjvi 1/4 fvM cÖwZ‡ewk‡`i w`j| Gici hv
Aewkó wQj Zvi 3/5 fvM Zvi mšÍvb‡`i †`Iqvi ci Zvi Kv‡Q 6wU Kgjv iBj| Zvwbg KZwU Kgjv wK‡bwQj?
[BB AD 10, MTB MTO 19] Answer: 40 oranges.
4. A Group of students has hired a bus for Taka 3000 for going to a picnic. They had an understanding
that each participant would share the charge in equal amounts. But because of 10 students not turning
up, the charged per student increased by Taka 10 over the initial estimates. What was the number of
students who originally registered for the picnic? [BB AD 06, MTB MTO 19] Answer: 60
Abyev`t GK `j QvÎ wcKwb‡K hvevi Rb¨ 3,000 UvKv w`‡q GKwU evm fvov K‡i| Zviv cÖ‡Z¨‡K mg-cwigvb Puv`v †`evi wm×všÍ
†bq| wKš‘ 10 Rb QvÎ bv hvIqvq AvbygvwbK cÖv_wgK wnmve †_‡K RbcÖwZ Puv`vi cwigvb 10 UvKv e„w× cvq| cÖK…Zc‡¶ KZRb
QvÎ wcKwb‡K hvevi Rb¨ †iwR‡óªkb K‡iwQj?

Mutual Trust Bank Ltd.


Post name: Management Trainee Officer Exam Date : 2019
1. Tanim bought some oranges. He gave ½ of them to his sister, 1/4th of the remainder to his neighbor,
3/5th of those left to his children and had 6 left in the end. How many oranges did Tanim buy?
Abyev`t Zvwbg wKQz Kgjv‡jey wKbj, hvi A‡a©K †m Zvi †evb‡K w`j| Aewkó Kgjvi 1/4 fvM cÖwZ‡ewk‡`i w`j| Gici hv
Aewkó wQj Zvi 3/5 fvM Zvi mšÍvb‡`i †`Iqvi ci Zvi Kv‡Q 6wU Kgjv iBj| Zvwbg KZwU Kgjv wK‡bwQj?
[BB AD 10, MTB MTO 19] Answer: 40 oranges.
2. A Group of students has hired a bus for Taka 3000 for going to a picnic. They had an understanding
that each participant would share the charge in equal amounts. But because of 10 students not turning
up, the charged per student increased by Taka 10 over the initial estimates. What was the number of
students who originally registered for the picnic? [BB AD 06, MTB MTO 19] Answer: 60
Abyev`tGK `j QvÎ wcKwb‡K hvevi Rb¨ 3,000 UvKv w`‡q GKwU evm fvov K‡i| Zviv cÖ‡Z¨‡K mg-cwigvb Puv`v †`evi wm×všÍ
†bq| wKš‘ 10 Rb QvÎ bv hvIqvq AvbygvwbK cÖv_wgK wnmve †_‡K RbcÖwZ Puv`vi cwigvb 10 UvKv e„w× cvq| cÖK…Zc‡¶ KZRb
QvÎ wcKwb‡K hvevi Rb¨ †iwR‡óªkb K‡iwQj?

Written Math Solution (BIBM) FB Group : BD job Info. (BCS & Bank)
16

 Created by Sumon K. Sarkar  FB Group : BD job Info. (BCS & Bank)  Created by Sumon K. Sarkar 
Bank Job Solution (BIBM)
 Created by Sumon K. Sarkar  FB Group : BD job Info. (BCS & Bank)  Created by Sumon K. Sarkar 

Premier Bank Ltd.


Post name: Trainee Junior Officer (General) Exam Date : 2019
1. The ratio of the ages of a man and his wife is 4:3. After 4 years, this ratio will be 9:7. If the time of
marriage, the ratio was 5:3, then how many years ago were they married? Answer: 12 years
Abyev`t GK e¨w³ Ges Zvi ¯¿xÕi eq‡mi AbycvZ 4 t 3| 4 eQi c‡i Zv‡`i eq‡mi AbycvZ n‡e 9t 7| hw` we‡qi mgq Zv‡`i
eq‡mi AbycvZ 5 t 3 n‡j, Zviv KZ eQi Av‡M we‡q K‡iwQj?
2. ABC is a triangle in which AB = 3cm, BC = 5cm, and AC = 4cm, AD is a perpendicular from A to
BC. Find the length of AD. [Modhumoti MTO 16, Premier Bank JO 19] Answer: 2.4 cm

3. A trader, while selling an item, was asking for such a price that would enable him to offer a 20%
discounts and still make a profit of 30% on cost. If the cost of the item was Tk. 50 what was his asking
price? [BB AD 08, Standard Bank TAO 16, Premier Bank JO 19] Answer: tk 81.25
Abyev`t GKRb we‡µZv GKwU c‡Y¨i Ggb GKwU `vg e‡jb hv‡Z cY¨wU‡Z 20% wWmKvD›U w`‡jI we‡µZvi 30%
jvf nq| hw` cY¨wUi g~j¨ 50 UvKv nq Zvn‡j we‡µZv KZ `vg e‡jwQ‡jb?
4. If x+y=a, x2+ y2 =b2 and x3+ y3 = c3 , then show that a3 + 2c3 = 3ab2
[Modhumoti MTO 16, Premier Bank JO 19, Board Book]

NRB Bank Ltd.


Post Name: Management Trainee Officer Exam Date : 2019
10 1 18
1. Solve the Equation: + = [SJIB MTO 16, SJIB TSO 18, Premier Bank TJO 18, NRB Bank MTO 19]
2𝑥−5 𝑥+5 3𝑥−5
Answer: x =15
2. Mr. Karim borrowed Tk. 500 at 5% simple interest per year. After some time, he borrowed Tk. 400 at
1
3 % simple interest per year for the second time. Six months after the second time borrowing, he
2
repaid both borrowed money along with interest and the amount repaid was Tk. 994.50. How many
years after the first time borrowing Mr. Karim repaid the borrowed money? Answer : 3.5 years
(Bank Asia MTO 17, NRB Bank MTO 19)
Abyev`t wgt Kwig evwl©K 5% mij my‡` 500 UvKv FY †bb | wKQz mgq ci wZwb evwl©K 3% mij my‡` 400 UvKv FY †bb|
wØZxqevi FYMÖn‡Yi 6 gvm ci wZwb my`vm‡j Dfq FY †kva K‡ib| wZwb †gvU 994.50 UvKv †kva K‡ib cÖ_gevi FYMÖn‡Yi KZ
eQi ci wZwb FY‡kva K‡ib?
3. A team of 2 men and 5 women complete 1/4th of a job in 3 days. After that another man joins them
and they all complete the next 1/4th of the job in 2 days. How many men can complete the whole job
in 4 days? Answer: 6 men.
[BB AD 14, PKSF AM 09, NCC Bank Officer 02, One Bank SCO 18, NRB Bank MTO 19]
Abyev`t 2 Rb cyiæl Ges 5 Rb gwnjvi GKwU `j 3 w`‡b GKwU Kv‡Ri 1/4 Ask m¤c~Y© K‡i| 3w`b ci GKRb AwZwi³ cyiæl
Kv‡R †hvM w`‡j Zviv 2 w`‡b AviI 1/4 Ask m¤cbœ K‡ib| KZRb cyiæl wg‡j evwK KvR 4 w`‡b m¤cbœ Ki‡Z cvi‡eb?
4. The total price of cricket bat and ball is Tk. 1350. If the price of bat increases 25% and the price of ball
decreases 20% then the total price of bat and ball remains same. Find the price of bat and ball.
অনুবােঃ নক্রতকট বযাট ও বতলর সবণতমাট োম 1350 টাকা। বযাতটর োম 25% বাডতল এবং বতলর োম 20% কতম পগতল বযাট ও বতলর
পমাট োম একই র্াতক। বযাট এবং বতলর োম পবর করুন। Answer: 600, 750

Written Math Solution (BIBM) FB Group : BD job Info. (BCS & Bank)
Bank Job Solution (BIBM) 17

Created by Sumon K. Sarkar  FB Group : BD job Info. (BCS & Bank)  Created by Sumon K. Sarkar 
Modhumoti Bank Ltd.
Created by Sumon K. Sarkar  FB Group : BD job Info. (BCS & Bank)  Created by Sumon K. Sarkar 

Post Name: Probationary Officer Exam Date : 2018


1. Mr B invests Tk 2400 in the bank at 5% interest. How much additional money must be invested at 8%
interest so that the total interest will be equal to 6% of his entire investment? Answer: Tk1200
[UCBL Officer 11, Modhumoti PO 18, SJIB TSO 18]
Abyev`t B mv‡ne evwl©K kZKiv 5 UvKv mij gybvdvq 2400 UvKv wewb‡qvM K‡ib| evwl©K kZKiv 8 UvKv mij gybvdvqAvi KZ
UvKv wewb‡qvM Ki‡j wZwb †gvU wewb‡qv‡Mi Dci GK‡Î evwl©K kZKiv 6 UvKv mij gybvdv cv‡eb?
2. Salesman is paid a monthly salary of tk 15,000 plus 12.5% commission on all of his
sales .What should be his total annual sales in TK so that his annual earning from salary and
commission is tk 2,65,000 ? Answer:680,000
Abyev`t GKRb weµq Kg©x‡K cÖwZ gv‡m 15000 UvKv I Zvi †gvU weµ‡qi Dci 12.5% Kwgkb †`qv nq| Zv‡K eQ‡i KZ
UvKvi cb¨ wewµ Ki‡Z n‡e hv‡Z Zvi †eZb I Kwgkb wg‡j 265000 UvKv nq?
3. The perimeter of a square field is equal to the perimeter of a rectangle field. Length of the rectangle
fields is 3 times of its width and the area is 768 square meter. How many square sized tiles of 80
centimetre wide will be required to cover the square field? [Social Islamic Bank-PO-2017] Answer:1600
Abyev`t GKwU eM©vKvi gv‡Vi cwimxgv GKwU AvqZKvi gv‡Vi cwimxgvi mgvb| AvqZ‡¶ÎwUi ˆ`N©¨ Gi cÖ‡¯’i wZb¸Y Ges Gi
†¶Îdj 768 eM©wgUvi| eM©vKvi gvVwU‡K m¤ú~Y©iƒ‡c XvK‡Z 80 †m.wg KZ¸‡jv eM©vKvi UvBjm jvM‡e?
4 9 25
4. Solve the equation: + = Answer: -3/5
2𝑥+1 3𝑥+2 5𝑥+4

Standard Bank Ltd.


Trainee Assistant Officer(Cash) Exam Date : 2018
1. A merchant purchased a jacket for Tk. 60 and then determined a selling price that equaled the purchase
price of the jacket plus a markup that was 25 percent of the selling price. During a sale, the merchant
discounted the selling price by 20 percent and sold the jacket. What was the merchant's gross profit on
this sale? [Standard Bank TAO Cash 18, Dhaka Bank TO 18] Answer: Tk 4
Abyev`t GK e¨emvqx GKwU R¨v‡KU 60 UvKvq µq K‡i Ges weµq g~j¨ wba©viY K‡i †hUvq µqg~j¨ mv‡_ weµqg~‡j¨I 25%
Av‡Q| weµ‡qi mgq e¨emvqx 20% Qvo w`‡q R¨v‡KUwU weµq K‡i| weµq Gi Dci e¨emvqxi †gvU jvf KZ wQj?
2. If 12 candies are sold for Tk. 10 then there is a loss of x%. If 12 candies are sold for Tk. 12 then there
is a profit of x%. What is the value of x? [Dhaka Bank Cash 18, South East Bank PO 17. Standard Bank TAO Cash 18] Answer:9.09
Abyev`t hw` 10 UvKvq 12wU PK‡jU weµq Kiv nq Zvn‡j X% ¶wZ nq| hw` 12 UvKvq 12wU PK‡jU weµq Kiv nq Zvn‡j X%
jvf nq| X Gi gvb KZ?

Standard Bank Ltd.


Post Name: Trainee Assistant Officer Exam Date : 2018
1. In a mixture 60 liters, the ratio of milk and water 2:1. If this ratio is to be 1: 2, then estimate the quantity
of water in liter to be further added in the mixture. Answer: 60 liters
Abyev`t 60 wjUv‡ii GKwU wgkª‡b `ya Avi cvwbi AbycvZ 2:1| GB wgkª‡Yi AbycvZ 1:2 n‡Z n‡j Avi KZ wjUvi AwZwi³ cvwb
†hvM Ki‡Z n‡e?
1
2. If x = 3+22, then the value of √𝑥 − [Standard Bank TAO 18, Janata AEO (RO) 17] Answer: 2
√𝑥

Written Math Solution (BIBM) FB Group : BD job Info. (BCS & Bank)
18

 Created by Sumon K. Sarkar  FB Group : BD job Info. (BCS & Bank)  Created by Sumon K. Sarkar 
Bank Job Solution (BIBM)
 Created by Sumon K. Sarkar  FB Group : BD job Info. (BCS & Bank)  Created by Sumon K. Sarkar 

Premier Bank Ltd.


Post: Trainee Assistant Officer Exam Date : 2018
10 1 18
1. Solve the Equation: + = Answer:15
2𝑥−5 𝑥+5 3𝑥−5
[SJIB MTO 16, SJIB TSO 18, Premier Bank TJO 18, NRB Bank MTO 19]
2. Karim and Rahim have equal amount of money. Radha has half of Rahim's money and Amena has half
of Radha's money. If you add one taka with all the money they have, it will be Tk. 100. How much
Rahim has? [Premier Bank TJO 18, BB AD 04, Standard Bank TAO Cash 16] Answer: 36
Abyev`t Kwig Ges iwn‡gi Kv‡Q mgcwigvb UvKv Av‡Q| iwn‡gi UvKvi A‡a©K ivavi Av‡Q Ges ivavi UvKvi A‡a©K Av‡gbvi Av‡Q|
hw` Zv‡`i mK‡ji UvKvi mgwói m‡½ 1 †hvM K‡ib Zvn‡j 100 UvKv n‡e| iwn‡gi Kv‡Q KZ UvKv Av‡Q?
3. TV was marked by 120% of its cost price. Then shopkeeper sold the TV at 10% discount. After that
his profit was 2400. Find the cost price. Answer: Tk. 30,000
Abyev`t wUwfi Mv‡q weµqg~j¨, µq g~‡j¨i 120% †jLv wQj| Zvici †`vKvb`vi 10% wWmKvD›U G wUwf wewµ Ki‡jb| Gi ci
Zvi 2400 UvKv jvf n‡qwQj| wUwfi µqg~j¨ †ei Kiæb|
4. A garden was 60 meter long and 20 meter wide. There was 5 meter wide 4 sided path inside the garden.
If it needs 20 taka per square meter to cover with grass, how much it will need to cover the path with
grass? [Premier Bank TJO 18, SJIB TSO 18] Answer: 14,000
Abyev`t GKwU evMvb 60 wgUvi j¤^v Ges 20 wgUvi cÖk¯Í wQj| evMv‡bi wfZ‡ii w`‡K Pvicv‡k 5 wgUvi cÖk¯Í iv¯Ív wQj|hw`
evMv‡b Nvm jvMv‡Z cÖwZ eM© wgUv‡i 20 UvKv cÖ‡qvRb nq, Zvn‡j iv¯ÍvwU Nvm jvMv‡Z †gvU KZ UvKvi cÖ‡qvRb n‡e?

NRBC Bank Ltd.


Post name: Management Trainee Officer Exam Date : 2018
8 9 7
1. Solve the equation: + = [NRBC Bank MTO 18, National Bank PO 17] Answer: 2/5
2𝑥−1 3𝑥−1 𝑥+1
2. A, B and C can do a piece of work in 16, 32 and 48 days respectively. They started working together
but C left after working 4 days and B left 2 days before the completion of work. How many days it
took to complete the work? Answer:10(4/9)
[SJIB MTO 11, SIBL PO 17, NRBC Bank MTO 18, National Bank PO 17. Langka Bangla Finance MTO 17]
Abyev`t A, B I C GKwU KvR h_vµ‡g 16, 32 Ges 48 w`‡b Ki‡Z cv‡i| Zviv GKmv‡_ KvR Ki‡Z ïiæ K‡i wKš‘ KvR ïiæ
nIqvi 4 w`‡b ci C KvR †Q‡o †`q Ges KvR †kl nIqvi 2 w`b Av‡M B KvR †Q‡o †`q| m¤ú~Y© KvRwU †kln‡Z KZw`b mgq
jv‡M?
3. The cost price of two watches taken together is Tk. 840. If by selling one at a profit of 16% and the
other at a loss of 12%, there is no loss or gain in the whole transaction, find the cost price of the two
watches. [SJIB MTO 11, SIBL PO 17, NRBC Bank MTO 18] Answer: Tk. 360 and Tk. 480
Abyev`t GKm‡½ †Kbv 2wU Nwoi µqg~j¨ GK‡Î 840 UvKv| hw` GwU‡K 16% jv‡f Ges Ab¨wU‡K 12% ¶wZ‡Z weµq Ki‡j
†gv‡Ui Dci †Kvb jvf ev ¶wZ nq bv Zvn‡j Nwo `zBwUi cÖwZwUi µqg~j¨ wbY©q Ki|
4. The perimeter of a square field is equal to the perimeter of a rectangle field. The length of the rectangle
is thrice the width of it and the area is 768 square meters. How many square sized tiles of 80 centimeters
will be required to cover the square filed? Answer:1600
[Al-Arafah MTO 16, National Bank PO 17, NRBC Bank MTO 18, SIBL PO 17, Modhumoti PO 18]
Abyev`t GKwU eM©vKvi gv‡Vi cwimxgv GKwU AvqZKvi gv‡Vi cwimxgvi mgvb| AvqZ‡¶ÎwUi ˆ`N©¨ Gi cÖ‡¯’i wZb¸Y Ges Gi
†¶Îdj 768 eM©wgUvi| eM©vKvi gvVwU‡K m¤ú~Y©iƒ‡c XvK‡Z 80 †m.wg KZ¸‡jv eM©vKvi UvBjm jvM‡e?

Written Math Solution (BIBM) FB Group : BD job Info. (BCS & Bank)
Bank Job Solution (BIBM) 19

Created by Sumon K. Sarkar  FB Group : BD job Info. (BCS & Bank)  Created by Sumon K. Sarkar 
Shahjalal Islami Bank Ltd.
Created by Sumon K. Sarkar  FB Group : BD job Info. (BCS & Bank)  Created by Sumon K. Sarkar 

Post name: Trainee Officer (Cash) Exam Date : 2018


4 9 25
1. Solve the equation: + = [Al-Arafah MTO 16, Modhumoti PO 18, SJIB TO Cash 18] Answer: -3/5
2𝑥+1 3𝑥+2 5𝑥+4
2. Train A leaves Dhaka at 4 pm & travels at the rate of 20 miles/h. Train B leaves Dhaka at 9pm &
travels on parallel line at the rate of 30 miles/h. How many hours after starting journey & at what
distance train B will overtake train A? Answer: 10 hours and 300 miles
অনুবােঃ পট্রন A ঢাকা পর্তক নবকাল 4 টায পছতড যায এবং 20 মাইল/ ন্টা পবতগ ভ্রমর্ কতর। পট্রন B ঢাকা পর্তক রাে 9 টায পছতড যায
এবং সমান্তরাল লাইতন 30 মাইল/ ন্টা পবতগ ভ্রমর্ কতর। যাত্রা শুরু করার কে ন্টা পতর এবং কে েূরতত্ব B পট্রন A পক ছানডতয যাতব?
3. A sum of Tk 2000 was invested in two parts one at 5% interest rate and another one at 10% interest
rate. If the total annual interest earned was Tk. 160, then how much money was invested at 10% interest
rate? Answer: Tk. 1200
অনুবােঃ 2000 টাকা দুনট অংতশ্ একনট 5% সুতের হাতর এবং অনযনট 10% সুতের হাতর নবননতযাগ করা হতযনছল। যনে পমাট বানেণক সুে
অনেণে হতো টাকা। 160, োহতল 10% সুতের হাতর কে টাকা নবননতযাগ করা হতযনছল?
4. A fruit seller bought 20 kg mangoes for Tk 100 per kg and an equal amount of apples for Tk. 150 per
kg. If he sells all the fruits at Tk. 140 per kg. What will be his profit or loss percentage? Answer: 12%
Abyev`t GKRb dj we‡µZv 20 †KwR Avg cÖwZ †KwR 100 UvKv `‡i Ges cÖwZ †KwR 150 UvKv `‡i mgvb msL¨K Av‡cj µq
K‡ib| hw` †m me¸‡jv dj 140 UvKv cÖwZ †KwR wn‡m‡e weµq K‡i Zvn‡j Zvi kZKiv KZ jvf ev ¶wZ n‡e?

Shahjalal Islami Bank Ltd.


Post Name: Trainee Officer Exam Date : 2018
10 1 18
1. Solve the equation: + = Answer: 15
2𝑥−5 𝑥+5 3𝑥−5
[SJIB MTO 16, SJIB TSO 18, Premier Bank TJO 18, NRB Bank MTO 19]
2. A team of 2 men and 5 women completed 1/4th of a job in 3 days. After 3 days, another man joined the
team and they took 2 days to complete another 1/4 th of the job. How many men can complete the
remaining job in 4 days? [BB AD 14, PKSF AM 09, NCC Bank Officer 02, One Bank SCO 18, NRB Bank MTO 19] Answer: 3 men.
Abyev`t 2 Rb cyiæl Ges 5 Rb gwnjvi GKwU `j 3 w`‡b GKwU Kv‡Ri 1/4 Ask m¤c~Y© K‡i| 3w`b ci GKRb AwZwi³ cyiæl
Kv‡R †hvM w`‡j Zviv 2 w`‡b AviI 1/4 Ask m¤cbœ K‡ib| KZRb cyiæl wg‡j evwK KvR 4 w`‡b m¤cbœ Ki‡Z cvi‡eb?
3. Mr. B invests Tk. 2400 at 5% interest annually. How much additional money needs to invest at 8%
interest to earn overall interest at 6% on entire amount? Answer: Tk.1200
[UCBL Officer 11, Modhumoti PO 18, SJIB TSO 18]
Abyev`t B mv‡ne evwl©K kZKiv 5 UvKv mij gybvdvq 2400 UvKv wewb‡qvM K‡ib| evwl©K kZKiv 8 UvKv mij gybvdvqAvi KZ
UvKv wewb‡qvM Ki‡j wZwb †gvU wewb‡qv‡Mi Dci GK‡Î evwl©K kZKiv 6 UvKv mij gybvdv cv‡eb?
4. A garden was 60 meter long and 20 meter wide. There was 5 meter wide 4 sided path inside the garden.
If it needs 20 taka per square meter to cover with grass, how much it will need to cover the path with
grass? [Premier Bank TJO 18, SJIB TSO 18] Answer:Tk.14000
Abyev`t GKwU evMvb 60 wgUvi j¤^v Ges 20 wgUvi cÖk¯Í wQj| evMv‡bi wfZ‡ii w`‡K Pvicv‡k 5 wgUvi cÖk¯Í iv¯Ív wQj|hw`
evMv‡b Nvm jvMv‡Z cÖwZ eM© wgUv‡i 20 UvKv cÖ‡qvRb nq, Zvn‡j iv¯ÍvwU Nvm jvMv‡Z †gvU KZ UvKvi cÖ‡qvRb n‡e?

Written Math Solution (BIBM) FB Group : BD job Info. (BCS & Bank)
20

 Created by Sumon K. Sarkar  FB Group : BD job Info. (BCS & Bank)  Created by Sumon K. Sarkar 
Bank Job Solution (BIBM)
 Created by Sumon K. Sarkar  FB Group : BD job Info. (BCS & Bank)  Created by Sumon K. Sarkar 

One Bank Ltd.


Post Name: Special Cadre Officer Exam Date : 2018
1. A sum of money is to be distributed equally among a group of children. If there were 25 children less
than each would get Tk. 1.50 more, and if there 50 children more, each would get TK. 1.50 less. Find
the number of children and the amount of money distributed. Answer: Tk. 450
[One Bank SCO 18, National Bank PO 15]
Abyev`t KZ¸‡jv wkïi g‡a¨ wKQz UvKv mgvbfv‡e fvM K‡i †`qv nq| hw` 25wU wkï Kg _vKZ Zvn‡j cÖ‡Z¨‡K 1.50 UvKv †ewk
†cZ Ges hw` 50wU wkï †ewk _vKZ Zvn‡j cÖ‡Z¨‡K 1.50 UvKv Kg †cZ| KZwU wkï wQj Ges KZ UvKv †`qv n‡q‡Q, Zv wbY©q Ki|
2. A team of 2 men and 5 women completed 1/4th of a job in 3 days. After that another man joined them
and they all complete the next 1/4th of the job in 2days. How many men can complete the whole job
in 4 days? [BB AD 14, PKSF AM 09, NCC Bank Officer 02, One Bank SCO 18, NRB Bank MTO 19] Answer: 6 men.
Abyev`t 2 Rb cyiæl Ges 5 Rb gwnjvi GKwU `j 3 w`‡b GKwU Kv‡Ri 1/4 Ask m¤c~Y© K‡i| 3w`b ci GKRb AwZwi³ cyiæl
Kv‡R †hvM w`‡j Zviv 2 w`‡b AviI 1/4 Ask m¤cbœ K‡ib| KZRb cyiæl wg‡j evwK KvR 4 w`‡b m¤cbœ Ki‡Z cvi‡eb?
3. In 2017, the number of product that a company sold to retailers decreased by 20%, while the price per
unit increased by 20% from that of previous year. Company's revenue from the sales of the product in
2017 was Tk. 6,00,000. Find out the difference between the sales revenue of the year 2017 and 2016.
Abyev`t 2017 mv‡j LyPiv we‡µZv‡`i Kv‡Q wewµZ c‡Y¨i cwigvY 20% K‡g hvq, ‡hLv‡b weµqg~j¨ c~‡e©i eQ‡ii Zzjbvq 20%
e„w× cvq| cY¨ weµq n‡Z †Kv¤cvwb KZ©…K †gvU weµq ivR‡¯^i cwigvY 600000 UvKv wQj| 2016 I 2017 mv‡ji weµq
ivR‡¯^i g‡a¨ cv_©K¨ wbY©q Kiæb| Answer: 25,000
𝑥+3 𝑥+4 𝑥+5 𝑥+6
4. Solve the problem: − = − Answer: -7/2
𝑥+2 𝑥+3 𝑥+4 𝑥+5

Lanka Bangla Finance


Post Name: Management Trainee Officer Exam Date : 2017
1 1 1 1
1. Solve the problem: + = + [SIBL PO 17, Langka Bangla Finance MTO 17] Answer: -3(1/2)
𝑥+2 𝑥+5 𝑥+4 𝑥+3
2. A, B and C can do a piece of work in 16, 32 and 48 days respectively. They started working together
but C left after working 4 days and B left 2 days before the completion of work. How many days it
took to complete the work? Answer: 10 (4/9)
[SJIB MTO 11, SIBL PO 17, NRBC Bank MTO 18, National Bank PO 17, Langka Bangla Finance MTO 17]
Abyev`t A, B I C GKwU KvR h_vµ‡g 16, 32 Ges 48 w`‡b Ki‡Z cv‡i| Zviv GKmv‡_ KvR Ki‡Z ïiæ K‡i wKš‘ KvR ïiæ
nIqvi 4 w`‡b ci C KvR †Q‡o †`q Ges KvR †kl nIqvi 2 w`b Av‡M B KvR †Q‡o †`q| m¤ú~Y© KvRwU †kln‡Z KZw`b mgq jv‡M?
3. The length of a rectangular field is 1.5 times as large as its width. An amount of Tk. 10260 was needed
to cover the field with grass at 1.9Tk per sq. meter. How much would it cost to fence the four sides of
the rectangular field at the rate of Tk 2.5 per meter? Answer: 750
[ONE Bank SCO 17, Bank Asin MTO 17, SJIB MTO 16, Langka Bangla Finance MTO 17 Board Book]
Abyev`t GKwU AvqZKvi gv‡Vi ˆ`N¨© Gi cÖ‡¯’i †`o ¸Y| cÖwZ eM©wgUv‡i 1.9 UvKv wnmv‡e m¤ú~Y© gvVwU‡Z Nvm jvMv‡Z 10,260
UvKv cÖ‡qvRb| cÖwZ wgUvi 2.5 UvKv wnmv‡e gvVwUi Pvicv‡k †eov w`‡Z KZ UvKv LiP n‡e?
4. A man has to go 10 km to catch a bus. He walks part of the way at 7 km per hour and runs the rest of
the way at 12 km per hour. If he takes 1 hour 15 minutes to complete his journey. Find how far he
walked? [Modhumoti Bank MTO 16, ONE Bank SCO 17, Langka Bangla Finance MTO 17] Answer: 7 km
Abyev`t †Kvb e¨w³‡K evm aivi Rb¨ 10 wK.wg †h‡Z nq| c‡_i wKQz Ask wZwb 7 wK.wg/N›Uv †e‡M †nu‡U Ges evwK Ask
12wK.wg/N›Uv †e‡M †`Š‡o cvi K‡ib| hw` GB c_ †h‡Z Zvi †gvU 1 N›Uv 15 wgwbU mgq jv‡M Zvn‡j wZwb KZUzKz c_ nuv‡Ub Zv
wbY©q Ki|

Written Math Solution (BIBM) FB Group : BD job Info. (BCS & Bank)
Bank Job Solution (BIBM) 21

Created by Sumon K. Sarkar  FB Group : BD job Info. (BCS & Bank)  Created by Sumon K. Sarkar 
National Bank Ltd.
Created by Sumon K. Sarkar  FB Group : BD job Info. (BCS & Bank)  Created by Sumon K. Sarkar 

Post Name: Probationary Officer Exam Date : 2017


8 9 7
1. Solve the problem: + = [NRBC Bank MTO 18, National Bank PO 17] Answer: 2/5
2𝑥−1 3𝑥−1 𝑥+1
2. Aman, Belal and Chad started a small business with a total amount tk 28,000. Aman paid tk 4500 more
than Belal and Belal paid tk 7000 less than Chad. If the company made a profit of tk 5600 .How much
profit should Belal receive? [SUBB MTO 16, National Bank PO 17] Answer: Tk.1100
Abyev`t Avgvb, ‡ejvj I Pvu` †gvU 28000 UvKv wb‡q GKwU e¨emv ïiæ K‡i| ‡ejvj hZ UvKv †`q Avgvb Zvi †P‡q 4500 UvKv
†ewk †`q Ges Pvu` hZ UvKv †`q ‡ejvj Zvi †P‡q 7000 UvKv Kg †`q| hw` Zv‡`i †Kv¤úvwbi 5600 UvKv jvf nq Zvn‡j ‡ejvj
KZ UvKv jf¨vsk cv‡e?
3. A, B and C can do a piece of work in 16, 32 and 48 days respectively. They started working together
but C left after working 4 days and B left 2 days before the completion of work. How many days it
took to complete the work? Answer: 10 (4/9)
[SJIB MTO 11, SIBL PO 17, NRBC Bank MTO 18, National Bank PO 17. Langka Bangla Finance MTO 17]
Abyev`t A, B I C GKwU KvR h_vµ‡g 16, 32 Ges 48 w`‡b Ki‡Z cv‡i| Zviv GKmv‡_ KvR Ki‡Z ïiæ K‡i wKš‘ KvR ïiæ
nIqvi 4 w`‡b ci C KvR †Q‡o †`q Ges KvR †kl nIqvi 2 w`b Av‡M B KvR †Q‡o †`q| m¤ú~Y© KvRwU †kln‡Z KZw`b mgq
jv‡M?
4. The perimeter of a square field is equal to the perimeter of a rectangle field. The length of the rectangle
is thrice the width of it and the area is 768 square meters. How many square sized tiles of 80 centimeters
will be required to cover the square filed? Answer: 1600
[Al-Arafah MTO 16, National Bank PO 17, NRBC Bank MTO 18, SIBL PO 17, Modhumoti PO 18]
Abyev`t GKwU eM©vKvi gv‡Vi cwimxgv GKwU AvqZKvi gv‡Vi cwimxgvi mgvb| AvqZ‡¶ÎwUi ˆ`N©¨ Gi cÖ‡¯’i wZb¸Y Ges Gi
†¶Îdj 768 eM©wgUvi| eM©vKvi gvVwU‡K m¤ú~Y©iƒ‡c XvK‡Z 80 †m.wg KZ¸‡jv eM©vKvi UvBjm jvM‡e?

Bank Asia Ltd.


Post name: Management Trainee Officer Exam Date : 2017
1. Mr. Karim borrowed Tk. 500 at 5% simple interest per year. After some time, he borrowed Tk. 400 at
1
3 % simple interest per year for the second time. Six months after the second time borrowing, he repaid
2
both borrowed money along with interest and the amount repaid was Tk. 994.50. How many years
after the first time borrowing Mr. Karim repaid the borrowed money? Answer: 3.5 years
[Bank Asia MTO 17, NRB Bank MTO 19]
Abyev`t wgt Kwig evwl©K 5% mij my‡` 500 UvKv FY †bb | wKQz mgq ci wZwb evwl©K 3% mij my‡` 400 UvKv FY †bb|
wØZxqevi FYMÖn‡Yi 6 gvm ci wZwb my`vm‡j Dfq FY †kva K‡ib| wZwb †gvU 994.50 UvKv †kva K‡ib cÖ_gevi FYMÖn‡Yi KZ
eQi ci wZwb FY‡kva K‡ib?
2. Rahim can do a piece of work in 80 days. He works for 10 days and then Karim alone finishes the rest
of the work in 42 days. How much time would it take for the two of them together to complete the
whole work? [BB AD 06, Modlumoti Bank MTO 16, Bank Asia MTO 17] Answer: 30 days
Abyev`t Avwei GKwU KvR 80 w`‡b Ki‡Z cv‡i| †m 10 w`b KvR Kivi ci evkvi GKvB evwK KvR 42 w`‡b †kl K‡i| Zviv
`yÕR‡b GKmv‡_ KvR Ki‡j m¤ú~b© KvRwU Ki‡Z Zv‡`i KZw`b mgq jvM‡e?
3. The length of a rectangular field is 1.5 times as large as its width. An amount of Tk. 10260 was needed
to cover the field with grass at 1.9Tk per sq. meter. How much would it cost to fence the four sides of
the rectangular field at the rate of Tk. 2.5 per meter? Answer: 750
[ONE Bank SCO 17, Bank Asia MTO 17, SJIB MTO 16, Langka Bangla Finance MTO 17 Board Book]

Written Math Solution (BIBM) FB Group : BD job Info. (BCS & Bank)
22

 Created by Sumon K. Sarkar  FB Group : BD job Info. (BCS & Bank)  Created by Sumon K. Sarkar 
Bank Job Solution (BIBM)
 Created by Sumon K. Sarkar  FB Group : BD job Info. (BCS & Bank)  Created by Sumon K. Sarkar 

Abyev`t GKwU AvqZKvi gv‡Vi ˆ`N¨© Gi cÖ‡¯’i †`o ¸Y| cÖwZ eM©wgUv‡i 1.9 UvKv wnmv‡e m¤ú~Y© gvVwU‡Z Nvm jvMv‡Z 10,260
UvKv cÖ‡qvRb| cÖwZ wgUvi 2.5 UvKv wnmv‡e gvVwUi Pvicv‡k †eov w`‡Z KZ UvKv LiP n‡e?
𝑥−4 𝑥−7 𝑥−2
4. Solve the problem: + + =3 Answer: 2
𝑥−1 𝑥−3 𝑥−9

Modhumoti Bank Ltd.


Post Name: Probationary Officer Exam Date : 2017
1. A shirt sold at 6% profits.If the purchase price was 4% less and selling price was Tk 4 more,the profit
be 12.5%.What was the purchase price of the shirt. Answer: Tk. 200
[Modhumoti Bank PO-2017] [Bangladesh Bank AD-2017][SBC AM -2016] [BKB -2012][NBL-2014][BCS Written] [Non cader Written]
[BAPEX][Islamic Bank-2013] [BRC-2008], 16th BCS, MBA-2007-08;1999;1997, NCC BANK-2002, PBL-2000
অনুবােঃ একনট শ্াটণ 6% লাতভ নবনক্র হয। যনে ক্রয মূলয 4% কম হয এবং নবক্রয মূলয 4 টাকা পবনশ্ হয, োহতল লাভ 12.5%। শ্াটণনটর
ক্রয মূলয কে নছল।
2. A man's running speed is 3 times of his walking speed.He runs a distance and come back by walking
total time taken 2 hrs.what was the distance if he runs 9 miles per hour. Answer: 4.5 miles
[Modhumoti Bank PO-2017][PBL -2008]
Abyev`tGKRb †jv‡Ki †`Šov‡bvi MwZ Zvi nvUvi MwZi 3 ¸Y| †jvKwU GKwU wbw`©ó `~iZ¡ †`Š‡o wM‡q †n‡U †diZ Av‡mb| Zvi
†gvU 2 N›Uv mgq †j‡MwQj| hw` Zvi †`Šov‡bvi MwZ 9 gvBj/N›Uv nq Z‡e `~iZ¡ (gvBj) wbY©q Kiæb?
3. A,B,C started a job which they can complete in 2 days.B can do the job in 5 days and C can do it in 4
days.After working for 1 day, both B & C left. A complete testbed of the work how many days?
[Modhumoti Bank PO-2017] Answer: 10 days
Abyev`tA, B Ges C GKwU KvR GK‡Î `yB w`‡b †kl Ki‡Z cv‡i| B Ges C GB KvR h_vµ‡g 5 I 4 w`‡b †klKi‡Z cv‡i|
1 w`b KvR Kivi ci B Ges C `y'R‡bB KvR †Q‡o P‡j †Mj| evwK KvR †kl Ki‡Z A Gi KZ w`b mgq jvM‡e?
1 1 1 1
4. Solve the equation: + = + [Modhumoti Bank PO-2017] Answer:4
2𝑥−5 2𝑥−11 2𝑥−7 2𝑥−9

Social Islami Bank Ltd.


Post Name: Probationary Officer Exam Date : 2017
1. The cost price of two watches taken together is Tk. 840. If by selling one at a profit of 16% and the
other at a loss of 12%, there is no loss or gain in the whole transaction, find the cost price of the two
watches. [SJIB MTO 11, SIBL PO 17, NRBC Bank MTO 18] Answer: Tk. 360 and Tk. 480
Abyev`t GKm‡½ †Kbv 2wU Nwoi µqg~j¨ GK‡Î 840 UvKv| hw` GwU‡K 16% jv‡f Ges Ab¨wU‡K 12% ¶wZ‡Z weµq Ki‡j
†gv‡Ui Dci †Kvb jvf ev ¶wZ nq bv Zvn‡j Nwo `zBwUi cÖwZwUi µqg~j¨ wbY©q Ki|
2. A, B and C can do a piece of work in 16, 32 and 48 days respectively. They started working together
but C left after working 4 days and B left 2 days before the completion of work. How many days it
took to complete the work? Answer: 10(4/9)
[SJIB MTO 11, STBL PO 17, NRBC Bank MTO 18, National Bank PO 17. Langka Bangla Finance MTO 17]
Abyev`t A, B I C GKwU KvR h_vµ‡g 16, 32 Ges 48 w`‡b Ki‡Z cv‡i| Zviv GKmv‡_ KvR Ki‡Z ïiæ K‡i wKš‘ KvR ïiæ
nIqvi 4 w`‡b ci C KvR †Q‡o †`q Ges KvR †kl nIqvi 2 w`b Av‡M B KvR †Q‡o †`q| m¤ú~Y© KvRwU †kln‡Z KZw`b mgq
jv‡M?
3. The perimeter of a square field is equal to the perimeter of a rectangle field. The length of the rectangle
is thrice the width of it and the area is 768 square meters. How many square sized tiles of 80 centimeters
will be required to cover the square filed? Answer: 1600
[Al-Arafah MTO 16, National Bank PO 17, NRBC Bank MTO 18, SIBL PO 17, Modhumoti PO 18]

Written Math Solution (BIBM) FB Group : BD job Info. (BCS & Bank)
Bank Job Solution (BIBM) 23

Created by Sumon K. Sarkar  FB Group : BD job Info. (BCS & Bank)  Created by Sumon K. Sarkar 
Abyev`t GKwU eM©vKvi gv‡Vi cwimxgv GKwU AvqZKvi gv‡Vi cwimxgvi mgvb| AvqZ‡¶ÎwUi ˆ`N©¨ Gi cÖ‡¯’i wZb¸Y Ges Gi
Created by Sumon K. Sarkar  FB Group : BD job Info. (BCS & Bank)  Created by Sumon K. Sarkar 

†¶Îdj 768 eM©wgUvi| eM©vKvi gvVwU‡K m¤ú~Y©iƒ‡c XvK‡Z 80 †m.wg KZ¸‡jv eM©vKvi UvBjm jvM‡e?
1 1 1 1
4. Solve the problem: + = + [SIBL PO 17, Langka Bangla Finance MTO 17] Answer: -3(1/2)
𝑥+2 𝑥+5 𝑥+4 𝑥+3

One Bank Ltd.


Post Name: Special Cadre Officer Exam Date : 2017
4 15 35
1. Solve the equation: + = [ONE Bank SCO 17, Board Book] Answer: -9/13
2𝑥+3 5𝑥+4 7𝑥+6
2. Salam used a part of Tk. 1,00,000 to purchase a Television. Of the remaining portion, he invested 1/3
of it at 4% simple annual interest and 2/3 of it at 6% simple annual interest. If, after a year, the income
from two investments totaled Tk. 320, what was the purchase price of the television? Answer:94000
Abyev`t mvjvg 1,00,000 UvKvi wKQz Ask w`‡q GKwU †Uwjwfkb µq Kij| evwK UvKvi 1/3 Ask 4% my‡` Ges 2/3 Ask 6%
my‡` e¨vs‡K Rgv ivLj| eQi c‡i †m e¨vsK †_‡K GK‡Î 320 UvKv gybvdv cvq Zvn‡j †UwjwfkbwUi µqg~j¨ KZ?
3. The length of a rectangular field is 1.5 times as large as its width. An amount of Tk. 10260 was needed
to cover the field with grass at 1.9Tk per sq. meter. How much would it cost to fence the four sides of
the rectangular field at the rate of Tk 2.5 per meter? Answer: Tk 750
[ONE Bank SCO 17, Bank Asia MTO 17, SJIB MTO 16, Langka Bangla Finance MTO 17 Board Book]
Abyev`t GKwU AvqZKvi gv‡Vi ˆ`N¨© Gi cÖ‡¯’i †`o ¸Y| cÖwZ eM©wgUv‡i 1.9 UvKv wnmv‡e m¤ú~Y© gvVwU‡Z Nvm jvMv‡Z 10,260
UvKv cÖ‡qvRb| cÖwZ wgUvi 2.5 UvKv wnmv‡e gvVwUi Pvicv‡k †eov w`‡Z KZ UvKv LiP n‡e?
4. A man has to go 10 km to catch a bus. He walks part of the way at 7 km per hour and runs the rest of
the way at 12 km per hour. If he takes 1 hour 15 minutes to complete his journey. Find how far he
walked? [Modhumoti Bank MTO 16, ONE Bank SCO 17, Langka Bangla Finance MTO 17] Answer: 7km
Abyev`t †Kvb e¨w³‡K evm aivi Rb¨ 10 wK.wg †h‡Z nq| c‡_i wKQz Ask wZwb 7 wK.wg/N›Uv †e‡M †nu‡U Ges evwK Ask
12wK.wg/N›Uv †e‡M †`Š‡o cvi K‡ib| hw` GB c_ †h‡Z Zvi †gvU 1 N›Uv 15 wgwbU mgq jv‡M Zvn‡j wZwb KZUzKz c_ nuv‡Ub Zv
wbY©q Ki|

Modhumoti Bank Ltd.


Post Name: Probationary Officer Exam Date : 2016
2 2 2 3 3 3 3 3 2
1. If x+y=a, x + y =b and x + y = c , then show that a + 2c = 3ab
[Modhumoti MTO 16, Premier Bank JO 19, Board Book]
1 5
2. In a school, there are equal number of boys and girls. Among the students of the girls and th of the
8 6
boys are residing in the hostel. What percent of the Students consists of boys who do not reside in the
hostel among all students? [Modhumoti Bank MTO 16, SJIB MTO 10] Answer: 8.33%
অনুবােঃ একনট স্কুতল পছতল ও পমতয সমান সংখযক। ছাত্রতের মতযয 1/8 অংশ্ পমতয এবং 5/6 অংশ্ পছতল ছাত্রাবাতস র্াতক। ছাত্রতের কে
শ্োংশ্ পছতলরা সকল ছাত্রতের মতযয পহাতস্টতল র্াতক না?
3. A man has to go 10 km to catch a bus. He walks part of the way at 7 km per hour and runs the rest of
the way at 12 km per hour. If he takes 1 hour 15 minutes to complete his journey. Find how far he
walked? [Modhumoti Bank MTO 16, ONE Bank SCO 17, Langka Bangla Finance MTO 17] Answer:7 km
Abyev`t †Kvb e¨w³‡K evm aivi Rb¨ 10 wK.wg †h‡Z nq| c‡_i wKQz Ask wZwb 7 wK.wg/N›Uv †e‡M †nu‡U Ges evwK Ask
12wK.wg/N›Uv †e‡M †`Š‡o cvi K‡ib| hw` GB c_ †h‡Z Zvi †gvU 1 N›Uv 15 wgwbU mgq jv‡M Zvn‡j wZwb KZUzKz c_ nuv‡Ub Zv
wbY©q Ki|

Written Math Solution (BIBM) FB Group : BD job Info. (BCS & Bank)
24

 Created by Sumon K. Sarkar  FB Group : BD job Info. (BCS & Bank)  Created by Sumon K. Sarkar 
Bank Job Solution (BIBM)
 Created by Sumon K. Sarkar  FB Group : BD job Info. (BCS & Bank)  Created by Sumon K. Sarkar 

4. Abir can do a piece of work in 80 days. He works for 10 days and then Basher alone finishes the rest
of the work in 42 days. How much time would it take for the two of them together to complete the
whole work? [BB AD 06, Modlumoti Bank MTO 16, Bank Asia MTO 17] Answer:30 days
Abyev`t Avwei GKwU KvR 80 w`‡b Ki‡Z cv‡i| †m 10 w`b KvR Kivi ci evkvi GKvB evwK KvR 42 w`‡b †kl K‡i| Zviv
`yÕR‡b GKmv‡_ KvR Ki‡j m¤ú~b© KvRwU Ki‡Z Zv‡`i KZw`b mgq jvM‡e?
5. ABC is a triangle in which AB = 3cm, BC = 5cm, and AC = 4cm, AD is a perpendicular from A to
BC. Find the length of AD. [Modbumoti MTO 16, Premier Bank JO 19] Answer:2.4 cm

Modhumoti Bank Ltd.


Post Name: Probationary Officer Exam Date : 2016
1. A, B, C started a partnership business in where A's share: B's share = 1:2 and B's share: C's share =
3:4. If the total profit is Tk. 1105 then how much money will A get? Answer: Tk 195
Abyev`t A, B Ges C GKwU e¨emv ïiæ K‡i †hLv‡b A Gi wewb‡qvM t B Gi wewb‡qvM = 1 t 2 Ges B Gi wewb‡qvM t C Gi
wewb‡qvM = 3 t 4| hw` †gvU jv‡fi cwigvb 1,105 UvKv nq Zvn‡j A KZ UvKv cv‡e?
2. Daily earnings of two persons are in the ratio 4:5 and their daily expense 7:9, if each saves Tk. 50 per
day. What is their income? Answer: Tk. 400 and Tk. 500
Abyev`t 2 Rb e¨w³i ˆ`wbK Av‡qi AbycvZ 4 t 5 Ges Li‡Pi AbycvZ 7 t 9| hw` cÖ‡Z¨‡KB cÖwZw`b 50 UvKv Rgv K‡ib Zvn‡j
Zv‡`i Avq KZ?
3. A boat can take 8 hour to go 32 km against the current and take 4 hour for same distance with the
current, what is the speed of the boat and current? [IFIC Bank PO 11, Modhunoti Bank MTO 16]
Abyev`t †mªv‡Zi wecix‡Z GKwU †bŠKv 8 N›Uvq 32 wK.wg hvq| †hLv‡b †mªv‡Zi AbyK~‡j H GKB c_ †h‡Z gvÎ 4 N›Uv mgq jv‡M|
†bŠKv I †mªv‡Zi †eM KZ? Answer: 2 km/h and 6 km/h

Standard Bank Ltd.


Post Name: Trainee Assistant Officer(Cash) Exam Date : 2016
1. A can dig a pond in 30 days and B can dig the same pond in 20 days. In how many days A and B can
dig the pond if they work together? [Combined Cash 08, Standard Bank TAO Cash 16] Answer: 12 days
Abyev`t A GKwU cyKzi 30 w`‡b Ges B GKB cyKzi 20 w`‡b Lbb Ki‡Z cv‡i| Zviv GK‡Î H cyKziwU KZ w`‡b Lbb Ki‡Z
cvi‡e?
2. Karim and Rahim have equal amount of money. Radha has half of Rahiin's money and Amena bas half
of Radha's money. If you add one taka with all the money they have, it will be Tk. 100. How much
Rahim has?[Preinier Bank TJO 18, BB AD 04, Standard Bank TAO Cash 16] Answer: Tk. 36
Abyev`t Kwig Ges iwn‡gi Kv‡Q mgcwigvb UvKv Av‡Q| iwn‡gi UvKvi A‡a©K ivavi Av‡Q Ges ivavi UvKvi A‡a©K Av‡gbvi Av‡Q|
hw` Zv‡`i mK‡ji UvKvi mgwói m‡½ 1 †hvM K‡ib Zvn‡j 100 UvKv n‡e| iwn‡gi Kv‡Q KZ UvKv Av‡Q?

Written Math Solution (BIBM) FB Group : BD job Info. (BCS & Bank)
Bank Job Solution (BIBM) 25

Created by Sumon K. Sarkar  FB Group : BD job Info. (BCS & Bank)  Created by Sumon K. Sarkar 
3. A casher received a total amount of Tk 10,000 from depositor in a total of 140 notes of Tk. 50 and Tk.
Created by Sumon K. Sarkar  FB Group : BD job Info. (BCS & Bank)  Created by Sumon K. Sarkar 

100 denominations. How many notes of Tk. 50 denominations did he receive? Answer: 80
4. The simple interest rate of a bank was reduced to 5% from 7%. As a consequences Mr. B's income was
reduced by Tk. 2100 in 5 years. How much is Mr. B's initial deposit in the bank? Answer: Tk. 21,000
[BB AD 01, Standard Bank TAO Cash 16]
Abyev`t my‡`i nvi 7% †_‡K K‡g 5% nIqvq Mr. B e¨w³i my` 5 eQ‡i 2,100 UvKv K‡g †Mj| Mr. B Zvi g~jab KZ?
5. A garden is 60 meter long and 20 meter wide. Inside the garden there is a 5 meter wide path around it.
What is the area of the path in square meter? [Standard Bank TAO Cash 16] Answer: 700
Abyev`t GKwU evMvb 60 wgUvi j¤^v Ges 20 wgUvi cÖk¯Í wQj| evMv‡bi wfZ‡ii w`‡K Pvicv‡k 5 wgUvi cÖk¯Í iv¯Ív wQj|hw`
evMv‡b Nvm jvMv‡Z cÖwZ eM© wgUv‡i 20 UvKv cÖ‡qvRb nq, Zvn‡j iv¯ÍvwU Nvm jvMv‡Z †gvU KZ UvKvi cÖ‡qvRb n‡e?

Standard Bank Ltd.


Post Name: Trainee Assistant Officer Exam Date : 2016
1. A sales person earns Tk. 250 fixed salary. He also gets 15% commission on total sales. What will be
sales volume, if he wants to earn total Tk. 1000? Answer: Tk. 5000
Abyev`t GKRb weµqKg©xi g~j †eZb 250 UvKv| wZwb Zvi †gvU wewµi Dci 15% Kwgkb cvb| hw` wZwb 1,000 UvKv Avq
Ki‡Z Pvb Zvn‡j Zv‡K wK cwigvb wewµ Ki‡Z n‡e?
2. A trader, while selling an item, was asking for such a price that would enable him to offer a 20%
discounts and still inake a profit of 30% on cost. If the cost of the item was Tk. 50 wbat was his asking
price? [BB AD 08, Standard Bank TAO 16, Premier Bank JO 19] Answer: Tk 81.25
Abyev`t GKRb we‡µZv GKwU c‡Y¨i Ggb GKwU `vg e‡jb hv‡Z cY¨wU‡Z 20% wWmKvD›U w`‡jI we‡µZvi 30% jvf nq| hw`
cY¨wUi g~j¨ 50 UvKv nq Zvn‡j we‡µZv KZ `vg e‡jwQ‡jb?
3. Two banks offer interest rate of 6% and 7% respectively, on Fixed Deposit. Mr. Rahman deposited a
total amount of Tk. 4000 in the banks and in one year his interest income was Tk. 250. How much
money was deposited in the bank with 7% interest? Answer: Tk. 1000
Abyev`t ingvb mv‡ne 4,000 UvKv 6% I 7% my‡` Avjv`v fv‡e mÂq K‡ib| 1 eQi ci wZwb hw` GK‡Î 250 UvKv gybvdv cvb
Zvn‡j 7% nv‡i KZ UvKv Rgv †i‡LwQ‡jb?
4. A pipe can fill a tank in 30 minutes and other can drain out the tank in 40 minutes. If both pipes are
started same time, how much time will be required to fill half of the tank? Answer: 60 minutes
Abyev`t GKwU bj 30 wgwb‡U GKwU †PŠev”Pv cvwbc~Y© K‡i| Aci GKwU bj 40 wgwb‡U †PŠev”PvwU‡K cvwb k~b¨ K‡i| 2wU bj
GK‡Î Pvjy Kiv n‡j, †PŠev”PvwUi A‡a©K cvwbc~Y© Ki‡Z KZ mgq jvM‡e?
5. The length of a rectangular field is 3 feet long than its breadth. If the perimeter of rectangular field is
380 feet. What will be area of the field in square feet? Answer: 9022.75 Sq. feet
[SJIB TSO 16]
Abyev`t GKwU AvqZKvi gv‡Vi ˆ`N¨© Gi cÖ‡¯’i †P‡q 3 dzU j¤^v| AvqZ‡¶ÎwUi cwimxgv 380 dzU n‡j Gi †¶Îdj KZ eM©dzU?

Shahjalal Islami Bank Ltd.


Post Name: Trainee Officer (Cash) Exam Date : 2016
1 𝑥
1. If 𝑥 + = 2 , then the value of Answer: 1
𝑥 𝑥 2 +𝑥−1
2. Age of three persons is now in the proportion 2: 3: 4 and in 5 years from now, the proportion will be
5: 7: 9. What is the present age of the youngest person? Answer: 20 years

Written Math Solution (BIBM) FB Group : BD job Info. (BCS & Bank)
26

 Created by Sumon K. Sarkar  FB Group : BD job Info. (BCS & Bank)  Created by Sumon K. Sarkar 
Bank Job Solution (BIBM)
 Created by Sumon K. Sarkar  FB Group : BD job Info. (BCS & Bank)  Created by Sumon K. Sarkar 

Abyev`t wZb e¨w³i eZ©gvb eq‡mi AbycvZ 2: 3: 4 Ges GLb †_‡K 5 eQi ci AbycvZ n‡e 5: 7: 9| me‡P‡q Kg eqmx e¨w³i
eZ©gvb eqm KZ?
3. Mr. X can finish a work in 6 days and Mr. Y can finish the same work in 8 days. How many days will
it take to finish the work it they work together? Answer: 24/7 days
Abyev`t Mr. X GKwU KvR 6 w`‡b Ges Y GKB KvR 8 w`‡b Ki‡Z cv‡i| Zviv GK‡Î H KvR KZ w`‡b †kl Ki‡Z cvi‡e?
4. A shirt was sold for Tk 171 and the gain was as much percent as it costs in Taka amount. What was
the purchase price of the shirt? Answer: Tk. 90
Abyev`t GKwU kvU© 171 UvKvq weµq Kivq µqg~‡j¨i mgcwigvb kZKiv jvf Kiv nj| kvU©wUi µqg~j¨ KZ?

Shahjalal Islami Bank Ltd.


Post Name: Trainee Senior Officer Exam Date : 2016
1 2 1
1. If √𝑥 + = 𝑎, then 𝑥 + =? Answer: a4 – 4a2+ 2
√𝑥 𝑥2
2. A borrower pays 8.5% interest per year on the first Tk 600 he borrows and 7.25% per year on the part
of the loan in excess of Tk. 600. How much interest will the borrower pay on a loan of Tk. 6500 for
one year? Answer: Tk. 478.75
Abyev`t †Kvb FYMÖnxZv F‡Yi 1g 600 UvKvi Rb¨ 8.5% Ges 600 UvKvi AwZwi³ UvKvi 7.25% my` †`q| Zvi F‡Yi cwigvb
6,500 UvKv n‡j wZwb eQ‡i KZ UvKv my` w`‡eb?
3. A man's running speed is 3 times of his walking speed. The inan runs a distance and comes back
walking. Total time taken by him 2 hours. What is the distance in miles if he runs 9 miles per hour?
[SJIB TSO 16. Modhumoti Bank PO 17] Answer: 4.5 miles
Abyev`t GKRb †jv‡Ki †`Šov‡bvi MwZ Zvi nvUvi MwZi 3 ¸Y| †jvKwU GKwU wbw`©ó `~iZ¡ †`Š‡o wM‡q †n‡U †diZ Av‡mb| Zvi
†gvU 2 N›Uv mgq †j‡MwQj| hw` Zvi †`Šov‡bvi MwZ 9 gvBj/N›Uv nq Z‡e `~iZ¡ (gvBj) wbY©q Kiæb?
4. The length of a rectangular field is 30 feet long than its width and the perimeter is 380 feet. How much
would it cost to cover the field with grass at the rate of Tk. 20 per square? Answer: Tk. 176,000
[Standard Bank TAO 16]

Shahjalal Islami Bank Ltd.


Post name: Management Trainee Officer Exam Date : 2016
1. Aman, Belal and Chad started a small business with a total amount tk 28,000. Aman paid tk 4500 more
than Belal and Belal paid tk 7000 less than Chad. If the company made a profit of tk 5600 .How much
profit should Belal receive? [SUBB MTO 16, National Bank PO 17] Answer: Tk.1100
Abyev`t Avgvb, ‡ejvj I Pvu` †gvU 28000 UvKv wb‡q GKwU e¨emv ïiæ K‡i| ‡ejvj hZ UvKv †`q Avgvb Zvi †P‡q 4500 UvKv
†ewk †`q Ges Pvu` hZ UvKv †`q ‡ejvj Zvi †P‡q 7000 UvKv Kg †`q| hw` Zv‡`i †Kv¤úvwbi 5600 UvKv jvf nq Zvn‡j ‡ejvj
KZ UvKv jf¨vsk cv‡e?
10 1 18
2. Solve the Equation: + = Answer: 15
2𝑥−5 𝑥+5 3𝑥−5
(SJIB MTO 16, SJIB TSO 18, Premier Bank TJO 18, NRB Bank MTO 19]
3. The length of a rectangular field is 1.5 times as large as its width. An amount of Tk. 10260 was needed
to cover the field with grass at Tk 1.9 per sq. ineter. How much would it cost to fence the four sides of
the rectangular field at the rate of Tk 2.5 per meter? Answer: Tk. 750
[ONE Bank SCO 17, Bank Asin MTO 17, SJIB MTO 16, Langka Bangla Finance MTO 17 Board Book]
Abyev`t GKwU AvqZKvi gv‡Vi ˆ`N¨© Gi cÖ‡¯’i †`o ¸Y| cÖwZ eM©wgUv‡i 1.9 UvKv wnmv‡e m¤ú~Y© gvVwU‡Z Nvm jvMv‡Z 10,260
UvKv cÖ‡qvRb| cÖwZ wgUvi 2.5 UvKv wnmv‡e gvVwUi Pvicv‡k †eov w`‡Z KZ UvKv LiP n‡e?

Written Math Solution (BIBM) FB Group : BD job Info. (BCS & Bank)
Bank Job Solution (BIBM) 27

Created by Sumon K. Sarkar  FB Group : BD job Info. (BCS & Bank)  Created by Sumon K. Sarkar 
4. Rahim can do a piece of work in 80 days. After working 10 days he left and rest of the work is done
Created by Sumon K. Sarkar  FB Group : BD job Info. (BCS & Bank)  Created by Sumon K. Sarkar 

by Karim in 42 days. If they worked together how many days they need to complete the work?
অনুবােঃ রনহম ৮০ নেতন একটা কাে করতে পাতর। 10 নেন কাে করার পর নেনন তল যান এবং বানক কাে 42 নেতন কনরম কতরন। োরা
একসতে কাে করতল কে নেতন কাে পশ্ে করতে হতব? Answer: 30 days

Al-Arafah Bank Ltd.


Post name: Management Trainee Officer Exam Date : 2016
4 9 25
1. Solve the equation: + = Answer: -3 /5
2𝑥+1 3𝑥+2 5𝑥+4
[Al-Arafah MTO 16, Modhumoti PO 18, SJIB TO Cash 18]
2. The average weight of three men A, B and C is 84 kg. Another man D joins the group and the average
now becomes 80 kg. If a fifth man E, whose weight is 3 kg more than that of D, replaces A, then the
average weight of B, C, D and E becomes 79 kg. What is the weight of A? Answer: 75 kg
[AB Bank PO 02, One Bank PO 07, RAKUB SO 14, Al-Arafah MTO 16, Janata Bank EO(Cancelled) 17]
Abyev`t wZb e¨w³ A, B I C Gi Mo IRb 84Kg| Zv‡`i mv‡_ AviI GKRb D †hvM †`Iqvq bZzb Mo nq 80Kg|hw` A
Gi ¯’‡j D Gi †P‡q 3Kg †ewk IR‡bi cÂg e¨w³ E Zv‡`i mv‡_ †hvM †`q, Zvn‡j B, C, D Ges E Gi Mo IRb nq 79Kg|
A Gi IRb KZ?
3. The perimeter of a square field is equal to the perimeter of a rectangle field. The length of the rectangle
is thrice the width of it and the area is 768 square meters. How many square sized tiles of 80 centimeters
will be required to cover the square filed? Answer: 1600
[Al-Arafah MTO 16, National Bank PO 17, NRBC Bank MTO 18, Social Islami Bank PO 17, Modhumoti PO 18]
Abyev`t GKwU eM©vKvi gv‡Vi cwimxgv GKwU AvqZKvi gv‡Vi cwimxgvi mgvb| AvqZ‡¶ÎwUi ˆ`N©¨ Gi cÖ‡¯’i wZb¸Y Ges Gi
†¶Îdj 768 eM©wgUvi| eM©vKvi gvVwU‡K m¤ú~Y©iƒ‡c XvK‡Z 80 †m.wg KZ¸‡jv eM©vKvi UvBjm jvM‡e?
4. A, B and C started a business jointly with a total amount Tk. 28000. A paid Tk. 4500 more than B and
B paid Tk. 7000 less than C. If the company made a profit of Tk 5600, how much profit should C
receive? [Al-Arafah MTO 16] Answer: Tk. 2500
Abyev`t A, B Ges C †gvU 28,000 UvKv wb‡q GKwU e¨emv ïiæ K‡i| C hZ UvKv †`q B Zvi †P‡q 7,000 UvKv Kg †`q Ges
B hZ UvKv †`q A Zvi †P‡q 4,500 UvKv †ewk †`q| Zv‡`i †Kv¤úvwbi 5,600 UvKv jvf n‡j, C KZ UvKv jf¨vsk cv‡e?

Bank Asia LTD.


Post name: Trainee Officer Exam Date : 2016
1. Average mark obtained by 300 students in a class is 80. When 10% of the students are excluded from
the group, the average mark of remaining students is 85. Find average mark of the 10% of students
excluded from the group? Answer: 35
অনুবােঃ একনট ক্লাতস 300 েন নশ্োর্ণীর প্রাপ্ত গড নম্বর হল 80। যখন 10% ছাত্রতক েল পর্তক বাে পেওযা হয, েখন বানক ছাত্রতের গড
নম্বর 85 হয। গ্রুপ পর্তক বাে পডা 10% ছাতত্রর গড নম্বর কে?
2. The ratio of gold and silver in an ornament weighing 42 gram 4:3. How much gold will need to be
added to make the ratio of gold and silver 5:3? Answer:6gm gold
[Sonali, Janata, Agrani & Rupali Bank (BRC) (Senior Officer) -2008]
অনুবােঃ 42 গ্রাম ওেতনর একনট অলঙ্কাতর পসানা ও পরৌতপযর অনুপাে 4:3। স্বর্ণ ও রূপার অনুপাে 5:3 হওযার েনয কে পসানা পযাগ
করতে হতব?
3. A cistern can be filled in 10 hours but it takes 12 hours due to a leak in its bottom. If the cistern is full,
then how much time the leak will take to empty the cistern? Answer: 60 hours.

Written Math Solution (BIBM) FB Group : BD job Info. (BCS & Bank)
28

 Created by Sumon K. Sarkar  FB Group : BD job Info. (BCS & Bank)  Created by Sumon K. Sarkar 
Bank Job Solution (BIBM)
 Created by Sumon K. Sarkar  FB Group : BD job Info. (BCS & Bank)  Created by Sumon K. Sarkar 

অনুবােঃ একনট প ৌবাচ্চা 10 ন্টায ভরাট করা যায েতব এনটর নীত ফুতটা হওযার কারতর্ এনট 12 ন্টা সময পনয। োহতল পূর্ণ প ৌবাচ্চানট
খানল হতে কে সময লাগতব?
𝟖 𝟗 𝟕
4. Solve the equation: + = Answer: x=2/5
𝟐𝒙−𝟏 𝟑𝒙−𝟏 𝒙+𝟏

Bangladesh Bank
Post Name: Officer (General) Exam Date : 2015
√2+𝑥+√2−𝑥
1. If = 2 , 𝑓𝑖𝑛𝑑 𝑡ℎ𝑒 𝑣𝑎𝑙𝑢𝑒 𝑜𝑓 𝑥. Answer: 8/5
√2+𝑥−√2−𝑥
2. If the sum of five consecutive integers is S, what is the largest of those integers in terms of S?
[BB Officer 15, PKB EO Cash 18] Answer: (S/5)+2
Abyev`t 5wU µwgK msL¨vi mgwó S n‡j, e„nËg msL¨vwU‡K S Gi gva¨‡g cÖKvk Kiæb|
3. Of the three numbers, second is twice the first and is also thrice the third. If the average of the three
numbers is 44, then what will be the largest number? Answer: 72
Abyev`t wZbwU msL¨vi g‡a¨ 2q msL¨vwU 1g msL¨vi wظY Ges 3q msL¨vi wZb¸Y| hw` msL¨v wZbwUi Mo 44 nq, Zvn‡j e„nËg
msL¨vwU KZ?
4. The difference between two numbers is five and the difference of their squares is 65. What is the larger
number? Answer:9
Abyev`t `ywU msL¨vi AšÍi 5 Ges Zv‡`i e‡M©i AšÍi 65| eo msL¨vwU KZ?
5. The annual incomes and expenditures of a man and his wife are in the ratios 5:3 and 3:1, respectively.
If they decide to save equally and find a balance of Tk. 4000 at the end of the year, what was their
income? Answer: Tk. 5000 and Tk. 3000
Abyev`t GKRb e¨w³ Ges Zvi ¯¿x evwl©K Avq Ges e¨‡qi AbycvZ h_vµ‡g 5 t 3 Ges 3 t 1| hw` Zviv mgvb fv‡e UvKv Rgv‡bvi
wm×všÍ †bq Ges eQi †k‡l †gvU 4,000 UvKv mÂq K‡i Zvn‡j Zv‡`i Avq KZ?
6. A shop stocks four types of caps, there are 1/3 as many red caps as blue caps and ½ as many green
caps as red caps. There are equal number of green caps and yellow caps. If there are 42 blue caps, then
what percent of the total caps in the shop are blue? Answer:60%
Abyev`t GKwU †`vKv‡b 4 ai‡bi K¨vc Av‡Q| hZ¸‡jv bxj K¨vc Av‡Q Zvi 1/3 fvM jvj K¨vc Av‡Q Ges hZ¸‡jv jvj K¨vc
Av‡Q Zvi 1/2 fvM meyR K¨vc Av‡Q| hZ¸‡jv meyR K¨vc Av‡Q ZZ¸‡jv njy` K¨vc Av‡Q | hw` 42wU bxj K¨vc _v‡K Zvn‡j
†`vKv‡bi †gvU K¨v‡ci kZKiv KZwU bxj K¨vc Av‡Q ?
7. A person sold two articles. Each for the same price of Tk. 1040. He incurs 20% loss on the first and
10% loss on the second. Find his overall percentage of loss. Answer: 15.29%
Abyev`t †Kvb e¨w³ 2 ai‡bi wRwbm weµq K‡ib| cÖ‡Z¨KwUi g~j¨ 1,040 UvKv| cÖ_gwU‡Z 20% Ges 2q wU‡Z 10% ¶wZ nq|
Zvi †gvU kZKiv ¶wZi cwigvb KZ?
8. Robi drove 100 miles to visit a friend. If he had driven 8 miles per hour faster than he did, he would
have arrived in 5/6 of the time, he actually took. How many minutes did the trip take? Answer: 150
Abyev`t iwe 100 gvBj Mvwo Pvwj‡q Zvi eÜzi mv‡_ †`Lv Ki‡Z hvq| hw` †m †h ‡e‡M Mvwo Pvjvq Zvi †P‡q AviI 8 gvBj/N›Uv
†ewk †e‡M Mvwo PvjvZ Z‡e †m †h mg‡q eÜzi evmvq †cŠQvq Zvi †P‡q 5/6 ¸Y Kg mg‡q †cŠQv‡Z cvi‡Zv| eÜzi evmvq †h‡Z Zvi
KZ wgwbU mgq †j‡MwQj?

Written Math Solution (BIBM) FB Group : BD job Info. (BCS & Bank)
Bank Job Solution (BIBM) 29

Created by Sumon K. Sarkar  FB Group : BD job Info. (BCS & Bank)  Created by Sumon K. Sarkar 
Bangladesh Bank
Created by Sumon K. Sarkar  FB Group : BD job Info. (BCS & Bank)  Created by Sumon K. Sarkar 

Assistant Director (FF) Exam Date : 2015


1 6 1
1. If 𝑥 + = 3, then 𝑥 + =? [BB AD(ff) 15] [Senior Officer -2018] Answer: 322
𝑥 𝑥6
2. The average age of students of a class is 15.8 years. The average age of boys in the class is 16.4 years
and of the girls is 15.4 years. Find the ratio of number of boys to the number of girls in the class.
Abyev`t GKwU †kªbx‡Z wk¶v_©x‡`i Mo eqm 15.8 eQi| H †kªYxi evjK‡`i Mo eqm 16.4 eQi Ges evwjKv‡`i Mo eqm 15.4
eQi| H †kªbx‡Z evjK Ges evwjKv‡`i msL¨vi AbycvZ wbY©q Ki| Answer: 2 : 3
3. The percentage profit earned by selling an article for Taka 1920 is equal to the percentage loss incurred
by selling the same article for Taka 1280. At what price should the article be sold to make a 25% profit?
[South East Bank MTO 2013, BB AD(freedom fighter) 2015, SJIB MTO 2013, IBBL PO 2019, Sadharan Bima JO 2019, Sadharan Bima AM
2019, BB AD -AME - 2023] Answer: Tk. 2000
Abyev`t GKwU cY¨ 1,920 UvKvq weµq Ki‡j kZKiv hZ jvf nq, cY¨wU 1,280 UvKvq weµq Ki‡j kZKiv ZZ UvKv ¶wZ
nq| 25% jvf Ki‡Z n‡j cY¨wU‡K KZ `v‡g weµq Ki‡Z n‡e?
4. A bus hired at the cost of Tk. 2400 and it was decided that every student would share the cost equally.
But 10 more students jointed and as a result the fare decreased by Tk. 8 per person. How many students
were travelling in the bus? Answer: 60
Abyev`t 2,400 UvKv w`‡q GKwU evm fvov Kiv nq Ges cÖ‡Z¨K QvÎ mgcwigvb Puv`v †`‡e wVK Kiv nq| wKš‘ 10 Rb AwZwi³
QvÎ †hvM †`qvq Puv`vi cwigvb 8 UvKv n«vm cvq| KZRb QvÎ ev‡m hv‡”Q?
5. A can do a work in 10 days, while B alone can do it in 15 days. They work together for 5 days and rest
of the work is done by C in 2 days. If they get Tk. 4500 for whole work, how should they divide
money? [BASIC Bank AO Cash 14, BB AD(ft) 15] Answer: Tk. 2250, Tk. 1500 and Tk. 750
Abyev`t A GKv GKwU KvR 10 w`‡b Ki‡Z cv‡i †hLv‡b B GKv H KvRwU 15 w`‡b K‡i| Zviv GKm‡½ 5 w`b KvR K‡i Ges
evKx KvR C GKv 2w`‡b m¤úbœ K‡i| m¤ú~Y© Kv‡Ri Rb¨ 4,500 UvKv †`qv n‡j Zviv H UvKv wb‡R‡`i g‡a¨ wKfv‡e fvM K‡i
‡b‡e?

Midland Bank Ltd.


Post name: Trainee Officer Exam Date : 2015
1. A person earns yearly interest of Tk. 920 by investing Tk. X at 4% and Tk. Y at 5% simple interest
rate. If he had invested Tk. X at 5% and Tk. Y at 4% simple interest rate, then his yearly interest earning
would have been reduced by Tk. 40. Find out the amount of X and Y.
[ONE Bank PO 10, NCC Bank MTO 11, Premier Bank MTO 12, Midland Bank TO 15] Answer: X=Tk 8000 and Y= Tk12,000
Abyev`t GK e¨w³ 4% my‡` x UvKv Ges 5% my‡` y UvKv wewb‡qvM K‡i eQ‡i 920 UvKv gybvdv AR©b K‡ib| wZwb hw` 5% my‡`
x UvKv Ges 4% my‡` y UvKv wewb‡qvM K‡ib Zvn‡j gybvdvi cwigvb 40 UvKv K‡g hv| x Ges y Gi cwigvb wbY©q Ki|
2. A basketball team has won 15 games and lost 9. If these games represent 16.67% of the games to the
played, then how many more games must the team win to average 75% for the season?
[ONE Bank PO 10, NCC Bank MTO 11, Premier Bank MTO 12, Midland Bank TO 15, Bangladesh bank AP 2023] Answer: 93 games
Abyev`t GKwU ev‡¯‹Uej wUg 15wU †Ljvq Rqjvf K‡i Ges 9wU †Ljvq civwRZ nq| hw` GB †Ljv¸‡jv Øviv Zv‡`i †gvU ‡Ljvi
16.67% †Ljv‡K †evSv‡bv nq Zvn‡j H wmR‡b M‡o 75% †Ljvq †RZvi Rb¨ Zv‡`i‡K KZwU †Ljvq wRZ‡Z n‡e?

Written Math Solution (BIBM) FB Group : BD job Info. (BCS & Bank)
30

 Created by Sumon K. Sarkar  FB Group : BD job Info. (BCS & Bank)  Created by Sumon K. Sarkar 
Bank Job Solution (BIBM)
 Created by Sumon K. Sarkar  FB Group : BD job Info. (BCS & Bank)  Created by Sumon K. Sarkar 

Midland Bank Ltd.


Post name: Management Trainee Officer Exam Date : 2015
1. In a country, 60% of the male citizen and 70% of the female citizen are eligible to vote. 70% of the
male and 60% of female citizen are eligible to cast their vote. What fraction of citizens voted during
their election? [Janata Bank EO 12, South East Bank MTO 13, SJIB MTO 13, Midland Bank MTO 15] Answer: 21/50
Abyev`t GKwU †`‡ki 60% cyiæl Ges 70% gwnjv †fvU w`‡Z cvi‡eb| G‡`i g‡a¨ 70% cyiæl Ges 60% gwnjv †fvU `vb
K‡ib| KZRb bvMwiK wbe©vP‡bi mgq †fvU cÖ`vb K‡ib?
2. A father has divided his property between his two sons A and B. A invests the amount at a compound
profit of 8%. B invests the amount of 10% simple profit. At the end of 2 years, the profit received by
Bis Tk. 1336 more than A. Find the amount of both. Total amount of his father is Tk. 25000.
[Midland Bank MTO 15, SIBL PO 13, BB AME-2023] Answer: Tk 10,000 and Tk. 15,000
Abyev`t †Kvb wcZv Zvi m¤úwË Zvi `yB cyÎ A I B Gi g‡a¨ fvM K‡i †`q| A Zvi UvKv 8% Pµe„w× gybvdvq Ges B Zvi UvKv
10% mij gybvdvq wewb‡qvM K‡i| 2 eQi c‡i A Gi †P‡q B 1,336 UvKv †ewk gybvdv AR©b K‡i| Df‡qi UvKvi cwigvb wbY©q
Ki| Zv‡`i wcZvi Kv‡Q 25,000 UvKv wQj|

National Bank Ltd.


Post name: Probationary Officer Exam Date : 2015
2 1
1. If 𝑥 = 3 + √8 then find the value of 𝑥 + Answer: 34
𝑥2
2. A sum of money is to be distributed equally among a group of children. If there were 25 children less
than each would get Tk. 1.50 more, and if there 50 children inore, each would get TK 1.50 less. Find
the number of children and the amount of money distributed. Answer: 100 children and Tk. 450
[One Bank SCO 18, National Bank PO 15]
Abyev`t KZ¸‡jv wkïi g‡a¨ wKQz UvKv mgvbfv‡e fvM K‡i †`qv nq| hw` 25wU wkï Kg _vKZ Zvn‡j cÖ‡Z¨‡K 1.50 UvKv †ewk
†cZ Ges hw` 50wU wkï †ewk _vKZ Zvn‡j cÖ‡Z¨‡K 1.50 UvKv Kg †cZ| KZwU wkï wQj Ges KZ UvKv †`qv n‡q‡Q, Zv wbY©q
Ki|
3. A video magazine distributor made 3500 copies of the May issue of the magazine at a cost of Tk.
4,00,000. He gave 500 cassettes free to some key video libraries. He also allowed a 25% discount on
the market price of the cassette. In this manner, he was able to sell all the 3500 cassettes that were
produced. If the market price of a cassettes was Tk. 160, what is his gain or loss for the May issue of
the video magazine? Answer: 40,000
Abyev`t GKwU wfwWI g¨vMvwRb cwi‡ekK g¨vMvwRw›Ui †g msL¨vi 3500 Kwc Qvcvb 400000 UvKvq| wZwb 500wU K¨v‡mU
webvg~‡j¨ w`‡jb wKQz cÖavb cÖavb wfwWI jvB‡eªix‡Z| GQvovI wZwb g~j weµqg~‡j¨i Dci AviI 25% gyj¨ Qvo w`‡jb| Gfv‡e
wZwb me¸‡jv K¨v‡mU wewµ Ki‡Z m¶g n‡jb| hw` g~j weµqg~j¨ 160 UvKv nq, Zvn‡j H gv‡m g¨vMvwRb wewµ †_‡K Zvi KZ
jvf ev ¶wZ n‡qwQj?

National Bank Ltd.


Post name: Probationary Officer Exam Date : 2014
1. The speed of a railway engine is 42 Km per hour when no compartment is attached, and the reduction
in speed is directly proportional to the square root of the number of compartments attached. If the speed
of the train carried by this engine is 24 km per hour when 9 compartments are attached, the maximum
number of compartments that can be carried by the engine? Answer: 48

Written Math Solution (BIBM) FB Group : BD job Info. (BCS & Bank)
Bank Job Solution (BIBM) 31

Created by Sumon K. Sarkar  FB Group : BD job Info. (BCS & Bank)  Created by Sumon K. Sarkar 
Abyev`t hLb †Kvb ewM †ijI‡q Bwćbi mv‡_ jvMv‡bv Ae¯’vq _v‡Kbv ZLb Gi MwZ N›Uvq 42 wK‡jvgUvi| Bwćbi MwZ nªv‡mi
Created by Sumon K. Sarkar  FB Group : BD job Info. (BCS & Bank)  Created by Sumon K. Sarkar 

cwigvY jvMv‡bv ewMi msL¨vi eM©g~‡ji mgvbycvwZK| 9wU ewM jvMv‡bv Ae¯’vq hw` Bwćbi MwZ N›Uvq 24 wK‡jvwgUvi nq, Z‡e
m‡e©v”P KZwU ewM jvMv‡bv hv‡e?
2. A shopkeeper lost 7.5% by selling an article. If he had bought it at 10% less and sold it for 31 taka
more, he would have gained 20%. Find the cost price of the article. Answer: Tk 200
[National Bank PO 14, BKB Supervisor 12]
Abyev`t GKRb †`vKvb`vi GKwU `ªe¨ 7.5% ¶wZ‡Z weµq K‡ib| hw` wZwb GwU 10% Kg `v‡g wKb‡Z cvi‡Zb Ges 31 UvKv
†ewk w`‡q wewµ Ki‡Z cvi‡Zb Zvn‡j Zvi 20% jvf n‡Zv| `ªe¨wUi µqg~j¨ wbY©q Ki|
3. Two tanks, X and Y, are filled to capacity with jet fuel. Tank X holds 600 gallons more than tank Y.
If 100 gallons of fuel were to be pumped from each tank, tank X would then contain 3 times as much
fuel as tank Y. What is the total number of gallons of fuel in the two full tanks? Answer: 1400
[Mercantile Band Officer 09, IFIC Bank PO 10, National Bank PO 14]
Abyev`t 2wU U¨v¼ X I Y R¡vjvbx †Zj Øviv c~Y© Kiv hvq| X U¨v¼wU Y U¨v‡¼i †P‡q 600 M¨vjb †ewk avib Ki‡Z cv‡i| Dfq
U¨v¼ †_‡K 100 M¨vjb K‡i †Zj †ei K‡i wb‡j, X U¨v†¼ Y U¨v‡¼i wZb¸Y †Zj _v‡K| U¨v¼ `ywU‡Z †Zjc~Y© Ae¯’vq †gvU KZ
M¨vjb †Zj _v‡K?

Basic Bank Ltd.


Post name: Assistant Officer (Cash) Exam Date : 2014
1. A can do a work in 10 days, while B alone can do it in 15 days. They work together for 5 days and rest
of the work is done by C in 2 days. If they get Tk. 4500 for whole work, how should they divide
money? [BASIC Bank AO Cash 14, BB AD(ff) 15] Answer: Tk. 2250, Tk. 1500 and Tk. 750
Abyev`t A GKv GKwU KvR 10 w`‡b Ki‡Z cv‡i †hLv‡b B GKv H KvRwU 15 w`‡b K‡i| Zviv GKm‡½ 5 w`b KvR K‡i Ges
evKx KvR C GKv 2w`‡b m¤úbœ K‡i| m¤ú~Y© Kv‡Ri Rb¨ 4,500 UvKv †`qv n‡j Zviv H UvKv wb‡R‡`i g‡a¨ wKfv‡e fvM K‡i
‡b‡e?
2. A sum of Tk. 1260 is borrowed from a money lender at 10% p.a. compounded annually. If the amount
is to be paid in two equal annual installinents, find the annual installments. Answer: Tk 726
Abyev`t hw` 10% Pµe„w× my‡` 1,260 UvKv FY †bqv nq Ges hw` 2 eQ‡i m¤ú~Y© UvKv cwi‡kva Ki‡Z nq Zvn‡j cÖ‡Z¨K eQi
KZ UvKv cwi‡kva Ki‡Z n‡e?
3. In the figure, lines I and mare parallel. If y-z = 60°, then what is the value of x? Answer: 120°

Rajshahi Krisi Unnayan Bank Ltd.


Post name: Officer Exam Date : 2014
4 √1+𝑥+√1−𝑥
1. If 𝑥 = , then what is the value of =? [SJIB TO 13, RAKUB Officer 14] Answer: 2
5 √1+𝑥−√1−𝑥
1. An amount of money is invested in a savings account for two years. It increases by Tk. 420 in two
years after annual compounding at the rate of 10 % per year. What the amount in Taka, invested
initially? Answer: Tk. 2000

Written Math Solution (BIBM) FB Group : BD job Info. (BCS & Bank)
32

 Created by Sumon K. Sarkar  FB Group : BD job Info. (BCS & Bank)  Created by Sumon K. Sarkar 
Bank Job Solution (BIBM)
 Created by Sumon K. Sarkar  FB Group : BD job Info. (BCS & Bank)  Created by Sumon K. Sarkar 

অনুবােঃ দুই বছতরর েনয একনট সঞ্চযী অযাকাউতন্ট একনট পনরমার্ অর্ণ নবননতযাগ করা হয। এনট প্রনে বছর 10% হাতর বানেণক ক্রবৃনির
পর দুই বছতর 420 টাকা বৃনি পায। প্রার্নমকভাতব নবননতযাগকৃে করা টাকার পনরমার্ কে?
2. A certain college has students to teacher ratio of 11 to 1. The average annual salary for teachers is Tk.
26000. If the college pays a total of Tk. 33,80,000 in annual salaries to its teachers. How many students
does the college have? Answer: 1430
Abyev`t †Kvb K‡j‡Ri wk¶v_©x I wk¶‡Ki AbycvZ 11 t 1| wk¶‡Ki evwl©K Mo †eZb 26,000 UvKv| hw` K‡jR †_‡K
wk¶K‡`i‡K eQ‡i †gvU 33,80,000 UvKv †eZb ‡`qv nq Zvn‡j K‡j‡R KZRb wk¶v_©x Av‡Q?
4. Tickets to cricket game cost Tk. 40 for reserved seats and Tk. 30 for general seats. If all 500 tickets
were sold for Tk. 17600. How many reserved seats were sold? Answer: 260
Abyev`t wµ‡KU †Ljvi wiRvf© wUwK‡Ui g~j¨ 40 UvKv Ges mvaviY wUwK‡Ui g~j¨ 30 UvKv| hw` 500wU wUwKU 17,600 UvKvq wewµ
nq Zvn‡j KZ¸‡jv wiRvf© wUwKU wewµ nq?
5. In the ABCD is a square. If the length of the square is 10ft, then what will be the area of the triangle
OCD? Answer: 25 Sq. ft
Abyev`t ABCD GKwU eM©‡¶Î hvi GK evûi •`N¨© 10 dzU| OCD wÎfz‡Ri †¶Îdj KZ?

Rajshahi Krisi Unnayan Bank Ltd.


Post name: Senior Officer Exam Date : 2014
3
1. √8𝑥 2 √32𝑥√4𝑥 2 = 4, then the value of x? Answer:1
[Premier Bank MTO 12, RAKUB SO 14, IFIC Bank Premier Bank Officer Cash 13]
2. The average weight of three men A, B and C is 84 kg. Another man D joins the group and the average
now becomes 80 kg. If a fifth man E, whose weight is 3 kg more than that of D, replaces A, then the
average weight of B, C, D and E becomes 79 kg. What is the weight of A ? Answer: 75 kg
[AB Bank PO 02, One Bank PO 07, RAKUB SO 14, Al-Arafah MTO 16, Janata Bank EO(Cancelled) 17]
Abyev`t wZb e¨w³ A, B I C Gi Mo IRb 84Kg| Zv‡`i mv‡_ AviI GKRb D †hvM †`Iqvq bZzb Mo nq 80Kg|hw` A
Gi ¯’‡j D Gi †P‡q 3Kg †ewk IR‡bi cÂg e¨w³ E Zv‡`i mv‡_ †hvM †`q, Zvn‡j B, C, D Ges E Gi Mo IRb nq 79Kg|
A Gi IRb KZ?
3. A trader bought some mangoes for Tk. 150 per dozen and equal number of apples for Tk. 100 per
dozen. If he sells all the fruits Tk. 140 per dozen, what will be his profit/loss in percentage?
[BASIC Bank PO 99, RAKUB SO 14, SJIB MTO 10] Answer: Profit 12%
Abyev`t GKRb e¨emvqx cÖwZ WRb 150 UvKv `‡i wKQz Avg Ges cÖwZ WRb 100 UvKv `‡i mgvb msL¨K Av‡cj µq
K‡ib| hw` †m me¸‡jv dj 140 UvKv WRb wn‡m‡e weµq K‡i Zvn‡j Zvi kZKiv KZ jvf ev ¶wZ n‡e?
Two partners A and B have 70% and 30% shares respectively in a business. After sometimes, a third
partner C joined by investing Tk. 10 lakh and thus having 20% share in the business. What is the
percentage of share of A's now in the business? [RAKUB SO 14, SJIB MTO 10] Answer: 56%
Abyev`t `yBRb cvU©bvi A I B GKwU e¨emvq h_vµ‡g 70% I 30% †kqv‡ii Askx`vi| wKQzw`b ci C Zv‡`i mv‡_ †hvM †`q
Ges †m 10 j¶ UvKv wewb‡qvM Kivq e¨emvq Zvi †kqv‡ii cwigvb nq 20%| eZ©gv‡b e¨emvq A Gi kZKiv †kqv‡ii cwigvb
KZ?

Written Math Solution (BIBM) FB Group : BD job Info. (BCS & Bank)
Bank Job Solution (BIBM) 33

Created by Sumon K. Sarkar  FB Group : BD job Info. (BCS & Bank)  Created by Sumon K. Sarkar 
4. In the figure, rectangle PQRS inscribed in a circle and PQ = 6. If the area of the rectangular region is
Created by Sumon K. Sarkar  FB Group : BD job Info. (BCS & Bank)  Created by Sumon K. Sarkar 

48, what is the area of the circular region? Answer: 25π

United Commercial Bank Ltd.


Post name: Senior Officer Exam Date: 2014
1. A pipe is turned on to fill water into a cistern at the rate of 4 liters per minute. The cistern has a leak
which would empty it in 6 hours and the cistern is now emptied in 10 hours. Determine the capacity of
the cistern. Answer: 3600 liters
অনুবাে : একনট পাইপ নেতয একনট cistern প্রনে নমনটতট 4 নলটার কতর পানন পূর্ণ কতর। নকন্তু cistern (নপপা) নটতে নছদ্র র্াকায এনট
6 ণ্টায খানল হয। বেণমাতন পাইপনট পখালা র্াকায এনট 10 ণ্টায খানল হয। Cistern-নটর পানন যারর্ েমো ননর্ণয করুন।
2. If I walk at the rate of 4 kmph I miss the train by 10 minutes. If I walk at the rate of 5 kmph, I reach 5
minutes before the arrival of the train. How far is the station from my house assuming that I start
walking from my house? Answer: 5 km.
অনুবাে : আনম যনে 4 নকনম প্রনে ণ্টা পবতগ হাাঁনট োহতল আনম 10 নমননতটর মতযয পট্রন নমস করব। আনম যনে 5 নকনম প্রনে ন্টা গনেতে
হাাঁনট েতব আনম পট্রন আসার 5 নমননট আতগ পপৌাঁতছ যাই। আমার বানড পর্তক পস্টশ্ন কেেূতর যতর নননচ্ছ পয আনম আমার বানড পর্তক হাাঁটতে
শুরু কনর?
3. In the figure, PS=SQ=RS and ∠P = 30°. Find x. Answer: 90°.

United Commercial Bank Ltd.


Post name: Management Trainee Officer Exam Date: 2013
1. A system of equation is given below: x+I= 6; x-m=5; x+p=4; x-q=3; What is the value of I+ m+p+q? ] Answer: 2
2. A series has 3 numbers a, ar, ar2. In the series, the first term is twice of the second term. What is the
ratio of the sum of the first 2 terms to the sum of the last 2 terms? [UCBL MTO 13, BB AD 12] Answer: 2:1
Abyev`t GKwU mvwi‡Z 3wU msL¨v nj a, ar, ar2| mvwiwUi 1g msL¨vwU, 2q msL¨vi wظY| 1g 2wU msL¨vi †hvMdj Ges †kl `ywU
msL¨vi †hvMd‡ji AbycvZ KZ n‡e?
3. A man travels from A to B at a speed x km/hr. He then rests at B for 1 hours. He then travels from B
to C at a speed 2x kin/hr and rests for 2x hours. He moves further to D at a speed twice as that between
B and C. He thus reaches D in 16 hr. If distance A-B, B-C and C-D are all equal to 12 km, then find
the time for which he rested at B. Answer: 3 hours
Abyev`t GKRb e¨vw³ A †_‡K B †Z cÖwZ N›Uvq x wKwg MwZ‡Z ågb K‡ib| Zvici wZwb B †Z x N›Uvi Rb¨ wekªvg K‡ib|
Gici wZwb cÖwZ N›Uvq 2x wKwg MwZ‡Z B †_‡K C †Z ågY K‡ib Ges †mLv‡b Avevi 2x N›Uvi Rb¨ wekªvg †bb| wZwb AviI

Written Math Solution (BIBM) FB Group : BD job Info. (BCS & Bank)
34

 Created by Sumon K. Sarkar  FB Group : BD job Info. (BCS & Bank)  Created by Sumon K. Sarkar 
Bank Job Solution (BIBM)
 Created by Sumon K. Sarkar  FB Group : BD job Info. (BCS & Bank)  Created by Sumon K. Sarkar 

wKQz`yi wM‡q D †Z †c․Qvb| Gmgq Zvi Mvwoi MwZ wQj B †_‡K C †Z åg‡bi MwZi wظY| wZwb Gfv‡e †gvU 16 N›Uvq D †Z
†c․Qvb| hw` A-B, B-C Ges C-D Gi me©‡gvU `~iZ¡ 12 wKwg nq Zvn‡j B †Z Zvi wekªv‡gi mgq wbY©q Kiæb|
4. The side length of a square inscribed in a circle is 2. What is the area of the circle? Answer: 2π

National Bank Ltd.


Post name: Probationary Officer Exam Date : 2013
1. In a class of 40 students, the number of students who passed the math exam is equal to half the number
of students who passed the science exam. Each student in the class passed at least one of the two exam.
If 5 students passed both exams, then how many students passed the math exam? Answer:15
Abyev`t GKwU K¬v‡m 40 Rb Qv‡Îi g‡a¨ hZRb QvÎ weÁv‡b cvk K‡i Zvi A‡a©K QvÎ MwY‡Z cvk K‡i| cÖ‡Z¨K QvÎB `ywU
cix¶vi AšÍZ GKwU‡Z cvk K‡i| hw` 5 Rb QvÎ Dfq cix¶v‡ZB cvk K‡i Zvn‡j KZRb QvÎ MwY‡Z cvk K‡i?
0.0015×10𝑚
2. If = 5 × 107 then find out the value of ( m – k ) Answer:9
0.03×10𝑘
3. Sheela and Meela are 4 miles apart. If Sheela starts walking toward Meela at 3 miles per hour and at
the same time Meela starts walking toward Sheela at 2 miles per hours, how much time will pass before
they meet? Answer:48 minutes
Abyev`t wkjv Ges wgjv 4 gvBj `~i‡Z¡ Av‡Q| hw` wkjv 3 gvBj/N›Uv †e‡M wgjvi w`‡K Ges GKB mg‡q wgjv 2 gvBj/N›Uv †e‡M
wkjvi w`‡K nuvU‡Z _v‡K| KZ mgq ci Zviv GKB hvqMvq G‡m †cŠQv‡e?
4. In the figure, if AB = 8, BC = 6, AC = 10 and CD = 9, then AD =? Answer: 17

Shahjalal Islami Bank Ltd.


Post name: Trainee Officer (cash) Exam Date : 2013
1. Two equal amount of money are deposited in two banks, each at 15% per annum, for 3.5 years and 5
years. If difference between their profits is Ik. 144, what is the each amount of money deposited?
Abyev`t 15% gybvdvq mgcwigvb UvKv 3.5 eQi Ges 5 eQ‡ii Rb¨ 2wU Avjv`v e¨vs‡K ivLv n‡jv| hw` G‡`i gybvdvi cv_©K¨
144 UvKv nq Zvn‡j cÖ‡Z¨K e¨vs‡K KZ UvKv Rgv ivLv n‡qwQj? Answer:Tk 640
2. An old man distributed all the gold coins he had to his two sons into two different numbers such that
the difference between the squares of the two numbers is 36 times the difference between the two
numbers. How many coins did the old man have? Answer:36
Abyev`t GKRb e„× †jvK Zvi mg¯Í ¯^Y©gy`ªv Zvi `yB cy‡Îi g‡a¨ Ggb msL¨vq fvM K‡i †`b †hb msL¨v `ywUi e‡M©i AšÍi, msL¨v
`ywUi AšÍ‡ii †P†q 36 ¸Y eo nq| e„× ‡jvKwUi Kv‡Q KZwU ¯^Y©gy`ªv wQj?
3. If a number of two digits is divided by the product of its digits, the quotient is 3. When 18 is added to
the number, the digits of the number change their places. Find the number. Answer:24
Abyev`t GKRb e„× †jvK Zvi mg¯Í ¯^Y©gy`ªv Zvi `yB cy‡Îi g‡a¨ Ggb msL¨vq fvM K‡i †`b †hb msL¨v `ywUi e‡M©i AšÍi,
msL¨v `ywUi AšÍ‡ii †P†q 36 ¸Y eo nq| e„× ‡jvKwUi Kv‡Q KZwU ¯^Y©gy`ªv wQj?
4. In the following diagram, if BC = CD = BD =1 and angle ADC is a right angle, what is the perimeter
of triangle ABD? Answer: 2+√2

Written Math Solution (BIBM) FB Group : BD job Info. (BCS & Bank)
Bank Job Solution (BIBM) 35

Created by Sumon K. Sarkar  FB Group : BD job Info. (BCS & Bank)  Created by Sumon K. Sarkar 
Created by Sumon K. Sarkar  FB Group : BD job Info. (BCS & Bank)  Created by Sumon K. Sarkar 

Shahjalal Islami Bank Ltd.


Post name: Trainee Officer Exam Date : 2013
4 √1+𝑥+√1−𝑥
1. If 𝑥 = , then what is the value of =? Answer: 2
5 √1+𝑥−√1−𝑥
[SJIB TO 13, RAKUB Officer 14]
2. In a certain Accounting class, the ratio of the number of Accounting majors to the under of students
who are not Accounting major is 2 to 5. If 2 more Accounting majors were to enter the class, the ratio
would be 1 to 2. How many students are in the class? Answer: 28
Abyev`t †Kvb GKwU A¨vKvDw›Us K¬v‡m hv‡`i A¨vKvDw›Us †gRi Ges hv‡`i A¨vKvDw›Us †gRi bq Zv‡`i AbycvZ 2 t 5| 2
Rb bZzb A¨vKvDw›Us †gRi K¬v‡m cÖ‡ek Kivq AbycvZ nq 1 t 2| K¬v‡m KZRb wk¶v_©x Av‡Q?
3. What is perimeter of AABC show in the figure? Answer: 4+ 2√2

3. The percentage profit earned by selling an article for Tk. 1920 is equal to the percentage loss incurred
by selling the same article for Tk 1280. At what price should the article be sold to make 25% profit?
[South East Bank MTO 2013, BB AD(freedom fighter) 2015, SJIB MTO 2013, IBBL PO 2019, Sadharan Bima JO 2019, Sadharan Bima AM
2019, BB AD -AME - 2023] Answer: Tk. 2000
Abyev`t GKwU cY¨ 1,920 UvKvq weµq Ki‡j kZKiv hZ jvf nq, cY¨wU 1,280 UvKvq weµq Ki‡j kZKiv ZZ UvKv ¶wZ
nq| 25% jvf Ki‡Z n‡j cY¨wU‡K KZ `v‡g weµq Ki‡Z n‡e?

Shahjalal Islami Bank Ltd.


Post name: Trainee Senior Officer Exam Date : 2013
4(√6+√2) 2+√3
1. What is the value of - Answer:1
√6−√2 2−√3
2. In a set of three numbers, the average of first two numbers is 2, the average of the last two numbers is 3, and the
average of the first and last number's is 4. What is the average of three numbers? Answer: 3
Abyev`t wZbwU msL¨vi g‡a¨ 1g msL¨v `ywUi Mo 2, †kl msL¨v `ywUi Mo 3 Ges 1g I †kl msL¨vi Mo 4| msL¨v wZbwUi Mo KZ?
3. The total income of Mr. Atig in the years 2008, 2009 and 2010 was Tk. 36,40,000. His income
increased by 20% each year. What was his income in 2010? Answer:Tk14,40,000
Abyev`t Rbve AvwZ‡Ki 2008, 2009 Ges 2010 mv‡ji †gvU Avq 36,40,000 UvKv| Zvi Avq cÖ‡Z¨K eQi 20% e„w× cvq|
2010 mv‡j Zvi Avq KZ wQj?
4. What is the area of the equilateral triangle, if the base BC=6? Answer: 9√3

Written Math Solution (BIBM) FB Group : BD job Info. (BCS & Bank)
36

 Created by Sumon K. Sarkar  FB Group : BD job Info. (BCS & Bank)  Created by Sumon K. Sarkar 
Bank Job Solution (BIBM)
 Created by Sumon K. Sarkar  FB Group : BD job Info. (BCS & Bank)  Created by Sumon K. Sarkar 

Shahjalal Islami Bank Ltd.


Post name: Management Trainee Officer Exam Date : 2013
1. In a country, 60% of the male citizen and 70% of the female citizen are eligible to vote. 70% of the
male and 60% of female citizen are eligible to cast their vote. What fraction of citizens voted during
their election? [Janata EO 12, South East Bank MTO 13, SJIB MTO 13, Midland Bank MTO 15] Answer: 21/50
Abyev`t GKwU †`‡ki 60% cyiæl Ges 70% gwnjv †fvU w`‡Z cvi‡eb| G‡`i g‡a¨ 70% cyiæl Ges 60% gwnjv †fvU `vb
K‡ib| KZRb bvMwiK wbe©vP‡bi mgq †fvU cÖ`vb K‡ib?
2. The percentage profit earned by selling an article for Tk. 1920 is equal to the percentage loss incurred
by selling the same article for Tk 1280. At what price should the article be sold to make 25% profit?
[South East Bank MTO 2013, BB AD(freedom fighter) 2015, SJIB MTO 2013, IBBL PO 2019, Sadharan Bima JO 2019, Sadharan Bima AM
2019, BB AD -AME - 2023] Answer: Tk. 2000
Abyev`t GKwU cY¨ 1,920 UvKvq weµq Ki‡j kZKiv hZ jvf nq, cY¨wU 1,280 UvKvq weµq Ki‡j kZKiv ZZ UvKv ¶wZ
nq| 25% jvf Ki‡Z n‡j cY¨wU‡K KZ `v‡g weµq Ki‡Z n‡e?
3. A, B and Center into a partnership in the ratio 7/2 : 4/3 : 6/5. After 4 months, A increases his share by
50%. If the total profit at the end of one year is Tk. 21,600, then what is B's share in the profit?
[SJIB MTO 13, South East Bank MTO 13] Answer: Tk. 4000
Abyev`t A, B I C 7/2 : 4/3 : 6/5 Abycv‡Z GKwU Askx`vix Kvievi ïiæ K‡i| 4 gvm ci A Zvi †kqvi 50% e„w× K‡i| hw`
1 eQi c‡i †gvU jv‡fi cwigvb 21,600 UvKv nq Zvn‡j B Gi jf¨vsk KZ n‡e?
4. The ratio between the perimeter and the breadth of rectangle is 5:1. If the area of the rectangle is 216
sq. cm, what is the length of the rectangle? [SJIB MTO 13, South East Bank MTO 13] Answer: 18cm
Abyev`t GKwU AvqZKvi †¶‡Îi cwimxgv Ges cÖ‡¯’i AbycvZ 5 t 1| AvqZ‡¶ÎwUi †¶Îdj 216 eM© †m.wg n‡j, Gi ˆ`N©¨ KZ?

Social Islami Bank Ltd.


Post name: Probationary Officer Exam Date : 2013
1. A father has divided his property between his two sons A and B. A invests the amount at a compound
profit of 8%. B invests the amount of 10% simple profit. At the end of 2 years, the profit received by
B is Ik. 1336 more than A. Find the amount of both. Total amount of his father is Tk. 25000.
[Midland Bank MTO 15, SIBL PO 13, BB AME-2023] Answer: Tk. 10,000 and Tk. 15,000
Abyev`t †Kvb wcZv Zvi m¤úwË Zvi `yB cyÎ A I B Gi g‡a¨ fvM K‡i †`q| A Zvi UvKv 8% Pµe„w× gybvdvq Ges B Zvi UvKv
10% mij gybvdvq wewb‡qvM K‡i| 2 eQi c‡i A Gi †P‡q B 1,336 UvKv †ewk gybvdv AR©b K‡i| Df‡qi UvKvi cwigvb wbY©q
Ki| Zv‡`i wcZvi Kv‡Q 25,000 UvKv wQj| Answer: Tk 10,000 and Tk. 15,000
2. Two equal glasses are respectively 1/3 and 1/4 full of milk. They are then filled up with water and the
contents inixed in a tumbler. What is the ratio of milk and water in the tumbler? Answer: 7:17
Abyev`t `yBwU mg cwigvb Møvm 1/3 Ges 1/4 Ask `ya Øviv c~Y©| Zvici †m¸‡jv cvwb Øviv c~Y© K‡i c‡i GKUv eo cv‡Î me¸‡jv
wgkv‡bv nj| IB eo cv‡Î `ya I cvwbi AbycvZ KZ?
3. One day, Mr. Wahid started 30 minutes late from home and reached his office 50 minutes late, while
driving 25% slower than his usual speed. How much time in minutes does Mr. Wahid usually take to
reach her office from home? Answer: 60 minutes
Abyev`t `yBwU mg cwigvb Møvm 1/3 Ges 1/4 Ask `ya Øviv c~Y©| Zvici †m¸‡jv cvwb Øviv c~Y© K‡i c‡i GKUv eo cv‡Î me ¸‡jv
wgkv‡bv nj| H eo cv‡Î `ya I cvwbi AbycvZ KZ?
4. AD is the longest side of the right triangle ABD shown in the figure. What is the length of longest
side of △ABC? Answer: √41

Written Math Solution (BIBM) FB Group : BD job Info. (BCS & Bank)
Bank Job Solution (BIBM) 37

Created by Sumon K. Sarkar  FB Group : BD job Info. (BCS & Bank)  Created by Sumon K. Sarkar 
Created by Sumon K. Sarkar  FB Group : BD job Info. (BCS & Bank)  Created by Sumon K. Sarkar 

Al-Arafah Bank Ltd.


Post name: Management Trainee Officer Exam Date : 2013
√3+1 √3−1 2 2
1. If 𝑥 = and 𝑦 = then find the value of 𝑥 + 𝑦 Answer:5/2
√3−1 √3+1
2. According to a car dealer's sale report, 1/3 of the cars sold during a certain period were Sedans and 1/5
of the other cars sold were station wagons. If N station wagons were sold during that period, how many
Sedans, in terms of N, were sold? Answer: 5N/2
Abyev`t GKRb Mvwoi wWjv‡ii weµq ZvwjKv †_‡K †`Lv hvq †h, wbw`©ó mg‡q hZ Mvwo wewµ nq Zvi 1/3 Ask wm`vbm Ges
Ab¨vb¨ wewµZ Mvwoi 1/5 Ask †÷kb IqvMbm| hw` H mg‡q N msL¨K †÷kb IqvMb wewµ n‡q _v‡K Zvn‡j hZ¸‡jv wm`vbm
wewµ nq Zv N Øviv cÖKvk Ki?
𝑥 𝑥
3. If years ago Samad was 12, and years from now he will be 2x years old, how will he be 3x years
2 2
from now? Answer:54 years
x x
Abyev`t hw` eQi c~‡e© mvgv‡`i eqm wQj 12 Ges eQi c‡i Zvi eqm 2x nq, Z‡e 3x eQi c‡i Zvi eqm KZ n‡e?
2 2
4. The ratio between the length and breadth of a rectangular park is 3:2. If a man cycling along the
boundary of the park at the speed of 12 km per hour completes one round in 8 minutes then what is the
area of the park? [Al-Arafah MTO 13, City Bank MTO 17] Answer:153600
Abyev`t GKwU AvqZKvi cv‡K©i ˆ`N©¨ I cÖ‡¯’i AbycvZ 3 t 2| †Kvb e¨w³ mvB‡Kj wb‡q N›Uvq 12 wK.wg †e‡M 8 wgwb‡U HcvK©wUi
Pvicv‡k GKevi Ny‡i Av‡mb| cvK©wUi †¶Îdj KZ?

Premier Bank Ltd.


Post name: Trainee Junior Officer Exam Date : 2013
1. A man borrows Tk. 2500 at 4% p.a. and Tk. 1800 at 5% simple interest for the same period. If he pays
Tk. 570 as total interest, find the time for which the sums were borrowed. Answer: 3 years
Abyev`t †Kvb e¨w³ GKwU wbw`©ó mg‡qi Rb¨ 4% my‡` 2,500 UvKv Ges 5% my‡` 1,800 UvKv FY MÖnY K‡ib| hw` wZwb GK‡Î
570 UvKv my` cÖ`vb K‡ib, Zvn‡j mgq wbY©q Ki|
2. The price of sugar and rice are in the ratio 4:5. If the price of sugar is increased by 10% and rice by
20% find the ratio between increased prices of sugar and rice. Answer: 11 : 15
Abyev`t wPwb Ges Pv‡ji `v‡gi AbycvZ 4 t 5| hw` wPwbi `vg 10% Ges Pv‡ji `vg 20% e„w× cvq Zvn‡j e„w× cvIqvi ci
wPwb I Pv‡ji `v‡gi AbycvZ wbY©q Kiæb|
3. The length of each side of a triangle is 12 cm. Find the height of the triangle. Answer: 6√𝟑
অনুবােঃ একনট নত্রভুতের প্রনেনট বাহুর দে ণয 12 পসনম। নত্রভুতের উচ্চো ননর্ণয কর
𝑎 1 3𝑎+2𝑏
4. If = then the value of Answer: - 3
𝑏 3 3𝑎−2𝑏

Written Math Solution (BIBM) FB Group : BD job Info. (BCS & Bank)
38

 Created by Sumon K. Sarkar  FB Group : BD job Info. (BCS & Bank)  Created by Sumon K. Sarkar 
Bank Job Solution (BIBM)
 Created by Sumon K. Sarkar  FB Group : BD job Info. (BCS & Bank)  Created by Sumon K. Sarkar 

Premier Bank Ltd.


Post name: Management Trainee Officer Exam Date : 2012
3
1. √8𝑥 2 √32𝑥√4𝑥 2 = 4, then the value of x? Answer:1
[Premier Bank MTO 12, RAKUB SO 14, IFIC Bank Premier Bank Officer Cash 13]
2. A person earns yearly interest of Tk. 920 by investing Tk. X at 4% and Tk. Y at 5% simple interest
rate. If he had invested Tk. X at 5% and Tk. Y at 4% simple interest rate, then his yearly interest earning
would have been reduced by Tk. 40. Find out the amount of X and Y.
[ONE Bank PO 10, NCC Bank MTO 11, Premier Bank MTO 12, Midland Bank TO 15] Answer: X=Tk 8000 and Y= Tk12,000
Abyev`t GK e¨w³ 4% my‡` x UvKv Ges 5% my‡` y UvKv wewb‡qvM K‡i eQ‡i 920 UvKv gybvdv AR©b K‡ib| wZwb hw` 5% my‡`
x UvKv Ges 4% my‡` y UvKv wewb‡qvM K‡ib Zvn‡j gybvdvi cwigvb 40 UvKv K‡g hv| x Ges y Gi cwigvb wbY©q Ki|
3. A basketball team has won 15 games and lost 9. If these games represent 16.67% of the games to the
played, then how many more gaines must the team win to average 75% for the season?
[ONE Bank PO 10, NCC Bank MTO 11, Premier Bank MTO 12, Midland Bank TO 15, Bangladesh bank AP 2023] Answer: 93 games
Abyev`t GKwU ev‡¯‹Uej wUg 15wU †Ljvq Rqjvf K‡i Ges 9wU †Ljvq civwRZ nq| hw` GB †Ljv¸‡jv Øviv Zv‡`i †gvU ‡Ljvi
16.67% †Ljv‡K †evSv‡bv nq Zvn‡j H wmR‡b M‡o 75% †Ljvq †RZvi Rb¨ Zv‡`i‡K KZwU †Ljvq wRZ‡Z n‡e?

One Bank Ltd.


Post name: Special Cadre Officer Exam Date : 2012
1. Suppose 81p + 62q=138 and 62p + 819 = 5, find out the value of p & q. Answer: p = 4 and q = -3
2. A person spends 1/3rd of the money with him on food, 1/5th of the remaining on education, 1/4th of the
remaining on treatment. Now he is left with Tk 200. How much did he have with him in the beginning?
Abyev`t GK e¨w³ Zvi UvKvi 1/3 Ask Lvev‡i e¨q K‡ib| Aewkó UvKvi 1/5 Ask covïbvq e¨q K‡ib| Gici hv Aewkó _v‡K
Zvi 1/4 Ask wPwKrmvLv‡Z e¨q Kivi ci Zvi nv‡Z 200 UvKv Aewkó _v‡K| Zvi Kv‡Q KZ UvKv wQj? Answer: TK. 500

Shahjalal Islami Bank Ltd.


Post: Trainee Officer Exam Date : 2011
1. The length of each side of a triangle is 12 cm. Find the height of the triangle. Answer: 6√𝟑
অনুবােঃ একনট নত্রভুতের প্রনেনট বাহুর দে ণয 12 পসনম। নত্রভুতের উচ্চো ননর্ণয কর
2. The average income of A for 15 days is Tk 70. The average for first five days is Tk. 60 and that for the
last nine days is Tk. 80. What is the income for the sixth day? Answer: Tk 30
Abyev`t A Gi 15 w`‡bi Mo Avq 70 UvKv| cÖ_g 5 w`‡bi Mo Avq 60 UvKv Ges †kl 9 w`‡bi Mo Avq 80 UvKv| 6ô w`‡b Zvi
Avq KZ wQj?
3. A man sells articles at 5% profit. If had bought it at 5% less and sold it for Tk. 1 less, he would have
gained 10%. What is the cost price of the article? Answer: Tk. 200
Abyev`t †Kvb e¨w³ 5% jvf cY¨ weµq K‡ib| hw` wZwb 5% Kg `v‡g wK‡b 1 UvKv K‡g weµq Ki‡Zb Zvn‡j Zvi 10% jvf
n‡Zv| cY¨wUi µqg~j¨ KZ?
4. Nipu, Nila and Dipa formed a partnership with investments of Tk. 75000, Tk. 60000 and Tk. 40000
respectively. After 3 years of operation, the partnership had a net profit of Tk 26250. What was the
share of Dipa in the profit? Answer: Tk. 11,250, Tk. 9000 and Tk. 6000
Abyev`t wbcy , bxjv Ges w`cy GKwU Askx`vwi‡Z¡ h_vµ‡g 75000, 60000 Ges 40000 UvKv wewb‡qvM K‡i| 3 eQi ci†gvU
jv‡fi cwigvY wQj 26250 UvKv | GB gybvdvq `xcvi Ask KZ UvKv wQj?

Written Math Solution (BIBM) FB Group : BD job Info. (BCS & Bank)
Bank Job Solution (BIBM) 39

Created by Sumon K. Sarkar  FB Group : BD job Info. (BCS & Bank)  Created by Sumon K. Sarkar 
Shahjalal Islami Bank Ltd.
Created by Sumon K. Sarkar  FB Group : BD job Info. (BCS & Bank)  Created by Sumon K. Sarkar 

Post: Trainee Senior Officer Exam Date : 2011


𝑎 1 3𝑎+2𝑏
1. If = then the value of Answer: -3
𝑏 3 3𝑎−2𝑏
2. A club has 20 members. They are electing a principal and a vice president. How inany different
outcome of the election are possible? (Assume the president and the vice president must be different
members of the club.) Answer:380
3. Tk. 1500 is invested at a rate of 10% simple interest and interest is added to the principal after every 5
years in how many years will it amount to Tk. 2500? Answer:6.11 years
Abyev`t hw` 10% mij gybvdvq 1,500 UvKv wewb‡qvM Kiv nq Ges cÖ‡Z¨K 5 eQi ci hw` gybvdv Avm‡ji mv‡_ †hvM Kiv nq
Zvn‡j KZ eQ‡i UvKvi cwigvb 2,500 n‡e?
4. The base of a rectangle exceeds three times the height by 8. Find the dimensions of the rectangle if-
(a) the semi-perimeter is 32 (b) the perimeter is 100
Answer: a) Base = 6 and height =26 b) Base = 10.5 and height = 39.5

Shahjalal Islami Bank Ltd.


Post: Management Trainee Officer Exam Date : 2011
1. Two Cans have the same height equal to 21 m. One Can is cylindrical; the diameter of whose base is
10 cmn. The other can has square base of side 10 cm. What is the difference in their capacities.
Abyev`t `ywU cv‡Îi cÖwZwUi D”PZv 21 wgUvi| GKwU cvÎ wmwjÛvi AvK…wZi hvi f~wgi e¨vm 10 †m.wg| Ab¨wU eM©vK…wZi hvi GK
av‡ii ˆ`N¨© 10 †m.wg| cvÎ `ywUi avib¶gZvi cv_©K¨ KZ? Answer: 45066 cm3
2. The cost price of two watches taken together is Tk. 840. If by selling one at a profit of 16% and the
other at a loss of 12%, there is no loss or gain in the whole transaction, find the cost price of the two
watches. [SJIB MTO 11, SIBL PO 17, NRBC Bank MTO 18] Answer: Tk. 360 & Tk. 480
Abyev`t GKm‡½ †Kbv 2wU Nwoi µqg~j¨ GK‡Î 840 UvKv| hw` GwU‡K 16% jv‡f Ges Ab¨wU‡K 12% ¶wZ‡Z weµq Ki‡j
†gv‡Ui Dci †Kvb jvf ev ¶wZ nq bv Zvn‡j Nwo 2wUi cÖwZwUi µqg~j¨ wbY©q Ki|
2. A train travelling at 48 km/h completely crosses another train having half its length and travelling in
opposite direction at 42 km/h, in 12 second. It also passes a railway platform in 45 second. What is the
length of the platform? Answer: 400 meters
Abyev`t 48 wK.wg/N›Uv †e‡Mi GKwU †Uªb wecixZ w`K †_‡K 42 wK.wg/N›Uv †e‡M Avmv Gi A‡a©K ˆ`‡N©¨i GKwU †Uªb‡K 12
†m‡K‡Û m¤ú~Y© iƒ‡c AwZµg K‡i| GwU †ijI‡q cøvUdg©wU‡K 45 †m‡K‡Û AwZµg K‡i| cøvUd‡g©i ˆ`N©¨ KZ?
3. A, B and C can do a piece of work in 16, 32 and 48 days respectively. They started working together
but C left after working 4 days and B left 2 days before the completion of work. How many days it
took to complete the work? Answer: 10 (4/9) days
[SJIB MTO 11, Social Islami Bank PO 17, NRBC Bank MTO 18, National Bank PO 17, Langka Bangla Finance MTO 17]
Abyev`t A, B I C GKwU KvR h_vµ‡g 16, 32 Ges 48 w`‡b Ki‡Z cv‡i| Zviv GKmv‡_ KvR Ki‡Z ïiæ K‡i wKš‘ KvR ïiæ
nIqvi 4 w`‡b ci C KvR †Q‡o †`q Ges KvR †kl nIqvi 2 w`b Av‡M B KvR †Q‡o †`q| m¤ú~Y© KvRwU †kln‡Z KZw`b mgq
jv‡M?

Written Math Solution (BIBM) FB Group : BD job Info. (BCS & Bank)
40

 Created by Sumon K. Sarkar  FB Group : BD job Info. (BCS & Bank)  Created by Sumon K. Sarkar 
Bank Job Solution (BIBM)
 Created by Sumon K. Sarkar  FB Group : BD job Info. (BCS & Bank)  Created by Sumon K. Sarkar 

Standard Bank Ltd.


Post name: Trainee Assistant Officer(Cash) Exam Date : 2011
𝑎1/2 +𝑎 −1/2 1−𝑎 −1/2
1. Simplify: + Answer: 2/(1-a)
1−𝑎 1+√𝑎
2. In a survey, 60% of those surveyed owned a car and 80% of those surveyed owned a TV. If 55% owned
both a car and a TV, what percent of those surveyed owned a car or a TV or both? Answer: 85%
Abyev`t GKwU cwimsL¨v‡b †`Lv hvq 60% †jv‡Ki GKwU Mvwo Av‡Q Ges 80% †jv‡Ki GKwU wUwf Av‡Q| hw` 55% †jv‡Ki
GKwU Mvwo Ges GKwU wUwf _v‡K Zvn‡j kZKiv KZRb †jv‡Ki GKwU Mvwo A_ev GKwU wUwf DfqB Av‡Q?
3. Water has been poured into an empty rectangular tank at the rate of 5 cubic feet per minute for 6
minutes. The length of the tank is 4 feet and the width is one half of the length. How deep is the water
in the tank? Answer: 3 Feet 9 inches
Abyev`t cvwbk~Y¨ GKwU AvqZKvi †PŠev”Pv cÖwZ wgwb‡U 5 Nb dzU nv‡i cvwbc~Y© Ki‡Z 6 wgwbU jv‡M| †PŠev”PvwUi ˆ`N©¨ 4 dzU Ges
cÖ¯’ ˆ`‡N¨©i A‡a©K| †PŠev”PvwUi MfxiZv KZ?

NCC Bank Ltd.


Post : Management Trainee Officer Exam Date : 2011
1. A person earns yearly interest of Tk. 920 by investing Tk. X at 4% and Tk. Y at 5% simple interest
rate. If he had invested Tk. X at 5% and Tk. Y at 4% simple interest rate, then his yearly interest earning
would have been reduced by Tk. 40. Find out the amount of X and Y.
[ONE Bank PO 10, NCC Bank MTO 11, Premier Bank MTO 12, Midland Bank TO 15 ] Answer: X=Tk 8000 and Y= Tk12,000
Abyev`t GK e¨w³ 4% my‡` x UvKv Ges 5% my‡` y UvKv wewb‡qvM K‡i eQ‡i 920 UvKv gybvdv AR©b K‡ib| wZwb hw` 5% my‡`
x UvKv Ges 4% my‡` y UvKv wewb‡qvM K‡ib Zvn‡j gybvdvi cwigvb 40 UvKv K‡g hv| x Ges y Gi cwigvb wbY©q Ki|
2. A basketball team has won 15 games and lost 9. If these games represent 16.67% of the games to the
played, then how many more games must the team win to average 75% for the season?
[ONE Bank PO 10, NCC Bank MTO 11. Premier Bank MTO 12, Midland Bank TO 15, Bangladesh bank AP 2023] Answer: 93 games
Abyev`t GKwU ev‡¯‹Uej wUg 15wU †Ljvq Rqjvf K‡i Ges 9wU †Ljvq civwRZ nq| hw` GB †Ljv¸‡jv Øviv Zv‡`i †gvU ‡Ljvi
16.67% †Ljv‡K †evSv‡bv nq Zvn‡j H wmR‡b M‡o 75% †Ljvq †RZvi Rb¨ Zv‡`i‡K KZwU †Ljvq wRZ‡Z n‡e?
3. A book sells for Tk. 65. This price gives the seller a profit of 30% on his cost. What will be the new
selling price if he cuts his profit to 10% of its cost? Answer: Tk 55
Abyev`t GKwU eB‡qi weµqg~j¨ 65 UvKv| G‡Z we‡µZvi 30% jvf nq| hw` wZwb 10% jvf K‡ib Zvn‡j Zvi bZzb weµqg~j¨
KZ n‡e?

UCB Bank Ltd.


Post: Management Trainee Officer Exam Date : 2011
1. A pension fund has a total of Tk. 1 million invested in Bangladesh Biman bond and DEF Corporation's
debenture. The Bangladesh Biman bond yields 12% in cash each year, and the DEF debenture pays
10% in cash each year. The pension fund received a total of Tk 115,000 in cash from Bangladesh
Biman bond and DEF debenture last year. How much money was invested in Bangladesh Biman bond?
Abyev`t GKwU †cbkb dv‡Ûi 1 wgwjqb (10 j¶) UvKv evsjv‡`k wegv‡bi Pzw³c‡Î Ges wWBGd K‡c©v‡ik‡bi FYc‡Î wewb‡qvM
Kiv n‡q‡Q| evsjv‡`k wegv‡bi Pzw³cÎ Ges wWBGd K‡c©v‡ik‡bi FYcÎ †_‡K cÖwZ eQi h_vµ‡g 12% I 10% nv‡i bM` gybvdv
Avq nq| MZ eQi Gfv‡e †cbkb dvÛwU bM` 115,000 UvKv Avq K‡i| evsjv‡`k wegv‡bi Pzw³c‡Î KZ UvKv wewb‡qvM Kiv
n‡qwQ‡jv? Answer: Tk. 7,50,000

Written Math Solution (BIBM) FB Group : BD job Info. (BCS & Bank)
Bank Job Solution (BIBM) 41

Created by Sumon K. Sarkar  FB Group : BD job Info. (BCS & Bank)  Created by Sumon K. Sarkar 
2. Company A owns 40% of the stock in ABC Company. Company B owns 15,000 shares. Company C
Created by Sumon K. Sarkar  FB Group : BD job Info. (BCS & Bank)  Created by Sumon K. Sarkar 

owns all the share not owned by Companies A or B. How many shares of stock does Company A own
if Company C has 25% more share than Company A? Answer: 60,000 share
[Premier Bank Officer 03, UCBL MTO 11]
Abyev`t XYZ Kc©‡ik‡bi 40% ÷K †Kv¤úvbx A Gi gvwjKvbvaxb| Company B Gi †kqv‡i cwigvb 15,000| A Ges B
Gi ev‡` evwK mg¯Í †kqv‡qi gvwjK C| C Gi hw` A Gi †P‡q 25% †ewk †kqvi _v‡K Zvn‡j A Gi †kqv‡ii cwigvb KZ?
3. A manufacturer of boxes wants to make a profit of x taka. When he sells 5,000 boxes it costs Ik. 5 a
box to make the first 1000 boxes and then it costs Tk. y a box to make the remaining 4,000 boxes.
What price in taka should be charge for the 5000 boxes? Answer:Tk.(4000y + 5000 +x)
Abyev`t GKRb ev· cÖ¯‘ZKvix x UvKv jvf Ki‡Z Pvb| hLb wZwb 5000 ev· weµq K‡ib ZLb wZwb cÖ_g 1000 ev‡·i cÖwZwU‡Z
LiP K‡ib 5 UvKv, Gici evwK 4000 ev· ˆZix‡Z LiP K‡ib y UvKv| 5000 ev‡·i g~j¨ KZ avh© nIqv DwPZ?
4. In an increasing sequence of 10 consecutive integers, the sum of the first 5 integers is 560. What is the
sum of the last 5 integers in the sequence? Answer: 585
Abyev`t 10wU µgea©gvb c~Y©msL¨vi, cÖ_g 5wUi †hvMdj 560| †kl 5wU msL¨vi †hvMdj KZ?

UCB Bank Ltd.


Post name: Officer Exam Date : 2011
2
1. If x + xy +1 = 16 and x-y=1, then x+y=? Answer: 5 or -6
2. Mr. B invests Ik. 2400 at 5% interest annually. How much additional money needs to invest at 8%
interest to earn overall interest at 6% on entire amount? Answer: Tk 1200
[UCBL Officer 11, Modhumoti PO 18, SJIB TSO 18]
Abyev`t B mv‡ne evwl©K kZKiv 5 UvKv mij gybvdvq 2400 UvKv wewb‡qvM K‡ib| evwl©K kZKiv 8 UvKv mij gybvdvqAvi KZ
UvKv wewb‡qvM Ki‡j wZwb †gvU wewb‡qv‡Mi Dci GK‡Î evwl©K kZKiv 6 UvKv mij gybvdv cv‡eb?
3. A square office, 1000 feet by 1000 feet, is to be partitioned into two offices by a single interior wall.
The difference between the perimeters of the resulting two officers be 400 feet. What are their
dimensions? [BB AD 06, UCBL Officer 11] Answer: 600ft & 400ft
অনুবােঃ ১০০নফট বাই ১০০০ নফট এর বগণাকার অনফস দুই ভাতগ নবভি করতে হতব পযন দুই অংতশ্র পনরসীমার পার্ণকয ৪০০ নফট হে।
তরর দে ণয অ প্রস্থ কে?
4. A boy covers a distance of 6 km partly by walking and partly by cycling. If he cycles at 18 km per
hour and walks at 6 km per hour and takes 35 minutes in all, find the distance he covers by walking.
Abyev`t GKRb evjK 6 wKwg `~i‡Z¡i wKQz Ask †nu‡U Ges wKQz Ask mvB‡Kj Pvwj‡q †M‡jv| hw` mvB‡K‡j N›Uvq 18 wKwg †e‡M
Ges N›Uvq 6 wKwg †e‡M †nu‡U hvq Ges †gvU 35 wgwbU mgq †bq, Zvn‡j †nu‡U AwZµvšÍ `~iZ¡ wbY©q Kiæb|
[Combined Cash 08, UCBL Officer 11] Answer: 2.25 km

IFIC Bank Ltd.


Post: Officer Exam Date : 2011
1. A trader bought 65 meter cloth and sold 20 meter at a profit of 15%, 30 meter at a profit of 19% and
the rest at a profit of 25%. IN total he made a profit of tk. 498.What was the purchase price of pa meter
of cloth?
অনুবােঃ একেন নবতক্রো ৬৫ নমটার কাপড ক্রয কতরন এবং ১৫% লাতভ ২০ নমটার, ১৯% লাতভ ৩০ নমটার এবং অবনশ্ষ্ট কাপড ২৫%
লাতভ নবক্রয করন! নেনন পমাট ৪৯৮ টাকা লাভ কতরন। প্রনে নমটার কাপতডর ক্রযমূলয কে টাকা ?

Written Math Solution (BIBM) FB Group : BD job Info. (BCS & Bank)
42

 Created by Sumon K. Sarkar  FB Group : BD job Info. (BCS & Bank)  Created by Sumon K. Sarkar 
Bank Job Solution (BIBM)
 Created by Sumon K. Sarkar  FB Group : BD job Info. (BCS & Bank)  Created by Sumon K. Sarkar 

2. A boat sailing against the current takes 8 hours to travel 32 kms, while sailing with the current it takes
only 4 hours to travels the same distance. What is the speed of the current?
অনুবােঃ পরাতের নবপরীতে একনট পনৌকা 32 নকতলানমটার পযতে 8 ন্টা সময পনয, যখন পরাতের সাতর্ যাত্রা কতর একই েূরত্ব পযতে 4
ন্টা লাতগ। পরাতের গনে কে?
𝑎 𝑥 𝑥 𝑏
3. If + = + then x=?
𝑥 𝑎 𝑏 𝑥

UCB Bank Ltd.


Post: Officer Exam Date : 2010
1 1 1
1. If 2x = 4y = 8z and + + = 4 then find the value of x Answer: 1/2
2𝑥 4𝑦 4𝑧
2. Fifteen years hence A will be twice as old as his son, but five years ago he was 4 times as old as his
son, Find their present age. Answer: 15, 45
অনুবােঃ ১৫ বছর পর A এর বেস হতব োর পুতত্রর বেতসর নিগুন। নকন্তু ৫ বছর পূতবণ পস োর পুতত্রর বেতসর ৪ গুন নছল। োতের বেণমান
বেস কে?
3. In a stream running at 2 kmph, a motor boat goes 10 km upstream and back again to the starting point
in 55 minutes. Find the speed of the motorboat in still water. Answer: 22kmph
Abyev`t GKwU †gvUi PvwjZ †bŠKv 2 wK.wg / N›Uv †e‡Mi †mªv‡Zi wecix‡Z 10 wK.wg wM‡q Avevi wd‡i Avm‡Z †gvU 55 wgwbU mgq
†bq| w¯’i cvwb‡Z †bŠKvwUi †eM wbY©q Ki|
4. Janaki spends 20% of her monthly salary for her son's education. When she will get her increment of
Tk. 170 next month, she plans to spend half of that also for her son's education. The total money she
would spend then, would be Tk. 645. What is her present salary? Answer: Tk. 2800
Abyev`t †RvbwK Zvi mšÍv‡bi cov‡jLvi †cQ‡b Zvi †eZ‡bi 20% e¨q K‡ib| wZwb wm×všÍ wb‡q‡Qb †h, AvMvgx gv‡m hLb Zvi
†eZb 170 UvKv e„w× cv‡e, ZLb wZwb GB UvKvi A‡a©KI Zvi mšÍv‡bi cov‡kvbvi †cQ‡b e¨q Ki‡eb | ZLb wZwb †gvU 645
UvKv e¨vq Ki‡eb | Zvi eZ©gvb †eZb KZ?
5. A wheel makes 1000 revolution in covering a distance of 88 km. Determine the diameter of the wheel.
Abyev`t GKwU PvKvi 80 wKwg `~iZ¡ cvwo w`‡Z 1000 N~Y©‡bi cÖ‡qvRb nq| PvKvwUi e¨vm wbY©q Kiæb| Answer: 28 meter
6. In an examination, a student was asked to find 3/4 of a certain number. By mistaken he found 3/14 of
it. His answer was 150 more than the correct answer. Find the number. Answer: - 280
Abyev`t GKwU cix¶vq GKRb Qv·K GKwU msL¨vi 3/4 Ask †ei Ki‡Z ejv nj| fzjekZ †m AsKwUi 3/14 Ask †ei K‡i
†djj| cÖvß djvdj mwVK djvdj †_‡K 150 †ewk wQj| msL¨vwU wbY©q Kiæb|

Shahjalal Islami Bank Ltd.


Post: Management Trainee Officer Exam Date : 2010
1. A man is standing on a railway bridge which is 180 m long. He finds that a train crosses the bridge in
20 seconds but him in 8 seconds. Find the length of the train. Answer: 120 m
[Sonali Officer IT 16, SJIB MTO 10]
Abyev`t †Kvb e¨w³ 180 wg. j¤^v GKwU †ijI‡q eªx‡Ri Dci `vwo‡q Av‡Q| wZwb †`L‡jb †h GKwU †Uªb 20 †m‡K‡Û eªxRwU
AwZµg K‡i wKš‘ Zv‡K 8 †m‡K‡Û AwZµg K‡i hvq| †Uª‡bi ˆ`N¨© wbY©q Ki|
2. A trader bought some mangoes for Tk. 150 per dozen and equal number of apples for Tk. 100 per
dozen. If he sells all the fruits Tk. 140 per dozen, what will be his profit/loss in percentage?
[BASIC Bank PO 99, RAKUB SO 14, SJIB MTO 10] Answer:12% profit
Abyev`t GKRb e¨emvqx cÖwZ WRb 150 UvKv `‡i wKQz Avg Ges cÖwZ WRb 100 UvKv `‡i mgvb msL¨K Av‡cj µq K‡ib| hw`
†m me¸‡jv dj 140 UvKv WRb wn‡m‡e weµq K‡i Zvn‡j Zvi kZKiv KZ jvf ev ¶wZ n‡e?

Written Math Solution (BIBM) FB Group : BD job Info. (BCS & Bank)
Bank Job Solution (BIBM) 43

Created by Sumon K. Sarkar  FB Group : BD job Info. (BCS & Bank)  Created by Sumon K. Sarkar 
1 5
3. In a school, there are equal number of boys and girls. Among the students of the girls and th of the
8 6
Created by Sumon K. Sarkar  FB Group : BD job Info. (BCS & Bank)  Created by Sumon K. Sarkar 

boys are residing in the hostel. What percent of the Students consists of boys who do not reside in the
hostel among all students? [Modhumoti Bank MTO 16, SJIB MTO 10] Answer:8.33%
Abyev`t GKwU we`¨vj‡q †Q‡j Ges †g‡qi msL¨v mgvb| wk¶v_©x‡`i g‡a¨ 1/8 Ask †g‡q Ges 5/6 Ask †Q‡j †nv‡÷‡j evm K‡i|
†gvU wk¶v_©xi g‡a¨ kZKiv KZRb †Q‡j †nv‡÷‡j evm K‡i bv?
4. Two partner's A and B have 70% and 30% shares respectively in a business. After sometimes, a third
partner C joined by investing Tk. 10 lakh and thus having 20% share in the business. What is the
percentage of share of A's now in the business? [RAKUB SO 14, SJIB MTO 10] Answer: 56%
Abyev`t `yBRb cvU©bvi A I B GKwU e¨emvq h_vµ‡g 70% I 30% †kqv‡ii Askx`vi| wKQzw`b ci C Zv‡`i mv‡_ †hvM †`q
Ges †m 10 j¶ UvKv wewb‡qvM Kivq e¨emvq Zvi †kqv‡ii cwigvb nq 20%| eZ©gv‡b e¨emvq A Gi kZKiv †kqv‡ii cwigvb
KZ?
5. Taxi fare is described by the following relationship:
Total taxi Fare = Fixed Charge: Tk. A up to 2 km + Tk. B per km run exceeding 2 kın + Tk 60 for per hour's waiting.
A person paid Tk. 432 for running 52 km and 2 hours of waiting charge. The same person paid Tk. 732 for running
102 km and 2 hours of waiting charge. Find the value of A and B Answer: A = 12 & B = 6
[ Bangladesh bank officer (electrical) 2023]
Abyev`t wb‡Pi m¤c‡K©i mvnvh¨ †Uw· fvov †`Lv‡bv hvq:
†gvU †Uw· fvov = ¯’vqx PvR© A UvKv 2wKwg ch©šÍ + B UvKv 2 wKwg c‡i +60 UvKv cÖwZ N›Uv A‡c¶v PvR©|
GKRb e¨w³ 52 wKwg Ges 2 N›Uv A‡c¶vi Rb¨ 432 UvKv w`j| GKB e¨w³ 102 wKwg I 2 N›Uv A‡c¶vi Rb¨ 732 UvKv cÖ`vb
K‡i| A Ges B wbb©q Ki|
6. In the figure below, AB is perpendicular to BC and BD = DC. If AD = √10 cm and AC = 4 cm, then
what is the value of BC? Answer: 2√𝟐 cm

Social Islami Bank Ltd.


Post name: Trainee Officer Exam Date : 2010
2 3
1. Solve the following equation: + =1 Answer: 2 ±√10
𝑥−2 𝑥+3
2. An article is sold for Tk. 190, hence gaining a certain amount. Had the article been sold for Tk. 175,
he would have suffered loss equal to 50% of the gain in the first case. Find cost price of the article.
Abyev`t GKUv cY¨ 190 UvKvq weµq K‡i wbw`©ô cwigvb jvf nj| hw` cY¨wU 175 UvKvq weµq Kiv n‡Zv Z‡e Zvi Av‡Mi jv‡fi
50% Gi mgvb ¶wZ n‡Zv| cY¨wUi µqg~j¨ KZ? Answer: Tk. 180
3. In an examination, a student gets 38% of the maximum marks and fails by 6 marks. Another student
who gets 45% of the maximum marks, gets 15 marks more than the required passing percentage. Find
passing percentage of the marks. Answer: 40%
Abyev`t †Kvb cix¶vq GKRb wk¶v_©x H cix¶vq cÖvß m‡e©v”P b¤^‡ii 38% †c‡q 6 b¤^‡ii e¨veav‡b †dj K‡i| Ab¨ GKRb
wk¶v_©x m‡e©v”P bv¤^v‡ii 45% †c‡q cvk b¤^‡ii †P‡q 15 b¤^i †ewk cvq| H cix¶vi kZKiv cvk b¤^i KZ wQj?

Written Math Solution (BIBM) FB Group : BD job Info. (BCS & Bank)
44

 Created by Sumon K. Sarkar  FB Group : BD job Info. (BCS & Bank)  Created by Sumon K. Sarkar 
Bank Job Solution (BIBM)
 Created by Sumon K. Sarkar  FB Group : BD job Info. (BCS & Bank)  Created by Sumon K. Sarkar 

4. A sum of Tk. 5500 is divided into two parts and invested at 4% p.a. for 3 years and at 5% p.a. for 6
years respectively. If interest earned on the second part is three times of that on the first part, find the
amount invested in second part. Answer:Tk. 3000
Abyev`t 5500 UvKv `yB fv‡M fvM K‡i, GK fvM evwl©K 4% gybvdvq 3 eQ‡ii Rb¨ Ges Aci fvM evwl©K 5% gybvdvq 6
eQ‡ii Rb¨| hw` wØZxq fv‡M cÖvß gybvdv cÖ_g fv‡Mi Zzjbvq 3 ¸Y †ewk nq, Zvn‡j wØZxq fv‡M wewb‡qvMK…Z
A‡_©i cwigvY wbY©q Kiæb|
5. The length of a rectangle is 7 more than its width. If the perimeter of the rectangle is the same as the
perimeter of a square of side 8.5, what is the length of a diagonal of the rectangle? Answer: 13
Abyev`t GKwU AvqZ‡¶‡Îi ˆ`N¨©, cÖ‡¯’i Zzjbvq 7 GKK †ewk| hw` AvqZ‡¶ÎwUi cwimxgv, 8.5 GKK avi wewkó eM©‡¶‡Îi
cwimxgvi mgvb nq; Zvn‡j AvqZ‡¶‡Îi K‡Y©i ˆ`N¨© KZ?

One Bank Ltd.


Post: Probationary Officer Exam Date : 2010
5 15
1. A snapshot measures inches by inches. It is to be enlarged so that the longer dimension will be 4
2 8
inches. What will be the length of the enlarged shorter dimension? Answer: 3 inches
5 15
Abyev`t GKwU mœ¨vc k‡Ui cwigvc Bw evB BwÂ| GUv‡K Ggbfv‡e evov‡bv nq †hb e„nËi cÖvšÍwU 4 Bw nq| ¶z`ªZi
2 8
cÖvšÍwUi ˆ`N¨© KZ n‡e?
2. A basketball team has won 15 games and lost 9. If these games represent 16.67% of the games to the
played, then how many more games must the team win to average 75% for the season?
[ONE Bank PO 10, NCC Bank MTO 11, Premier Bank MTO 12, Midland Bank TO 15 , Bangladesh bank AP 2023] Answer:93 games
Abyev`t GKwU ev‡¯‹Uej wUg 15wU †Ljvq Rqjvf K‡i Ges 9wU †Ljvq civwRZ nq| hw` GB †Ljv¸‡jv Øviv Zv‡`i †gvU
‡Ljvi 16.67% †Ljv‡K †evSv‡bv nq Zvn‡j H wmR‡b M‡o 75% †Ljvq †RZvi Rb¨ Zv‡`i‡K KZwU †Ljvq
wRZ‡Z n‡e?
4 √𝑝 ×16𝑝𝑞 ×2−2√𝑝
3. × 4−𝑝𝑞 = ? Answer: 1
8√𝑞 ×4 𝑝𝑞 ×2−3√𝑞
4. A person earns yearly interest of Tk. 920 by investing Tk. X at 4% and Tk. Y at 5% simple interest
rate. If he had invested Tk. X at 5% and Tk. Y at 4% simple interest rate, then his yearly interest earning
would have been reduced by Tk. 40. Find out the amount of X and Y.
[ONE Bank PO 10, NCC Bank MTO 11, Premier Bank MTO 12, Midland Bank TO 15]
Abyev`t GK e¨w³ 4% my‡` x UvKv Ges 5% my‡` y UvKv wewb‡qvM K‡i eQ‡i 920 UvKv gybvdv AR©b K‡ib| wZwb hw`
5% my‡` x UvKv Ges 4% my‡` y UvKv wewb‡qvM K‡ib Zvn‡j gybvdvi cwigvb 40 UvKv K‡g hv| x Ges y Gi
cwigvb wbY©q Ki| Answer: X = Tk. 8000 and Y = Tk. 12,000
5. 10% fruit of a seller was damaged during transportation, another 15% was rotten. At what profit in
percentage should he sell rest fruit so that he can make an overall profit of 20%? Answer: 60%
Abyev`t GKRb dj we‡µZvi 10% dj cwien‡bi mgq bó nq Ges 15% dj cu‡P hvq| kZKiv KZ jv‡f Aewkó
dj¸‡jv weµq Ki‡j †gv‡Ui Dci Zvi 20% jvf n‡e?

Bangladesh Bank
Post Name: Assistant Director Exam Date : 2010
1. Three partners A, B, and C start a business. Twice the capital of A is equal to thrice the capital of B,
and the capital of B is 4 times the capital of C. They share the profit in the ratio of their capital. In a

Written Math Solution (BIBM) FB Group : BD job Info. (BCS & Bank)
Bank Job Solution (BIBM) 45

Created by Sumon K. Sarkar  FB Group : BD job Info. (BCS & Bank)  Created by Sumon K. Sarkar 
particular year, the gross profit is Tk. 2,50,000 and the administrative expenses are 20% of the gross
Created by Sumon K. Sarkar  FB Group : BD job Info. (BCS & Bank)  Created by Sumon K. Sarkar 

profit. Find the share of profit of each partner. Answer: Tk 1,09,090 , Tk 72,727, Tk 18,183
অনুবােঃ নেন অংশ্ীোর A, B, এবং C একনট বযবসা শুরু কতর। A এর নিগুর্ মূলযন B এর নেনগুর্ মূলযতনর সমান এবং B এর মূলযন
C এর মূলযতনর 4 গুর্। োরা োতের মূলযতনর অনুপাতে মুনাফা ভাগ কতর পনয। একনট নননেণষ্ট বছতর পমাট মুনাফা হয ২৫০,০০০ টাকা।
এবং প্রশ্াসননক বযয পমাট লাতভর 20%। প্রনেনট অংশ্ীোতরর লাতভর অংশ্ কে?
2. Tanim bought some oranges. He gave ½ of them to his sister, 1/4th of the remainder to his neighbor,
3/5th of those left to his children and had 6 left in the end. How many oranges did Tanim buy?
Abyev`t Zvwbg wKQz Kgjv‡jey wKbj, hvi A‡a©K †m Zvi †evb‡K w`j| Aewkó Kgjvi 1/4 fvM cÖwZ‡ewk‡`i w`j| Gici hv
Aewkó wQj Zvi 3/5 fvM Zvi mšÍvb‡`i †`Iqvi ci Zvi Kv‡Q 6wU Kgjv iBj| Zvwbg KZwU Kgjv wK‡bwQj?
[BB AD 10, MTB MTO 19] Answer: 40 oranges.
3. The length of rectangular plot is greater than its breadth by 20 meters. If the perimeter of the plot is
160 meters, what is the area of the plot in square meters? Answer: 1500 square meters
অনুবােঃ আযেতেত্রাকার প্লতটর দে ণয োর প্রতস্থর প তয 20 নমটার পবনশ্। প্লতটর পনরসীমা 160 নমটার হতল, বগণ নমটাতর প্লতটর পেত্রফল কে?

Bangladesh Bank
Post Name: Assistant Director Exam Date : 2009
1. A boy purchased some chocolates from a shop for Tk. 120. If the price of chocolate was reduced by
20%, he could have purchased 2 more chocolates. How many chocolates did he buy?
অনুবােঃ একনট পছতল একনট পোকান পর্তক 120 টাকায নকছু কতলট নকতনতছ। যনে কতলতটর োম 20% কমাতনা পযে, োহতল নেনন
আরও 2নট কতলট নকনতে পারতেন। পস কযনট কতলট নকতনতছ? Answer: 8 chocolate
2. Recently Kamal's hourly wage has been increased by 10%. Before this increase, Kamal's total weekly
wage was Tk. 137. If his weekly working hours were to decrease by 10% from last week's total working
hours, what would be the change, if any, in Kamal's total weekly wage? Answer: Tk. 1.37
অনুবােঃ সম্প্রনে কামাতলর ণ্টাপ্রনে মেুনর ১০ শ্োংশ্ বৃনি করা হতযতছ। এই বৃনির আতগ কামাতলর পমাট সাপ্তানহক মেুনর নছল ১৩৭ টাকা।
যনে োর সাপ্তানহক কমণ ন্টা গে সপ্তাতহর পমাট কমণ ণ্টার পর্তক 10% কতম যায, োহতল কামাতলর পমাট সাপ্তানহক মেুনরতে নক পনরবেণন হতব?
3. A loss of 15% is incurred by selling a watch for Tk. 612. How much is the sum of money by which it
is sold to make a profit of 10%? Answer: Tk. 792
অনুবােঃ একনট নড 612 টাকায নবনক্র করতল 15% পলাকসান হয।10% মুনাফা করতে নবনক্র করা টাকার পযাগফল কে?

Basic Bank Ltd.


Post: Assistant Officer Exam Date : 2009
1 1
1. If a - = m, then show 𝑎4 + = 𝑚4 + 4𝑚2 + 2 [BASIC AO 09, Board Book 8 & 9/10]
𝑎 𝑎4
2. On a certain X-Y committee, 2/3rd of the members are men, and 3/8th of the men from country Y. If
3/5th of the committee members are from country X, what fraction of the members are women from
country Y? Answer: 3/20
Abyev`t GKwU x-y KwgwU‡Z, m`m¨‡`i 2/3 Ask cyiæl Ges cyiæl‡`i 3/8 Ask y †`‡ki| hw` KwgwUi m`m¨‡`i 3/5
Ask x †`‡ki nq Zvn‡j KZRb gwnjv m`m¨ y †`‡ki?
3. Malek spends 75% of his income. His income is increased by 20% and be increases his expenditure by
10%. Calculate the percentage of his increased amount of savings? Answer: 50%
Abyev`t gv‡jK Zvi Av‡qi 75% LiP K‡i| Zvi Avq 20% e„w× cvq Ges LiP 10% e„w× cvq| Zvi mÂq †h cwigvb e„w×
cvq Zvi kZKiv cwigvb wbY©q Ki|

Written Math Solution (BIBM) FB Group : BD job Info. (BCS & Bank)
46

 Created by Sumon K. Sarkar  FB Group : BD job Info. (BCS & Bank)  Created by Sumon K. Sarkar 
Bank Job Solution (BIBM)
 Created by Sumon K. Sarkar  FB Group : BD job Info. (BCS & Bank)  Created by Sumon K. Sarkar 

4. Three partners A, B and C start a business. Twice the investment of A is equal to thrice the capital of
B and the capital of B is 4 times the capital of C. Find the share of each out of a profit of Tk. 297,000.
Abyev`t wZbRb Askx`vi A, B Ges C GKwU e¨emv ïiæ K‡i| A Gi wewb‡qv‡Mi wظY B Gi wewb‡qv‡Mi wZb¸‡bi mgvb
Ges B Gi wewb‡qvM C Gi wewb‡qv‡Mi Pvi¸‡Yi mgvb| 2,97,000 UvKv jvf n‡j Zv‡`i cÖ‡Z¨‡Ki jf¨vsk wbY©q
Ki| [BASIC Bank AO 09. Rupali Officer 13] Answer:Tk. 1,62,000, Tk. 1,08,000 and Tk. 27,000
5. A tank can be filled by a tap in 20 minutes and by another tap in 60 minutes. Both the taps are kept
open for 10 minutes and then the first tap is shut off. After this, the tank will be completely filled in
what time? [PKB SEO 18, BASIC Bank AO 09] Answer: 20 min
Abyev`t GKwU †PŠev”Pv X bj Øviv cvwbc~Y© n‡Z 20 wgwbU Ges Y bj Øviv cvwb c~Y© n‡Z 60 wgwbU jv‡M| 2wU bj GKmv‡_
10 wgwbU Pvjy ivLvi ci X bjwU eÜ K‡i ‡`qv nq| Zvici, †PŠev”PvwU cvwbc~Y© Ki‡Z KZ mgq jvM‡e?
6. On a certain X-Y committee, 2/3 of the members are men, and 3/8 of the men are from country Y, If
3/5 of the committee members are from country X. What fraction of the members are Women from
country Y? Answer: 3/20
অনুবােঃ একনট নননেণষ্ট X-Y কনমনটতে, সেসযতের 2/3 পুরুে, এবং পুরুেতের 3/8 েন Y পেশ্ পর্তক, যনে কনমনটর সেসযতের 3/5 সেসয
পেশ্ X পর্তক হয। সেসযতের কে অংশ্ Y পেতশ্র মনহলা?
7. In the square ABCD, the four un-shaded parts are quarter circles each having the same radius. If the
side of the square is 16. What is the area of the shaded part? Express the answer in term of 𝜋 .
Answer:256 - 64 π

ICB
Post name: Assistant Programmer Exam Date : 2008
1. In a certain conference room, 2/3 of the male participants & 1/2 of the female participants speak
Bangla. If the ratio of male & female participants is 6:4, then what fraction of the participants of
conference room speak Bangla? Answer: 3/5
Abyev`t GKwU wbw`©ó Kbdv‡iÝ K‡¶, cyiæl AskMÖnYKvix‡`i 2/3 Ask Ges gwnjv AskMÖnYKvix‡`i 1/2 Ask evsjvq K_v
e‡j| hw` cyiæl I gwnjv AskMÖnYKvix‡`i AbycvZ 6t4 nq, Zvn‡j Kbdv‡iÝ K‡¶ AskMÖnYKvix‡`i KZ Ask
evsjvq K_v e‡j?
2. If 3𝑎 − 5𝑏 = 4 𝑎𝑛𝑑 3𝑎+2 − 5𝑏+1 = 56 then determine the values of a & b? Answer: a=2,b=1
3. A water tank has two taps (Tap-1 and tap-2). Tap-l can fill a tank in 8 hours and Tap-2 can empty the
tank in 16 hours. How long will they to fill the tank if both taps are opened simultaneously but tap-2
is closed after 8 hours? Answer: 12 hours
Abyev`t GKwU cvwbi U¨v‡¼i `yBwU U¨vc (U¨vc-1 I U¨vc -2)| U¨vc-1 GKwU U¨v¼ AvU NÈvq c~Y© K‡i Ges U¨vc-2 16
NÈvq U¨v¼wU Lvwj K‡i| hw` Dfq U¨vc GK‡Î †Lvjv _v‡K wKš‘ AvU NÈv ci U¨vc-2 eÜ K‡i †`Iqv nq, Z‡e
U¨v¼wU c~Y© n‡Z KZ mgq jvM‡e?
4. A certain principal amount including interest in 3 years is Tk. 390 and it will be Tk. 450 in 5 years.
Determine the simple interest rate & principle amount. [ Class 8 Board Book] Answer: 10%
Abyev`t †Kvb Avmj my` Avm‡j 3 eQ‡i 390 Ges 5 eQ‡i 450 UvKv nq| kZKiv my‡`i nvi I Avmj wbY©q Kiæb|

Written Math Solution (BIBM) FB Group : BD job Info. (BCS & Bank)
Bank Job Solution (BIBM) 47

Created by Sumon K. Sarkar  FB Group : BD job Info. (BCS & Bank)  Created by Sumon K. Sarkar 
Bangladesh Bank
Created by Sumon K. Sarkar  FB Group : BD job Info. (BCS & Bank)  Created by Sumon K. Sarkar 

Post Name: Assistant Director Exam Date : 2008


1. Mr. Rahim pays 10% tax on all income earned over Tk 60000 but he does not pay any tax on interest
on saving certificate. In 2006 he paid 7500 as tax & he earned 12000 as interest on savings certificates.
What is his total income in 2006. Answer: Tk. 1,47,000
অনুবােঃ েনাব রনহম 60000 টাকার উপতর অনেণে সমস্ত আতযর উপর 10% টযাক্স পেন েতব পসনভং সানটণনফতকতটর সুতের উপর নেনন
পকান টযাক্স পেন না। 2006 সাতল নেনন টযাক্স নহসাতব 7500 টাকা পেন এবং সঞ্চতযর সুে নহসাতব 12000 লাভ কতরন । 2006 সাতল
োর পমাট আয কে?
2. A trader, while selling an item, was asking for such a price that would enable him to offer a 20%
discounts and still make a profit of 30% on cost. If the cost of the item was Tk. 50 what was his asking
price? [BB AD 08, Standard Bank TAO 16, Premier Bank JO 19] Answer: Tk 81.25
Abyev`t GKRb we‡µZv GKwU c‡Y¨i Ggb GKwU `vg e‡jb hv‡Z cY¨wU‡Z 20% wWmKvD›U w`‡jI we‡µZvi 30%
jvf nq| hw` cY¨wUi g~j¨ 50 UvKv nq Zvn‡j we‡µZv KZ `vg e‡jwQ‡jb?
3. A man deposits Tk. 1000 in a bank at 8% interest rate compounded annually. At the end of the 3rd
year, what will be the total amount including interest? Answer: 1259.712
অনুবােঃ একেন মানুে 1000 টাকা একনট বযাতঙ্ক 8% বানেণক ক্রবৃনি সুতের হাতর েমা পেন । ৩য বছর পশ্তে সুেসহ পমাট মুনাফার
পনরমার্ কে হতব?

Bangladesh Commerce Bank


Post name: Junior officer Exam Date : 2008
1. A can do a piece of work in 40 days. He works at it for 8 days and then B finishes it in 16 days. How
long will they take to complete the work if they do it together? Answer: 13.33day
Abyev`t A GKwU KvR 40 w`‡b Ki‡Z cv‡i| †m 8 w`‡b KvR K‡i Ges Gici B KvRwU 16 w`‡b †kl K‡i| Zviv GK‡Î
KvRwU KZw`‡b †kl Ki‡Z cvi‡e?
2. A car gets 20 kms per gallon of gas when it travels at 50 kms per hour. The car gets 12% fewer kms to
the gallon at 60 kms per hour. How far can the car travel at 60 kms per hour on 11 gallons of gas?
Abyev`t GKwU Mvwoi N›Uvq 50 wK.wg †e‡M 20 wK.wg †h‡Z 1 M¨vjb M¨vm `iKvi nq| MvwowU N›Uvq 60 wK.wg †e‡M 1
M¨vjb M¨v‡m Av‡Mi †P‡q 12% Kg `~iZ¡ AwZµg K‡i| N›Uvq 60 wK.wg †e‡M 11 M¨vjb M¨v‡m MvwowU KZ`~i
†h‡Z cvi‡e? Answer: 196.96km
3. A man sells articles at a profit of 20%. If had bought it at 20% less and sold it for Tk. 5 less, he would
have gained 25%. Find the cost price of the article? Answer: Tk 25
Abyev`t GKRb e¨w³ 20% jv‡f GKwU AvwU©‡Kj weµq K‡ib| hw` wZwb 20% K‡g wK‡b Ges 5 UvKv K‡g weµq Ki‡Z
cvi‡Zb Zvn‡j 25% jvf n‡Zv| AvwU©‡KjwUi µqg~j¨ wbY©q Ki|
2𝑚 (2𝑚−1 )𝑚 22𝑚
4. Simplify the following algebraic expression: Answer: 1
2𝑚+1 2𝑚−1 ((2𝑚 )𝑚
5. In the accompanying diagram ABCD is a rectangle. The area of isosceles right triangle ABE = 7, and
EC = 2BE. What would be the area of ABCD? Answer: 42

Written Math Solution (BIBM) FB Group : BD job Info. (BCS & Bank)
48

 Created by Sumon K. Sarkar  FB Group : BD job Info. (BCS & Bank)  Created by Sumon K. Sarkar 
Bank Job Solution (BIBM)
 Created by Sumon K. Sarkar  FB Group : BD job Info. (BCS & Bank)  Created by Sumon K. Sarkar 

6. Arif and Mukul start running at the same time and from the same point around a circle of 216 meters
circumference. If Arif can complete one round in 40 seconds and Mukul in 50 seconds, how many
seconds will it be before they both reach the starting point simultaneously and how many rounds they
will complete by this time? Answer: 4 round
Abyev`t Avwid Ges gyKzj GKmv‡_ GKB ¯’vb †_‡K, GKB mg‡q 216 wgUvi cwiwa wewkó GKwU e„ËvKvi †¶‡Îi Pvwiw`‡K †`Šo
ïiæ K‡ib| Avwid hw` GK ivDÛ 40‡m‡K‡Û †kl K‡ib Zvn‡j gyKzj Zv K‡ib 50 †m‡K‡Û| GKmv‡_ Avevi †`Šo ïiæi ¯’v‡b
G‡m wgwjZ n‡Z Zv‡`i KZ †m‡KÛ mgq jvM‡e? ZZ¶‡Y Zviv cÖ‡Z¨‡K KZ ivDÛ mgvß Ki‡e?

Sonali, Janata, Agrani & Rupali Bank (BRC)


Post name: Senior Officer Exam Date : 2008
1. A manager has a budget of Tk. 60,000 for giving salary increments to 4 full-time and 2 part time
employees. Each of the full-time employees receives the same increment, which is twice the increment
that each of the part-time employees received. What is the amount of the increment that each of the
full time employees received? Answer: Tk. 12,000
অনুবােঃ একেন মযাতনোতরর 4 েন ফুলটাইম এবং 2 েন পাটণটাইম কমণ ারীতের পবেন পেোর েনয বাতেট 60,000 টাকা। ফুল-টাইম
কমণ ারীতের প্রতেযতক একই ইননক্রতমন্ট পায, যা প্রনেনট খণ্ডকালীন কমণ ারীর প্রাপ্ত ইননক্রতমতন্টর নিগুর্। পূর্ণকালীন কমণ ারীতের প্রতেযতক
পয ইননক্রতমন্ট প্রাপ্ত হয োর পনরমার্ কে?
2. A shopkeeper who sells, on an average, 18 pencils and 12 markers per day. The profit from a pencil is
times the profit made by selling a marker. If he makes a profit of Tk. 900 in a month (a month consists of 30 days)
by selling pencils, how much profit does he make per month by selling markers? Answer: Tk 1800
অনুবােঃ একেন নবতক্রো নেতন গতড ১৮নট পপনিল ও ১৫নট মারকার নবক্রয কতর। পস মারকাতর পয লাভ কতর পপনিতল োর গুর্ লাভ
কতর। পস পপনিল নবনক্র কতর প্রনেমাতস ৯০০ টাকা লাভ কতর (? ৩০ নেতন ১ মাস যরতে হতব। )। প্রনেমাতস মারকার নবনক্রয কতর কে টাকা
লাভ কতর?
3. The ratio of gold and silver in an ornament weighing 42 gm is 4:3. How much gold will need to be
added for the ratio of gold and silver to be 5: 3? [ Bank Asia (TO) 2016] Answer:6gm gold
অনুবােঃ 42 গ্রাম ওেতনর একনট অলঙ্কাতর পসানা ও পরৌতপযর অনুপাে 4:3। স্বর্ণ ও রূপার অনুপাে 5:3 হওযার েনয কে পসানা পযাগ
করতে হতব?

Sonali, Janata, Agrani & Rupali Bank (BRC)


Post name: Officer Exam Date : 2008
1. A good has been sold at a loss of 12%. If it could be sold by Tk. 1000 more, there would have been
profit of 8%. What is the cost price of that good? Answer: Tk. 5000
অনুবােঃ একনট পর্য 12% েনেতে নবনক্র করা হতযতছ। যনে আরও ১০০০ টাকা পবনশ্তে নবনক্র করা পযে েতব ৮% লাভ হে। পসই পতর্যর
ক্রেমূলয কে?
2. Two trains of equal length of 500 meters starts moving on two parallel railway tracks at a speed of 104
kilometers per hour and 80 kilometers per hour respectively. How much time the faster train will take
to cross the slower one. Answer: 75 second
অনুবােঃ 500 নমটার সমান দেত ণযর দুনট পট্রন দুনট সমান্তরাল পরলপতর্ যর্াক্রতম 104 নকতলানমটার প্রনে ন্টা এবং 80 নকতলানমটার প্রনে
ন্টা গনেতে লতে শুরু কতর। দ্রুেগামী পট্রননট যীরগনের পট্রননট অনেক্রম করতে কে সময লাগতব?
3. For making a cupboard it requires human labor three times the labor required to make a bench. Six
carpenters can make 36 benches and 5 cupboards in 12 days. How many days will 10 carpenters require
to make 61 beanches and 8 cupboards? Answer: 12 days

Written Math Solution (BIBM) FB Group : BD job Info. (BCS & Bank)
Bank Job Solution (BIBM) 49

Created by Sumon K. Sarkar  FB Group : BD job Info. (BCS & Bank)  Created by Sumon K. Sarkar 
অনুবােঃ একনট পবঞ্চ দেনরর েনয পয মানুতের শ্রতমর প্রতোেন হে, একনট আলমানর দেনরর েনয োর নেনগুর্ মানুতের শ্রতমর প্রতযােন হয।
Created by Sumon K. Sarkar  FB Group : BD job Info. (BCS & Bank)  Created by Sumon K. Sarkar 

ছযেন নমস্ত্রী 12 নেতন 36নট পবঞ্চ এবং 5নট আলমানর দেনর করতে পাতরন। 61নট পবঞ্চ এবং 8নট আলমানর দেনর করতে 10 েন নমস্ত্রীর
কে নেন লাগতব?

Sonali, Janata, Agrani & Rupali Bank (BRC)


Post name: Officer (Cash) Exam Date : 2008
1. A boy covers a distance of 6 km partly by walking and partly by cycling. If he cycles at 18 km per
hour and walks at 6km per hour and takes 35 minutes in all, find the distance he covers by walking.
Abyev`t GKRb evjK 6 wKwg `~i‡Z¡i wKQz Ask †nu‡U Ges wKQz Ask mvB‡Kj Pvwj‡q †M‡jv| hw` mvB‡K‡j N›Uvq 18 wKwg †e‡M
Ges N›Uvq 6 wKwg †e‡M †nu‡U hvq Ges †gvU 35 wgwbU mgq †bq, Zvn‡j †nu‡U AwZµvšÍ `~iZ¡ wbY©q Kiæb| Answer: 2.25 km
2. In an MBA class the ratio of number of commerce graduates to the number of science graduates is 2
to 5. If 2 more commerce graduates enter the class the ratio becomes 1 to 2. How many commerce
graduates are in the class? Answer: 8
অনুবােঃ একনট এমনবএ ক্লাতস নবজ্ঞান স্নােকতের সংখযার সাতর্ বানর্েয স্নােতকর সংখযার অনুপাে 2 পর্তক 5। যনে আরও 2 েন বানর্েয
স্নােক ক্লাতস প্রতবশ্ কতর েতব অনুপাে 1 পর্তক 2 হতব। কেেন বানর্েয স্নােক পশ্রনর্তে রতযতছ?
3. A can dig a pond in 30 days and B can dig the same pond in 20 days. In how many days A and B can
dig the pond if they work together? Answer: 12 day
Abyev`t A GKwU cyKzi 30 w`‡b Ges B GKB cyKzi 20 w`‡b Lbb Ki‡Z cv‡i| Zviv GK‡Î H cyKziwU KZ w`‡b Lbb Ki‡Z
cvi‡e?

Shahjalal Islami Bank Ltd.


Post name: Trainee Senior Officer Exam Date : 2007
1. The sum of square of two numbers is 80 and the square of their difference is 16. Determine the product
of the two numbers. Answer: 32
অনুবােঃ দুনট সংখযার বগণতেতত্রর পযাগফল 80 এবং োতের বতগণর পার্ণকয হল 16। দুনট সংখযার গুর্ফল ননর্ণয কর।
2. A number consists of two digits. The sum of the digits is 15. If 27 is subtracted from the number, its
digits are interchanged. Find the number. Answer:96
Abyev`t GKwU msL¨v `ywU A¼ wb‡q MwVZ| A¼Ø‡qi †hvMdj 15| hw` msL¨vwU †_‡K 27 we‡qvM †`qv nq Zvn‡j A¼Øq ¯’vb
wewbgq K‡i| msL¨vwU wbY©q Kiæb|
3. Masum has twice as much money as Selim and Selim has 50% more money than what Badal has. The
average money with thein is Tk. 110, then determine the amount of Masum's money?
Abyev`t gvmy‡gi Kv‡Q †mwj‡gi Zzjbvq wظY UvKv Av‡Q Ges †mwj‡gi Kv‡Q ev`‡ji Zzjbvq 50% †ewk UvKv Av‡Q| Zv‡`i
cÖ‡Z¨‡Ki M‡o 110 UvKv Av‡Q| gvmy‡gi Kv‡Q KZ UvKv Av‡Q? Answer: Tk180
4. If 2𝑎 + 3𝑏 = 17 𝑎𝑛𝑑 2𝑎+2 − 3𝑏+1 = 5 then determine the values of a and b. Answer: a=3,b=2
5. A and B are two stations 195 km. apart. A train starts from A at 8 a.in, and travels towards B at 60
km/hr. Another train starts from B at 9 a.m. and travels towards A at 75 km/hr. At what time do they
meet? Answer: 10AM
Abyev`t 2wU †÷kb A I B Gi `~iZ¡ 195 wK.wg | GKwU †Uªb †ejv 8 Uvi mgq A †_‡K hvÎv ïiæ K‡i Ges B Gi w`‡K 60
wK.wg / N›Uv †e‡M hvq| Aci GKwU †Uªb B †_‡K †ejv 9 Uvi mgq hvÎv ïiæ K‡i A Gi w`‡K 75 wK.wg/ N›Uv †e‡M †h‡Z _v‡K|
KLb Zviv gy‡LvgyLx wgwjZ n‡e?

Written Math Solution (BIBM) FB Group : BD job Info. (BCS & Bank)
50

 Created by Sumon K. Sarkar  FB Group : BD job Info. (BCS & Bank)  Created by Sumon K. Sarkar 
Bank Job Solution (BIBM)
 Created by Sumon K. Sarkar  FB Group : BD job Info. (BCS & Bank)  Created by Sumon K. Sarkar 

One Bank Ltd.


Post: Probationary Officer Exam Date : 2007
1. Three workers can do a job in 20 days. Two of the workers work twice as fast as the third. How long
would it take one of the faster workers to do the job himself? Answer: 50 days
Abyev`t 3 Rb kªwgK GKwU KvR 20 w`‡b Ki‡Z cv‡i| kªwgK‡`i 2 RbB 3q Rb A‡c¶v wØMyY MwZ‡Z KvR Ki‡Z cv‡i| `ªæZ
MwZ m¤úbœ‡`i 1 R‡bi H KvRwU GKv Ki‡Z KZ mgq jvM‡e?
2. A labor is paid Tk. 8 per hour for an 8 hour day and 1.5 times that rate for each hour in excess of 8
hours in a single day. If the labor received Tk. 80 for a single day's work, how long did he work on
that day? Answer: 9 hours 20 min
Abyev`t GKRb kªwgK w`‡b 8 NÈv KvR Kivi Rb¨ N›Uv cÖwZ 8 UvKv K‡i cvb Ges 8 N›Uvi AwZwi³ KvR Kivi Rb¨ N›Uv cÖwZ
1.5 ¸Y †ewk UvKv cvb| hw` †m GK w`‡b 80 UvKv cvq Zvn‡j H w`‡b †m KZ¶Y KvR K‡i?
3. In the figure below, ABC is a right angle and AB = BD = AD = 20 feet. Find out the length of BC.

Answer: 20√𝟑
4. Mr. X sold two properties Pl and P2 for Tk. 100,000 each. He sold property Pl for 20% loss than what
he paid for it. What is the percentage of profit of property P2 so that he is not in gain or loss on the sale
of the two properties? Answer: 33.33%
Abyev`t Rbve X Zvi m¤úwË P1 Ges P2 Gi cÖwZwU 1,00,000 UvKvq weµq K‡ib| wZwb P1 †K †h `v‡g µq
K‡iwQ‡jb Zvi †P‡q 20% Kg `v‡g weµq K‡ib| P2 †K kZKiv KZ jv‡f weµq Ki‡j †gvU c‡Y¨i Dci jvf
ev ¶wZ †KvbwUB n‡e bv?
9(4𝑥 )2 𝟗
5. Simplify the following algebraic expression. Answer:
16𝑥+2 −2𝑥+1 (8𝑥 ) 𝟐𝟓𝟒
2 3
6. Solve the following equation: + = 1 [One Bank PO 2007, SIBL TO 10] Answer: 2 ±√10
𝑥−2 𝑥+3
7. 200 children came to the park last Sunday. All of the older children rode bicycles into the park and all
of the younger children came on tricycles. 480 wheels rode into the park that day, all of them
functioning on the children's bicycles or tricycle. How many young children came to the park last
Sunday? Answer: 80
Abyev`t MZ iweevi GKwU cv‡K© 200wU wkï ågY Ki‡Z Av‡m| hviv eq‡m eo Zviv mevB evB mvB‡K‡j Ges hviv †QvU Zviv UªvB
mvB‡Kj wb‡q Av‡m| H w`b wkï‡`i evB mvB‡Kj Ges UªvB mvB‡Kj wgwj‡q cv‡K© †gvU 480 wU PvKv wQj|MZ iweevi KZRb
Kgeqmx wkï G‡mwQj?
8. Mr. Karim deposited certain amount of money for a fixed period of time. On maturity he received a
total of Tk. 45000 when the ratio of interest and investinent became 3:6. If the interest rate was 5%
(Simple) per annum, calculate the time period for which the money was invested. Answer: 10 years
[BASIC Bank PO 1999, ONE Bank PO 07]
Abyev`t Kwig mv‡ne GKwU wbw`©ó mg‡qi Rb¨ wKQz UvKv e¨vs‡K Rgv iv‡Lb| H mgq ci wZwb e¨vsK †_‡K †gvU 45,000 UvKv
jvf K‡ib G‡Z gybvdv Ges Avm‡ji AbycvZ nq 3 t 6| hw` gybvdvi nvi 5% nq Zvn‡j mgq wbY©q Ki|
9. A is two years older than B, who is twice as old as C. If the average age of the three is 9 years, how
old is B? Answer: 10 years

Written Math Solution (BIBM) FB Group : BD job Info. (BCS & Bank)
Bank Job Solution (BIBM) 51

Created by Sumon K. Sarkar  FB Group : BD job Info. (BCS & Bank)  Created by Sumon K. Sarkar 
Abyev`t Kwig mv‡ne GKwU wbw`©ó mg‡qi Rb¨ wKQz UvKv e¨vs‡K Rgv iv‡Lb| H mgq ci wZwb e¨vsK †_‡K †gvU 45,000
Created by Sumon K. Sarkar  FB Group : BD job Info. (BCS & Bank)  Created by Sumon K. Sarkar 

UvKv jvf K‡ib G‡Z gybvdv Ges Avm‡ji AbycvZ nq 3 t 6| hw` gybvdvi nvi 5% nq Zvn‡j mgq wbY©q Ki|
10. The average weight of three men A, B and C is 84 kg. Another man D joins the group and the average
now becomes 80 kg. If a fifth man E, whose weight is 3 kg more than that of D, replaces A, then the
average weight of B, C, D and E becomes 79 kg. What is the weight of A? Answer: 10 kg
[AB Bank PO 02, One Bank PO 07, RAKUB SO 14, Al-Arafah MTO 16, Janata Bank EO(Cancelled) 17]
Abyev`t wZb e¨w³ A, B I C Gi Mo IRb 84Kg| Zv‡`i mv‡_ AviI GKRb D †hvM †`Iqvq bZzb Mo nq 80Kg|hw` A
Gi ¯’‡j D Gi †P‡q 3Kg †ewk IR‡bi cÂg e¨w³ E Zv‡`i mv‡_ †hvM †`q, Zvn‡j B, C, D Ges E Gi Mo IRb nq 79Kg|
A Gi IRb KZ?

Bangladesh Bank
Post Name: Assistant Director Exam Date : 2001
1. A total of 50 employees work in a bank branch of these 22 have taken the accounting course, 15 have
taken finance, 14 marketing, 9 of them taken exactly 2 of the courses, 1 of them has taken all. How
many of the 50 employees have taken none of the course? Answer: 10
Abyev`t GKwU e¨vsK Gi kvLvq 50 Rb PvKyix K‡i| Zv‡`i g‡a¨ 22 Rb accounting, 15 Rb finance Ges 14 Rb
marketing ‡Kvm© wb‡qwQj| 9 Rb gvÎ 2 wU K‡i ‡Kvm© wb‡qwQj Ges 1 Rb me KqwU ‡Kvm© wb‡qwQj| 50 Rb Kg©Pvixi g‡a¨
KZRb ‡Kvb ‡Kvm©B ‡bqwb|
2. Mr. X, a sales person earns 5% commission on all sales between Tk.20000 and 40000 and 8% on all
sales exceeding Tk.40000 in a month. He does not earn any commission if sales in a month amount to
less than Tk. 20000. His monthly salary is Tk.60000, he has to pay tax 20% tax on his basic salary, but
no tax on commission, in April 2001, the total net income (salary + commission) of the sales person
was Tk 65000. How much were the sales in April? Answer: Tk. 227500
Abyev`t নমঃ X একেন নবতক্রো ২০,০০০ টাকা পর্তক ৪০,০০০ টাকা পযণন্ত নবক্রতযর েনয ৫% এবং ৪০,০০০ টাকার অনযক নবক্রতযর
েনয ৮% কনমশ্ন প্রনে মাতস পায । পয যনে ২০,০০০ টাকার ননত নবক্রয কতর োহতল পকান কনমশ্ন পায না । োর মানসক পবেন ৬০,০০০
টাকা । োর টযাক্স নেতে হয োর পবেতনর উপর ২০% নকন্তু কনমশ্তনর উপর পকান টযাক্স নেতে হয না । এনপ্রল ২০০১ সাতল োর মূল আয
(পবেন + কনমশ্ন) নেল ৬৫,০০০ টাকা । এনপ্রল মাতস পস কে নবক্রয কতর নেল?
3. A trader sells on an average 18 pencils and 12 pens per day. The profit comes from pencil is 1/3rd of
the profit made from selling a pen. If he makes profits of tk 900 in a month by selling pencils, how much profit
does he make per month by selling pens? The trader sells 30days in a month. Answer: Tk 1800
অনুবােঃ একেন নবতক্রো নেতন গতড ১৮নট পপনিল ও ১৫নট কলম নবক্রয কতর। পস কলতম পয লাভ কতর পপনিতল োর গুর্ লাভ কতর। পস
পপনিল নবনক্র কতর প্রনেমাতস ৯০০ টাকা লাভ কতর। প্রনেমাতস কলম নবনক্রয কতর কে টাকা লাভ কতর? ৩০ নেতন ১ মাস যরতে হতব।
4. Mr. A purchased a house for tk 1000000 tk in 1995, he spent 100000 tk for routine maintenance &
upkeep of the house. In 1999 he sold the house for 25% of more then what he paid for it. He paid 5%
of the proceeds as gain tax & he has to pay 50% of his net profit to the broker, what is his net income?
অনুবােঃ নমঃ A 1995 সাতল 1000000 টাকা নেতয একনট বানড ক্রয করতলন। বানড রের্াতবেতর্র েনয নেনন 100000 টাকা খর
করতলন । 1999 সাতল নেনন োর খরত র 25% লাতভ বানডনট নবক্রে করতলন । পস 5% sales tax নেতলন এবং লাতভর 50% broker
(োলাল) পক নেতলন । োর কাতছ (লাতভর) কে রইল? Answer: Tk. 1,30,625
5. A simple interest rate of a bank was reduced to 5% from 7%. As a consequences Mr. B’s income was
reduced by tk 2100 in 5 years. How much is Mr. B’s initial deposit in the bank? Answer: Tk. 21,000
Abyev`t e¨vs‡Ki mijmy‡` 7% ‡_‡K 5% K‡g ‡Mj| d‡j wgt B Gi Avq e¨vsK ‡_‡K my‡`i Avq 5 eQ‡i 2100 UvKv K‡g ‡Mj|
ïiz‡Z wgt B KZ UvKv e¨vs‡K Rgv ‡i‡LwQj?

Written Math Solution (BIBM) FB Group : BD job Info. (BCS & Bank)
52

 Created by Sumon K. Sarkar  FB Group : BD job Info. (BCS & Bank)  Created by Sumon K. Sarkar 
Bank Job Solution (BIBM)
 Created by Sumon K. Sarkar  FB Group : BD job Info. (BCS & Bank)  Created by Sumon K. Sarkar 

6. One fifth of the products made by a company are defective. Four-fifth of the defectives was rejected
and one-twentieth of the products were rejected by mistake. What percent of the products sold by the
company is defective? Answer: 5%
অনুবােঃ একনট পকাম্পানীর উৎপানেে পতর্যর এক পঞ্চমাংশ্ ত্রুনটপূর্ণ । ঐ ত্রুনটপূর্ণ পতর্যর ার-পঞ্চমাংশ্ বাে যায এবং ভাতলা পতনযর অংশ্
ভুলক্রতম বাে যায। যনে পয সকল পর্য বাে যাযনন ো সবটাই নবনক্র হয েতব ত্রুনটপূর্ণ পর্য কে শ্োংশ্ নবনক্র হয?

Bangladesh Bank
Post Name: Officer (general) Exam Date : 2001
1. A trader sells on an average 18 pencils and 12 pens per day. The profit comes from pencil is 1/3 times
the profit made by selling a pen. If he makes a profit of Tk 900 in a month by selling pencils, how much pr ofit
does he make per month by selling pens? The trader sells 30days in a month. Answer: Tk 1800
অনুবােঃ একেন নবতক্রো নেতন গতড ১৮নট পপনিল ও ১৫নট কলম নবক্রয কতর। পস কলতম পয লাভ কতর পপনিতল োর গুর্ লাভ কতর। পস
পপনিল নবনক্র কতর প্রনেমাতস ৯০০ টাকা লাভ কতর। প্রনেমাতস কলম নবনক্রয কতর কে টাকা লাভ কতর? ৩০ নেতন ১ মাস যরতে হতব।
2. Mr. X pays 10% tax on all income over 60000Tk but he does not pay any tax on interest on postal
saving certificate. In 2000 he paid 7500 as tax & he earned 12000 as interest on postal savings account.
What is his net income in 2000? Answer: Tk 1,39,500
অনুবােঃ নমঃ X 60,000 টাকার উপর আতযর 10% টযাক্স পেন নকন্তু নেনন Saving certificates এর উপর পকাতনা টযাক্স পেন না ।
2,000 সাতল নেনন 7,500 টাকা টযাক্স নেতলন এবং 12,000 টাকা Saving certificate হতে সুে বাবে পপতলন । 2,000 সাতল
োর নীট আে কে?
3. A candidate answered all 22 questions in a test & received 63.5 marks. If the total marks were derived
by adding 3.5 marks each correct answer and deducting 1 mark for each incorrect answer, how many
questions did the student answer incorrectly? Answer: 3
অনুবােঃ একেন পনরোর্ণী 22নট প্রশ্নই উত্তর করল এবং 63.5 নম্বর পপল । প্রনেনট শুি উত্ততরর েনয যনে 3.5 ও ভুল উত্ততরর েনয 1
নম্বর কাটা যায েতব পয কেগুতলা প্রতশ্নর উত্তর ভুল কতরনছল?
4. In an organization 30% of all employees live over 10 miles away from the place of work & 60% of
worker who live who live over 10miles use company transport. If 40% of employees of the company
use company transport, what percent of the employees live 10miles or less from work and use company
transport? Answer: 31(3/7)%
অনুবােঃ পকান একনট প্রনেষ্ঠাতনর 30% কমণেীবী 10 মাইতলর পবনশ্ েূতর র্াতক । 10 মাইতলর পবনশ্ েূতর র্াতক এমন কমণেীবীর 60%
পকাম্পাননর পনরবহর্ বযবহার কতর । যনে পকাম্পানীর পমাট কমণেীবীর 40% পকাম্পানন পনরবহর্ বযবহার কতর োহতল কমণেীবীতের শ্েকরা
কে ভাগ কমণস্থল পর্তক েশ্মাইল বা এর কম েূরতত্ব র্াতক এবং পকাম্পানীর পনরবহর্ বযবহার কতর?
5. Mr. Rich sold two properties P1 & P2 for Tk 50000 each. He sold property P1 for 20% more then what
he paid for it & sold P2 less than 20% what he paid for it. What was his total gain or loss, if any, on
the scale of two properties? Answer: Tk. 4,166.67
অনুবােঃ নমঃ নর P1, ও P2 দুনট সম্পনত্ত 50,000 টাকা কতর নবক্রে করতলন । P1 নেনন োর ক্রযমূতলযর প তয 20% পবনশ্ P2 োর
ক্রযমূতলযর প তয 20% কতম নবক্রে করতলন । োতে োর পমাট কে লাভ/েনে হতলা?
6. Mr. X has a investable amount of Tk 100,000 he will invest the amount for two years. He has two
options. He can invest at simple interest rate of 12% per annum; alternatively he can invest at
compound rate of 10% (compounded semi annually). Calculate the earnings at two option and advice
him accordingly. Answer: Simple interest
অনুবােঃ নমঃ X দুই বছতরর েনয 100000 টাকা 12% হাতর সরল সুতে নকংবা 10% হাতর ক্রবৃনি হাতর খাটাতে পাতরন । দুনট নবকতল্প
আে গর্না করুন এবং পসই অনুযাযী োতক পরামশ্ণ নেন।

Written Math Solution (BIBM) FB Group : BD job Info. (BCS & Bank)
Bank Job Solution (BIBM) 53

Created by Sumon K. Sarkar  FB Group : BD job Info. (BCS & Bank)  Created by Sumon K. Sarkar 
Combined Officer General (2020 year Based)
Created by Sumon K. Sarkar  FB Group : BD job Info. (BCS & Bank)  Created by Sumon K. Sarkar 

Post: officer (General) Exam Date: 31– 03 – 2023


1. A man engaged a worker on the condition that he would pay him Tk. 30000 and one uniform
after one year of service. The worker served only for 9 months and got Tk. 22000 and a uniform.
What is the price of the uniform?
অনুবােঃ এক বছতরর াকনরর পতর এক বযনি এক শ্রনমকতক 30000 টাকা এবং একনট ইউননফমণ প্রোতনর শ্তেণ ননতযাগ পেন। শ্রনমক মাত্র ৯
মাস াকনর কতরতছন এবং 22000 টাকা ও একনট ইউননফমণ পপতযতছন । ইউননফতমণর োম কে?
Solution:
Let, the price of the uniform be Tk. x
So, Total income for 12 months = Tk. (30,000 + x)
30,000+𝑥 90,000+3𝑥 3
∴ Income for 9 months = ( )×9 = = x + 22,500
12 4 4
According to the condition,
3
22,000 + x = x + 22,500
4
3
⇒ x – x = 22,500 – 22,000
4
4𝑥−3𝑥
⇒ = 500
4
∴ x = Tk. 20,000
Answer: Tk. 20,000

2. The cost of 3 horses is same as the cost of 5 cows. If total cost of 4 horses and 6 cows is Tk. 1900,
find the cost of one horse.
অনুবােঃ ৩নট প াডার োম ৫নট গরুর োতমর সমান। 4নট প াডা ও 6নট গরুর পমাট খর হতল 1900 টাকা, একনট প াডার োম কে?
Solution:
Given that, The cost of 5 cows = The cost of 3 horses
3
∴ The cost of 1 cows = The cost of horses
5
3×6 18
So, The cost of 6 cows = The cost of = horses
5 5
Again Given that,
4 horses + 6 cows = 1900
18
⇒ 4 horses + horses = 1900
5
18
⇒ (4 + ) horses = 1900
5
20+18
⇒( ) horses = 1900
5
38
⇒ ( ) horses = 1900
5
38
⇒ 1 horses = 1900 ÷
5
5
⇒ 1 horses = 1900 ×
38
∴ 1 horses = 250
The cost of one horse Tk. 250
Answer: Tk. 250

Written Math Solution (BIBM) FB Group : BD job Info. (BCS & Bank)
54

 Created by Sumon K. Sarkar  FB Group : BD job Info. (BCS & Bank)  Created by Sumon K. Sarkar 
Bank Job Solution (BIBM)
 Created by Sumon K. Sarkar  FB Group : BD job Info. (BCS & Bank)  Created by Sumon K. Sarkar 

Alternative way,
Let, The cost of one cows be Tk. x & the cost of one horses be Tk. y
According to the question,
5x = 3y
⇒ 30x = 18y ------------------------------ (1) ( Both side multiply by 6)
& 6x + 4y = 1900
⇒ 30x + 20y = 1900 × 5 ---------------------(2) ( Both side multiply by 5)
Put the value of 30x in equation (2)
18y + 20y = 1900 × 5
⇒ 38y = 1900 × 5
1900×5
∴y= = 250
38
The cost of one horse Tk. 250
Answer: Tk. 250

3. A man won a lottery of Tk. 2000000. He gave 2/5 of the amount to his wife and 1/4 of the balance
to his son. He donated 2/9 of the balance amount. How much money was donated?
অনুবােঃ এক বযনি 2000000 টাকার একনট লটানর নেতেতছ। নেনন োর স্ত্রীতক 2/5 অংশ্ এবং বানক টাকার ¼ অংশ্ োর পছতলতক নেতযতছন।
নেনন অবনশ্ষ্ট টাকার 2/9 অংশ্ োন কতরতছন। কে টাকা োন করা হতযনছল?
Solution:
Given that, Total lottery amount Tk. 20,00,000
2
Wife got = 20,00,000 × = 8,00,000
5
∴ Remaining balance = 20,00,000 – 8,00,000 = 12,00,000
1
Son got = 12,00,000 × = 3,00,000
4
∴ Remaining balance = 12,00,000 – 3,00,000 = 9,00,000
2
Donated Amount = 9,00,000 × = 2,00,000
9
Answer: Tk. 2,00,000

4. Reena took loan of Tk. 1200 at simple interest rate for as many years as the rate of interest. If
she paid Tk. 432 as interest at the end of the loan period, what is the rate of interest?
অনুবােঃ অনুবােঃ রীনা সহে সুতের হাতর 1200 টাকা ঋর্ পনন পযখাতন ঋতর্র সুতের হার ও ঋতর্র পমোতের সমে একই। যনে নেনন ঋতর্র
পমযাে পশ্তে 432 টাকা সুে প্রোন কতরন, োহতল সুতের হার কে?
Solution:
Given that, Principle ( P )= Tk.1200
Total interest ( I ) = Tk. 432
Rate of interest (r) = r %
According to the question,
Number of year (n) = Rate of interest = r
We know,
Total interest I = Pnr
432 = 1200 × r × r %
𝑟
⇒ 432 = 1200 × r ×
100

Written Math Solution (BIBM) FB Group : BD job Info. (BCS & Bank)
Bank Job Solution (BIBM) 55

Created by Sumon K. Sarkar  FB Group : BD job Info. (BCS & Bank)  Created by Sumon K. Sarkar 
⇒ 432 = 12 × r2
Created by Sumon K. Sarkar  FB Group : BD job Info. (BCS & Bank)  Created by Sumon K. Sarkar 

⇒ 12 × r2 = 432
432
⇒ r2 =
12
⇒ r2 = 36
⇒ r = ±6
∴r=6 ( Here, On the condition, Interest rate would not be negative)
Answer: Interest rate 6%

5. A square and an equilateral triangle have equal perimeters. If the diagonal of the 12√𝟐 cm, then
what is the area of the triangle?
অনুবােঃ একনট বগণতেত্র এবং একনট সমবাহু নত্রভুতের সমান পনরসীমা রতযতছ। কর্ণ 12√2 পসনম হতল, নত্রভুতের পেত্রফল কে?
6. Solution:
Let, one side of a square be x cm x
& one side of a equilateral triangle a cm
The perimeters of a square = 4x cm
We know, the diagonal of square √2𝑥 x x a
According to the question, a
√2𝑥 = 12√2
∴ x = 12
x a
The perimeters of a square = 4x = 4 × 12 = 48 cm
According to the question,
The perimeters of a equilateral triangle = 48 cm
48
So, one side of a equilateral triangle, a = = 16 cm
3
√3 2 √3
∴ The area of the triangle = a = 162 = 64√3 square cm
4 4
Answer: 64√𝟑 square cm

এ রকম গুরুত্বপূর্ণ ফাইল পপতে আমাতের গ্রুতপ প াখ রাখুন


এবং অবশ্যই ননতের বন্ধুতের ইনভাইট করুন

Written Math Solution (BIBM) FB Group : BD job Info. (BCS & Bank)
56

 Created by Sumon K. Sarkar  FB Group : BD job Info. (BCS & Bank)  Created by Sumon K. Sarkar 
Bank Job Solution (BIBM)
 Created by Sumon K. Sarkar  FB Group : BD job Info. (BCS & Bank)  Created by Sumon K. Sarkar 

Combined Senior officer (2020 year Based)


Post: Senior officer Exam Date: 24– 02 – 2023
1. Certain amount of money is divided among A, B, and C in such a way that A gets 3 times as much
as B and B receives 2 times as much as C. If A receives Tk. 1500 more than C, what is the total
amount of money?
অনুবােঃ নননেণষ্ট পনরমার্ অর্ণ A, B, এবং C এর মতযয এমনভাতব ভাগ করা হতযতছ যাতে A ,B এর 3 গুর্ পায এবং B , C এর 2 গুর্ পবনশ্
পায । যনে A, C এর প তয 1500 পবনশ্ টাকা পায, েতব পমাট টাকার পনরমার্ কে?
Solution:
Let, A, B & C receives Tk. a, Tk. b, & Tk. c respectively
A gets 3 times as much as B so, a = 3b,
𝑏 2 𝑏
B receives 2 times as much as C So, b =2c ⇒ = ⇒c=
𝑐 1 2
A receives Tk. 1500 more than C So, a = 1500 + c ------------- (1)
Put the value of a & c in equation (1)
𝑏
3b = 1500 +
2
𝑏
⇒ 3b − = = 1500
2
6𝑏−𝑏
⇒ = 1500
2
5𝑏
⇒ = 1500
2
1500×2
∴ 𝑏= = 600
5
B receives Tk. 600
So, A receives 3 ×Tk. 600 = Tk. 1800
600
C receives = Tk. = Tk. 300
2
Total amount of money = Tk. ( 1800 + 600 + 300) = Tk. 2,700
Answer: Tk. 2700

2. The population of a village is 10,000. In one year, male population increase by 6% and female
population by 4%. If the population at the end of the year is 10520, find the original size of the
male population in that village.
অনুবােঃ একনট গ্রাতমর েনসংখযা 10,000। এক বছতর পুরুে েনসংখযা 6% এবং মনহলা েনসংখযা 4% বৃনি পায। বছতরর পশ্তে েনসংখযা
10,520 হতল, পসই গ্রাতমর পুরুে েনসংখযার কে?
Solution:
Let, the number of male population be ‘x’
& number of female population is (10000 - x)
According to the question,
𝑥 × 6% +(10,000 − 𝑥) × 4 = 10,520 − 10,000
6 4
⇒𝑥 × +(10,000 − 𝑥) × = 520
100 100
6𝑥 (10,000−𝑥)4
⇒ + = 520
100 100
6𝑥+40,000−4𝑥
⇒ = 520
100
⇒ 2𝑥 + 40,000 = 52,000

Written Math Solution (BIBM) FB Group : BD job Info. (BCS & Bank)
Bank Job Solution (BIBM) 57

Created by Sumon K. Sarkar  FB Group : BD job Info. (BCS & Bank)  Created by Sumon K. Sarkar 
⇒ 2x = 52,000 – 40,000
Created by Sumon K. Sarkar  FB Group : BD job Info. (BCS & Bank)  Created by Sumon K. Sarkar 

⇒ 2x = 12,000
∴ x = 6000
The number of male population is 6000
Answer: 6000

3. A bag contains coins of Tk.1, Tk.2, and Tk.5 denomination in the ratio 2:3:4. If the total amount
is Tk.280, find the number of coins of Tk.5 denomination in the bag.
অনুবােঃ একনট বযাতগ 2:3:4 অনুপাতে 1 টাকা, 2 টাকা এবং 5 টাকা মূতলযর কতযন রতযতছ। যনে পমাট 280 টাকা হয, োহতল বযাতগ 5 টাকার
কতযতনর সংখযা পবর করুন।
Solution:
Given that, The ratio of Tk.1, Tk.2, and Tk.5 coins is 2:3:4
Let, Number of Tk.1 coins = 2x.
Number of Tk.2 coins = 3x
& Number of Tk.5 coins = 4x.
So, Value of 2x Tk.1 coins = Tk.1 × 2x = Tk.2x
Value of 3x Tk.2 coins = Tk.2 × 3x = Tk.6x
∴ Value of 4x Tk.5 coins = Tk.5 × 4x = Tk.20x
According to the question,
2x + 6x + 20x = 280
⇒ 28x =280
∴ x = 10
∴ Number of Tk.5 coins = 4x=4×10= 40
Answer: 40

4. 3 years ago, father's age was twice the age of his 4 sons. In 3 years time, the father's will be equal
to the sum of the ages of the sons. Find the present age of the father.
অনুবােঃ 3 বছর পূতবণ, বাবার বযস োর 4 পছতলর বযতসর নিগুর্ নছল। ৩ বছতর নপোর বযস পুত্রতের বযতসর সমনষ্টর সমান হতব। নপোর বেণমান
বযস ননর্ণয করুন।
Solution:
Let, the present age of the father be 'x'
and the sum of the present ages of four sons be 'y'
3 years ago, Age of the father = x-3
3 years ago, Sum of ages of four sons = y -3×4 = y - 12
According to the 1st condition,
x-3 = 2(y-12)
⇒ x -3 = 2y -24
⇒ x = 2y -24 +3
⇒ x = 2y - 21-------------------(1)
After 3 years, Age of father = x+3
Sum of ages of four sons = y + 3×4 = y+12
According to the 2nd condition,
x+3 = y+12
⇒ x = y+12-3

Written Math Solution (BIBM) FB Group : BD job Info. (BCS & Bank)
58

 Created by Sumon K. Sarkar  FB Group : BD job Info. (BCS & Bank)  Created by Sumon K. Sarkar 
Bank Job Solution (BIBM)
 Created by Sumon K. Sarkar  FB Group : BD job Info. (BCS & Bank)  Created by Sumon K. Sarkar 

⇒x = y+9 ---------------------(2)
Substituting the value of x in equation (1) we get,
y+9 = 2y -21
⇒ y = 21+9
⇒ y = 30
Put the value of y in equation (2)
x = y+9
⇒ x = 30 +9
⇒ x= 39
The present age of the father = 39 years
Answer: 39 years

5. If the perimeter of an isosceles right triangle is (6+3√2) meter, then what is the area of the
triangle?
অনুবােঃ একনট সমনিবাহু সমতকার্ী নত্রভুতের পনরনয (6+3√2) নমটার হতল, নত্রভুতের পেত্রফল কে?
Solution:
Let, the equal sides of an isosceles right triangle be a.
In an isosceles right angled triangle, the two sides on the right angle are equal.
Hence, hypotenuse of the isosceles right angled triangle = √𝑎2 + 𝑎2 = √2𝑎2 = √2a
Given, perimeter of the isosceles right triangle = (6+3√2) meter
A
According to the question,
⇒ a+a+√2a =(6+3√2)

a =3 meter
⇒ 2a+√2a =3(2+√2)
⇒ a(2+√2 )= 3(2+√2)
⇒ a=3
In the isosceles right triangle, the base and height, a=3 meter C
1 1 B a=3 meter
Hence, area of the triangle = ×base×height = ×3×3=4.5 square meter
2 2
Answer: 4.5 square meter

এ রকম গুরুত্বপূর্ণ ফাইল পপতে আমাতের গ্রুতপ প াখ রাখুন


এবং অবশ্যই ননতের বন্ধুতের ইনভাইট করুন

Written Math Solution (BIBM) FB Group : BD job Info. (BCS & Bank)
Bank Job Solution (BIBM) 59

Created by Sumon K. Sarkar  FB Group : BD job Info. (BCS & Bank)  Created by Sumon K. Sarkar 
Bangladesh Bank
Created by Sumon K. Sarkar  FB Group : BD job Info. (BCS & Bank)  Created by Sumon K. Sarkar 

Post: Assistant Maintenance Engineer Exam Date: 04– 02 – 2023


𝟒(√𝟔+√𝟐) 𝟐+√𝟑
4. Solve the problem: −
(√𝟔−√𝟐) 𝟐−√𝟑
Solution:
4(√6+√2) 2+√3
Given that, −
(√6−√2) 2−√3

4(√6+√2)(√6+√2) (2+√3)(2+√3)
⇒ −
(√6−√2)(√6+√2) (2−√3)(2+√3)
2
4(√6+√2) (2+√3)2
⇒ −
(√6)2 −(√2)2 (2)2 −(√3)2

4{(√6)2 +2√6.√2+(√2)2 } (2)2 +2.2.√3+(√3)2


⇒ −
6−2 4−3
4(6+2√6.√2+2) 4+4√3+3
⇒ −
4 1

⇒ 8 + 2√12 - (7 + 4√3)
⇒ 8 + 4√3 - 7 - 4√3
⇒1
Answer: 1

5. A father divides his property between his two sons A and B. A invests the amount at compound
interest of 8% p.a. B invests the amount at 10% p.a simple interest. At the end of 2 years, the
interest received by B is Taka 1336 more than the interest received by A. Find A 's share in the
father's father's property of Taka 25,000.
[Midland Bank MTO 15, SIBL PO 13, Bangladesh Bank-AME-2023]
Abyev`t †Kvb wcZv Zvi m¤úwË Zvi `yB cyÎ A I B Gi g‡a¨ fvM K‡i †`q| A Zvi UvKv 8% Pµe„w× gybvdvq Ges B Zvi UvKv
10% mij gybvdvq wewb‡qvM K‡i| 2 eQi c‡i A Gi †P‡q B 1,336 UvKv †ewk gybvdv AR©b K‡i| Df‡qi UvKvi cwigvb wbY©q
Ki| Zv‡`i wcZvi Kv‡Q 25,000 UvKv wQj|
Solution:
Let, Share of A be x Tk
& Share of B be (25000-x) Tk
A's profit = 𝑃(1 + 𝑟)2 − 𝑃
= 𝑥(1 + 8%)2 − 𝑥
8 2
= 𝑥 (1 + ) −𝑥
100
100+8 2 108 2
=𝑥( ) −𝑥 =𝑥( ) − 𝑥= 1.1664x – x = 0.1664x
100 100
B's profit = (25000-x) ×2×10%
10
=(25000-x) ×2 ×
100
1
=(25000-x) × = 5000 – 0.2x
5
According to the question,
5000 – 0.2x - 0.1664x =1336

Written Math Solution (BIBM) FB Group : BD job Info. (BCS & Bank)
60

 Created by Sumon K. Sarkar  FB Group : BD job Info. (BCS & Bank)  Created by Sumon K. Sarkar 
Bank Job Solution (BIBM)
 Created by Sumon K. Sarkar  FB Group : BD job Info. (BCS & Bank)  Created by Sumon K. Sarkar 

⇒ 0.2x+0.1664x = 5000 – 1336


⇒ 0.3664x = 3664
⇒ x =10000
So, Share of A=10,000 Tk and share of B =25,000 - 10,000 = 15,000 Tk
Answer: 10,000 Tk and 15,000 Tk

6. The percentage profit earned by selling an article for Taka 1920 is equal to the percentage loss
incurred by selling the same article for Taka 1280. At what price should the article be sold to
make a 25% profit?
[South East Bank MTO 2013, BB AD(freedom fighter) 2015, SJIB MTO 2013, IBBL PO 2019, Sadharan Bima JO 2019, Sadharan Bima
AM 2019, BB AD -AME - 2023]
Abyev`t GKwU cY¨ 1,920 UvKvq weµq Ki‡j kZKiv hZ jvf nq, cY¨wU 1,280 UvKvq weµq Ki‡j kZKiv ZZ UvKv ¶wZ
nq| 25% jvf Ki‡Z n‡j cY¨wU‡K KZ `v‡g weµq Ki‡Z n‡e?
Solution:
Let, cost price an article be Tk. x
According to the question,
1920-x = x-1280 [ Selling Price – Cost price ]
⇒ 1920+1280 = x+ x
⇒ 2x = 3200
∴ x = 1600
Cost price an article is Tk. 1600
At 25% profit, new selling price an article = Tk. (1600+25% of 1600) = Tk. 2000
Answer: Tk. 2000
Alternative Way,
Let, Cost Price be Tk. x
Then, According to the question,
1920−𝑥 x−1280
×100 = ×100
𝑥 𝑥
⇒ 1920 - x = x - 1280
⇒2x = 3200 ⇒ x = 1600
Cost Price be Tk. 1600
125
∴ Required Selling Price = 125% of Tk. 1600 = × 1600 = Tk. 2000
100
Answer: Tk. 2000

7. AD is the longest side of the right triangle ABD shown in the figure. What is the length of the
longest side of ∆ABC? [ Social Islami Bank Ltd. (PO) 2013 ]
Solution: A
If AD to be the longest side of right triangle, angle B
must be 90º because the side opposite to the biggest 4
angle is biggest i.e.
AD = Hypotenuse of triangle ABC =√42 + 52 =√41
B 5 C D
Answer: √𝟒𝟏

Written Math Solution (BIBM) FB Group : BD job Info. (BCS & Bank)
Bank Job Solution (BIBM) 61

Created by Sumon K. Sarkar  FB Group : BD job Info. (BCS & Bank)  Created by Sumon K. Sarkar 
Bangladesh Bank
Created by Sumon K. Sarkar  FB Group : BD job Info. (BCS & Bank)  Created by Sumon K. Sarkar 

Post: Assistant Programmar Exam Date: 03– 02 – 2023


𝟏 𝟏𝟕 𝟏
1. If x is an integer and 𝒙 + = , then value of 𝒙 − =?
𝒙 𝟒 𝒙
Solution:
Given that,
1 17
𝒙+ =
𝑥 4
1 2 17 2
⇒(𝑥 + ) = ( )
𝑥 4
1 2 1 289
⇒(𝑥 − ) + 4. 𝑥. =
𝑥 𝑥 16
1 2 289
⇒(𝑥 − ) + 4 =
𝑥 16
1 2 289
⇒(𝑥 − ) = −4
𝑥 16
1 2 289−64
⇒(𝑥 − ) =
𝑥 16
1 2 225
⇒(𝑥 − ) =
𝑥 16
1 225
⇒𝑥+ = √
𝑥 16
1 15
⇒𝑥+ =
𝑥 4
𝟏𝟓
𝐀𝐧𝐬𝐰𝐞𝐫:
𝟒

𝟐
2. A basket ball team has won 15 games and lost 9. If these games represent 16 percent of the
𝟑
games to be played, then how many more games must the team win to average 75 percent for the
season?
[ONE Bank PO 10, NCC Bank MTO 11, Premier Bank MTO 12, Midland Bank TO 15, Bangladesh bank AP 2023]
Abyev`t GKwU ev‡¯‹Uej wUg 15wU †Ljvq Rqjvf K‡i Ges 9wU †Ljvq civwRZ nq| hw` GB †Ljv¸‡jv Øviv Zv‡`i †gvU ‡Ljvi
𝟐
16 % †Ljv‡K †evSv‡bv nq Zvn‡j H wmR‡b M‡o 75% †Ljvq †RZvi Rb¨ Zv‡`i‡K KZwU †Ljvq wRZ‡Z n‡e?
𝟑
Solution:
Total game that have already been played=15+9=24.
According to the question,
2 50
16 = % = 24
3 3
24×100
Or,100% = =144.
50/3
24×100×3
Or,100% = =144.
50
To win 75% of the game the team need to win = 144×75% = 108.
So it needs to win (108-15) = 93 more games.
Answer: 93 more games

Written Math Solution (BIBM) FB Group : BD job Info. (BCS & Bank)
62

 Created by Sumon K. Sarkar  FB Group : BD job Info. (BCS & Bank)  Created by Sumon K. Sarkar 
Bank Job Solution (BIBM)
 Created by Sumon K. Sarkar  FB Group : BD job Info. (BCS & Bank)  Created by Sumon K. Sarkar 

3. Students of a class are made to stand in rows. If 4 students are extra in each row, then there
would be 2 rows less. If four students are less in each row, then there would be 4 more rows.
What is the number of students in the class?
অনুবােঃ একনট ক্লাতসর নশ্োর্ণীতের সানরবিভাতব োাঁড করাতনা হয। প্রনে সানরতে ৪ েন নশ্োর্ণী অনেনরি হতল ২ সানর কম হতব। যনে
প্রনে সানরতে ারেন নশ্োর্ণী কম র্াতক, োহতল আরও 4নট সানর হতব। ক্লাতস নশ্োর্ণীর সংখযা কে?
Solution:
Let, the number of students in each row =x & the number of rows =y.
So, the number of students =xy.
Then, by the first condition,
(x+4)(y−2)=xy
⟹−2x+4y−8+xy=xy
⟹−2x+4y=8 .......(i)
Also by the second condition,
(x−4)(y+4)=xy
⟹4x−4y−16+xy=xy
⟹4x−4y=16 .......(ii)
Adding the equation (i) & (ii)
⟹ 2x=24
⟹x=12
Put the value of x in eqution (i)
−2×12+4y=8
⟹ −24+4y=8
⟹ 4y=8+24
⟹ 4y=32
y=8
So, number of students in class, xy = 12 × 8 = 96
Answer: 96

4. In the given figure, PQT is an right triangle, then what is the area of square QRST.
Solution:
Here, PQT is an right angel triangle & ∠QPT=900
∆ PQT QT2 = PQ2 + PT2 R
QT2=52+52
QT2 = 50

Q S
2
The area of square QRST= QT = 50

5
Answer: 50
P 5 T

Written Math Solution (BIBM) FB Group : BD job Info. (BCS & Bank)
Bank Job Solution (BIBM) 63

Created by Sumon K. Sarkar  FB Group : BD job Info. (BCS & Bank)  Created by Sumon K. Sarkar 
Bangladesh Bank
Created by Sumon K. Sarkar  FB Group : BD job Info. (BCS & Bank)  Created by Sumon K. Sarkar 

Post: Assistant Director (Research) Exam Date: 03– 02 – 2023

1. Monthly incomes of two persons are in the ratio of 5: 4 and their monthly expenditures are in
the ratio of 9:7. If each person saves Tk. 500 per month, then what are their monthly incomes?
অনুবােঃ দুই বযনির মানসক আতের অনুপাে 5:4 এবং োতের মানসক বযতের অনুপাে 9:7। যনে প্রতেযক বযনি মাতস ৫০০ টাকা সঞ্চে
কতর, োহতল োতের মানসক আয কে?
Solution:
Let, their monthly income be 5x,4x & monthly expenditure be 9y,7y respectively.
5x−9y=500 ……………….(1)
4x−7y=500 .………………(2)
From (1) and (2)
5x−9y = 4x−7y
⇒ 5x – 4x = 9y – 7y
∴ x = 2y ……………………(3)
put the value of ‘x’ in equation (1),
⇒ 5×2y − 9y = 500
⇒ 10y − 9y = 500
∴ y = 500
put the value of ‘y’ in equation (3),
⇒ x = 2 × 500
∴ x = 1000
Their monthly income is 5000 and 4000 respectively.
Answer: 5000 and 4000 respectively.

2. One man wanted to fence one side of his garden. If he uses posts 6 feet apart, the ends and start
of the side ends with posts. If he buys certain no of posts from market they are 5 less than
required. If the post are 8 feet apart the no of posts brought satisfied the requirements. What is
the length of the garden? How many posts were brought?
অনুবােঃ একেন পলাক একনট বাগাতনর একপাতশ্ পবড়া লাগাতে ান। নেনন বাগাতনর ৬ ফুট অন্তর এক প্রান্ত পর্তক পশ্ে প্রান্ত পযণন্ত পপাস্ট
লাগাতবন। নেনন যেগুতলা পপাস্ট নকতনতছন ো প্রতোেতনর প তে ৫ নট পপাস্ট কম হে। নকন্তু যনে ৮ ফুট অন্তর পপাস্ট লাগান েতব পপাস্ট সংখযা
কম-পবনশ্ নকছুই হে না। বাগাতনর দে ণয কে ফুট? কেগুতলা পপাস্ট ক্রে করা হতেনছল?
Solution:
Let, Number of total posts were bought ‘x’
If he uses posts 6 feet apart, then total lenth be 6(x – 1) feet
If he uses posts 8 feet apart, then total lenth be 8(x – 5 – 1) = 8(x – 6) feet
According to the condition,
8(x – 6) = 6(x – 1)
⇒ 8(x – 6) = 6x – 6
⇒ 8x – 48 = 6x – 6
⇒ 8x – 6x = 48 – 6
⇒ 2x = 42

Written Math Solution (BIBM) FB Group : BD job Info. (BCS & Bank)
64

 Created by Sumon K. Sarkar  FB Group : BD job Info. (BCS & Bank)  Created by Sumon K. Sarkar 
Bank Job Solution (BIBM)
 Created by Sumon K. Sarkar  FB Group : BD job Info. (BCS & Bank)  Created by Sumon K. Sarkar 

∴ x = 21
So, Number of total posts were bought = 21 - 5 = 16
And length of the garden = 6(21 − 1) = 120 feet
Answer: Length 120 feet and Posts bought 16

3. An executive goes on a business trip. His daily schedule has a definite pattern. If he is busy with
a meeting in the morning, he is free in the afternoon. When he returns, he realises that he
attended 15 meetings altogether. There were 12 free mornings and 13 free afternoons. What was
the duration of his trip?
অনুবােঃ একেন ননবণাহী একনট বযবসানযক সফতর যায। োর প্রনেনেতনর সমযসূ ীর একনট নননেণষ্ট পযাটানণ রতযতছ। যনে পস সকাতল নমনটং
ননতয বযস্ত র্াতক েতব পস নবতকতল নি র্াতকন। নেনন যখন নফতর আতসন, েখন নেনন বুঝতে পাতরন পয নেনন পমাট 15নট নমনটংতয অংশ্
ননতযতছন। 12নট মুি সকাল এবং 13নট মুি নবকাল নছল। োর সফতরর সমযকাল কে নছল?
Solution:
Let, number of meeting at morning be x
number of meeting at afternoons be y
According to the question,
x + y = 15 --------------- (1)
Total morning in the trip = x+12
Total Afternoon in the trip = y+13
According to the Condition,
x+12 = y +13
⇒ x – y = 13 – 12
⇒ x – y = 1--------------- (2)
Adding the equation (1) & (2)
x + y + x – y = 15 + 1
⇒ 2x = 16
⇒x=8
The duration of his trip = (8 + 12) = 20 days
Answer: 20 days

4. A team of 2 men and 5 women complete 1/4th of a job in 3 days. After that another man joins
them and they all complete the next 1/4th of the job in 2 days. How many men can complete the
whole job in 4 days?
[BB AD 14, PKSF AM 09, NCC Bank Officer 02, One Bank SCO 18, NRB Bank MTO 19]
Abyev`t 2 Rb cyiæl Ges 5 Rb gwnjvi GKwU `j 3 w`‡b GKwU Kv‡Ri 1/4 Ask m¤c~Y© K‡i| 3w`b ci GKRb AwZwi³ cyiæl
Kv‡R †hvM w`‡j Zviv 2 w`‡b AviI 1/4 Ask m¤cbœ K‡ib| KZRb cyiæl wg‡j evwK KvR 4 w`‡b m¤cbœ Ki‡Z cvi‡eb?
Solution:
Let, Man and Women be m & w
Work rate of Man and Women,
2 5 1
+ =
𝑚 𝑤 4×3
2 5 1
⇒ + = ....................... (i)
𝑚 𝑤 12
3 5 1
+ =
𝑚 𝑤 4×2

Written Math Solution (BIBM) FB Group : BD job Info. (BCS & Bank)
Bank Job Solution (BIBM) 65

Created by Sumon K. Sarkar  FB Group : BD job Info. (BCS & Bank)  Created by Sumon K. Sarkar 
3 5 1
⇒ + = ............................... (ii)
𝑚 𝑤 8
Created by Sumon K. Sarkar  FB Group : BD job Info. (BCS & Bank)  Created by Sumon K. Sarkar 

Subtract equation (i) from equation (ii)


3 2 1 1
⇒ − = −
𝑚 𝑚 8 12
1 3−2
⇒ =
𝑚 24
⇒ m = 24
It means one man needs 24 days to complete the work so work completed in 4 days ,
24
we need = 6 men.
4
Answer: 6 men.

Alternative Way,
A team of 2 men and 5 women can complete a job in 4 × 3 = 12 days
A team of 2 + 1 = 3 men and 5 women can complete the whole thing in 4 × 2 = 8 days
A man can do the same amount of work in 8 days as a team of 2 men and 5 women can in 12 - 8 = 4
days
12
Therefore, a man can complete a job (solo) in 8 × = 24 days
4
24
A job can be completed within 4 days by = 6 men.
4
Answer: 6 men.

Bangladesh Bank
Post: Officer (Electrical) Exam Date: 13– 01 – 2023

4. Taxi fare is described by the following relationship:


Total taxi Fare = Tk. A upto 2 km+Tk. B per km (exceeding 2 km)+Tk.60 for per hour waiting
A person paid Tk. 432 for running 52 km and 2 hours of waiting charge. The same person paid
Tk.732 for running 102 km and 2 hours of waiting charge. Find the value of 'A' and 'B'.
[Shahjalal Islami Bank Ltd. (MTO) 2010 ]
Abyev`t wb‡Pi m¤c‡K©i mvnvh¨ †Uw· fvov †`Lv‡bv hvq:
†gvU †Uw· fvov = ¯’vqx PvR© A UvKv 2wKwg ch©šÍ + B UvKv 2 wKwg c‡i +60 UvKv cÖwZ N›Uv A‡c¶v PvR©|
GKRb e¨w³ 52 wKwg Ges 2 N›Uv A‡c¶vi Rb¨ 432 UvKv w`j| GKB e¨w³ 102 wKwg I 2 N›Uv A‡c¶vi Rb¨ 732 UvKv cÖ`vb
K‡i| A Ges B wbb©q Ki|
Solution:
Given that,
Total taxi Fare = Tk. A upto 2 km+Tk. B per km (exceeding 2 km)+Tk.60 for per hour waiting
According to the 1st condition,
A + (52 – 2 )B + 2× 60 = 432 Let, Total distance d km & time t hours
⇒ A + 50B + 120 = 432 Total fare = A + (d – 2 )B + t × 60
⇒ A + 50B = 432 – 120
⇒ A + 50B = 312 --------------------------(1)
According to the 2nd condition,
A + (102 – 2 )B + 2× 60 = 732

Written Math Solution (BIBM) FB Group : BD job Info. (BCS & Bank)
66

 Created by Sumon K. Sarkar  FB Group : BD job Info. (BCS & Bank)  Created by Sumon K. Sarkar 
Bank Job Solution (BIBM)
 Created by Sumon K. Sarkar  FB Group : BD job Info. (BCS & Bank)  Created by Sumon K. Sarkar 

⇒ A + 100B + 120 = 732


⇒ A + 100B = 732 – 120
⇒ A + 100B = 612 ------------------------(2)
Subtract equation (1) from (2)
A + 100B – A – 50B = 612 – 312
⇒ 50B = 300
⇒B=6
Put the value of B in Equation (1)
⇒ A + 50 × 6 = 312
⇒ A + 300 = 312
⇒ A = 312 – 300
⇒ A = 12
Answer: A = 12 & B = 6

5. Three friends invest equal shares in a business. After careful calculation, they realized that each
of them would have invested Tk. 3,000 less if they had found 2 more friends with equal shares.
What is the amount of total investment in the business?
অনুবােঃ নেন বন্ধু একনট বযবসায সমান পশ্যার নবননতযাগ কতর। সেকণোর সনহে নহসাব করার পর োরা বুঝতে পারতলন পয, োতের প্রতেযতক
৫০ হাোর টাকা নবননতযাগ কতরতছ। 3,000 কম যনে োরা সমান পশ্যার সহ আরও 2 বন্ধু খুাঁতে পপে। বযবসায পমাট নবননতযাতগর পনরমার্
কে?
Solution:
Let, each of patner invested Tk. x
So, Total invested Tk ( x + x + x ) = Tk. 3x
Each of them invested less Tk. 3,000 , then the investment of each person Tk. (x – 3,000)
According to the question,
5(x – 3,000) = 3x
⇒ 5x – 15,000 = 3x
⇒ 5x – 3x = 15,000
⇒ 2x = 15,000
⇒ x = 7,500
∴ 3x = 22,500
So, Total invested Tk. 22,500
Answer: Tk. 22,500

এ রকম গুরুত্বপূর্ণ ফাইল পপতে আমাতের গ্রুতপ প াখ রাখুন


এবং অবশ্যই ননতের বন্ধুতের ইনভাইট করুন

Written Math Solution (BIBM) FB Group : BD job Info. (BCS & Bank)
Bank Job Solution (BIBM) 67

Created by Sumon K. Sarkar  FB Group : BD job Info. (BCS & Bank)  Created by Sumon K. Sarkar 
6. A trader bought some mangoes for Tk. 150 per dozen and an equal number of apples for Tk.100
Created by Sumon K. Sarkar  FB Group : BD job Info. (BCS & Bank)  Created by Sumon K. Sarkar 

per dozen. If he sells all the fruits for Tk. 140 per dozen, what would be his profit/loss in
percentage?
[Same as DBBl PO 12, BASIC Bank PO 99, RAKUB SO 14, SJIB MTO 10]
Abyev`t GKRb e¨emvqx cÖwZ WRb 150 UvKv `‡i wKQz Avg Ges cÖwZ WRb 100 UvKv `‡i mgvb msL¨K Av‡cj µq K‡ib| hw`
†m me¸‡jv dj 140 UvKv WRb wn‡m‡e weµq K‡i Zvn‡j Zvi kZKiv KZ jvf ev ¶wZ n‡e?
Solution:
Given that, Cost Price of per dozen mangoes Tk. 150
Cost Price of per dozen apples Tk. 100
Total Cost Price of two dozen mangoes & apples = Tk. (150 +100) = Tk. 250
Selling Price of two dozen mangoes & apples = Tk. (140×2) = Tk. 280
Profit = Tk. (280 – 250) = Tk. 30
30
Percentage of Profit = ( × 100)% = 12%
250
Answer: 12%

7. The figure shows a circular flowerbed, with its center at O having radius of 8 feet. The flowerbed
is surrounded by a circular path, which is 3 feet wide. What is the area of the path in square
feet?
অনুবােঃ ন ত্রনট একনট বৃত্তাকার ফুতলর নবছানা পেখায, যার পকন্দ্র O-পে রতযতছ 8 ফুট বযাসাযণ। ফুতলর নবছানা একনট বৃত্তাকার পর্ িারা
পবনষ্টে, যা 3 ফুট ওডা। পতর্র পেত্রফল কে বগণফুট?
Solution:
Here, The radius of the smaller circle r = 8 feet
thus its area of the smaller circle 𝜋𝑟 2 = 𝜋82 = 64𝜋 square feet
The radius of the bigger circle R = 8 + 3 = 11 feet
thus its area of the bigger circle 𝜋𝑅2 = 𝜋112 = 121𝜋 square feet
The area of the path = 121π − 64π = 57π square feet
Answer: 57π

এ রকম গুরুত্বপূর্ণ ফাইল পপতে আমাতের গ্রুতপ প াখ রাখুন


এবং অবশ্যই ননতের বন্ধুতের ইনভাইট করুন

Written Math Solution (BIBM) FB Group : BD job Info. (BCS & Bank)
68

 Created by Sumon K. Sarkar  FB Group : BD job Info. (BCS & Bank)  Created by Sumon K. Sarkar 
Bank Job Solution (BIBM)
 Created by Sumon K. Sarkar  FB Group : BD job Info. (BCS & Bank)  Created by Sumon K. Sarkar 

Bangladesh Bank
Post Name: Officer Exam Date: 18– 11 – 2022

1. A labour is paid Tk 100 per hour for working 8 hours in a day as regular pay. His payment is
1½ times than the rate of regular pay in excess of 8 hours in a single day as overtime. It the labour
received a total of Tk 1000 for a single day, how long did he work on that day?
Same As [ One Bank Ltd. Probationary Officer Exam Date : 2010]
অনুবােঃ ননযনমে পবেন নহতসতব নেতন ৮ ণ্টা কাে করার েনয একেন শ্রনমকতক ণ্টায ১০০ টাকা পেওযা হয। এক নেতন ৮ ন্টার পবনশ্
কাে করতল োর ওভারটাইম পপতমন্ট ননযনমে পবেতনর হাতরর প তয ১.৫ গুর্। এই শ্রনমক এক নেতনর েনয সবণতমাট ১০০০ টাকা পপল, পস
নেতন কেনেন কাে করল?
Solution:
Let, Total working x hours
Overtime Salary Rate per hour = 100 × 1½
= 150
According to the question,
⇒ 100 × 8 + 150(𝑥 − 8) = 1000
⇒ 800 + 150x – 1200 = 1000
⇒ 150x – 400 = 1000
⇒ 150x = 1400
1400
⇒x=
150
28 1
⇒x= = 9 ℎ𝑜𝑢𝑟𝑠
3 3
1
⇒ x = 9 ℎ𝑜𝑢𝑟𝑠 (60 × ) 𝑚𝑖𝑛𝑢𝑡𝑖𝑒𝑠
3
⇒ x = 9 ℎ𝑜𝑢𝑟𝑠 20 𝑚𝑖𝑛𝑢𝑡𝑖𝑒𝑠
Answer: 𝟗 𝒉𝒐𝒖𝒓𝒔 𝟐𝟎 𝒎𝒊𝒏𝒖𝒕𝒊𝒆𝒔

2. In a certain office, 1/3 of workers are women, 1/2 of the women are married and 1/3 of the
married women have children. If 3/4 of the men are married and 2/3 of the married men have
children, what part of the workers are without children?
অনুবােঃ একনট অনফতস, 1/3 অংশ্ কমণী মনহলা, 1/2 অংশ্ মনহলা নববানহে এবং 1/3 অংশ্ নববানহে মনহলাতের সন্তান রতযতছ। যনে 3/4
অংশ্ পুরুে নববানহে এবং 2/3 অংশ্ নববানহে পুরুতের সন্তান র্াতক, োহতল শ্রনমকতের পকান অংশ্ সন্তানহীন?
Solution: 1
Let, Total number of workers x
1 𝑥
Number of women = 𝑥 × =
3 3
𝑥 3𝑥−𝑥 2𝑥
Number of men = 𝑥 − = =
3 3 3
𝑥 1 𝑥
Number of Married women × =
3 2 6
𝑥 1 𝑥
Number of Married women who have children = × =
6 3 18
2𝑥 3 𝑥
Number of Married men = × =
3 4 2
𝑥 2 𝑥
Number of Married men who have children = × =
2 3 3

Written Math Solution (BIBM) FB Group : BD job Info. (BCS & Bank)
Bank Job Solution (BIBM) 69

Created by Sumon K. Sarkar  FB Group : BD job Info. (BCS & Bank)  Created by Sumon K. Sarkar 
𝑥 𝑥 𝑥+6𝑥 7𝑥
Number of workers are without children = + = =
18 3 18 18
Created by Sumon K. Sarkar  FB Group : BD job Info. (BCS & Bank)  Created by Sumon K. Sarkar 

7𝑥 18𝑥−7𝑥 11𝑥
Number of workers are without children = 𝑥 − = =
18 18 18
11𝑥
18 11𝑥 11𝑥 1 11
Part of the workers are without children= = ÷𝑥 = × =
𝑥 18 18 𝑥 18
𝟏𝟏
Answer:
𝟏𝟖

3. Mr. A deposited a certain amount of money for a fixed period of time. On maturity, he received
a total of Tk. 60,000 and the ratio of interest and investment was 3:6. If the interest rate was
6.25% (simple). calculate the time period for which the money was invested?
Same as [BASIC Bank PO 1999, ONE Bank PO 07]
Abyev`t Mr. A GKwU wbw`©ó mg‡qi Rb¨ wKQz UvKv e¨vs‡K Rgv iv‡Lb| H mgq ci wZwb e¨vsK †_‡K †gvU 60,000 UvKv jvf
K‡ib G‡Z gybvdv Ges Avm‡ji AbycvZ nq 3 t 6| hw` gybvdvi nvi 6.25% nq Zvn‡j mgq wbY©q Ki|
Solution:
Given that, the ratio of interest and investment was 3:6
Let, the ratio of interest (I) and investment (P) 3x & 6x
6.25 625 1
Here, interest rate, r = 6.25% = = =
100 100×100 16
Number the time period n
We know, I = Pnr
𝐼
𝑜𝑟, 𝑛 =
𝑃𝑟
3𝑥
𝑜𝑟, 𝑛 = 1
6𝑥×
16
3𝑥 ×16
𝑜𝑟, 𝑛 = =8
6𝑥
Number the time 8
Answer:8

4. 200 children came to the park last Sunday. All of the older children rode bicycles into the park
and all of the younger children came on tricycles. 480 Wheels rode into the park that day, all of
them functioning on the children's bicycles of tricycles. How many younger children came to the
park last Sunday? [ One Bank Ltd. Probationary Officer Exam Date : 2010]
অনুবােঃ এক অনফতস ১/৩ অংশ্ নারী কমণী , ১/২ অংশ্ নারী নববানহে এবং ১/৩ অংশ্ নববানহে নারীর সন্তান আতছ। যনে ৩/৪ অংশ্ পুরুে
নববানহে হে এবং ২/৩ অংশ্ নববানহে পুরুতের সন্তান র্াতক েতব কে অংশ্ কমণীর সন্তান নছল না?
Solution:
Let, the number of younger children to the park last Sunday who rode tricycle be x
So, the number of older children to the park last Sunday who rode bicycle = 200-x
According to the question,
3x+2(200-x)=480
⇒ 3x+400-2x = 480
⇒ 3x-2x =480-400
⇒x=80
So, the number of younger children who rode tricycle = 80
Answer: 80

Written Math Solution (BIBM) FB Group : BD job Info. (BCS & Bank)
70

 Created by Sumon K. Sarkar  FB Group : BD job Info. (BCS & Bank)  Created by Sumon K. Sarkar 
Bank Job Solution (BIBM)
 Created by Sumon K. Sarkar  FB Group : BD job Info. (BCS & Bank)  Created by Sumon K. Sarkar 

5. The length and width of a garden is 60 meter and 20 meter respectively. Inside the garden, there
is a 5- meter-wide path around it. What is the area of the path in square meter ?
অনুবােঃ একনট বাগাতনর দে ণয ৬০ নমটার এবং প্রস্থ ২০ নমটার। বাগাতনর পভেতরর ারনেতক ৫ নমটার প্রস্থ নবনশ্ষ্ট একনট রাস্তা আতছ। েতব
রাস্তার পেত্রফল কে?
[Premier Bank TJO 18, SJIB TSO 18] [Standard Bank TAO Cash 16]
Solution:
Given that,
The length of a garden = 60 meter
The width of a garden = 20 meter
Area of a garden including path = 60 × 20 = 1200 square meter
Length of a garden without path = 60 - (5× 2) = 50 meter
Width of a garden without path = 20 - (5× 2) = 10 meter
Area of a garden without path = 50 × 10 = 500 square meter
Area of the path = 1200 - 500 = 700 square meter
Answer: 700 square meter

Bangladesh Bank
Post Name: Assistant Director Exam Date: 18– 11 – 2022
𝟒√𝒑 ×𝟏𝟔𝒑𝒒 ×𝟐−𝟐√𝒑
1. × 𝟒−𝒑𝒒
𝟖√𝒒 ×𝟒𝒑𝒒 ×𝟐−𝟑√𝒒
Solution:
4 √𝑝 ×16𝑝𝑞 ×2−2√𝑝
× 4−𝑝𝑞
8√𝑞 ×4 𝑝𝑞 ×2−3√𝑞
22√𝑝 ×4 2𝑝𝑞 ×2−2√𝑝
= × 2−2𝑝𝑞
23√𝑞 ×4 𝑝𝑞 ×2−3√𝑞
22√𝑝−2√𝑝 ×24𝑝𝑞
= × 2−2𝑝𝑞
23√𝑞−3√𝑞 ×22𝑝𝑞
20 ×24𝑝𝑞−2𝑝𝑞
= × 2−2𝑝𝑞
20
= 22𝑝𝑞 × 2−2𝑝𝑞
= 22𝑝𝑞−2𝑝𝑞
= 20 = 1
Answer: 1

2. Every year Mr. Kalam saves Tk 5400 which is 15 percent of his annual income. Mr. Rahim
spends Tk 2800 a month which is 87.5 percent of his monthly income. Who is earning more?
অনুবােঃ প্রনে বছর নমঃ কালাম ৫৪০০ টাকা সঞ্চয কতরন যা োর বানেণক আতযর ১৫ শ্োংশ্। েনাব রনহম মাতস ২৮০০ টাকা খর কতরন
যা োর মানসক আতযর ৮৭.৫ শ্োংশ্। পক পবনশ্ আয করতছ?
Solution:
Let, Mr. Kalam’s annual salary be Tk. x
According to the Question,
𝑥 × 15% = 5400
15
⇒𝑥× = 5400
100

Written Math Solution (BIBM) FB Group : BD job Info. (BCS & Bank)
Bank Job Solution (BIBM) 71

Created by Sumon K. Sarkar  FB Group : BD job Info. (BCS & Bank)  Created by Sumon K. Sarkar 
100
⇒ 𝑥 = 5400 ×
15
Created by Sumon K. Sarkar  FB Group : BD job Info. (BCS & Bank)  Created by Sumon K. Sarkar 

⇒ 𝑥 = 36,000
Mr. Rahim spends Tk. 2800 a month
Mr. Rahim annual spends = Tk. (2800 × 12) = Tk. 33,600
Let, Mr. Rahim’s annual salary be Tk. y
According to the Question,
𝑦 × 87.5% = 33,600
87.5
⇒𝑥× = 33,600
100
100
⇒ 𝑥 = 33,600 ×
87.5
100×10
⇒ 𝑥 = 33,600 ×
875
⇒ 𝑥 = 38,400
We see that, Mr. Rahim’s annual salary is greater than Mr. Kalam’s annual salary.
So, Mr. Rahim is earning more than Mr. Kalam.
Anwser: Mr. Rahim is earning more

3. The average weight of A, B and C is 40 kg. Weight of C is 24 kg more than A's weight and 3 kg
less than B's weight. What will be the average weight of A, B, C and D, if D weights 15 kg less
than C?
অনুবােঃ A, B এবং C এর গড ওেন 40 পকনে। C এর ওেন A এর ওেতনর প তয 24 পকনে পবনশ্ এবং B এর ওেতনর পর্তক 3 পকনে
কম। যনে D-এর ওেন C পর্তক 15 পকনে কম হয, A, B, C এবং D-এর গড ওেন কে হতব?
Solution:
Total weight of A, B and C = 40 × 3 = 120
Let, A's weight be x kg
So, C's weight = x + 24
B's weight = x + 24 +3 = x +27
According to the Condition,
x + x + 24 + x + 27 = 120
⇒ 3x + 51 = 120
⇒ 3x = 120 – 51
⇒ 3x = 69
⇒ x = 23
∴ C's weight = x + 24 = 23 + 24 = 47
So, D's weight = 47 – 15 = 32
120+32 152
Required Average weight= = = 38
4 4
Answer: 38

এ রকম গুরুত্বপূর্ণ ফাইল পপতে আমাতের গ্রুতপ প াখ রাখুন

Written Math Solution (BIBM) FB Group : BD job Info. (BCS & Bank)
72

 Created by Sumon K. Sarkar  FB Group : BD job Info. (BCS & Bank)  Created by Sumon K. Sarkar 
Bank Job Solution (BIBM)
 Created by Sumon K. Sarkar  FB Group : BD job Info. (BCS & Bank)  Created by Sumon K. Sarkar 

4. Matin visited his cousin Raju during the summer vacation. In the morning, they both would go
for swimming. In the evening, they would play tennis. They would engage in at most one activity
per day. There were days when they took rest and stayed home all day long. There were 32
mornings when they did nothing. 18 evenings when they stayed at home, and a total of 28 days
when they swam or played tennis. What duration of the summer vacation did Matin stay with
Raju?
অনুবােঃ মানটণন গ্রীষ্মকালীন ছুনটর সময োর খালাতো ভাই রােুর বাসায পবডাতে যায। সকাতল োরা একসাতর্ সাোর কাটতে পযে, নবকাতল
পটননস পখলতে পযে। োরা প্রনেনেন বডতোর একনট কাে করে। এমন নকছু নেন নছল যখন োরা পুতরানেন বাসায বতস নবশ্রাম ননে। ৩২
নেন সকালতবলা োরা নকছু কতরনন, ১৮ নেন নবকালতবলা োরা বাসায নছল এবং পমাট ২৮ নেন োরা হয পটননস পখতলনছল অর্বা সাোর
পকতটনছল। অবকাশ্ যাপতনর কেনেন মানটণন রােুর সাতর্ নছল?
Solution:
Let, Matin stay with Raju in x days.
According to the Question,
( x -32 ) + ( x – 18 ) = 28
⇒ x -32 + x – 18 = 28
⇒ 2x - 50 = 28
⇒ 2x = 28 + 50
⇒ 2x = 78
⇒ x = 39
Matin stay with Raju in 39 days
Answer: 39 days

5. In the figure below, AB is perpendicular to BC and BD=DC, If AD= √𝟏𝟎 cm and AC=4cm, then
what is the valu of BC?

Solution:
Let, AB be x and BD=DC be y
Given that, BD=DC & AD= √10 cm AC = 4cm So, BD =DC = y
∴ BC = y+y = 2y
According to the 1st Question,
∆𝐴𝐵𝐷 𝐴𝐵2 + 𝐵𝐷2 = 𝐴𝐷2 [ According to Pythagoras theorem ]
𝑥 2 + 𝑦 2 = (√10)2
⇒ 𝑥 2 + 𝑦 2 = 10 --------------------- (1)
According to the 2nd Question,
∆𝐴𝐵𝐶 - 𝐴𝐵2 + 𝐵𝐶 2 = 𝐴𝐶 2 [ According to Pythagoras theorem ]
2 2 2
𝑥 + (2𝑦) = 4

Written Math Solution (BIBM) FB Group : BD job Info. (BCS & Bank)
Bank Job Solution (BIBM) 73

Created by Sumon K. Sarkar  FB Group : BD job Info. (BCS & Bank)  Created by Sumon K. Sarkar 
⇒ 𝑥 2 + 4𝑦 2 = 16 ------------------(2)
Created by Sumon K. Sarkar  FB Group : BD job Info. (BCS & Bank)  Created by Sumon K. Sarkar 

Subtract equation (2) from equation (1)


𝑥 2 + 4𝑦 2 − 𝑥 2 − 𝑦 2 = 16 − 10
⇒ 3𝑦 2 = 6
⇒ 𝑦2 = 2
⇒ 𝑦 = √2
∴ BC = √2 + √2 = 2√2
Answer: 𝟐√𝟐

এ রকম গুরুত্বপূর্ণ ফাইল পপতে আমাতের গ্রুতপ প াখ রাখুন


এবং অবশ্যই ননতের বন্ধুতের ইনভাইট করুন

Bank Asia Ltd.


Post: Business Officer Exam Date: 25 – 03 – 2022
1. The price of 20 articles is the same as the selling price of X articles. If the profit is 25%, Then
what is the value of X ?
অনুবােঃ 20নট ননবতন্ধর মূলয X ননবতন্ধর নবক্রয মূতলযর সমান। যনে লাভ হয 25%, োহতল X এর মান কে?
Solution:
Let, Cost price of each article be tk. 1
So, Cost price of x articles = tk. x.
Selling price of x articles = Tk. 20.
Profit = Tk. (20 - x).
20 − x
× 100 = 25
𝑥
⇒ 2000 - 100x = 25x
⇒ 125x = 2000
⇒ x = 16.
Answer: 16

Alternative Way,
Cost price of 20 articles = selling price of x articles
⇒20× Cost price = x× selling price
20 selling price
⇒ = …….........(1)
𝑥 Cost price
Profit =25%
We know, Profit = selling price − Cost price
selling price −Cost price
Profit % = × 100
Cost price
Accoding to the Condition,

Written Math Solution (BIBM) FB Group : BD job Info. (BCS & Bank)
74

 Created by Sumon K. Sarkar  FB Group : BD job Info. (BCS & Bank)  Created by Sumon K. Sarkar 
Bank Job Solution (BIBM)
 Created by Sumon K. Sarkar  FB Group : BD job Info. (BCS & Bank)  Created by Sumon K. Sarkar 

selling price −Cost price


× 100 = 25
Cost price
selling price 25
⇒ −1=
Cost price 100
selling price 25
⇒ = +1
Cost price 100
20 25+100
⇒ = (∵ from (1))
𝑥 100
20 125
⇒ =
𝑥 100
20 5
⇒ =
𝑥 4
4 1
⇒ =
𝑥 4
⇒x =16.
Answer: 16

3. A widow took 1/3rd of the estate. Each son of the twos took the remaining estate. If the widow
and one son took 3,60,000 Tk, then what is the value of the estate?
অনুবােঃ একেন নবযবা সম্পনত্তর 1/3 অংশ্ ননতযনছতলন। দুেতনর প্রতেযক পছতল অবনশ্ষ্ট সম্পনত্ত ননতযনছল। যনে নবযবা ও এক পছতল
3,60,000 টাকা পনন, োহতল সম্পনত্তর মূলয কে?
Solution:
Let, the total value of the estate = 3x
1
The widow received = 3𝑥 × = x
3
Remaining estate = 3x – x = 2x
2𝑥
Each son received = =𝑥
2
According to the Question,
widow +One son = 360000
⇒ x + x = 360000
⇒2x = 360000
⇒x = 180000
⇒3x = 180,000 × 3
⇒3x = 540,000
Thus total estate value tk. 540,000
Answer: tk. 540,000

এ রকম গুরুত্বপূর্ণ ফাইল পপতে আমাতের গ্রুতপ প াখ রাখুন


এবং অবশ্যই ননতের বন্ধুতের ইনভাইট করুন

Written Math Solution (BIBM) FB Group : BD job Info. (BCS & Bank)
Bank Job Solution (BIBM) 75

Created by Sumon K. Sarkar  FB Group : BD job Info. (BCS & Bank)  Created by Sumon K. Sarkar 
Standrad Bank Ltd.
Created by Sumon K. Sarkar  FB Group : BD job Info. (BCS & Bank)  Created by Sumon K. Sarkar 

Post Name: Trainee Assistant Officer Exam Date: 11 – 03 – 2022


1. The ratio, by volume, of bleach to detergent to water in a certain solution is 2 : 40 : 100. The
solution will be altered so that the ratio of bleach to detergent is tripled while the ratio of
detergent to water is halved. If the altered solution will contain 300 liters of water, how many
liters of detergent will it contain?
অনুবােঃ একনট নননেণষ্ট দ্রবতর্ নডটারতেন্ট পর্তক নিত র েতলর অনুপাে হল 2 : 40 : 100৷ দ্রবর্নট পনরবেণন করা হতব যাতে নি পর্তক
নডটারতেতন্টর অনুপাে নেনগুর্ হয এবং নডটারতেতন্টর েতলর অনুপাে অতযণক হয৷ পনরবনেণে দ্রবতর্ 300 নলটার েল র্াকতল, কে নলটার
নডটারতেন্ট র্াকতব?
Solution:
Given ratio, bleach : detergent : water = 2:40:100 = 1:20:50
The new ratio bleach : detergent = 2:40 = 1: 20
the ratio of bleach to detergent is tripled =1:20×3 = 1 : 60
The new ratio detergent : water = 40:100
1
the ratio of detergent to water is halved = 40× : 100 = 20 : 100 = 60 : 300
2
Thus, the new ratio is : 1:60:300
Let, bleach, detergent & water = x , 60x & 300x
According to the condition,
300x = 300
Or, x = 1
Or, 60x = 60 litres
Detergent = 60 litres.
Answer: 60 litres

3. Alif bought a bike, but he paid 15000 taka directly & remaining money with 8% interest in 2
years. In total he paid 28,920 taka. What was the actual cost of the bike?
অনুবােঃ আনলফ একনট বাইক নকতননছল, নকন্তু পস 15000 টাকা সরাসনর পনরতশ্ায কতরতছ এবং বানক টাকা 8% সুতে 2 বছতর। সব
নমনলতয নেনন ২৮,৯২০ টাকা পনরতশ্ায কতরন। বাইকনটর প্রকৃে োম কে নছল?
Solution:
Let, Alif paid Tk. x remaining money
Interest in remaining money = 𝑥 × 2 × 8%
8
=𝑥 ×2 ×
100
4𝑥
=
25
According to the condition,
4𝑥
15,000 + x + = 28,920
25
4𝑥
⇒x+ = 28,920 – 15,000
25
25𝑥+4𝑥
⇒ = 13,920
25
29𝑥
⇒ = 13,920
25
13,920×25
⇒𝑥=
29

Written Math Solution (BIBM) FB Group : BD job Info. (BCS & Bank)
76

 Created by Sumon K. Sarkar  FB Group : BD job Info. (BCS & Bank)  Created by Sumon K. Sarkar 
Bank Job Solution (BIBM)
 Created by Sumon K. Sarkar  FB Group : BD job Info. (BCS & Bank)  Created by Sumon K. Sarkar 

⇒ 𝑥 = 12,000
Actual cost of the bike = 15000 + 12000 = 27,000
Answer: Tk. 27,000

Modhumoti Bank Ltd.


Post Name:Probationary Officer Exam Date : 11 – 03 – 2022
1. A, B, and C can do a work on 10, 30 and 60 days respectively. A is assisted by B and C on every
alternative days. How many days required to complete the work?
অনুবােঃ A, B, এবং C যর্াক্রতম 10, 30 এবং 60 নেতন একনট কাে করতে পাতর। প্রনে নবকল্প নেতন A-পক B এবং C সাহাযয কতর।
কাে পশ্ে করতে কে নেন লাগতব?
Solution:
1
Let, Work done by A in 1 day = Part
10
1
Work done by B in 1 day = Part
30
1
Work done by C in 1 day = Part
60
1 1 1
Work done by A, B and C in 1 day = + +
10 30 60
6+2+1 9 3
= = = Part
60 60 20
3 1 3+2 5 1
In 2 day Work done = + = = = part
20 10 20 20 4
1
part work is done in 2 days
4
∴ 1 part work is done in 2× 4 = 8 days
Answer: 8 days

2. X can travel from village to school at 3km/h and return at 2 km/h. Total time taken 5 hours.
What is the distance from Village to School?
অনুবােঃ X গ্রাম পর্তক স্কুতল পযতে পাতর 3 নকনম/ ন্টা পবতগ এবং নফরতে পাতর 2 নকনম/ ন্টা পবতগ। পমাট সময পলতগতছ 5 ন্টা। গ্রাম
পর্তক স্কুতলর েূরত্ব কে?
Solution:
Let, the required distance be x km.
𝑥
Then time taken during the first journey = hr.
3
𝑥
and time taken during the second journey = hr.
2
According to the condition,
𝑥 𝑥
+ =5
3 2
2𝑥+3𝑥
⇒ =5
6
5𝑥
⇒ =5
6
⇒5x=30
⇒x=6
Required distance = 6 km.
Answer: 6 km.

Written Math Solution (BIBM) FB Group : BD job Info. (BCS & Bank)
Bank Job Solution (BIBM) 77

Created by Sumon K. Sarkar  FB Group : BD job Info. (BCS & Bank)  Created by Sumon K. Sarkar 
3. A rectangular Field with length 112 m and width 78 m. Inside the garden there is 2.5 m path.
Created by Sumon K. Sarkar  FB Group : BD job Info. (BCS & Bank)  Created by Sumon K. Sarkar 

Find the area of path and cost for constructing it at 4 tk per sqm.
অনুবােঃ একনট আযেতেত্রাকার পেত্র যার দে ণয 112 নমটার এবং প্রস্থ 78 নমটার। বাগাতনর নভেতর 2.5 নমটার পর্ রতযতছ। পতর্র
পেত্রফল ননর্ণয করুন এবং প্রনে বগণনমটাতর 4 টাকা এনট ননমণাতর্র েনয খর কে হতব?
Solution:

Solution:
Given that, Length of rectangular grassy plot =112 m long and Breadth =78 m
Therefore, the outside area of rectangular grassy plot =112×78 = 8736 m2
Total Area of Plot = 8736 m2
It has a gravel path 2.5 m wide all around it on the inside.
Then, Grassy area = (112−2.5×2)(78−2.5×2) = 7811 m2
Area of Path = Area of total plot − area of inside (grassy area)
=8736−7811= 925 m2
Cost of constructing road = 925× tk.4 = Tk. 3700
Answer: Tk. 3700

5. An article when sells at Tk 75 something loss. If it sells at Tk.96. gain would be double the former
loss. find the Cost price of that article.
অনুবােঃ একটা article ৭৫ টাকায নবনক্র করতল নকছু েনে হয।যনে ৯৬ টাকায নবনক্র হে েতব লাভ আতগর েনের নিগুর্ হতব। article
এর ক্রে মূলয কে?
Solution:
Let , Cost price of that article be Tk x.
According to the Question,
2(x-75) = 96 – x
⇒ 3x = 246
⇒ x = 82
Answer: Tk. 82

এ রকম গুরুত্বপূর্ণ ফাইল পপতে আমাতের গ্রুতপ প াখ রাখুন


এবং অবশ্যই ননতের বন্ধুতের ইনভাইট করুন

Written Math Solution (BIBM) FB Group : BD job Info. (BCS & Bank)
78

 Created by Sumon K. Sarkar  FB Group : BD job Info. (BCS & Bank)  Created by Sumon K. Sarkar 
Bank Job Solution (BIBM)
 Created by Sumon K. Sarkar  FB Group : BD job Info. (BCS & Bank)  Created by Sumon K. Sarkar 

Premier Bank Ltd.


Post name:Trainee Junior Officer (Cash) Exam Date : 19 – 03 – 2021
1. The average age of a committee of eight members is 40 years. A member aged 55 years retired
and his place was taken by another member aged 39 years. Find the average age of the present
committee.
অনুবােঃ আট সেতসযর একনট কনমনটর গড বযস ৪০ বছর। ৫৫ বছর বযসী একেন সেসয অবসর ননতযতছন এবং োর স্থতল ৩৯ বছর বযসী
অনয সেসয গ্রহর্ কতরতছন। বেণমান কনমনটর গড বযস ননর্ণয কর।
Solution:
Difference in age of last person = 55-39 = 16
This change will be distributed among all 8 members,
Hence, the average of group reduced by 16/8 = 2 years
So average age of present committee = 40 – 2 = 38 years
Answer: 38 years

Alternative Way,
Total age of a committee of eight members = 40 × 8 = 320 years
A member replace an another member , then
Total age of a committee of eight members = 320 − 55 + 39 = 304 years
304
So avg age of present committee = = 38 𝑦𝑒𝑎𝑟𝑠
8
Answer: 38 years

2. The ratio of the salary of A, B & C is 7:5:3. If B gets Tk 222 more than what C gets, then what is
the salary of A?
অনুবােঃ A, B এবং C এর পবেতনর অনুপাে 7:5:3। B যনে C এর প তে 222 টাকা পবনশ্ পায, োহতল A-এর পবেন কে?
Solution:
Let, the salary of A, B & C is 7x, 5x & 3x
According to the Question,
5x = 3x +222
⇒ 5x - 3x = 222
⇒ 2x =222
⇒ x = 111
⇒ 7x = 777
Thus,The salary of A Tk. 777
Answer: Tk. 777

এ রকম গুরুত্বপূর্ণ ফাইল পপতে আমাতের গ্রুতপ প াখ রাখুন


এবং অবশ্যই ননতের বন্ধুতের ইনভাইট করুন

Written Math Solution (BIBM) FB Group : BD job Info. (BCS & Bank)
Bank Job Solution (BIBM) 79

Created by Sumon K. Sarkar  FB Group : BD job Info. (BCS & Bank)  Created by Sumon K. Sarkar 
UCB Bank Ltd.
Created by Sumon K. Sarkar  FB Group : BD job Info. (BCS & Bank)  Created by Sumon K. Sarkar 

Post name:Probationary Officer Exam Date : 19 – 11 – 2021


1. Mr. Salam has an open terrace adjacent to his apartment where his children play football. To
protect the children from filing, Mr. Salam wants to put up railing around three sides of his
rectangular terrace and leave a side of 20 feet unfenced as that side is covered by his apartment
and needs no railing. If the terrace has an area of 680 square feet, how many feet of railing does
he need?
অনুবােঃ েনাব সালাতমর োর অযাপাটণতমতন্টর পাতশ্ একনট পখালা বারান্দা রতযতছ পযখাতন োর সন্তানরা ফুটবল পখতল। বাচ্চাতের পখলার
সমে যাতে না পতড় যাে পসেনয আযেতেত্রাকার বারান্দার ারপাতশ্ নেননেতক পরনলং লাগাতে ান এবং 20 ফুতটর একনট পাশ্ পরনলংনবহীন
রাখতে ান কারর্ পসই নেকনট োর অযাপাটণতমন্ট িারা আচ্ছানেে এবং পকান পরনলংতযর প্রতযােন পনই। বারান্দার পেত্রফল 680 বগণফুট
হতল, োর কে ফুট পরনলং লাগতব?
Solution:
Given that, rectangular terrace has an area of 680 square feet leave a side of 20 feet unfenced
Let, terrace’s another side x feet
According to the Question,
20x = 680
⇒ x = 680/20
⇒ x = 34
∴ terrace’s another side 34 feet
So, Total feet of railing = 34 +34 +20 = 88 feet
Answer: 88 feet

এ রকম গুরুত্বপূর্ণ ফাইল পপতে আমাতের গ্রুতপ প াখ রাখুন


এবং অবশ্যই ননতের বন্ধুতের ইনভাইট করুন

Premier Bank Ltd.


Post Name: Trainee Junior Officer (General) Exam Date : 21 – 10 – 2020

1. A man has Tk.100,000 invest. He invests Tk. 40,000 at 5% and Tk. 35,000 at 4% interest rate. In
order to have a yearly income of Tk. 5000, at what rate of interest he must invest the reminder
amount.
অনুবােঃ একেন মানুতের 100,000 টাকা নবননতযাগ আতছ। নেনন টাকা 5% সুে হাতর 40,000 টাকা এবং 4% সুে হাতর 35,000
নবননতযাগ কতরন । যাতে বাৎসনরক আয 5000 টাকা। নক সুতের হাতর োর বানক টাকার উপর নবননতযাগ করতে হতব।
Solution:
Let, A man invested at x%
40,000×5
40000 at 5% gives = 2,000
100
35,000×4
35000 at 4 % gives = 1400
100
To have 500, he needs 5000−(2000+1400) =1600

Written Math Solution (BIBM) FB Group : BD job Info. (BCS & Bank)
80

 Created by Sumon K. Sarkar  FB Group : BD job Info. (BCS & Bank)  Created by Sumon K. Sarkar 
Bank Job Solution (BIBM)
 Created by Sumon K. Sarkar  FB Group : BD job Info. (BCS & Bank)  Created by Sumon K. Sarkar 

Money left to invest =100,000−(40,000+35,000) =25,000.


According to the condition,
25,000 × 𝑥% = 1600
𝑥
⇒ 25,000 × = 1600
100
1600×100
⇒x= = 6.4%
25,000
A man invested at 6.4%
Answer: 6.4%

2. M can do a piece of work in 40 days. He works at it for 5 days and then N alone finishes the
remaining work in 42 days. The two together could complete the work in how many days?
অনুবােঃ M 40 নেতনর মতযয একনট কাে করতে পাতর। নেনন এনটতে 5 নেন কাে কতরন এবং োরপর N একাই 42 নেতন বানক কাে পশ্ে
কতরন। দুেন নমতল কে নেতন কাে পশ্ে করতে পারতেন?
Solution:
Let, A can do a piece of work in 40 days
1
Work of A in 1 day =
40
5 1
Work of A in 5 days = =
40 8
1
th part of work is done by A
8
1 7
∴ Remaining work =(1− ) =
8 8
7
th part is done by B in 42 days
8
42×8
So, Total work will be done by B in = 48
7
1
Work done by B in one day =
48
1 1 6+5 11
Work done by A and B together in one day will be ( + )= =
40 48 240 240
240
A and B will take = 21.82 days to complete the work together.
11
Answer: 21.82 days

3. In a shop, the cost of 4 shirts, 4 pairs of trousers and 2 hats is Tk. 560. The cost of 9 shirts, 9 pairs
of trousers and 6 hats is Tk. 1290. What is the total cost of 1 shirt, 1 pair of trousers and 1 hat?
[Janata AEO (RC) 17]
অনুবােঃ একনট পোকাতন 4 নট শ্াটণ, 4 পোডা ট্রাউোর এবং 2 টুনপর োম 560 টাকা। 9 নট শ্াটণ, 9 পোডা ট্রাউোর এবং 6 নট টুনপর োম
1290। ১নট শ্াটণ, ১ পোডা পযান্ট ও ১নট টুনপর পমাট োম কে
Solution:
Let, the cost of 1 shirt, 1 pair of trousers and 1 hat be Tk. s, Tk. t and Tk. h respectively.
According to the question,
4s + 4t + 2h = 560 …………… (i)
9s + 9t + 6h = 1290
∴ 3s + 3t + 2h = 430 …………..(ii) [Dividing by 3]
Now, subtraction the equation (i) & (ii)
∴ s+t = 130 ………………….. (iii)

Written Math Solution (BIBM) FB Group : BD job Info. (BCS & Bank)
Bank Job Solution (BIBM) 81

Created by Sumon K. Sarkar  FB Group : BD job Info. (BCS & Bank)  Created by Sumon K. Sarkar 
Put the value of (s+t) in equation (ii)
Created by Sumon K. Sarkar  FB Group : BD job Info. (BCS & Bank)  Created by Sumon K. Sarkar 

3s + 3t + 2h = 430
⇒ 3(s + t) + 2h = 430
⇒ 3×130 + 2h = 430
⇒ 390 + 2h = 430
⇒2h = 430 – 390
⇒2h = 40
∴ h = 20 …………………….. (iv)
Add the equation (iii) & (iv)
s + t + h = 130+20 = 150
∴ The total cost of 1 shirt, 1 pair of trousers and 1 hat = Tk. 150
Answer: Tk. 150

Bank Asia Ltd.


Post Name: Probationary Officer Exam Date : 2020
1. Raju can finish a work in 15 hours and Saju can finish the same work in 9 hours. They work
together firsty at 6 am but Saju stopped at 9 am. What the time required to finish the work by
Raju?
অনুবােঃ রােু একনট কাে 15 ন্টায পশ্ে করতে পাতর এবং সােু একই কাে 9 ন্টায পশ্ে করতে পাতর। সকাল ৬টায একসতে কাে
করতলও সােু সকাল ৯টায পর্তম যায। রােুর কাে পশ্ে করতে কে সময লাগতব?
Solution:
Here, 6am to 9am = 3hours
1 1 3+5 8
In 1 hour, Raju & saju can do + = =
15 9 45 45
8 8
In 3 hour, Raju & saju can do ×3=
45 15
8 7
Remaining work = 1− =
15 15
Raju can do 1 part of the work in 15 hours
7 7
Raju can do part of the work in 15 × = 7 hours
15 15
So work finish time = 6am+(3+7)h = 4:00pm
Answer: 4.00 PM

2. Mr. Zahir received Eid bonus of Tk. 7875. He distributed the money among his three daughters
Dame Lipi, Rini and Shahnaz. Lipi got 3/5 of what Shahnaz got. Shahnaz got double of what
Rimi got. Determine the actual amount of money that each of the sisters got.
[AB Bank AO 14, BISIC Officer 14, IFIC Bank MTO 13, DBBL MTO(2) 09, Bank Asia PO 20]
Abyev`t Rwni mv‡ne 7,875 UvKv C‡`i †evbvm cvb| GB UvKv Zvi wZb Kb¨v wjwc, wiwg I kvnbv‡Ri g‡a¨ fvM K‡i †`b| wjwc,
kvnbv†Ri Ask cvq I kvnbvR, wiwgi wظY UvKv cvq| cÖ‡Z¨‡K KZ UvKv K‡i cvb, wbY©q Ki|
Solution:
Given that, Shahnaz got double of what Rimi got
Rimi : Shahnaz = 1 : 2 = 5 : 10
Again, Lipi got 3/5 of what Shahnaz got
Shahnaz : Lipi = 5 : 3 = 10 : 6

Written Math Solution (BIBM) FB Group : BD job Info. (BCS & Bank)
82

 Created by Sumon K. Sarkar  FB Group : BD job Info. (BCS & Bank)  Created by Sumon K. Sarkar 
Bank Job Solution (BIBM)
 Created by Sumon K. Sarkar  FB Group : BD job Info. (BCS & Bank)  Created by Sumon K. Sarkar 

Rimi : Shahnaz : Lipi = 5 : 10 : 6


Sum of the ratio = 5 + 10 + 6 = 21
5
Rimi got = 7875 × = 1875
21
10
Shahaz got =7875 × = 3750
21
6
Lipi got = 7875 × = =2250
21
Answer: tk. 1875 , tk. 3750, tk. 2250

3. Mr. Arif have been invested two schemes, first amount Tk. 5000 in 2 years and second amount
Tk. 3000 in 4 years at the same rate of interest. Total interest he received Tk. 2200 at the end of
a year. Calculate the rate of interest.
অনুবােঃ েনাব আনরফ দুনট নস্কম নবননতযাগ কতরতছন, একই সুতের হাতর প্রর্মনট 2 বছতর 5000 টাকা এবং নিেীয 4 বছতর 3000 টাকা।
এক বছর পশ্তে নেনন পমাট সুে 2200 টাকা পান । সুতের হার গর্না করুন।
Solution:
Let, Rate of interest r%
According to the question,
5000 × 2 × 𝑟% + 3000 × 4 × 𝑟% = 2200
𝑟 𝑟
⇒ 5000 × 2 × + 3000 × 4 × = 2200
100 100
⇒ 100𝑟 + 120𝑟 = 2200
⇒ 220𝑟 = 2200
∴ 𝑟 = 10
Rate of interest 10%
Answer: 10%

Alternative way:
Let, the rate of interest per annum be r%
Interest at Tk. 5000 for 2 years = prn = 5000×r%×2 = Tk. 100r
Interest at Tk. 3000 for 4 years = prn = 3000×r%×4 = Tk. 120r
According to the question,
100r+120r = 2200
⇒ 220r = 2200
⇒ r = 10
Rate of interest = 10%
Answer: 10%

এ রকম গুরুত্বপূর্ণ ফাইল পপতে আমাতের গ্রুতপ প াখ রাখুন


এবং অবশ্যই ননতের বন্ধুতের ইনভাইট করুন

Written Math Solution (BIBM) FB Group : BD job Info. (BCS & Bank)
Bank Job Solution (BIBM) 83

Created by Sumon K. Sarkar  FB Group : BD job Info. (BCS & Bank)  Created by Sumon K. Sarkar 
Mutual Trust Bank Ltd.
Created by Sumon K. Sarkar  FB Group : BD job Info. (BCS & Bank)  Created by Sumon K. Sarkar 

Post name: Management Trainee Officer Exam Date : 2019


1. Tanim bought some oranges. He gave ½ of them to his sister, 1/4th of the remainder to his
neighbor, 3/5th of those left to his children and had 6 left in the end. How many oranges did
Tanim buy?
Abyev`t Zvwbg wKQz Kgjv‡jey wKbj, hvi A‡a©K †m Zvi †evb‡K w`j| Aewkó Kgjvi 1/4 fvM cÖwZ‡ewk‡`i w`j| Gici hv
Aewkó wQj Zvi 3/5 fvM Zvi mšÍvb‡`i †`Iqvi ci Zvi Kv‡Q 6wU Kgjv iBj| Zvwbg KZwU Kgjv wK‡bwQj?
[BB AD 10, MTB MTO 19]
Solution:
Let, Tanim bought x oranges.
𝑥 𝑥
After giving ½ to her sister, she has x - =
2 2
he give 1/4th of the remainder to her neighbour,
𝑥 𝑥 𝑥 𝑥 4𝑥−𝑥 3𝑥
so he now has = - = - = =
2 2×4 2 8 8 8
3𝑥 3 9𝑥
children got = × =
8 5 40
3𝑥 9𝑥 15𝑥−9𝑥 6𝑥 3𝑥
Now, she has = − = = =
8 40 40 40 20
According to the Question,
3𝑥
⇒ =6
20
6×20
⇒𝑥 = = 40
3
⇒x = 40
So he bought 40 oranges.
Answer: 40 oranges.

2. A Group of students has hired a bus for Taka 3000 for going to a picnic. They had an
understanding that each participant would share the charge in equal amounts. But because of
10 students not turning up, the charged per student increased by Taka 10 over the initial
estimates. What was the number of students who originally registered for the picnic?
[BB AD 06, MTB MTO 19]
Abyev`tGK `j QvÎ wcKwb‡K hvevi Rb¨ 3,000 UvKv w`‡q GKwU evm fvov K‡i| Zviv cÖ‡Z¨‡K mg-cwigvb Puv`v †`evi wm×všÍ
†bq| wKš‘ 10 Rb QvÎ bv hvIqvq AvbygvwbK cÖv_wgK wnmve †_‡K RbcÖwZ Puv`vi cwigvb 10 UvKv e„w× cvq| cÖK…Zc‡¶ KZRb
QvÎ wcKwb‡K hvevi Rb¨ †iwR‡óªkb K‡iwQj?
Solution:
Let, total number of students = x
3000
Per head proposed cost for the picnic = Tk
𝑥
Due to absence of 10 students,
3000
average cost charged = Tk
𝑥−10
According to the Question,
3000 3000
− = 10
𝑥 𝑥 − 10
3000x – 3000x ✛ 30000
⇒ = 10
𝑥(𝑥−10)

Written Math Solution (BIBM) FB Group : BD job Info. (BCS & Bank)
84

 Created by Sumon K. Sarkar  FB Group : BD job Info. (BCS & Bank)  Created by Sumon K. Sarkar 
Bank Job Solution (BIBM)
 Created by Sumon K. Sarkar  FB Group : BD job Info. (BCS & Bank)  Created by Sumon K. Sarkar 

30000
⇒ = 10
𝑥(𝑥−10)
3000
⇒ =1
𝑥(𝑥−10)
⇒ 𝑥(𝑥 − 10) = 3000
⇒ 𝑥 2 − 10𝑥 − 3000 = 0
⇒ 𝑥 2 – 60x ✛ 50x − 3000 = 0
⇒ 𝑥(𝑥– 60) ✛ 50(x − 60) = 0
⇒ (𝑥 + 50)(𝑥– 60) = 0
Here, x = -50 & x = 60
So, x = 60 (number of students cannot be negative)
So, Total number of students 60.
Answer: 60

Premier Bank Ltd.


Post name: Trainee Junior Officer (General) Exam Date : 2019

1. The ratio of the ages of a man and his wife is 4:3. After 4 years, this ratio will be 9:7. If the time
of marriage, the ratio was 5:3, then how many years ago were they married?
Abyev`t GK e¨w³ Ges Zvi ¯¿xÕi eq‡mi AbycvZ 4 t 3| 4 eQi c‡i Zv‡`i eq‡mi AbycvZ n‡e 9t 7| hw` we‡qi mgq Zv‡`i
eq‡mi AbycvZ 5 t 3 n‡j, Zviv KZ eQi Av‡M we‡q K‡iwQj?
Solution:
Let, a man and his wife present age be 4x & 3x.
Then, after 4years the husband's age = 4x+4
and the wife's age = 3x+4.
According to the condition,
4𝑥+4
=79
3x+4
⟹28x+28=27x+36
⟹x=8.
Then the present age of the husband = 4×8years = 32years
& the wife's age = 3×8 years= 24 years.
Let, their marriage took place P years back.
According to the condition,
32−𝑃
=35
24−P
⟹96−3P = 120−5P
⟹2P = 24
⟹P =12
12 years ago were they married.
Answer: 12 years

Written Math Solution (BIBM) FB Group : BD job Info. (BCS & Bank)
Bank Job Solution (BIBM) 85

Created by Sumon K. Sarkar  FB Group : BD job Info. (BCS & Bank)  Created by Sumon K. Sarkar 
2. ABC is a triangle in which AB = 3cm, BC = 5cm, and AC = 4cm, AD is a perpendicular from A
Created by Sumon K. Sarkar  FB Group : BD job Info. (BCS & Bank)  Created by Sumon K. Sarkar 

to BC. Find the length of AD.

[Modhumoti MTO 16, Premier Bank JO 19]


Solution:
Given that, ABC is a triangle in which AB = 3cm, BC = 5cm, and AC = 4cm
According to the condition,
∠𝐵𝐴𝐶 = 90𝑜
𝑆𝑜, ∆𝐵𝐴𝐶 𝑟𝑖𝑔ℎ𝑡 𝑎𝑛𝑔𝑒𝑙 𝑇𝑟𝑖𝑎𝑛𝑔𝑙𝑒
1
Area of the , ∆𝐵𝐴𝐶 = × 𝐴𝐵 × 𝐴𝐶
2
1
= ×3×4
2
=3×2=6
1
Area of the , ∆𝐴𝐵𝐶 = × 𝐴𝐷 × 𝐵𝐶
2
According to the condition,
1
× 𝐴𝐷 × 𝐵𝐶 = 6
2
1
⟹ × 𝐴𝐷 × 5 = 6
2
12
⟹ 𝐴𝐷 = = 2.4
5
Find the length of AD 2.4 cm
Answer: 2.4 cm

3. A trader, while selling an item, was asking for such a price that would enable him to offer a 20%
discounts and still make a profit of 30% on cost. If the cost of the item was Tk. 50 what was his
asking price?
[BB AD 08, Standard Bank TAO 16, Premier Bank JO 19]
Abyev`t GKRb we‡µZv GKwU c‡Y¨i Ggb GKwU `vg e‡jb hv‡Z cY¨wU‡Z 20% wWmKvD›U w`‡jI we‡µZvi 30%
jvf nq| hw` cY¨wUi g~j¨ 50 UvKv nq Zvn‡j we‡µZv KZ `vg e‡jwQ‡jb?
Solution:
Given that, cost price = 50 Tk
30
At 30% profit, price will be = 50 + (50 × ) = 50 + 15 = Tk.65
100
Let, asking price of the item = 100 Tk.
At 20% discount price = (100-20) = 80 Tk.
So, when discounted price is 80 Tk then asking price is 100 Tk
100×65
When discounted price is 65 Tk asking price is = = tk81.25
80
Answer: tk 81.25

Written Math Solution (BIBM) FB Group : BD job Info. (BCS & Bank)
86

 Created by Sumon K. Sarkar  FB Group : BD job Info. (BCS & Bank)  Created by Sumon K. Sarkar 
Bank Job Solution (BIBM)
 Created by Sumon K. Sarkar  FB Group : BD job Info. (BCS & Bank)  Created by Sumon K. Sarkar 

4. If x+y=a, x2+ y2 =b2 and x3+ y3 = c3 , then show that a3 + 2c3 = 3ab2
[Modhumoti MTO 16, Premier Bank JO 19, Board Book]
Solution:
Given That,
x+y=a, x2+ y2 =b2 and x3+ y3 = c3
L.H.S.
a3 + 2c3
(x+y)3 + 2(x3+ y3)
⇒ 𝑥 3 + 3𝑥 2 𝑦 + 3𝑥𝑦 2 + 𝑦 3 + 2𝑥 3 + 2𝑦 3
⇒ 3𝑥 3 + 3𝑦 3 + 3𝑥 2 𝑦 + 3𝑥𝑦 2
⇒ 3(𝑥 3 + 𝑦 3 + 𝑥 2 𝑦 + 𝑥𝑦 2 )
⇒ 3(𝑥 3 + 𝑥 2 𝑦 + 𝑥𝑦 2 + 𝑦 3 )
⇒ 3{𝑥 2 (𝑥 + 𝑦) + 𝑦 2 (𝑥 + 𝑦)}
⇒ 3(𝑥 + 𝑦)(𝑥 2 + 𝑦 2 )
⇒ 3ab2
So, a3 + 2c3 = 3ab2 (showed)

NRB Bank Ltd.


Post Name: Management Trainee Officer Exam Date : 2019
𝟏𝟎 𝟏 𝟏𝟖
1. Solve the Equation: + =
𝟐𝒙−𝟓 𝒙+𝟓 𝟑𝒙−𝟓
[SJIB MTO 16, SJIB TSO 18, Premier Bank TJO 18, NRB Bank MTO 19]
Solution:
𝟏𝟎 𝟏 𝟏𝟖
A + =
𝟐𝒙−𝟓 𝒙+𝟓 𝟑𝒙−𝟓
𝟏𝟎 𝟏 𝟏𝟓+𝟑
⇒ + =
𝟐𝒙−𝟓 𝒙+𝟓 𝟑𝒙−𝟓
𝟏𝟎 𝟏 𝟏𝟓 𝟑
⇒ + = +
𝟐𝒙−𝟓 𝒙+𝟓 𝟑𝒙−𝟓 𝟑𝒙−𝟓
𝟏𝟎 𝟏𝟓 𝟑 𝟏
⇒ − = −
𝟐𝒙−𝟓 𝟑𝒙−𝟓 𝟑𝒙−𝟓 𝒙+𝟓
𝟑𝟎𝒙−𝟓𝟎−𝟑𝟎𝒙+𝟕𝟓 𝟑𝒙+𝟏𝟓−𝟑𝒙+𝟓
⇒ =
(𝟐𝒙−𝟓)(𝟑𝒙−𝟓) (𝟑𝒙−𝟓)(𝒙+𝟓)
𝟐𝟓 𝟐𝟎
⇒ =
(𝟐𝒙−𝟓)(𝟑𝒙−𝟓) (𝟑𝒙−𝟓)(𝒙+𝟓)
𝟓 𝟒
⇒ =
(𝟐𝒙−𝟓) (𝒙+𝟓)
⇒ 8x – 20 = 5x +25
⇒ 8x –5x = 25 + 20
⇒ 3x = 45
⇒ x = 15
Answer: x =15
এ রকম গুরুত্বপূর্ণ ফাইল পপতে আমাতের গ্রুতপ প াখ রাখুন
এবং অবশ্যই ননতের বন্ধুতের ইনভাইট করুন

Written Math Solution (BIBM) FB Group : BD job Info. (BCS & Bank)
Bank Job Solution (BIBM) 87

Created by Sumon K. Sarkar  FB Group : BD job Info. (BCS & Bank)  Created by Sumon K. Sarkar 
2. Mr. Karim borrowed Tk. 500 at 5% simple interest per year. After some time, he borrowed Tk.
𝟏
Created by Sumon K. Sarkar  FB Group : BD job Info. (BCS & Bank)  Created by Sumon K. Sarkar 

400 at 𝟑 % simple interest per year for the second time. Six months after the second time
𝟐
borrowing, he repaid both borrowed money along with interest and the amount repaid was Tk.
994.50. How many years after the first time borrowing Mr. Karim repaid the borrowed money?
(Bank Asia MTO 17, NRB Bank MTO 19)
Abyev`t wgt Kwig evwl©K 5% mij my‡` 500 UvKv FY †bb | wKQz mgq ci wZwb evwl©K 3% mij my‡` 400 UvKv FY †bb|
wØZxqevi FYMÖn‡Yi 6 gvm ci wZwb my`vm‡j Dfq FY †kva K‡ib| wZwb †gvU 994.50 UvKv †kva K‡ib cÖ_gevi FYMÖn‡Yi KZ
eQi ci wZwb FY‡kva K‡ib?
Solution:
Let, Mr. Karim repaid x years after the first time borrowing.
According to the Question,
6 1
500 × 𝑥 × 5% + 400 × × 3 % = 994.5 − (500 + 400)
12 2
5 1 7
⇒ 500 × 𝑥 × + 400 × × % = 994.5 − 900
100 2 2
5 1 7
⇒ 500 × 𝑥 × + 400 × × = 94.5
100 2 200
⇒ 25𝑥 + 7 = 94.5
⇒ 25𝑥 = 94.5 − 7
⇒ 25𝑥 = 87.5
⇒ 𝑥 = 3.5
Mr. Karim repaid 3.5 years after the first time borrowing.
Answer : 3.5 years

4. A team of 2 men and 5 women complete 1/4th of a job in 3 days. After that another man joins
them and they all complete the next 1/4th of the job in 2 days. How many men can complete the
whole job in 4 days?
[BB AD 14, PKSF AM 09, NCC Bank Officer 02, One Bank SCO 18, NRB Bank MTO 19]
Abyev`t 2 Rb cyiæl Ges 5 Rb gwnjvi GKwU `j 3 w`‡b GKwU Kv‡Ri 1/4 Ask m¤c~Y© K‡i| 3w`b ci GKRb AwZwi³ cyiæl
Kv‡R †hvM w`‡j Zviv 2 w`‡b AviI 1/4 Ask m¤cbœ K‡ib| KZRb cyiæl wg‡j evwK KvR 4 w`‡b m¤cbœ Ki‡Z cvi‡eb?
Solution:
Let, Man and Women be m & w
Work rate of Man and Women,
2 5 1
+ =
𝑚 𝑤 4×3
2 5 1
⇒ + = ....................... (i)
𝑚 𝑤 12
3 5 1
+ =
𝑚 𝑤 4×2
3 5 1
⇒ + = ............................... (ii)
𝑚 𝑤 8
Subtract equation (i) from equation (ii)
3 2 1 1
⇒ − = −
𝑚 𝑚 8 12
1 3−2
⇒ =
𝑚 24
⇒ m = 24
It means one man needs 24 days to complete the work so work completed in 4 days ,

Written Math Solution (BIBM) FB Group : BD job Info. (BCS & Bank)
88

 Created by Sumon K. Sarkar  FB Group : BD job Info. (BCS & Bank)  Created by Sumon K. Sarkar 
Bank Job Solution (BIBM)
 Created by Sumon K. Sarkar  FB Group : BD job Info. (BCS & Bank)  Created by Sumon K. Sarkar 

24
we need = 6 men.
4
Alternative Way,
A team of 2 men and 5 women can complete a job in 4 × 3 = 12 days
A team of 2 + 1 = 3 men and 5 women can complete the whole thing in 4 × 2 = 8 days
A man can do the same amount of work in 8 days as a team of 2 men and 5 women can in 12 - 8 = 4
days
12
Therefore, a man can complete a job (solo) in 8 × = 24 days
4
24
A job can be completed within 4 days by = 6 men.
4
Answer: 6 men.

5. The total price of cricket bat and ball is Tk. 1350. If the price of bat increases 25% and the price
of ball decreases 20% then the total price of bat and ball remains same. Find the price of bat and
ball.
অনুবােঃ নক্রতকট বযাট ও বতলর সবণতমাট োম 1350 টাকা। বযাতটর োম 25% বাডতল এবং বতলর োম 20% কতম পগতল বযাট ও বতলর
পমাট োম একই র্াতক। বযাট এবং বতলর োম পবর করুন।
Solution:
Let, the price of bat and ball be 100x & 100y
According to the 1st condition,
100x + 100y = 1350
2x + 2y = 27 …………………(1)
At, 25% increases new price of bat,
25
100x + 100x × 25% = 100x + 100x × = 125x
100
At, 20% decreases new price of ball,
20
100y - 100y × 20% = 100y - 100y × = 80y
100
According to the 2nd condition,
25x + 16y = 270 …………..…(2)
Subtract equation (i) × 8 from equation (ii)
25x + 16y - 16x - 16y = 270 – 216
⇒ 9x = 54
⇒x=6
⇒ 100x = 600
Putting the value of x in equation (1)
2×6 + 2y = 27
⇒ 2y = 27 – 12 = 15
⇒ 100y = 750
Answer: 600, 750

এ রকম গুরুত্বপূর্ণ ফাইল পপতে আমাতের গ্রুতপ প াখ রাখুন


এবং অবশ্যই ননতের বন্ধুতের ইনভাইট করুন

Written Math Solution (BIBM) FB Group : BD job Info. (BCS & Bank)
Bank Job Solution (BIBM) 89

Created by Sumon K. Sarkar  FB Group : BD job Info. (BCS & Bank)  Created by Sumon K. Sarkar 
Modhumoti Bank Ltd.
Created by Sumon K. Sarkar  FB Group : BD job Info. (BCS & Bank)  Created by Sumon K. Sarkar 

Post Name: Probationary Officer Exam Date : 2018


1. Mr B invests Tk 2400 in the bank at 5% interest. How much additional money must be invested
at 8% interest so that the total interest will be equal to 6% of his entire investment?
[UCBL Officer 11, Modhumoti PO 18, SJIB TSO 18]
Abyev`t B mv‡ne evwl©K kZKiv 5 UvKv mij gybvdvq 2400 UvKv wewb‡qvM K‡ib| evwl©K kZKiv 8 UvKv mij gybvdvqAvi KZ
UvKv wewb‡qvM Ki‡j wZwb †gvU wewb‡qv‡Mi Dci GK‡Î evwl©K kZKiv 6 UvKv mij gybvdv cv‡eb?
Solution:
Let, Mr B Additional money invested be tk. x
According to the question,
8% of x + 5% of 2400 = 6%(x+2400)
8𝑥 5×2400 6(𝑥+2400)
⇒ + =
100 100 100
⇒ 8x+12000 = 6x+14400
⇒ 8x - 6x = 14400 – 12000
⇒ 2x = 2400
⇒ x = 1200
Mr B Additional money invest is tk. 1200
Answer: tk. 1200

2. Salesman is paid a monthly salary of tk 15000 plus 12.5% commission on all of his
sales .What should be his total annual sales in TK so that his annual earning from salary and
commission is tk 265000 ?
Abyev`t GKRb weµq Kg©x‡K cÖwZ gv‡m 15000 UvKv I Zvi †gvU weµ‡qi Dci 12.5% Kwgkb †`qv nq| Zv‡K eQ‡i KZ
UvKvi cb¨ wewµ Ki‡Z n‡e hv‡Z Zvi †eZb I Kwgkb wg‡j 265000 UvKv nq?
Solution:
Given that, Monthly salary= tk. 15000
So, Annual salary = (15000×12) = tk. 180,000
So,Annual commission he got = (265000-180000) = tk. 85000
Let, His annual sales be tk. x
According to the question,
12.5% of x = 85,000
12.5 𝑥
Or, = 85,000
100
Or, x = 6,80,000
Answer: tk. 6,80,000

এ রকম গুরুত্বপূর্ণ ফাইল পপতে আমাতের গ্রুতপ প াখ রাখুন


এবং অবশ্যই ননতের বন্ধুতের ইনভাইট করুন

Written Math Solution (BIBM) FB Group : BD job Info. (BCS & Bank)
90

 Created by Sumon K. Sarkar  FB Group : BD job Info. (BCS & Bank)  Created by Sumon K. Sarkar 
Bank Job Solution (BIBM)
 Created by Sumon K. Sarkar  FB Group : BD job Info. (BCS & Bank)  Created by Sumon K. Sarkar 

3. The perimeter of a square field is equal to the perimeter of a rectangle field. Length of the
rectangle fields is 3 times of its width and the area is 768 square meter. How many square sized
tiles of 80 centimetre wide will be required to cover the square field? [Social Islamic Bank-PO-2017]
Abyev`t GKwU eM©vKvi gv‡Vi cwimxgv GKwU AvqZKvi gv‡Vi cwimxgvi mgvb| AvqZ‡¶ÎwUi •`N©¨ Gi cÖ‡¯’i wZb¸Y Ges Gi
†¶Îdj 768 eM©wgUvi| eM©vKvi gvVwU‡K m¤ú~Y©iƒ‡c XvK‡Z 80 †m.wg KZ¸‡jv eM©vKvi UvBjm jvM‡e?
Solution:
Let, The width of the rectangle be x meter
So, The length of the rectangle be 3x meter
According to the question,
3x × x=768
or, 𝑥 2 = 256
or, x = 16
So,The perimeter of the rectangle = 2(3x + x)
=2(3×16+16)
=128
So, the perimeter of the square is 128 meters
Let, Each Side of square is y
According to the question,
4y=128
128
or, 𝑦 =
4
or, y = 32
So, area of the square be 𝑦 2 = 322
=1024 squar meter
=1024×10000 square cm
=10240000 square cm
Again, area of the tiles = 802
= 6400 square cm
102,40,000
Number of tiles =
6400
=1600 unit
Answer:1600 unit

𝟒 𝟗 𝟐𝟓
4. Solve the equation: + =
𝟐𝒙+𝟏 𝟑𝒙+𝟐 𝟓𝒙+𝟒
Solution:
4 9 25
+ =
2𝑥+1 3𝑥+2 5𝑥+4
4 25 9
⇒ = −
2𝑥+1 5𝑥+4 3𝑥+2
4 75𝑥+50−45𝑥−36
⇒ =
2𝑥+1 (5𝑥+4)(3𝑥+2)
4 30𝑥+14
⇒ =
2𝑥+1 (5𝑥+4)(3𝑥+2)
4 2(15𝑥+7)
⇒ =
2𝑥+1 (5𝑥+4)(3𝑥+2)

Written Math Solution (BIBM) FB Group : BD job Info. (BCS & Bank)
Bank Job Solution (BIBM) 91

Created by Sumon K. Sarkar  FB Group : BD job Info. (BCS & Bank)  Created by Sumon K. Sarkar 
2 15𝑥+7
⇒ =
2𝑥+1 (5𝑥+4)(3𝑥+2)
Created by Sumon K. Sarkar  FB Group : BD job Info. (BCS & Bank)  Created by Sumon K. Sarkar 

⇒ 2(5x + 4)(3x + 2)=( 15x + 7)(2x + 1)


⇒ 30𝑥 2 +44x+16 = 30𝑥 2 +29x+7
⇒ 30𝑥 2 +44x - 30𝑥 2 - 29x = 7 – 16
⇒ 15x = 9
−9 3
⇒𝑥= =−
15 5
𝟑
Answer: 𝒙 = −
𝟓

Standard Bank Ltd.


Trainee Assistant Officer(Cash) Exam Date : 2018
1. A merchant purchased a jacket for Tk. 60 and then determined a selling price that equaled the
purchase price of the jacket plus a markup that was 25 percent of the selling price. During a sale,
the merchant discounted the selling price by 20 percent and sold the jacket. What was the
merchant's gross profit on this sale?
[Standard Bank TAO Cash 18, Dhaka Bank TO 18]
Abyev`t GK e¨emvqx GKwU R¨v‡KU 60 UvKvq µq K‡i Ges weµq g~j¨ wba©viY K‡i †hUvq µqg~j ¨ mv‡_ weµqg~‡j¨I 25%
Av‡Q| weµ‡qi mgq e¨emvqx 20% Qvo w`‡q R¨v‡KUwU weµq K‡i| weµq Gi Dci e¨emvqxi †gvU jvf KZ wQj?
Solution:
Let, Determined a selling price tk. x
According to the question,
⇒ x = 60 + x × 25%
25
⇒ x = 60 + x ×
100
𝑥
⇒ x = 60 +
4
𝑥
⇒ x − = 60
4
4𝑥−𝑥
⇒ = 60
4
3𝑥
⇒ = 60
4
⇒ 3𝑥 = 60 × 4
⇒ 𝑥 = 80
So, Determined a selling price tk. 80
At 20% discount, selling price = 80 - 80×20%
= tk 64
Gross profit = Tk. (64 -60) = Tk. 4
Answer: Tk. 4

Alternative Way,
Let, Cost of a jacket = Tk. 60.
At 25% markup on selling price, So, the marked price is =(60+ 60 of 20%) = Tk. 80.
At 20% discount,
100−20
actual selling price is =80× ( ) = Tk. 64.
100

Written Math Solution (BIBM) FB Group : BD job Info. (BCS & Bank)
92

 Created by Sumon K. Sarkar  FB Group : BD job Info. (BCS & Bank)  Created by Sumon K. Sarkar 
Bank Job Solution (BIBM)
 Created by Sumon K. Sarkar  FB Group : BD job Info. (BCS & Bank)  Created by Sumon K. Sarkar 

So, gross profit on this sale = Tk. (64-60)


= Tk. 4.
Answer: Tk. 4.

2. If 12 candies are sold for Tk. 10 then there is a loss of x%. If 12 candies are sold for Tk. 12 then
there is a profit of x%. What is the value of x?
[Dhaka Bank Cash 18, South East Bank PO 17. Standard Bank TAO Cash 18]
Abyev`t hw` 10 UvKvq 12wU PK‡jU weµq Kiv nq Zvn‡j X% ¶wZ nq| hw` 12 UvKvq 12wU PK‡jU weµq Kiv nq Zvn‡j
X% jvf nq| X Gi gvb KZ?
Solution:
Let, cost price of candies be Tk. 100.
At x% loss, selling price = Tk. (100-x)
If selling price Tk (100-x) then cost price Tk. 100
100×10
If selling price Tk. 10 then cost price Tk.
100−X
Similarly,
At x% profit Selling price = Tk. (100+x)
If selling price Tk.(100+x) then cost price Tk.10
100×12
If selling price Tk. 12 then cost price Tk.
100+X
According to the condition,
100×10 100×12
⇒ =
100−X 100+X
5 6
⇒ =
100−X 100+X
⇒ 500 + 5x = 600 − 6x
⇒ 5x + 6x = 600 − 500
⇒ 11x = 100
⇒ x = 9.09
Answer: 9.09

Standard Bank Ltd.


Post Name: Trainee Assistant Officer Exam Date : 2018
1. In a mixture 60 liters, the ratio of milk and water 2:1. If this ratio is to be 1: 2, then estimate the
quantity of water in liter to be further added in the mixture.
Abyev`t 60 wjUv‡ii GKwU wgkª‡b `ya Avi cvwbi AbycvZ 2:1| GB wgkª‡Yi AbycvZ 1:2 n‡Z n‡j Avi KZ wjUvi AwZwi³ cvwb
†hvM Ki‡Z n‡e?
Solution:
2
Here, Quantity of milk = 60 × litres = 40 litres
3
Quantity of water =(60−40) litres =20 litres
Let,
The quantity of water to be added further be x litres
According to the question,
40 1
=
20+x 2

Written Math Solution (BIBM) FB Group : BD job Info. (BCS & Bank)
Bank Job Solution (BIBM) 93

Created by Sumon K. Sarkar  FB Group : BD job Info. (BCS & Bank)  Created by Sumon K. Sarkar 
⇒ 20+x = 80
Created by Sumon K. Sarkar  FB Group : BD job Info. (BCS & Bank)  Created by Sumon K. Sarkar 

⇒ x = 80 – 20 = 60
∴ Quantity of water to be added = 60 litres
Answer: 60 litres

𝟏
2. If x = 3+22, then the value of √𝒙 − [Standard Bank TAO 18, Janata AEO (RO) 17]
√𝒙
Solution:
Given that, x=3+22=25
1 1 1 25−1 24
So, √𝑥 − = √25 − =5− = =
√𝑥 √25 5 5 5
প্রশ্ন ভূল হতল,
𝟏
If x = 3+2√𝟐, then the value of √𝒙 −
√𝒙
Given that, x = 3+2√2,
1 1
⇒ =
x 3+2√2

1 (3−2√2)
⇒ =
x (3+2√2)(3−2√2)

1 (3−2√2)
⇒ =
x 32 −(2√2)2
1
⇒ = 3 − 2√2
x
1
So, x + = 3 + 2√2 + 3 − 2√2 = 6
x

1 2
(√X − )
√x

2 1 1 2
= (√x) − 2√x. + ( )
√x √x
1
= x+ −2
x

= 6−2
=4
Answer: 4

এ রকম গুরুত্বপূর্ণ ফাইল পপতে আমাতের গ্রুতপ প াখ রাখুন


এবং অবশ্যই ননতের বন্ধুতের ইনভাইট করুন

Written Math Solution (BIBM) FB Group : BD job Info. (BCS & Bank)
94

 Created by Sumon K. Sarkar  FB Group : BD job Info. (BCS & Bank)  Created by Sumon K. Sarkar 
Bank Job Solution (BIBM)
 Created by Sumon K. Sarkar  FB Group : BD job Info. (BCS & Bank)  Created by Sumon K. Sarkar 

Premier Bank Ltd.


Post: Trainee Assistant Officer Exam Date : 2018
𝟏𝟎 𝟏 𝟏𝟖
1. Solve the Equation: + =
𝟐𝒙−𝟓 𝒙+𝟓 𝟑𝒙−𝟓
[SJIB MTO 16, SJIB TSO 18, Premier Bank TJO 18, NRB Bank MTO 19]
Solution:
𝟏𝟎 𝟏 𝟏𝟖
+ =
𝟐𝒙−𝟓 𝒙+𝟓 𝟑𝒙−𝟓
𝟏𝟎 𝟏 𝟏𝟓+𝟑
⇒ + =
𝟐𝒙−𝟓 𝒙+𝟓 𝟑𝒙−𝟓
𝟏𝟎 𝟏 𝟏𝟓 𝟑
⇒ + = +
𝟐𝒙−𝟓 𝒙+𝟓 𝟑𝒙−𝟓 𝟑𝒙−𝟓
𝟏𝟎 𝟏𝟓 𝟑 𝟏
⇒ − = −
𝟐𝒙−𝟓 𝟑𝒙−𝟓 𝟑𝒙−𝟓 𝒙+𝟓
𝟑𝟎𝒙−𝟓𝟎−𝟑𝟎𝒙+𝟕𝟓 𝟑𝒙+𝟏𝟓−𝟑𝒙+𝟓
⇒ =
(𝟐𝒙−𝟓)(𝟑𝒙−𝟓) (𝟑𝒙−𝟓)(𝒙+𝟓)
𝟐𝟓 𝟐𝟎
⇒ =
(𝟐𝒙−𝟓)(𝟑𝒙−𝟓) (𝟑𝒙−𝟓)(𝒙+𝟓)
𝟓 𝟒
⇒ =
(𝟐𝒙−𝟓) (𝒙+𝟓)

⇒ 8x – 20 = 5x +25
⇒ 8x –5x = 25 + 20
⇒ 3x = 45
⇒ x = 15
Answer: x =15

2. Karim and Rahim have equal amount of money. Radha has half of Rahim's money and Amena
has half of Radha's money. If you add one taka with all the money they have, it will be Tk. 100.
How much Rahim has?
[Premier Bank TJO 18, BB AD 04, Standard Bank TAO Cash 16]
Abyev`t Kwig Ges iwn‡gi Kv‡Q mgcwigvb UvKv Av‡Q| iwn‡gi UvKvi A‡a©K ivavi Av‡Q Ges ivavi UvKvi A‡a©K Av‡gbvi Av‡Q|
hw` Zv‡`i mK‡ji UvKvi mgwói m‡½ 1 †hvM K‡ib Zvn‡j 100 UvKv n‡e| iwn‡gi Kv‡Q KZ UvKv Av‡Q?
Solution:
Let, Karim & Rahim everyone has tk. x
𝑥 𝑥
So, Radha has = k and Amena has =
2 4
According to Question,
𝑥 𝑥
x ✛ x ✛ ✛ ✛ 1 = 100
2 4
𝑥 𝑥
⇒ 2x ✛ ✛ = 100 – 1
2 4
8𝑥+2𝑥+𝑥
⇒ = 99
4
11𝑥
⇒ = 99
4
99×4
⇒𝑥 =
11
⇒x = 36
Rahim has tk 36

Written Math Solution (BIBM) FB Group : BD job Info. (BCS & Bank)
Bank Job Solution (BIBM) 95

Created by Sumon K. Sarkar  FB Group : BD job Info. (BCS & Bank)  Created by Sumon K. Sarkar 
Answer: tk 36
Created by Sumon K. Sarkar  FB Group : BD job Info. (BCS & Bank)  Created by Sumon K. Sarkar 

Alternative Way,
Radha : Rahim = 1 : 2 = (1 : 2)×2 = 2 : 4
Amena : Radha = 1 : 2 = (1 : 2)×1 = 1 : 2
So , Radha : Rahim : Amena : Karim ⇒ 2 : 4 : 1 : 4
Let , Radha , Rahim , Amena and Karim have be 2T , 4T , T , 4T
According to Question,
(2T+4T+T+4T) + 1 = 100
⇒ 11T = 99 ⇒ T = 9
Therefore , Rahim has TK. (9×4) = 36
Answer: 36

3. TV was marked by 120% of its cost price. Then shopkeeper sold the TV at 10% discount. After
that his profit was 2400. Find the cost price.
Abyev`t wUwfi Mv‡q weµqg~j¨, µq g~‡j¨i 120% †jLv wQj| Zvici †`vKvb`vi 10% wWmKvD›U G wUwf wewµ Ki‡jb| Gi ci
Zvi 2400 UvKv jvf n‡qwQj| wUwfi µqg~j¨ †ei Kiæb|
Solution:
Let, Cost price of TV be 100x
Mark price = 100x + 100x × 120% = 120x
At 10% discount Selling Price be = 120x – (120x × 10%)
= 120x – 12x
= 108x
Profit = ( 108x – 100x) = 8x
According to Question,
8x = 2400
⇒ x = 300
⇒ 100x = 30, 000
Cost price of TV tk. 30,000
Answer: 30,000

4. A garden was 60 meter long and 20 meter wide. There was 5 meter wide 4 sided path inside the
garden. If it needs 20 taka per square meter to cover with grass, how much it will need to cover
the path with grass?
[Premier Bank TJO 18, SJIB TSO 18]
Abyev`t GKwU evMvb 60 wgUvi j¤^v Ges 20 wgUvi cÖk¯Í wQj| evMv‡bi wfZ‡ii w`‡K Pvicv‡k 5 wgUvi cÖk¯Í iv¯Ív wQj|hw`
evMv‡b Nvm jvMv‡Z cÖwZ eM© wgUv‡i 20 UvKv cÖ‡qvRb nq, Zvn‡j iv¯ÍvwU Nvm jvMv‡Z †gvU KZ UvKvi cÖ‡qvRb n‡e?
Solution:
Given that,
The length of a garden = 60 meter
The width of a garden = 20 meter
Area of a garden including path = 60 × 20 = 1200 square meter
Length of a garden without path = 60 - (5× 2) = 50 meter

Written Math Solution (BIBM) FB Group : BD job Info. (BCS & Bank)
96

 Created by Sumon K. Sarkar  FB Group : BD job Info. (BCS & Bank)  Created by Sumon K. Sarkar 
Bank Job Solution (BIBM)
 Created by Sumon K. Sarkar  FB Group : BD job Info. (BCS & Bank)  Created by Sumon K. Sarkar 

Width of a garden without path = 20 - (5× 2) = 10 meter


Area of a garden without path = 50 × 10 = 500 square meter
Area of the path = 1200 - 500 = 700 square meter
Total Cost = 700 × 20 = tk 14, 000
Answer: tk 14, 000

NRBC Bank Ltd.


Post name: Management Trainee Officer Exam Date : 2018
𝟖 𝟗 𝟕
1. Solve the equation: + =
𝟐𝒙−𝟏 𝟑𝒙−𝟏 𝒙+𝟏
[NRBC Bank MTO 18, National Bank PO 17]
Solution:
𝟖 𝟗 𝟕
+ =
𝟐𝒙−𝟏 𝟑𝒙−𝟏 𝒙+𝟏
𝟖 𝟗 𝟒+𝟑
⇒ + =
𝟐𝒙−𝟏 𝟑𝒙−𝟏 𝒙+𝟏
𝟖 𝟗 𝟒 𝟑
⇒ + = +
𝟐𝒙−𝟏 𝟑𝒙−𝟏 𝒙+𝟏 𝒙+𝟏
𝟖 𝟒 𝟑 𝟗
⇒ − = −
𝟐𝒙−𝟏 𝒙+𝟏 𝒙+𝟏 𝟑𝒙−𝟏
𝟖𝒙+𝟖−𝟖𝒙+𝟒 𝟗𝒙−𝟑−𝟗𝒙−𝟗
⇒ =
(𝟐𝒙−𝟏)(𝒙+𝟏) (𝒙+𝟏)(𝟑𝒙−𝟏)
𝟏𝟐 −𝟏𝟐
⇒ =
(𝟐𝒙−𝟏)(𝒙+𝟏) (𝒙+𝟏)(𝟑𝒙−𝟏)
𝟏 −𝟏
⇒ =
(𝟐𝒙−𝟏) (𝟑𝒙−𝟏)

⇒ 3x -1 = - 2x + 1
⇒ 3x + 2x = 1 + 1
⇒ 5x = 2
⇒ x = 2/5
Answer: 2/5

এ রকম গুরুত্বপূর্ণ ফাইল পপতে আমাতের গ্রুতপ প াখ রাখুন


এবং অবশ্যই ননতের বন্ধুতের ইনভাইট করুন

Written Math Solution (BIBM) FB Group : BD job Info. (BCS & Bank)
Bank Job Solution (BIBM) 97

Created by Sumon K. Sarkar  FB Group : BD job Info. (BCS & Bank)  Created by Sumon K. Sarkar 
2. A, B and C can do a piece of work in 16, 32 and 48 days respectively. They started working
Created by Sumon K. Sarkar  FB Group : BD job Info. (BCS & Bank)  Created by Sumon K. Sarkar 

together but C left after working 4 days and B left 2 days before the completion of work. How
many days it took to complete the work?
[SJIB MTO 11, SIBL PO 17, NRBC Bank MTO 18, National Bank PO 17. Langka Bangla Finance MTO 17]
Abyev`t A, B I C GKwU KvR h_vµ‡g 16, 32 Ges 48 w`‡b Ki‡Z cv‡i| Zviv GKmv‡_ KvR Ki‡Z ïiæ K‡i wKš‘ KvR ïiæ
nIqvi 4 w`‡b ci C KvR †Q‡o †`q Ges KvR †kl nIqvi 2 w`b Av‡M B KvR †Q‡o †`q| m¤ú~Y© KvRwU †kln‡Z KZw`b mgq
jv‡M?
Solution:
Let, the work be done in x days.
A worked for x days, B for (x-2) and C for 4 days.
𝑥 𝑥−2 4
+ + =1
16 32 48
𝑥 𝑥−2 1
⇒ + + =1
16 32 12
6𝑥+3𝑥−6+8
⇒ =1
96

⇒ 9x +2 = 96
⇒ 9x = 96 – 2
⇒ 9x = 94
94
⇒x=
9
4
⇒ x = 10 𝑑𝑎𝑦𝑠
9
𝟒
Answer: 𝟏𝟎 𝒅𝒂𝒚𝒔
𝟗

3. The cost price of two watches taken together is Tk. 840. If by selling one at a profit of 16% and
the other at a loss of 12%, there is no loss or gain in the whole transaction, find the cost price of
the two watches.
[SJIB MTO 11, SIBL PO 17, NRBC Bank MTO 18]
Abyev`t GKm‡½ †Kbv 2wU Nwoi µqg~j¨ GK‡Î 840 UvKv| hw` GwU‡K 16% jv‡f Ges Ab¨wU‡K 12% ¶wZ‡Z weµq Ki‡j
†gv‡Ui Dci †Kvb jvf ev ¶wZ nq bv Zvn‡j Nwo `zBwUi cÖwZwUi µqg~j¨ wbY©q Ki|
Solution:
Let, the cost of one watch = Tk. x
Then cost of second watch = Tk. (840-x)
According to the question,
16% of x =12% of (840-x)
16 12
⇒x× = (840−x) ×
100 100
⇒ 16x = 5760 −12x
⇒ 16x + 12x = 12×840
⇒ 28x =12×840
𝟏𝟐×𝟖𝟒𝟎
⇒x= = 360
𝟐𝟖

Written Math Solution (BIBM) FB Group : BD job Info. (BCS & Bank)
98

 Created by Sumon K. Sarkar  FB Group : BD job Info. (BCS & Bank)  Created by Sumon K. Sarkar 
Bank Job Solution (BIBM)
 Created by Sumon K. Sarkar  FB Group : BD job Info. (BCS & Bank)  Created by Sumon K. Sarkar 

Hence ,cost of one watch Tk. 360


Cost of another watch = Tk. (840 – 360)
=Tk. 480
Answer: Tk. 360 & Tk. 480

4. The perimeter of a square field is equal to the perimeter of a rectangle field. The length of the
rectangle is thrice the width of it and the area is 768 square meters. How many square sized tiles
of 80 centimeters will be required to cover the square filed?
[Al-Arafah MTO 16, National Bank PO 17, NRBC Bank MTO 18, SIBL PO 17, Modhumoti PO 18]
Abyev`t GKwU eM©vKvi gv‡Vi cwimxgv GKwU AvqZKvi gv‡Vi cwimxgvi mgvb| AvqZ‡¶ÎwUi ˆ`N©¨ Gi cÖ‡¯’i wZb¸Y Ges Gi
†¶Îdj 768 eM©wgUvi| eM©vKvi gvVwU‡K m¤ú~Y©iƒ‡c XvK‡Z 80 †m.wg KZ¸‡jv eM©vKvi UvBjm jvM‡e?
Solution:
Let, The width of the rectangle be x meter
So, The length of the rectangle be 3x meter
According to the question,
3x × x=768
or, 𝑥 2 = 256
or, x = 16
So, The perimeter of the rectangle = 2(3x + x)
=2(3×16+16)
=128
So, the perimeter of the square is 128 meters
Let, Each Side of square is y
According to the question,
4y=128
128
or, 𝑦 =
4
or, y = 32
So, area of the square be 𝑦 2 = 322
=1024 squar meter
=1024×10000 square cm
=10240000 square cm
Again, area of the tiles = 802
= 6400 square cm
102,40,000
Number of tiles =
6400
=1600 unit
Answer:1600 unit

এ রকম গুরুত্বপূর্ণ ফাইল পপতে আমাতের গ্রুতপ প াখ রাখুন


এবং অবশ্যই ননতের বন্ধুতের ইনভাইট করুন

Written Math Solution (BIBM) FB Group : BD job Info. (BCS & Bank)
Bank Job Solution (BIBM) 99

Created by Sumon K. Sarkar  FB Group : BD job Info. (BCS & Bank)  Created by Sumon K. Sarkar 
Shahjalal Islami Bank Ltd.
Created by Sumon K. Sarkar  FB Group : BD job Info. (BCS & Bank)  Created by Sumon K. Sarkar 

Post name: Trainee Officer (Cash) Exam Date : 2018

𝟒 𝟗 𝟐𝟓
1. Solve the equation: + =
𝟐𝒙+𝟏 𝟑𝒙+𝟐 𝟓𝒙+𝟒
[Al-Arafah MTO 16, Modhumoti PO 18, SJIB TO Cash 18]
Solution:
4 9 25
+ =
2x+1 3x+2 5x+4
4 9 10+15
⇒ + =
2x+1 3x+2 5x+4
4 9 10+15
⇒ + =
2x+1 3x+2 5x+4
4 9 10 15
⇒ + = +
2x+1 3x+2 5x+4 5x+4
4 10 15 9
⇒ − = −
2x+1 5x+4 5x+4 3x+2
20x+16−20x−10 45x+30−45x−36
⇒ =
(2x+1)(5x+4) (5x+4)(3x+2)
6 −6
⇒ =
(2x+1)(5x+4) (5x+4)(3x+2)
1 −1
⇒ =
(2x+1) (3x+2)
⇒ 3𝑥 + 2 = −2𝑥 − 1
⇒ 3𝑥 + 2𝑥 = −2 − 1
⇒ 5𝑥 = −3
⇒ 𝑥 = −3/5
Answer: 𝒙 = −𝟑/𝟓

2. Train A leaves Dhaka at 4 pm & travels at the rate of 20 miles/h. Train B leaves Dhaka at 9pm
& travels on parallel line at the rate of 30 miles/h. How many hours after starting journey & at
what distance train B will overtake train A?
অনুবােঃ পট্রন A ঢাকা পর্তক নবকাল 4 টায পছতড যায এবং 20 মাইল/ ন্টা পবতগ ভ্রমর্ কতর। পট্রন B ঢাকা পর্তক রাে 9 টায পছতড যায
এবং সমান্তরাল লাইতন 30 মাইল/ ন্টা পবতগ ভ্রমর্ কতর। যাত্রা শুরু করার কে ন্টা পতর এবং কে েূরতত্ব B পট্রন A পক ছানডতয যাতব?
Solution:
From 4 pm to 9 pm, in 5 hours
train A covers = Speed × Time = (5 × 20)
= 100 miles.
Relative speed = (30 – 20) miles/h =10 miles/h.
So, train B needs to cover extra 100 miles to overtake train A.
𝐷𝑖𝑠𝑡𝑎𝑛𝑐𝑒 100
.: Required time = = = 10 hours.
𝑠𝑝𝑒𝑒𝑑 10
In 10 hours, train B goes = (30 ×10) miles
= 300 miles.
Answer: 10 hours and 300 miles.

Written Math Solution (BIBM) FB Group : BD job Info. (BCS & Bank)
100

 Created by Sumon K. Sarkar  FB Group : BD job Info. (BCS & Bank)  Created by Sumon K. Sarkar 
Bank Job Solution (BIBM)
 Created by Sumon K. Sarkar  FB Group : BD job Info. (BCS & Bank)  Created by Sumon K. Sarkar 

3. A sum of Tk 2000 was invested in two parts one at 5% interest rate and another one at 10%
interest rate. If the total annual interest earned was Tk. 160, then how much money was invested
at 10% interest rate?
অনুবােঃ 2000 টাকা দুনট অংতশ্ একনট 5% সুতের হাতর এবং অনযনট 10% সুতের হাতর নবননতযাগ করা হতযনছল। যনে পমাট বানেণক সুে
অনেণে হতো টাকা। 160, োহতল 10% সুতের হাতর কে টাকা নবননতযাগ করা হতযনছল?
Solution:
Let, At 10% interest rate invested Tk. x
At 5% interest rate invested Tk. (2000-x)
According to the question,
𝑥 × 10% + (2000 − 𝑥) × 5% = 160
10 (2000−𝑥)×5
⇒𝑥 × + = 160
100 100
𝑥 (2000−𝑥)
⇒ + = 160
10 20
2𝑥+2000−𝑥
⇒ = 160
20
⇒ 𝑥 + 2000 = 3200
⇒ 𝑥 = 3200 − 2000
⇒ 𝑥 = 1200
At 10% interest rate invested Tk. 1200
Answer: Tk. 1200

4. A fruit seller bought 20 kg mangoes for Tk 100 per kg and an equal amount of apples for Tk.
150 per kg. If he sells all the fruits at Tk. 140 per kg. What will be his profit or loss percentage?
Abyev`t GKRb dj we‡µZv 20 †KwR Avg cÖwZ †KwR 100 UvKv `‡i Ges cÖwZ †KwR 150 UvKv `‡i mgvb msL¨K Av‡cj µq
K‡ib| hw` †m me¸‡jv dj 140 UvKv cÖwZ †KwR wn‡m‡e weµq K‡i Zvn‡j Zvi kZKiv KZ jvf ev ¶wZ n‡e?
Solution:
Let, Cost Price =Tk. ( 100×20 + 150×20)= tk. 5000
Selling Price = 140 × 40 = Tk. 5600
Profit = Tk. (5600-5000) = Tk. 600
600×100
So, percentage of profit = % = 12%
5000
Answer: 12%

এ রকম গুরুত্বপূর্ণ ফাইল পপতে আমাতের গ্রুতপ প াখ রাখুন


এবং অবশ্যই ননতের বন্ধুতের ইনভাইট করুন

Written Math Solution (BIBM) FB Group : BD job Info. (BCS & Bank)
Bank Job Solution (BIBM) 101

Created by Sumon K. Sarkar  FB Group : BD job Info. (BCS & Bank)  Created by Sumon K. Sarkar 
Shahjalal Islami Bank Ltd.
Created by Sumon K. Sarkar  FB Group : BD job Info. (BCS & Bank)  Created by Sumon K. Sarkar 

Post Name: Trainee Officer Exam Date : 2018

𝟏𝟎 𝟏 𝟏𝟖
1. Solve the equation: + =
𝟐𝒙−𝟓 𝒙+𝟓 𝟑𝒙−𝟓
[SJIB MTO 16, SJIB TSO 18, Premier Bank TJO 18, NRB Bank MTO 19]
Solution:
10 1 18
+ =
2𝑥−5 𝑥+5 3𝑥−5
10 1 15+3
⇒ + 𝑥+5 = 3𝑥−5
2𝑥−5
10 1 15 3
⇒ + 𝑥+5 = 3𝑥−5 + 3𝑥−5
2𝑥−5
10 15 3 1
⇒ − = −
2𝑥−5 3𝑥−5 3𝑥−5 𝑥+5
30𝑥−50−30𝑥+75 3𝑥+15−3𝑥+5
⇒ =
(2𝑥−5)(3𝑥−5) (3𝑥−5)(𝑥+5)
25 20
⇒ = (3𝑥−5)(𝑥+5)
(2𝑥−5)(3𝑥−5)
5 4
⇒ = (𝑥+5)
(2𝑥−5)
⇒ 8x – 20 = 5x +25
⇒ 8x –5x = 25 + 20
⇒ 3x = 45
⇒ x = 15
Answer: x =15

5. A team of 2 men and 5 women completed 1/4 th of a job in 3 days. After 3 days, another man
joined the team and they took 2 days to complete another 1/4 th of the job. How many men can
complete the remaining job in 4 days?
[BB AD 14, PKSF AM 09, NCC Bank Officer 02, One Bank SCO 18, NRB Bank MTO 19]
Abyev`t 2 Rb cyiæl Ges 5 Rb gwnjvi GKwU `j 3 w`‡b GKwU Kv‡Ri 1/4 Ask m¤c~Y© K‡i| 3w`b ci GKRb AwZwi³ cyiæl
Kv‡R †hvM w`‡j Zviv 2 w`‡b AviI 1/4 Ask m¤cbœ K‡ib| KZRb cyiæl wg‡j evwK KvR 4 w`‡b m¤cbœ Ki‡Z cvi‡eb?
Solution:
Let, Man and Women be m & w
Work rate of Man and Women,
2 5 1
+ =
𝑚 𝑤 4×3
2 5 1
⇒ + = ....................... (i)
𝑚 𝑤 12
3 5 1
+ =
𝑚 𝑤 4×2
3 5 1
⇒ + = ............................... (ii)
𝑚 𝑤 8
Subtract equation (i) from equation (ii)
3 2 1 1
⇒ − = −
𝑚 𝑚 8 12

Written Math Solution (BIBM) FB Group : BD job Info. (BCS & Bank)
102

 Created by Sumon K. Sarkar  FB Group : BD job Info. (BCS & Bank)  Created by Sumon K. Sarkar 
Bank Job Solution (BIBM)
 Created by Sumon K. Sarkar  FB Group : BD job Info. (BCS & Bank)  Created by Sumon K. Sarkar 

1 3−2
⇒ =
𝑚 24
⇒ m = 24
It means one man needs 24 days to complete the work so work completed in 4 days ,
24
we need = 6 men.
4

Alternative Way,
A team of 2 men and 5 women can complete a job in 4 × 3 = 12 days
A team of 2 + 1 = 3 men and 5 women can complete the whole thing in 4 × 2 = 8 days
A man can do the same amount of work in 8 days as a team of 2 men and 5 women can in 12 - 8 = 4
days
12
Therefore, a man can complete a job (solo) in 8 × = 24 days
4
24
A job can be completed within 4 days by = 6 men.
4
Answer: 6 men.

6. Mr. B invests Tk. 2400 at 5% interest annually. How much additional money needs to invest at
8% interest to earn overall interest at 6% on entire amount?
[UCBL Officer 11, Modhumoti PO 18, SJIB TSO 18]
Abyev`t B mv‡ne evwl©K kZKiv 5 UvKv mij gybvdvq 2400 UvKv wewb‡qvM K‡ib| evwl©K kZKiv 8 UvKv mij gybvdvqAvi KZ
UvKv wewb‡qvM Ki‡j wZwb †gvU wewb‡qv‡Mi Dci GK‡Î evwl©K kZKiv 6 UvKv mij gybvdv cv‡eb?
Solution:
Let, Mr B Additional money invested be tk. x
According to the question,
8% of x + 5% of 2400 = 6%(x+2400)
8𝑥 5×2400 6(𝑥+2400)
⇒ + =
100 100 100
⇒ 8x+12000 = 6x+14400
⇒ 8x - 6x = 14400 – 12000
⇒ 2x = 2400
⇒ x = 1200
Mr B Additional money invest is tk. 1200
Answer: tk. 1200

এ রকম গুরুত্বপূর্ণ ফাইল পপতে আমাতের গ্রুতপ প াখ রাখুন


এবং অবশ্যই ননতের বন্ধুতের ইনভাইট করুন

Written Math Solution (BIBM) FB Group : BD job Info. (BCS & Bank)
Bank Job Solution (BIBM) 103

Created by Sumon K. Sarkar  FB Group : BD job Info. (BCS & Bank)  Created by Sumon K. Sarkar 
7. A garden was 60 meter long and 20 meter wide. There was 5 meter wide 4 sided path inside the
Created by Sumon K. Sarkar  FB Group : BD job Info. (BCS & Bank)  Created by Sumon K. Sarkar 

garden. If it needs 20 taka per square meter to cover with grass, how much it will need to cover
the path with grass?
[Premier Bank TJO 18, SJIB TSO 18]
Abyev`t GKwU evMvb 60 wgUvi j¤^v Ges 20 wgUvi cÖk¯Í wQj| evMv‡bi wfZ‡ii w`‡K Pvicv‡k 5 wgUvi cÖk¯Í iv¯Ív wQj|hw`
evMv‡b Nvm jvMv‡Z cÖwZ eM© wgUv‡i 20 UvKv cÖ‡qvRb nq, Zvn‡j iv¯ÍvwU Nvm jvMv‡Z †gvU KZ UvKvi cÖ‡qvRb n‡e?
Solution:
Given that,
The length of a garden = 60 meter
The width of a garden = 20 meter
Area of a garden including path = 60 × 20 = 1200 square meter
Length of a garden without path = 60 - (5× 2) = 50 meter
Width of a garden without path = 20 - (5× 2) = 10 meter
Area of a garden without path = 50 × 10 = 500 square meter
Area of the path = 1200 - 500 = 700 square meter
Total Cost = 700 × 20 = tk 14, 000
Answer: tk 14, 000

One Bank Ltd.


Post Name: Special Cadre Officer Exam Date : 2018
1. A sum of money is to be distributed equally among a group of children. If there were 25 children
less than each would get Tk. 1.50 more, and if there 50 children more, each would get TK. 1.50
less. Find the number of children and the amount of money distributed.
[One Bank SCO 18, National Bank PO 15]
Abyev`t KZ¸‡jv wkïi g‡a¨ wKQz UvKv mgvbfv‡e fvM K‡i †`qv nq| hw` 25wU wkï Kg _vKZ Zvn‡j cÖ‡Z¨‡K 1.50 UvKv †ewk
†cZ Ges hw` 50wU wkï †ewk _vKZ Zvn‡j cÖ‡Z¨‡K 1.50 UvKv Kg †cZ| KZwU wkï wQj Ges KZ UvKv †`qv n‡q‡Q, Zv wbY©q
Ki|
Solution:
Let, Number of Children be x
Each Children get y
The amount of money distributed xy
According to the 1st condition,
(x-25)(y+1.5) = xy
⇒ xy+1.5x -25y-37.5=xy
⇒ 1.5x - 25y - 37.5 = 0
⇒ 3x - 50y – 75 = 0 ………………….(1)
According to the 2nd condition,
(x+50)(y-1.5)= xy
⇒ xy - 1.5x + 50y – 75 = xy
⇒ -3x + 100y – 150 = 0 ………………….(2)
Now, adding the equation (i) & (ii)
50y – 225 = 0
⇒ y = 225/50

Written Math Solution (BIBM) FB Group : BD job Info. (BCS & Bank)
104

 Created by Sumon K. Sarkar  FB Group : BD job Info. (BCS & Bank)  Created by Sumon K. Sarkar 
Bank Job Solution (BIBM)
 Created by Sumon K. Sarkar  FB Group : BD job Info. (BCS & Bank)  Created by Sumon K. Sarkar 

⇒ y = 4.5
Put the value of y in equation (1)
3x - 50×4.5 – 75 = 0
⇒ 3x – 225 – 75 = 0
⇒ 3x – 300 = 0
⇒ 3x = 300
⇒ x = 100
No. of Children 100
The amount of money distributed = Tk. (100 × 4.5)
= Tk. 450
Answer: Tk. 450

2. A team of 2 men and 5 women completed 1/4th of a job in 3 days. After that another man joined
them and they all complete the next 1/4 th of the job in 2days. How many men can complete the
whole job in 4 days?
[BB AD 14, PKSF AM 09, NCC Bank Officer 02, One Bank SCO 18, NRB Bank MTO 19]
Abyev`t 2 Rb cyiæl Ges 5 Rb gwnjvi GKwU `j 3 w`‡b GKwU Kv‡Ri 1/4 Ask m¤c~Y© K‡i| 3w`b ci GKRb AwZwi³ cyiæl
Kv‡R †hvM w`‡j Zviv 2 w`‡b AviI 1/4 Ask m¤cbœ K‡ib| KZRb cyiæl wg‡j evwK KvR 4 w`‡b m¤cbœ Ki‡Z cvi‡eb?
Solution:
Let, Man and Women be m & w
Work rate of Man and Women,
2 5 1
+ =
𝑚 𝑤 4×3
2 5 1
⇒ + = ....................... (i)
𝑚 𝑤 12
3 5 1
+ =
𝑚 𝑤 4×2
3 5 1
⇒ + = ............................... (ii)
𝑚 𝑤 8
Subtract equation (i) from equation (ii)
3 2 1 1
⇒ − = −
𝑚 𝑚 8 12
1 3−2
⇒ =
𝑚 24
⇒ m = 24
It means one man needs 24 days to complete the work so work completed in 4 days ,
24
we need = 6 men.
4

Alternative Way,
A team of 2 men and 5 women can complete a job in 4 × 3 = 12 days
A team of 2 + 1 = 3 men and 5 women can complete the whole thing in 4 × 2 = 8 days
A man can do the same amount of work in 8 days as a team of 2 men and 5 women can in 12 - 8 = 4 days
12
Therefore, a man can complete a job (solo) in 8 × = 24 days
4
24
A job can be completed within 4 days by = 6 men.
4
Answer: 6 men.

Written Math Solution (BIBM) FB Group : BD job Info. (BCS & Bank)
Bank Job Solution (BIBM) 105

Created by Sumon K. Sarkar  FB Group : BD job Info. (BCS & Bank)  Created by Sumon K. Sarkar 
3. In 2017, the number of product that a company sold to retailers decreased by 20%, while the
Created by Sumon K. Sarkar  FB Group : BD job Info. (BCS & Bank)  Created by Sumon K. Sarkar 

price per unit increased by 20% from that of previous year. Company's revenue from the sales
of the product in 2017 was Tk. 6,00,000. Find out the difference between the sales revenue of the
year 2017 and 2016.
Abyev`t 2017 mv‡j LyPiv we‡µZv‡`i Kv‡Q wewµZ c‡Y¨i cwigvY 20% K‡g hvq, ‡hLv‡b weµqg~j¨ c~‡e©i eQ‡ii Zzjbvq 20%
e„w× cvq| cY¨ weµq n‡Z †Kv¤cvwb KZ©…K †gvU weµq ivR‡¯^i cwigvY 600000 UvKv wQj| 2016 I 2017 mv‡ji weµq
ivR‡¯^i g‡a¨ cv_©K¨ wbY©q Kiæb|
Solution:
Let, In 2016 the company sold the number of products be 100 units at the rate of Tk. 100x per unit.
In 2016, total revenue = Tk. (100×100x) = Tk. 10000x
In 2017, the number of products = 80% of 100
= 80 units.
Price per unit = 120% of 100x = Tk. 120x
Total revenue = Tk. (80×120x) = Tk. 9600x
Total revenue decreased = Tk.(10000x- 9600x)
= Tk. 400x
According to the question,
9600x = 6,00,000
6,00,000×400
∴ 400x = = Tk. 25,000
9600
The difference between the sales revenue of the year 2017 and 2016 is 25,000.
Answer: 25,000

𝒙+𝟑 𝒙+𝟒 𝒙+𝟓 𝒙+𝟔


4. Solve the problem: − = −
𝒙+𝟐 𝒙+𝟑 𝒙+𝟒 𝒙+𝟓
Solution:
𝑥+3 𝑥+4 𝑥+5 𝑥+6
− = −
𝑥+2 𝑥+3 𝑥+4 𝑥+5
𝑥+2+1 𝑥+3+1 𝑥+4+1 𝑥+5+1
⇒ − = −
𝑥+2 𝑥+3 𝑥+4 𝑥+5
𝑥+2 1 𝑥+3 1 𝑥+4 1 𝑥+5 1
⇒ + − − = + − −
𝑥+2 𝑥+2 𝑥+3 𝑥+3 𝑥+4 𝑥+4 𝑥+5 𝑥+5
1 1 1 1
⇒1 + −1− =1+ −1−
𝑥+2 𝑥+3 𝑥+4 𝑥+5
1 1 1 1
⇒ − = −
𝑥+2 𝑥+3 𝑥+4 𝑥+5
𝑥+3−𝑥−2 𝑥+5−𝑥−4
⇒ =
(𝑥+2)(𝑥+3) (𝑥+4)(𝑥+5)
1 1
⇒ =
(𝑥+2)(𝑥+3) (𝑥+4)(𝑥+5)

⇒ (𝑥 + 4)(𝑥 + 5) = (𝑥 + 2)(𝑥 + 3)
⇒𝑥 2 + 9𝑥 + 20 = 𝑥 2 + 5𝑥 + 6
⇒𝑥 2 + 9𝑥 + 20 − 𝑥 2 − 5𝑥 − 6 = 0
⇒ 4x + 14 = 0

Written Math Solution (BIBM) FB Group : BD job Info. (BCS & Bank)
106

 Created by Sumon K. Sarkar  FB Group : BD job Info. (BCS & Bank)  Created by Sumon K. Sarkar 
Bank Job Solution (BIBM)
 Created by Sumon K. Sarkar  FB Group : BD job Info. (BCS & Bank)  Created by Sumon K. Sarkar 

⇒ 4x = - 14
⇒ x = - 14/4
⇒ x = - 7/2
Answer: -7/2

Lanka Bangla Finance


Post Name: Management Trainee Officer Exam Date : 2017
𝟏 𝟏 𝟏 𝟏
1. Solve the problem: + = + [SIBL PO 17, Langka Bangla Finance MTO 17 Board Book]
𝒙+𝟐 𝒙+𝟓 𝒙+𝟒 𝒙+𝟑
Solution:
𝟏 𝟏 𝟏 𝟏
+ = +
𝒙+𝟐 𝒙+𝟓 𝒙+𝟒 𝒙+𝟑
𝟏 𝟏 𝟏 𝟏
⇒ − = −
𝒙+𝟐 𝒙+𝟒 𝒙+𝟑 𝒙+𝟓
𝒙+𝟒−𝒙−𝟐 𝒙+𝟓−𝒙−𝟑
⇒ =
(𝒙+𝟐)(𝒙+𝟒) (𝒙+𝟑)(𝒙+𝟓)
𝟐 𝟐
⇒ =
(𝒙+𝟐)(𝒙+𝟒) (𝒙+𝟑)(𝒙+𝟓)
𝟏 𝟏
⇒ =
(𝒙+𝟐)(𝒙+𝟒) (𝒙+𝟑)(𝒙+𝟓)

⇒ (𝑥 + 3)(𝑥 + 5) = (𝑥 + 2)(𝑥 + 4)
⇒ 𝑥 2 + 8𝑥 + 15 = 𝑥 2 + 6𝑥 + 8
⇒ 𝑥 2 + 8𝑥 + 15 − 𝑥 2 − 6𝑥 − 8 = 0
⇒ 2x + 7=0
⇒ 2x = - 7
⇒ x = - 7/2
Answer: -7/2

এ রকম গুরুত্বপূর্ণ ফাইল পপতে আমাতের গ্রুতপ প াখ রাখুন


এবং অবশ্যই ননতের বন্ধুতের ইনভাইট করুন

Written Math Solution (BIBM) FB Group : BD job Info. (BCS & Bank)
Bank Job Solution (BIBM) 107

Created by Sumon K. Sarkar  FB Group : BD job Info. (BCS & Bank)  Created by Sumon K. Sarkar 
2. A, B and C can do a piece of work in 16, 32 and 48 days respectively. They started working
Created by Sumon K. Sarkar  FB Group : BD job Info. (BCS & Bank)  Created by Sumon K. Sarkar 

together but C left after working 4 days and B left 2 days before the completion of work. How
many days it took to complete the work?
[SJIB MTO 11, SIBL PO 17, NRBC Bank MTO 18, National Bank PO 17, Langka Bangla Finance MTO 17]
Abyev`t A, B I C GKwU KvR h_vµ‡g 16, 32 Ges 48 w`‡b Ki‡Z cv‡i| Zviv GKmv‡_ KvR Ki‡Z ïiæ K‡i wKš‘ KvR ïiæ
nIqvi 4 w`‡b ci C KvR †Q‡o †`q Ges KvR †kl nIqvi 2 w`b Av‡M B KvR †Q‡o †`q| m¤ú~Y© KvRwU †kln‡Z KZw`b mgq
jv‡M?
Solution:
Let, the work be done in x days.
A worked for x days, B for (x-2) and C for 4 days.
𝑥 𝑥−2 4
+ + =1
16 32 48
𝑥 𝑥−2 1
⇒ + + =1
16 32 12
6𝑥+3𝑥−6+8
⇒ =1
96
⇒ 9x +2 = 96
⇒ 9x = 96 – 2
⇒ 9x = 94
94
⇒x=
9
4
⇒ x = 10 𝑑𝑎𝑦𝑠
9
𝟒
Answer: 𝟏𝟎 𝒅𝒂𝒚𝒔
𝟗

3. The length of a rectangular field is 1.5 times as large as its width. An amount of Tk. 10260 was
needed to cover the field with grass at 1.9Tk per sq. meter. How much would it cost to fence the
four sides of the rectangular field at the rate of Tk 2.5 per meter?
[ONE Bank SCO 17, Bank Asin MTO 17, SJIB MTO 16, Langka Bangla Finance MTO 17 Board Book]
Abyev`t GKwU AvqZKvi gv‡Vi ˆ`N¨© Gi cÖ‡¯’i †`o ¸Y| cÖwZ eM©wgUv‡i 1.9 UvKv wnmv‡e m¤ú~Y© gvVwU‡Z Nvm jvMv‡Z 10,260
UvKv cÖ‡qvRb| cÖwZ wgUvi 2.5 UvKv wnmv‡e gvVwUi Pvicv‡k †eov w`‡Z KZ UvKv LiP n‡e?
Solution:
Let, The width of the rectangular field be x meter
and length of the rectangular field = 1.5x meter
According to the question,
1.5x × x ×1.9= 10260 [ Total Cost ]
10,260
⇒ 𝑥2 =
1.5×1.9
⇒ 𝑥 2 = 3600
⇒ x = 60
The width of the rectangular field 60 meter
& length of the rectangular field = 1.5×60 = 90 meter
Perimeter of the rectangular field = 2(length + width)
= 2(60 + 90) meter
= 350 meter
So, total Cost to fence = 350×2.5 = Tk 750 Answer: Tk. 750

Written Math Solution (BIBM) FB Group : BD job Info. (BCS & Bank)
108

 Created by Sumon K. Sarkar  FB Group : BD job Info. (BCS & Bank)  Created by Sumon K. Sarkar 
Bank Job Solution (BIBM)
 Created by Sumon K. Sarkar  FB Group : BD job Info. (BCS & Bank)  Created by Sumon K. Sarkar 

4. A man has to go 10 km to catch a bus. He walks part of the way at 7 km per hour and runs the
rest of the way at 12 km per hour. If he takes 1 hour 15 minutes to complete his journey. Find
how far he walked?
[Modhumoti Bank MTO 16, ONE Bank SCO 17, Langka Bangla Finance MTO 17]
Abyev`t †Kvb e¨w³‡K evm aivi Rb¨ 10 wK.wg †h‡Z nq| c‡_i wKQz Ask wZwb 7 wK.wg/N›Uv †e‡M †nu‡U Ges evwK Ask
12wK.wg/N›Uv †e‡M †`Š‡o cvi K‡ib| hw` GB c_ †h‡Z Zvi †gvU 1 N›Uv 15 wgwbU mgq jv‡M Zvn‡j wZwb KZUzKz c_ nuv‡Ub Zv
wbY©q Ki|
Solution:
Let, A man walks x km and run (10 – x) km
15
Here, 1 hour 15 minutes = 1 hour + hour
60
15
= (1 + ) hour
60
60+15
=( ) hour
60
75
= hour
60
According to the question
𝑥 10−𝑥 75
+ =
7 12 60
12𝑥 + 70 – 7𝑥 5
⇒ =
84 4
5𝑥 + 70 5
⇒ =
84 4
5(𝑥 + 14) 5
⇒ =
84 4
𝑥 + 14
⇒ =1
21

⇒ 𝑥 + 14 = 21
⇒ 𝑥 = 21 – 14
⇒𝑥=7
A man walks 7 km
Answer : 7 km

এ রকম গুরুত্বপূর্ণ ফাইল পপতে আমাতের গ্রুতপ প াখ রাখুন


এবং অবশ্যই ননতের বন্ধুতের ইনভাইট করুন

Written Math Solution (BIBM) FB Group : BD job Info. (BCS & Bank)
Bank Job Solution (BIBM) 109

Created by Sumon K. Sarkar  FB Group : BD job Info. (BCS & Bank)  Created by Sumon K. Sarkar 
National Bank Ltd.
Created by Sumon K. Sarkar  FB Group : BD job Info. (BCS & Bank)  Created by Sumon K. Sarkar 

Post Name: Probationary Officer Exam Date : 2017


𝟖 𝟗 𝟕
1. Solve the problem: + =
𝟐𝒙−𝟏 𝟑𝒙−𝟏 𝒙+𝟏
[NRBC Bank MTO 18, National Bank PO 17]
Solution:
𝟖 𝟗 𝟕
+ =
𝟐𝒙−𝟏 𝟑𝒙−𝟏 𝒙+𝟏
𝟖 𝟗 𝟒+𝟑
⇒ + =
𝟐𝒙−𝟏 𝟑𝒙−𝟏 𝒙+𝟏
𝟖 𝟗 𝟒 𝟑
⇒ + = +
𝟐𝒙−𝟏 𝟑𝒙−𝟏 𝒙+𝟏 𝒙+𝟏
𝟖 𝟒 𝟑 𝟗
⇒ − = −
𝟐𝒙−𝟏 𝒙+𝟏 𝒙+𝟏 𝟑𝒙−𝟏
𝟖𝒙+𝟖−𝟖𝒙+𝟒 𝟗𝒙−𝟑−𝟗𝒙−𝟗
⇒ =
(𝟐𝒙−𝟏)(𝒙+𝟏) (𝒙+𝟏)(𝟑𝒙−𝟏)
𝟏𝟐 −𝟏𝟐
⇒ =
(𝟐𝒙−𝟏)(𝒙+𝟏) (𝒙+𝟏)(𝟑𝒙−𝟏)
𝟏 −𝟏
⇒ =
(𝟐𝒙−𝟏) (𝟑𝒙−𝟏)

⇒ 3x -1 = - 2x + 1
⇒ 3x + 2x = 1 + 1
⇒ 5x = 2
⇒ x = 2/5
Answer: 2/5

2. Aman, Belal and Chad started a small business with a total amount tk 28,000. Aman paid tk 4500
more than Belal and Belal paid tk 7000 less than Chad. If the company made a profit of tk 5600
.How much profit should Belal receive?
[SUBB MTO 16, National Bank PO 17]
Abyev`t Avgvb, ‡ejvj I Pvu` †gvU 28000 UvKv wb‡q GKwU e¨emv ïiæ K‡i| ‡ejvj hZ UvKv †`q Avgvb Zvi †P‡q 4500 UvKv
†ewk †`q Ges Pvu` hZ UvKv †`q ‡ejvj Zvi †P‡q 7000 UvKv Kg †`q| hw` Zv‡`i †Kv¤úvwbi 5600 UvKv jvf nq Zvn‡j ‡ejvj
KZ UvKv jf¨vsk cv‡e?
Solution:
Let, The amount of Belal be x tk
The amount of Aman be (x+4500) tk
The amount of Chad be (x+7000) tk
According to the question,
x+x+4500+x+7000 = 28,000
⇒ 3x + 11,500 =28,000
⇒ 3x =28,000 – 11,500
⇒ 3x =16,500
⇒ x = 5500

Written Math Solution (BIBM) FB Group : BD job Info. (BCS & Bank)
110

 Created by Sumon K. Sarkar  FB Group : BD job Info. (BCS & Bank)  Created by Sumon K. Sarkar 
Bank Job Solution (BIBM)
 Created by Sumon K. Sarkar  FB Group : BD job Info. (BCS & Bank)  Created by Sumon K. Sarkar 

The amount of Belal be tk 5,500


The amount of Aman be tk. (5500+4500) = tk 10,000
The amount of Chad be tk. ( 5500 + 7000) = tk. 12,500
Ratio of investment = Aman:Belal:Chad =10,000 : 5500 : 12500
=20 : 11 : 25
Sum of ratio = 20 + 11 + 25 = 56
Total profit =5600 tk
11
Belal should receive profit = 5600× = Tk. 1,100
56
Belal should receive profit Tk. 1100
Answer: Tk. 1100

3. A, B and C can do a piece of work in 16, 32 and 48 days respectively. They started working
together but C left after working 4 days and B left 2 days before the completion of work. How
many days it took to complete the work?
[SJIB MTO 11, SIBL PO 17, NRBC Bank MTO 18, National Bank PO 17. Langka Bangla Finance MTO 17]
Abyev`t A, B I C GKwU KvR h_vµ‡g 16, 32 Ges 48 w`‡b Ki‡Z cv‡i| Zviv GKmv‡_ KvR Ki‡Z ïiæ K‡i wKš‘ KvR ïiæ
nIqvi 4 w`‡b ci C KvR †Q‡o †`q Ges KvR †kl nIqvi 2 w`b Av‡M B KvR †Q‡o †`q| m¤ú~Y© KvRwU †kln‡Z KZw`b mgq
jv‡M?
Solution:
Let, the work be done in x days.
A worked for x days, B for (x-2) and C for 4 days.
𝑥 𝑥−2 4
+ + =1
16 32 48
𝑥 𝑥−2 1
⇒ + + =1
16 32 12
6𝑥+3𝑥−6+8
⇒ =1
96
⇒ 9x +2 = 96
⇒ 9x = 96 – 2
⇒ 9x = 94
94
⇒x=
9
4
⇒ x = 10 𝑑𝑎𝑦𝑠
9
𝟒
Answer: 𝟏𝟎 𝒅𝒂𝒚𝒔
𝟗

এ রকম গুরুত্বপূর্ণ ফাইল পপতে আমাতের গ্রুতপ প াখ রাখুন


এবং অবশ্যই ননতের বন্ধুতের ইনভাইট করুন

Written Math Solution (BIBM) FB Group : BD job Info. (BCS & Bank)
Bank Job Solution (BIBM) 111

Created by Sumon K. Sarkar  FB Group : BD job Info. (BCS & Bank)  Created by Sumon K. Sarkar 
4. The perimeter of a square field is equal to the perimeter of a rectangle field. The length of the
Created by Sumon K. Sarkar  FB Group : BD job Info. (BCS & Bank)  Created by Sumon K. Sarkar 

rectangle is thrice the width of it and the area is 768 square meters. How many square sized tiles
of 80 centimeters will be required to cover the square filed?
[Al-Arafah MTO 16, National Bank PO 17, NRBC Bank MTO 18, SIBL PO 17, Modhumoti PO 18]
Abyev`t GKwU eM©vKvi gv‡Vi cwimxgv GKwU AvqZKvi gv‡Vi cwimxgvi mgvb| AvqZ‡¶ÎwUi ˆ`N©¨ Gi cÖ‡¯’i wZb¸Y Ges Gi
†¶Îdj 768 eM©wgUvi| eM©vKvi gvVwU‡K m¤ú~Y©iƒ‡c XvK‡Z 80 †m.wg KZ¸‡jv eM©vKvi UvBjm jvM‡e?
Solution:
Let, The width of the rectangle be x meter
So, The length of the rectangle be 3x meter
According to the question,
3x × x=768
or, 𝑥 2 = 256
or, x = 16
So,The perimeter of the rectangle = 2(3x + x)
=2(3×16+16)
=128
So, the perimeter of the square is 128 meters
Let, Each Side of square is y
According to the question,
4y=128
128
or, 𝑦 =
4
or, y = 32
So, area of the square be 𝑦 2 = 322
=1024 squar meter
=1024×10000 square cm
=10240000 square cm
Again, area of the tiles = 802
= 6400 square cm
102,40,000
Number of tiles =
6400
=1600 unit
Answer:1600 unit

এ রকম গুরুত্বপূর্ণ ফাইল পপতে আমাতের গ্রুতপ প াখ রাখুন


এবং অবশ্যই ননতের বন্ধুতের ইনভাইট করুন

Written Math Solution (BIBM) FB Group : BD job Info. (BCS & Bank)
112

 Created by Sumon K. Sarkar  FB Group : BD job Info. (BCS & Bank)  Created by Sumon K. Sarkar 
Bank Job Solution (BIBM)
 Created by Sumon K. Sarkar  FB Group : BD job Info. (BCS & Bank)  Created by Sumon K. Sarkar 

Bank Asia Ltd.


Post name: Management Trainee Officer Exam Date : 2017

1. Mr. Karim borrowed Tk. 500 at 5% simple interest per year. After some time, he borrowed Tk.
𝟏
400 at 3 % simple interest per year for the second time. Six months after the second time
𝟐
borrowing, he repaid both borrowed money along with interest and the amount repaid was Tk.
994.50. How many years after the first time borrowing Mr. Karim repaid the borrowed money?
[Bank Asia MTO 17, NRB Bank MTO 19]
Abyev`t wgt Kwig evwl©K 5% mij my‡` 500 UvKv FY †bb | wKQz mgq ci wZwb evwl©K 3% mij my‡` 400 UvKv FY †bb|
wØZxqevi FYMÖn‡Yi 6 gvm ci wZwb my`vm‡j Dfq FY †kva K‡ib| wZwb †gvU 994.50 UvKv †kva K‡ib cÖ_gevi FYMÖn‡Yi KZ
eQi ci wZwb FY‡kva K‡ib?
Solution:
Let, Mr. Karim repaid x years after the first time borrowing.
According to the Question,
6 1
500 × 𝑥 × 5% + 400 × × 3 % = 994.5 − (500 + 400)
12 2
5 1 7
⇒ 500 × 𝑥 × + 400 × × % = 994.5 − 900
100 2 2
5 1 7
⇒ 500 × 𝑥 × + 400 × × = 94.5
100 2 200
⇒ 25𝑥 + 7 = 94.5
⇒ 25𝑥 = 94.5 − 7
⇒ 25𝑥 = 87.5
⇒ 𝑥 = 3.5
Mr. Karim repaid 3.5 years after the first time borrowing.
Answer : 3.5 years

2. Rahim can do a piece of work in 80 days. He works for 10 days and then Karim alone finishes
the rest of the work in 42 days. How much time would it take for the two of them together to
complete the whole work?
[BB AD 06, Modlumoti Bank MTO 16, Bank Asia MTO 17]
Abyev`t Avwei GKwU KvR 80 w`‡b Ki‡Z cv‡i| †m 10 w`b KvR Kivi ci evkvi GKvB evwK KvR 42 w`‡b †kl K‡i| Zviv
`yÕR‡b GKmv‡_ KvR Ki‡j m¤ú~b© KvRwU Ki‡Z Zv‡`i KZw`b mgq jvM‡e?
Solution:
Rahim can do a piece of work in 80 days
In 80 Days, Rahim can do 1 portion
1
In 1 Days, Rahim can do portion
80
1×10
In 10 Days, Rahim can do portion
80
1
= portion
8
1 8−1 7
Rest of the work = 1 − = = portion
8 8 8
Again,
7
In 42 Days, Karim can do portion
8

Written Math Solution (BIBM) FB Group : BD job Info. (BCS & Bank)
Bank Job Solution (BIBM) 113

Created by Sumon K. Sarkar  FB Group : BD job Info. (BCS & Bank)  Created by Sumon K. Sarkar 
7 1
In 1 Days, Karim can do = portion
8×42 48
Created by Sumon K. Sarkar  FB Group : BD job Info. (BCS & Bank)  Created by Sumon K. Sarkar 

In 1 Days, Rahim & Karim together can do


1 1
+ portion
40 48
3+5
= portion
240
8
= portion
240
1
= portion
30
1
So, they can do portion works in 1 day
30
30
They can do 1 part works in = 30 days
1
Answer: 30 Days

3. The length of a rectangular field is 1.5 times as large as its width. An amount of Tk. 10260 was
needed to cover the field with grass at 1.9Tk per sq. meter. How much would it cost to fence the
four sides of the rectangular field at the rate of Tk. 2.5 per meter?
[ONE Bank SCO 17, Bank Asia MTO 17, SJIB MTO 16, Langka Bangla Finance MTO 17 Board Book]
Abyev`t GKwU AvqZKvi gv‡Vi ˆ`N¨© Gi cÖ‡¯’i †`o ¸Y| cÖwZ eM©wgUv‡i 1.9 UvKv wnmv‡e m¤ú~Y© gvVwU‡Z Nvm jvMv‡Z 10,260
UvKv cÖ‡qvRb| cÖwZ wgUvi 2.5 UvKv wnmv‡e gvVwUi Pvicv‡k †eov w`‡Z KZ UvKv LiP n‡e?
Solution:
Let, The width of the rectangular field be x meter
and length of the rectangular field = 1.5x meter
According to the question,
1.5x × x ×1.9= 10260 [ Total Cost ]
10,260
⇒ 𝑥2 =
1.5×1.9
⇒ 𝑥 2 = 3600
⇒ x = 60
The width of the rectangular field 60 meter
& length of the rectangular field = 1.5×60 = 90 meter
Perimeter of the rectangular field = 2(length + width)
= 2(60 + 90) meter
= 350 meter
So, total Cost to fence = 350×2.5 = Tk 750
Answer: Tk. 750

𝒙−𝟒 𝒙−𝟕 𝒙−𝟐


4. Solve the problem: + + =𝟑
𝒙−𝟏 𝒙−𝟑 𝒙−𝟗
Solution:
𝑥−4 𝑥−7 𝑥−2
+ + =3
𝑥−1 𝑥−3 𝑥−9
𝑥−4 𝑥−7 𝑥−2
⇒ + + =1+1+1
𝑥−1 𝑥−3 𝑥−9
x−4 x−7 x−2
⇒ −1+ −1+ −1=0
x−1 x−3 x−9

Written Math Solution (BIBM) FB Group : BD job Info. (BCS & Bank)
114

 Created by Sumon K. Sarkar  FB Group : BD job Info. (BCS & Bank)  Created by Sumon K. Sarkar 
Bank Job Solution (BIBM)
 Created by Sumon K. Sarkar  FB Group : BD job Info. (BCS & Bank)  Created by Sumon K. Sarkar 

x−4−x+1 x−7−x+3 x−2−x+9


⇒ + + =0
x−1 x−3 x−9
−3 −4 7
⇒ + + =0
x−1 x−3 x−9
3 4 7
⇒− − + =0
𝑥−1 𝑥−3 𝑥−9
7 3 4
⇒ = +
𝑥−9 𝑥−1 𝑥−3
3 4 7
⇒ + =
𝑥−1 𝑥−3 𝑥−9
3 4 4+3
⇒ + =
𝑥−1 𝑥−3 𝑥−9
3 4 4 3
⇒ + = +
𝑥−1 𝑥−3 𝑥−9 𝑥−9
4 4 3 3
⇒ − = −
𝑥−3 𝑥−9 𝑥−9 𝑥−1
4𝑥−36−4𝑥+12 3𝑥−3−3𝑥+27
⇒ =
(𝑥−3)(𝑥−9) (𝑥−9)(𝑥−1)
−24 24
⇒ =
(𝑥−3)(𝑥−9) (𝑥−9)(𝑥−1)
−1 1
⇒ =
(𝑥−3) (𝑥−1)

⇒x–3=-x+1
⇒x+x= 1+3
⇒ 2x = 4
⇒x=2
Answer: x= 2

Modhumoti Bank Ltd.


Post Name: Probationary Officer Exam Date : 2017
1. A shirt sold at 6% profits.If the purchase price was 4% less and selling price was Tk 4 more,the
profit be 12.5%.What was the purchase price of the shirt.
[Modhumoti Bank PO-2017] [Bangladesh Bank AD-2017][SBC AM -2016] [BKB -2012][NBL-2014][BCS Written] [Non cader Written]
[BAPEX][Islamic Bank-2013] [BRC-2008], 16th BCS, MBA-2007-08;1999;1997, NCC BANK-2002, PBL-2000
অনুবােঃ একনট শ্াটণ 6% লাতভ নবনক্র হয। যনে ক্রয মূলয 4% কম হয এবং নবক্রয মূলয 4 টাকা পবনশ্ হয, োহতল লাভ 12.5%। শ্াটণনটর
ক্রয মূলয কে নছল।
Solution:
Let, the cost price is 100
At 6% profit, selling price = (100+6) = 106
At 4% less, cost = (100-4) = 96,
12.5
and earn 12.5% profit so, selling price= 96 + 96× = 108
100
So, at 4% less cost, profit is = Tk(108-106) =Tk. 2
When earing is 2 tk more the cost of the item is Tk. 100
100×4
So, when earning is 4 tk more cost of the item is =
2

Written Math Solution (BIBM) FB Group : BD job Info. (BCS & Bank)
Bank Job Solution (BIBM) 115

Created by Sumon K. Sarkar  FB Group : BD job Info. (BCS & Bank)  Created by Sumon K. Sarkar 
=Tk. 200
Created by Sumon K. Sarkar  FB Group : BD job Info. (BCS & Bank)  Created by Sumon K. Sarkar 

So, cost of the item is Tk. 200


Answer: Tk. 200

Alternative Way,
Let, Cost price be= x TK
First Selling price = (x+x of 6%)
6𝑥
= (𝑥 + )
100
100𝑥+6𝑥
=
100
106𝑥 53𝑥
= =
100 50
96𝑥 24𝑥
4% less cost price =(100-4) = 96% of x = =
100 25
24𝑥 24𝑥
Second Selling price =( + 𝑜𝑓 12.5%)
25 25
24𝑥 24𝑥 12.5
= + ×
25 25 100
216𝑥
=
200
According to the question,
216𝑥 53𝑥
− =4
200 50
216𝑥−212𝑥
⇒ =4
200
4𝑥
⇒ =4
200
⇒ x = 200
The cost price of the article Tk. 200
Answer: Tk. 200

2. A man's running speed is 3 times of his walking speed.He runs a distance and come back by
walking total time taken 2 hrs.what was the distance if he runs 9 miles per hour.
[Modhumoti Bank PO-2017][PBL -2008]
Abyev`tGKRb †jv‡Ki †`Šov‡bvi MwZ Zvi nvUvi MwZi 3 ¸Y| †jvKwU GKwU wbw`©ó `~iZ¡ †`Š‡o wM‡q †n‡U †diZ Av‡mb| Zvi
†gvU 2 N›Uv mgq †j‡MwQj| hw` Zvi †`Šov‡bvi MwZ 9 gvBj/N›Uv nq Z‡e `~iZ¡ (gvBj) wbY©q Kiæb?
Solution:
Let, Distance be = D miles
Runs speed = 9 m/h
Walking speed = 9/3 =3 m/h
𝐷 𝐷
According to the question, + =2
3 9
3𝐷+𝐷
Or, =2
9
4𝐷
Or, =2
9
Or, 𝐷 = 4.5
Answer: 4.5 miles

Written Math Solution (BIBM) FB Group : BD job Info. (BCS & Bank)
116

 Created by Sumon K. Sarkar  FB Group : BD job Info. (BCS & Bank)  Created by Sumon K. Sarkar 
Bank Job Solution (BIBM)
 Created by Sumon K. Sarkar  FB Group : BD job Info. (BCS & Bank)  Created by Sumon K. Sarkar 

3. A,B,C started a job which they can complete in 2 days.B can do the job in 5 days and C can do it
in 4 days.After working for 1 day, both B & C left. A complete testbed of the work how many
days?
[Modhumoti Bank PO-2017]
Abyev`tA, B Ges C GKwU KvR GK‡Î `yB w`‡b †kl Ki‡Z cv‡i| B Ges C GB KvR h_vµ‡g 5 I 4 w`‡b †klKi‡Z cv‡i|
1 w`b KvR Kivi ci B Ges C `y'R‡bB KvR †Q‡o P‡j †Mj| evwK KvR †kl Ki‡Z A Gi KZ w`b mgq jvM‡e?
Solution:
Let, Total portion work be 1 portion
1
(A+B+C) 1 day do the job = portion
2
1 1
(B & C) 1 day do the job = + portion
5 4
4+5
= portion
20
9
= portion
20
1 9
So, A, 1 day done the job = ( − ) portion
2 20
10−9
= portion
20
1
= portion
20
1 1
Remaining work = 1 − portion = portion
2 2
1
A, portion job done =1 day
20
1 20
A, portion job done = portion = 10 days
2 2
Answer:10 days

4. Solve the equation: [Modhumoti Bank PO-2017]


𝟏 𝟏 𝟏 𝟏
+ = +
𝟐𝒙−𝟓 𝟐𝒙−𝟏𝟏 𝟐𝒙−𝟕 𝟐𝒙−𝟗
Solution:
1 1 1 1
+ = +
2𝑥−5 2𝑥−11 2𝑥−7 2𝑥−9
1 1 1 1
Or, − = −
2𝑥−5 2𝑥−7 2𝑥−9 2𝑥−11
2𝑥−7−2𝑥+5 2𝑥−11−2𝑥+9
or, =
(2𝑥−5)(2𝑥−7) (2𝑥−9)(2𝑥−11)
−2 −2
or, =
(2𝑥−5)(2𝑥−7) (2𝑥−9)(2𝑥−11)
1 1
or, =
(2𝑥−5)(2𝑥−7) (2𝑥−9)(2𝑥−11)
𝑜𝑟, (2𝑥 − 9)(2𝑥 − 11) = (2𝑥 − 5)(2𝑥 − 7)
𝑜𝑟, 4𝑥 2 − 40𝑥 + 99 = 4𝑥 2 − 24𝑥 + 35
𝑜𝑟, 4𝑥 2 − 4𝑥 2 − 40𝑥 + 24𝑥 = 35 − 99
𝑜𝑟, −16𝑥 = −64
𝑜𝑟, 𝑥 = 4
Answer:4

Written Math Solution (BIBM) FB Group : BD job Info. (BCS & Bank)
Bank Job Solution (BIBM) 117

Created by Sumon K. Sarkar  FB Group : BD job Info. (BCS & Bank)  Created by Sumon K. Sarkar 
Social Islami Bank Ltd.
Created by Sumon K. Sarkar  FB Group : BD job Info. (BCS & Bank)  Created by Sumon K. Sarkar 

Post Name: Probationary Officer Exam Date : 2017


1. The cost price of two watches taken together is Tk. 840. If by selling one at a profit of 16% and
the other at a loss of 12%, there is no loss or gain in the whole transaction, find the cost price of
the two watches.
[SJIB MTO 11, SIBL PO 17, NRBC Bank MTO 18]
Abyev`t GKm‡½ †Kbv 2wU Nwoi µqg~j¨ GK‡Î 840 UvKv| hw` GwU‡K 16% jv‡f Ges Ab¨wU‡K 12% ¶wZ‡Z weµq Ki‡j
†gv‡Ui Dci †Kvb jvf ev ¶wZ nq bv Zvn‡j Nwo `zBwUi cÖwZwUi µqg~j¨ wbY©q Ki|
Solution:
Let, the cost of one watch = Tk. x
Then cost of second watch = Tk. (840-x)
According to the question,
16% of x =12% of (840-x)
16 12
⇒x× = (840−x) ×
100 100
⇒ 16x = 5760 −12x
⇒ 16x + 12x = 12×840
⇒ 28x =12×840
𝟏𝟐×𝟖𝟒𝟎
⇒x= = 360
𝟐𝟖
Hence ,cost of one watch Tk. 360
Cost of another watch = Tk. (840 – 360)
=Tk. 480
Answer: Tk. 360 & Tk. 480

2. A, B and C can do a piece of work in 16, 32 and 48 days respectively. They started working
together but C left after working 4 days and B left 2 days before the completion of work. How
many days it took to complete the work?
[SJIB MTO 11, STBL PO 17, NRBC Bank MTO 18, National Bank PO 17. Langka Bangla Finance MTO 17]
Abyev`t A, B I C GKwU KvR h_vµ‡g 16, 32 Ges 48 w`‡b Ki‡Z cv‡i| Zviv GKmv‡_ KvR Ki‡Z ïiæ K‡i wKš‘ KvR ïiæ
nIqvi 4 w`‡b ci C KvR †Q‡o †`q Ges KvR †kl nIqvi 2 w`b Av‡M B KvR †Q‡o †`q| m¤ú~Y© KvRwU †kln‡Z KZw`b mgq
jv‡M?
Solution:
Let, the work be done in x days.
A worked for x days, B for (x-2) and C for 4 days.
𝑥 𝑥−2 4
+ + =1
16 32 48
𝑥 𝑥−2 1
⇒ + + =1
16 32 12
6𝑥+3𝑥−6+8
⇒ =1
96

⇒ 9x +2 = 96
⇒ 9x = 96 – 2
⇒ 9x = 94

Written Math Solution (BIBM) FB Group : BD job Info. (BCS & Bank)
118

 Created by Sumon K. Sarkar  FB Group : BD job Info. (BCS & Bank)  Created by Sumon K. Sarkar 
Bank Job Solution (BIBM)
 Created by Sumon K. Sarkar  FB Group : BD job Info. (BCS & Bank)  Created by Sumon K. Sarkar 

94
⇒x=
9
4
⇒ x = 10 𝑑𝑎𝑦𝑠
9
𝟒
Answer: 𝟏𝟎 𝒅𝒂𝒚𝒔
𝟗

3. The perimeter of a square field is equal to the perimeter of a rectangle field. The length of the
rectangle is thrice the width of it and the area is 768 square meters. How many square sized tiles
of 80 centimeters will be required to cover the square filed?
[Al-Arafah MTO 16, National Bank PO 17, NRBC Bank MTO 18, SIBL PO 17, Modhumoti PO 18]
Abyev`t GKwU eM©vKvi gv‡Vi cwimxgv GKwU AvqZKvi gv‡Vi cwimxgvi mgvb| AvqZ‡¶ÎwUi ˆ`N©¨ Gi cÖ‡¯’i wZb¸Y Ges Gi
†¶Îdj 768 eM©wgUvi| eM©vKvi gvVwU‡K m¤ú~Y©iƒ‡c XvK‡Z 80 †m.wg KZ¸‡jv eM©vKvi UvBjm jvM‡e?
Solution:
Let, The width of the rectangle be x meter
So, The length of the rectangle be 3x meter
According to the question,
3x × x=768
or, 𝑥 2 = 256
or, x = 16
So,The perimeter of the rectangle = 2(3x + x)
=2(3×16+16)
=128
So, the perimeter of the square is 128 meters
Let, Each Side of square is y
According to the question,
4y=128
128
or, 𝑦 =
4
or, y = 32
So, area of the square be 𝑦 2 = 322
=1024 squar meter
=1024×10000 square cm
=10240000 square cm
Again, area of the tiles = 802
= 6400 square cm
102,40,000
Number of tiles =
6400
=1600 unit
Answer:1600 unit

এ রকম গুরুত্বপূর্ণ ফাইল পপতে আমাতের গ্রুতপ প াখ রাখুন


এবং অবশ্যই ননতের বন্ধুতের ইনভাইট করুন

Written Math Solution (BIBM) FB Group : BD job Info. (BCS & Bank)
Bank Job Solution (BIBM) 119

Created by Sumon K. Sarkar  FB Group : BD job Info. (BCS & Bank)  Created by Sumon K. Sarkar 
𝟏 𝟏 𝟏 𝟏
4. Solve the problem: + = + [SIBL PO 17, Langka Bangla Finance MTO 17 Board Book]
𝒙+𝟐 𝒙+𝟓 𝒙+𝟒 𝒙+𝟑
Created by Sumon K. Sarkar  FB Group : BD job Info. (BCS & Bank)  Created by Sumon K. Sarkar 

Solution:
𝟏 𝟏 𝟏 𝟏
+ = +
𝒙+𝟐 𝒙+𝟓 𝒙+𝟒 𝒙+𝟑
𝟏 𝟏 𝟏 𝟏
⇒ − = −
𝒙+𝟐 𝒙+𝟒 𝒙+𝟑 𝒙+𝟓
𝒙+𝟒−𝒙−𝟐 𝒙+𝟓−𝒙−𝟑
⇒ =
(𝒙+𝟐)(𝒙+𝟒) (𝒙+𝟑)(𝒙+𝟓)
𝟐 𝟐
⇒ =
(𝒙+𝟐)(𝒙+𝟒) (𝒙+𝟑)(𝒙+𝟓)
𝟏 𝟏
⇒ =
(𝒙+𝟐)(𝒙+𝟒) (𝒙+𝟑)(𝒙+𝟓)

⇒ (𝑥 + 3)(𝑥 + 5) = (𝑥 + 2)(𝑥 + 4)
⇒ 𝑥 2 + 8𝑥 + 15 = 𝑥 2 + 6𝑥 + 8
⇒ 𝑥 2 + 8𝑥 + 15 − 𝑥 2 − 6𝑥 − 8 = 0
⇒ 2x + 7=0
⇒ 2x = - 7
⇒ x = - 7/2
Answer: -7/2

One Bank Ltd.


Post Name: Special Cadre Officer Exam Date : 2017
𝟒 𝟏𝟓 𝟑𝟓
1. Solve the equation: + =
𝟐𝒙+𝟑 𝟓𝒙+𝟒 𝟕𝒙+𝟔
[ONE Bank SCO 17, Board Book]
Solution:
𝟒 𝟏𝟓 𝟑𝟓
+ =
𝟐𝒙+𝟑 𝟓𝒙+𝟒 𝟕𝒙+𝟔
𝟒 𝟏𝟓 𝟐𝟏+𝟏𝟒
⇒ + =
𝟐𝒙+𝟑 𝟓𝒙+𝟒 𝟕𝒙+𝟔
𝟒 𝟏𝟓 𝟐𝟏 𝟏𝟒
⇒ + = +
𝟐𝒙+𝟑 𝟓𝒙+𝟒 𝟕𝒙+𝟔 𝟕𝒙+𝟔
𝟒 𝟏𝟒 𝟐𝟏 𝟏𝟓
⇒ − = −
𝟐𝒙+𝟑 𝟕𝒙+𝟔 𝟕𝒙+𝟔 𝟓𝒙+𝟒
𝟐𝟖𝒙+𝟐𝟒−𝟐𝟖𝒙−𝟒𝟐 𝟏𝟎𝟓𝒙+𝟖𝟒−𝟏𝟎𝟓𝒙−𝟗𝟎
⇒ =
(𝟐𝒙+𝟑)(𝟕𝒙+𝟔) (𝟕𝒙+𝟔)(𝟓𝒙+𝟒)
−𝟏𝟖 −𝟔
⇒ =
(𝟐𝒙+𝟑)(𝟕𝒙+𝟔) (𝟕𝒙+𝟔)(𝟓𝒙+𝟒)
𝟑 𝟏
⇒ =
(𝟐𝒙+𝟑) (𝟓𝒙+𝟒)
⇒ 15𝑥 + 12 = 2𝑥 + 3
⇒ 15x – 2x = 3 – 12
⇒ 13x = - 9
⇒ x = - 9/13
Answer: - 9/13

Written Math Solution (BIBM) FB Group : BD job Info. (BCS & Bank)
120

 Created by Sumon K. Sarkar  FB Group : BD job Info. (BCS & Bank)  Created by Sumon K. Sarkar 
Bank Job Solution (BIBM)
 Created by Sumon K. Sarkar  FB Group : BD job Info. (BCS & Bank)  Created by Sumon K. Sarkar 

2. Salam used a part of Tk. 1,00,000 to purchase a Television. Of the remaining portion, he invested
1/3 of it at 4% simple annual interest and 2/3 of it at 6% simple annual interest. If, after a year,
the income from two investments totaled Tk. 320, what was the purchase price of the television?
Abyev`t mvjvg 1,00,000 UvKvi wKQz Ask w`‡q GKwU †Uwjwfkb µq Kij| evwK UvKvi 1/3 Ask 4% my‡` Ges 2/3 Ask 6%
my‡` e¨vs‡K Rgv ivLj| eQi c‡i †m e¨vsK †_‡K GK‡Î 320 UvKv gybvdv cvq Zvn‡j †UwjwfkbwUi µqg~j¨ KZ?
Solution:
Let, the purchase price of the TV tk x
and remaining amount of money tk. (100,000 – x)
According to the question,
1 2
(100,000 – x)× × 4% + (100,000 – x )× × 6% = 320
3 3
1 4 2 6
⇒ (100,000 – x)× × + (100,000 – x )× × = 320
3 100 3 100
1 4 2 6
⇒ (100,000 – x)× × + (100,000 – x) × × = 320
3 100 3 100
400,000 − 4𝑥 + 1200,000 − 12𝑥
⇒ = 320
300
⇒ 16,00,000 − 16𝑥 = 320 × 300
⇒ 100,000 − 𝑥 = 20 × 300
⇒ 100,000 − 𝑥 = 6000
⇒ 𝑥 = 100,000 − 6000
⇒ 𝑥 = 94,000
Answer: 94,000

3. The length of a rectangular field is 1.5 times as large as its width. An amount of Tk. 10260 was
needed to cover the field with grass at 1.9Tk per sq. meter. How much would it cost to fence the
four sides of the rectangular field at the rate of Tk 2.5 per meter?
[ONE Bank SCO 17, Bank Asia MTO 17, SJIB MTO 16, Langka Bangla Finance MTO 17 Board Book]
Abyev`t GKwU AvqZKvi gv‡Vi ˆ`N¨© Gi cÖ‡¯’i †`o ¸Y| cÖwZ eM©wgUv‡i 1.9 UvKv wnmv‡e m¤ú~Y© gvVwU‡Z Nvm jvMv‡Z 10,260
UvKv cÖ‡qvRb| cÖwZ wgUvi 2.5 UvKv wnmv‡e gvVwUi Pvicv‡k †eov w`‡Z KZ UvKv LiP n‡e?
Solution:
Let, The width of the rectangular field be x meter
and length of the rectangular field = 1.5x meter
According to the question,
1.5x × x ×1.9= 10260 [ Total Cost ]
10,260
⇒ 𝑥2 =
1.5×1.9
⇒ 𝑥 2 = 3600
⇒ x = 60
The width of the rectangular field 60 meter
& length of the rectangular field = 1.5×60 = 90 meter
Perimeter of the rectangular field = 2(length + width)
= 2(60 + 90) meter
= 350 meter
So, total Cost to fence = 350×2.5 = Tk 750

Written Math Solution (BIBM) FB Group : BD job Info. (BCS & Bank)
Bank Job Solution (BIBM) 121

Created by Sumon K. Sarkar  FB Group : BD job Info. (BCS & Bank)  Created by Sumon K. Sarkar 
Answer: Tk. 750
Created by Sumon K. Sarkar  FB Group : BD job Info. (BCS & Bank)  Created by Sumon K. Sarkar 

4. A man has to go 10 km to catch a bus. He walks part of the way at 7 km per hour and runs the
rest of the way at 12 km per hour. If he takes 1 hour 15 minutes to complete his journey. Find
how far he walked?
[Modhumoti Bank MTO 16, ONE Bank SCO 17, Langka Bangla Finance MTO 17]
Abyev`t †Kvb e¨w³‡K evm aivi Rb¨ 10 wK.wg †h‡Z nq| c‡_i wKQz Ask wZwb 7 wK.wg/N›Uv †e‡M †nu‡U Ges evwK Ask
12wK.wg/N›Uv †e‡M †`Š‡o cvi K‡ib| hw` GB c_ †h‡Z Zvi †gvU 1 N›Uv 15 wgwbU mgq jv‡M Zvn‡j wZwb KZUzKz c_ nuv‡Ub Zv
wbY©q Ki|
Solution:
Let, A man walks x km and run (10 – x) km
15
Here, 1 hour 15 minutes = 1 hour + hour
60
15
= (1 + ) hour
60
60+15
=( ) hour
60
75
= hour
60
According to the question
𝑥 10−𝑥 75
+ =
7 12 60
12𝑥 + 70 – 7𝑥 5
⇒ =
84 4
5𝑥 + 70 5
⇒ =
84 4
5(𝑥 + 14) 5
⇒ =
84 4
𝑥 + 14
⇒ =1
21

⇒ 𝑥 + 14 = 21
⇒ 𝑥 = 21 – 14
⇒𝑥=7
A man walks 7 km
Answer : 7 km

এ রকম গুরুত্বপূর্ণ ফাইল পপতে আমাতের গ্রুতপ প াখ রাখুন


এবং অবশ্যই ননতের বন্ধুতের ইনভাইট করুন

Written Math Solution (BIBM) FB Group : BD job Info. (BCS & Bank)
122

 Created by Sumon K. Sarkar  FB Group : BD job Info. (BCS & Bank)  Created by Sumon K. Sarkar 
Bank Job Solution (BIBM)
 Created by Sumon K. Sarkar  FB Group : BD job Info. (BCS & Bank)  Created by Sumon K. Sarkar 

Modhumoti Bank Ltd.


Post Name: Probationary Officer Exam Date : 2016
1. If x+y=a, x2+ y2 =b2 and x3+ y3 = c3 , then show that a3 + 2c3 = 3ab2
[Modhumoti MTO 16, Premier Bank JO 19, Board Book]
Solution:
Given That,
x+y=a, x2+ y2 =b2 and x3+ y3 = c3
L.H.S.
a3 + 2c3
(x+y)3 + 2(x3+ y3)
⇒ 𝑥 3 + 3𝑥 2 𝑦 + 3𝑥𝑦 2 + 𝑦 3 + 2𝑥 3 + 2𝑦 3
⇒ 3𝑥 3 + 3𝑦 3 + 3𝑥 2 𝑦 + 3𝑥𝑦 2
⇒ 3(𝑥 3 + 𝑦 3 + 𝑥 2 𝑦 + 𝑥𝑦 2 )
⇒ 3(𝑥 3 + 𝑥 2 𝑦 + 𝑥𝑦 2 + 𝑦 3 )
⇒ 3{𝑥 2 (𝑥 + 𝑦) + 𝑦 2 (𝑥 + 𝑦)}
⇒ 3(𝑥 + 𝑦)(𝑥 2 + 𝑦 2 )
⇒ 3ab2
So, a3 + 2c3 = 3ab2 (showed)

𝟏 𝟓
2. In a school, there are equal number of boys and girls. Among the students of the girls and th
𝟖 𝟔
of the boys are residing in the hostel. What percent of the Students consists of boys who do not
reside in the hostel among all students?
[Modhumoti Bank MTO 16, SJIB MTO 10]
অনুবােঃ একনট স্কুতল পছতল ও পমতয সমান সংখযক। ছাত্রতের মতযয 1/8 অংশ্ পমতয এবং 5/6 অংশ্ পছতল ছাত্রাবাতস র্াতক। ছাত্রতের কে
শ্োংশ্ পছতলরা সকল ছাত্রতের মতযয পহাতস্টতল র্াতক না?
Solution:
Let, the number of boys be ‘x’
So, the number of girls be ‘x’.
So, Total students = x + x = 2x
5 5x 6x−5x x
Non residing boys = x − of x = x − = =
6 6 6 6
x
6
Required percentage = × 100% = 8.33%
2x
Answer: 8.33%

Alternative Way,
Let, total student be 48
So, boys= 24 and girls= 24
5
Boys residing in the hostel =24× = 20
6
Boys do not reside in the hostel =24-20= 4
4
Required percentage = ×100 % = 8.33 %
48
Answer: 8.33 %

Written Math Solution (BIBM) FB Group : BD job Info. (BCS & Bank)
Bank Job Solution (BIBM) 123

Created by Sumon K. Sarkar  FB Group : BD job Info. (BCS & Bank)  Created by Sumon K. Sarkar 
Created by Sumon K. Sarkar  FB Group : BD job Info. (BCS & Bank)  Created by Sumon K. Sarkar 

3. A man has to go 10 km to catch a bus. He walks part of the way at 7 km per hour and runs the
rest of the way at 12 km per hour. If he takes 1 hour 15 minutes to complete his journey. Find
how far he walked?
[Modhumoti Bank MTO 16, ONE Bank SCO 17, Langka Bangla Finance MTO 17]
Abyev`t †Kvb e¨w³‡K evm aivi Rb¨ 10 wK.wg †h‡Z nq| c‡_i wKQz Ask wZwb 7 wK.wg/N›Uv †e‡M †nu‡U Ges evwK Ask
12wK.wg/N›Uv †e‡M †`Š‡o cvi K‡ib| hw` GB c_ †h‡Z Zvi †gvU 1 N›Uv 15 wgwbU mgq jv‡M Zvn‡j wZwb KZUzKz c_ nuv‡Ub Zv
wbY©q Ki|
Solution:
Let, A man walks x km and run (10 – x) km
15
Here, 1 hour 15 minutes = 1 hour + hour
60
15
= (1 + ) hour
60
60+15
=( ) hour
60
75
= hour
60
According to the question
𝑥 10−𝑥 75
+ =
7 12 60
12𝑥 + 70 – 7𝑥 5
⇒ =
84 4
5𝑥 + 70 5
⇒ =
84 4
5(𝑥 + 14) 5
⇒ =
84 4
𝑥 + 14
⇒ =1
21
⇒ 𝑥 + 14 = 21
⇒ 𝑥 = 21 – 14
⇒𝑥=7
A man walks 7 km
Answer : 7 km

4. Abir can do a piece of work in 80 days. He works for 10 days and then Basher alone finishes the
rest of the work in 42 days. How much time would it take for the two of them together to complete
the whole work?
[BB AD 06, Modlumoti Bank MTO 16, Bank Asia MTO 17]
Abyev`t Avwei GKwU KvR 80 w`‡b Ki‡Z cv‡i| †m 10 w`b KvR Kivi ci evkvi GKvB evwK KvR 42 w`‡b †kl K‡i| Zviv
`yÕR‡b GKmv‡_ KvR Ki‡j m¤ú~b© KvRwU Ki‡Z Zv‡`i KZw`b mgq jvM‡e?
Solution:
Abir can do a piece of work in 80 days
In 80 Days, Abir can do 1 portion
1
In 1 Days, Abir can do portion
80
1×10
In 10 Days, Abir can do portion
80

Written Math Solution (BIBM) FB Group : BD job Info. (BCS & Bank)
124

 Created by Sumon K. Sarkar  FB Group : BD job Info. (BCS & Bank)  Created by Sumon K. Sarkar 
Bank Job Solution (BIBM)
 Created by Sumon K. Sarkar  FB Group : BD job Info. (BCS & Bank)  Created by Sumon K. Sarkar 

1
= portion
8
1 8−1 7
Rest of the work = 1 − = = portion
8 8 8
7
Again, In 42 Days, Basher can do portion
8
7 1
In 1 Days, Basher can do = portion
8×42 48
1 1
In 1 Days, Abir & Basher together can do = + portion
40 48
3+5
= portion
240
8 1
= portion = portion
240 30
1
So, they can do portion works in 1 day
30
30
They can do 1 part works in = 30 days
1
Answer: 30 Days

5. ABC is a triangle in which AB = 3cm, BC = 5cm, and AC = 4cm, AD is a perpendicular from A


to BC. Find the length of AD. [Modbumoti MTO 16, Premier Bank JO 19]

Solution:
Given that,
ABC is a triangle in which
AB = 3cm, BC = 5cm, and AC = 4cm
According to the condition,
∠𝐵𝐴𝐶 = 90𝑜
𝑆𝑜, ∆𝐵𝐴𝐶 𝑟𝑖𝑔ℎ𝑡 𝑎𝑛𝑔𝑒𝑙 𝑇𝑟𝑖𝑎𝑛𝑔𝑙𝑒
1
Area of the , ∆𝐵𝐴𝐶 = × 𝐴𝐵 × 𝐴𝐶
2
1
= ×3×4
2
=3×2
=6
1
Area of the , ∆𝐴𝐵𝐶 = × 𝐴𝐷 × 𝐵𝐶
2
According to the condition,
1
× 𝐴𝐷 × 𝐵𝐶 = 6
2
1
⟹ × 𝐴𝐷 × 5 = 6
2
12
⟹ 𝐴𝐷 = = 2.4
5
Find the length of AD 2.4 cm Answer: 2.4 cm

Written Math Solution (BIBM) FB Group : BD job Info. (BCS & Bank)
Bank Job Solution (BIBM) 125

Created by Sumon K. Sarkar  FB Group : BD job Info. (BCS & Bank)  Created by Sumon K. Sarkar 
Modhumoti Bank Ltd.
Created by Sumon K. Sarkar  FB Group : BD job Info. (BCS & Bank)  Created by Sumon K. Sarkar 

Post Name: Probationary Officer Exam Date : 2016


1. A, B, C started a partnership business in where A's share: B's share = 1:2 and B's share: C's
share = 3:4. If the total profit is Tk. 1105 then how much money will A get?
Abyev`t A, B Ges C GKwU e¨emv ïiæ K‡i †hLv‡b A Gi wewb‡qvM t B Gi wewb‡qvM = 1 t 2 Ges B Gi wewb‡qvM t C Gi
wewb‡qvM = 3 t 4| hw` †gvU jv‡fi cwigvb 1,105 UvKv nq Zvn‡j A KZ UvKv cv‡e?
Solution:
A's share: B's share = 1:2 = 3:6
B's share: C's share = 3:4 = 6:8
A's share: B's share : C's share = 3: 6 : 8
Sum of the ratio = 3 + 6 + 8 = 17
3
A's share = 1105 × = Tk. 195
17
Answer: Tk. 195

2. Daily earnings of two persons are in the ratio 4:5 and their daily expense 7:9, if each saves Tk.
50 per day. What is their income?
Abyev`t 2 Rb e¨w³i ˆ`wbK Av‡qi AbycvZ 4 t 5 Ges Li‡Pi AbycvZ 7 t 9| hw` cÖ‡Z¨‡KB cÖwZw`b 50 UvKv Rgv K‡ib Zvn‡j
Zv‡`i Avq KZ?
Solution:
we know that, expense = (earning - saving)
Let, Daily earning of two persons be 4x and 5x.
According to the Question,
4𝑥−50 7
=
5𝑥−50 9
⟹ 36x – 450 = 35x – 350
⟹ 36x – 35x = 450 – 350
∴ x = 100
So, Daily earning of two persons be 4x = 4×100 = 400 and 5x =5×100=500
Answer: Tk. 400, Tk. 500

3. A boat can take 8 hour to go 32 km against the current and take 4 hour for same distance with
the current, what is the speed of the boat and current?
[IFIC Bank PO 11, Modhunoti Bank MTO 16]
Abyev`t †mªv‡Zi wecix‡Z GKwU †b․Kv 8 N›Uvq 32 wK.wg hvq| †hLv‡b †mªv‡Zi AbyK~‡j H GKB c_ †h‡Z gvÎ 4 N›Uv mgq jv‡M|
†bŠKv I †mªv‡Zi †eM KZ?
Solution:
Let, The speed of the boat be x km/hr
The speed of the current be y km/hr
According to the condition,
32
With the current, x + y = = 8 ………….(1)
4
32
Against the current , x - y = = 4……….(2)
8
Adding the equation (1) & (2)

Written Math Solution (BIBM) FB Group : BD job Info. (BCS & Bank)
126

 Created by Sumon K. Sarkar  FB Group : BD job Info. (BCS & Bank)  Created by Sumon K. Sarkar 
Bank Job Solution (BIBM)
 Created by Sumon K. Sarkar  FB Group : BD job Info. (BCS & Bank)  Created by Sumon K. Sarkar 

x+y+x–y=8+4
⇒2x = 12
⇒x=6
Put the value of x in equation (1)
x+y=8
⇒6+y=8
⇒y=8–6=2
The speed of the boat 6 km/hr
The speed of the current 2 km/hr
Answer: 6 km/hr, 2 km/hr

Standard Bank Ltd.


Post Name: Trainee Assistant Officer(Cash) Exam Date : 2016

1. A can dig a pond in 30 days and B can dig the same pond in 20 days. In how many days A and B
can dig the pond if they work together?
[Combined Cash 08, Standard Bank TAO Cash 16]
Abyev`t A GKwU cyKzi 30 w`‡b Ges B GKB cyKzi 20 w`‡b Lbb Ki‡Z cv‡i| Zviv GK‡Î H cyKziwU KZ w`‡b Lbb Ki‡Z
cvi‡e?
Solution:
In 30 Days, A can dig 1 portion of the pond
1
In 1 Days, A can dig portion of the pond
30
In 20 Days, B can dig 1 portion of the pond
1
In 1 Days, B can dig portion of the pond
20
1 1
In 1 Days, (A+B) can dig + portion of the pond
30 20
3+2
= portion of the pond
60
5
= portion of the pond
60
1
= portion of the pond
12
1
portion of the pond can dig in 1 day.
12
1 portion of the pond can dig in 12 day.
Answer: 12 day

এ রকম গুরুত্বপূর্ণ ফাইল পপতে আমাতের গ্রুতপ প াখ রাখুন


এবং অবশ্যই ননতের বন্ধুতের ইনভাইট করুন

Written Math Solution (BIBM) FB Group : BD job Info. (BCS & Bank)
Bank Job Solution (BIBM) 127

Created by Sumon K. Sarkar  FB Group : BD job Info. (BCS & Bank)  Created by Sumon K. Sarkar 
2. Karim and Rahim have equal amount of money. Radha has half of Rahiin's money and Amena
Created by Sumon K. Sarkar  FB Group : BD job Info. (BCS & Bank)  Created by Sumon K. Sarkar 

bas half of Radha's money. If you add one taka with all the money they have, it will be Tk. 100.
How much Rahim has?
[Preinier Bank TJO 18, BB AD 04, Standard Bank TAO Cash 16]
Abyev`t Kwig Ges iwn‡gi Kv‡Q mgcwigvb UvKv Av‡Q| iwn‡gi UvKvi A‡a©K ivavi Av‡Q Ges ivavi UvKvi A‡a©K Av‡gbvi Av‡Q|
hw` Zv‡`i mK‡ji UvKvi mgwói m‡½ 1 †hvM K‡ib Zvn‡j 100 UvKv n‡e| iwn‡gi Kv‡Q KZ UvKv Av‡Q?
Solution:
Let, Karim & Rahim everyone has tk. x
𝑥 𝑥
So, Radha has = and Amena has =
2 4
According to Question,
𝑥 𝑥
x ✛ x ✛ ✛ ✛ 1 = 100
2 4
𝑥 𝑥
⇒ 2x ✛ ✛ = 100 – 1
2 4
8𝑥+2𝑥+𝑥
⇒ = 99
4
11𝑥
⇒ = 99
4
99×4
⇒𝑥 = = 36
11
Rahim has tk 36
Answer: tk 36

Alternative Way,
Radha : Rahim ⇒ 1 : 2 = (1 : 2)×2 = 2 : 4
Amena : Radha ⇒ 1 : 2 = (1 : 2)×1 = 1 : 2
So, Radha : Rahim : Amena : Karim ⇒ 2:4 : 1 : 4
Let , Radha , Rahim , Amena and Karim have be 2T , 4T , T , 4T
ATQ , (2T+4T+T+4T) + 1 = 100
⇒ 11T = 99 ⇒ T = 9
Therefore , Rahim has TK. (9×4) = 36
Answer: tk 36

3. A casher received a total amount of Tk 10,000 from depositor in a total of 140 notes of Tk. 50
and Tk. 100 denominations. How many notes of Tk. 50 denominations did he receive?
Solution:
The number of Tk. 50 notes be x
& Tk. 100 notes be (140 – x)
According to the Question,
50x + 100(140 – x) = 10,000
⇒ 50x + 14,000 – 100x = 10,000
⇒ – 50x = 10,000 – 14,000
⇒ – 50x = – 4,000
⇒ x = 80
The number of Tk.50 notes be 80
Answer: 80

Written Math Solution (BIBM) FB Group : BD job Info. (BCS & Bank)
128

 Created by Sumon K. Sarkar  FB Group : BD job Info. (BCS & Bank)  Created by Sumon K. Sarkar 
Bank Job Solution (BIBM)
 Created by Sumon K. Sarkar  FB Group : BD job Info. (BCS & Bank)  Created by Sumon K. Sarkar 

4. The simple interest rate of a bank was reduced to 5% from 7%. As a consequences Mr. B's
income was reduced by Tk. 2100 in 5 years. How much is Mr. B's initial deposit in the bank?
[BB AD 01, Standard Bank TAO Cash 16]
Abyev`t my‡`i nvi 7% †_‡K K‡g 5% nIqvq Mr. B e¨w³i my` 5 eQ‡i 2,100 UvKv K‡g †Mj| Mr. B Zvi g~jab KZ?
Solution:
Let, Mr. B's initial deposit in the bank Tk. x
According to the question,
𝑥 × 5 × 7% − 𝑥 × 5 × 5% = 2100
35𝑥 25𝑥
⇒ − = 2100
100 100
35𝑥−25𝑥
⇒ = 2100
100
10𝑥
⇒ = 2100
100
𝑥
⇒ = 2100
10
⇒ 𝑥 = 21,000
Mr. B's initial deposit in the bank Tk. 21,000
Answer: Tk. 21,000

Alternative way:
The rate of interest rate reduced = 7% - 5% = 2%
In 5 years interest reduced = Tk. 2100
2100
In 1 year interest reduced = Tk. = Tk. 420
5
Tk. 2 reduce when deposit Tk. 100
100
Tk. 1 ” ” ” Tk.
2
100×420
Tk. 420 ” ” ” Tk. .
2
= Tk. 21,000
Answer: Tk. 21,000

5. A garden is 60 meter long and 20 meter wide. Inside the garden there is a 5 meter wide path
around it. What is the area of the path in square meter?
[Standard Bank TAO Cash 16]
Abyev`t GKwU evMvb 60 wgUvi j¤^v Ges 20 wgUvi cÖk¯Í wQj| evMv‡bi wfZ‡ii w`‡K Pvicv‡k 5 wgUvi cÖk¯Í iv¯Ív wQj|hw`
evMv‡b Nvm jvMv‡Z cÖwZ eM© wgUv‡i 20 UvKv cÖ‡qvRb nq, Zvn‡j iv¯ÍvwU Nvm jvMv‡Z †gvU KZ UvKvi cÖ‡qvRb n‡e?
Solution:
Given that, The length of a garden = 60 meter
The width of a garden = 20 meter
Area of a garden including path = 60 × 20 = 1200 square meter
Length of a garden without path = 60 - (5× 2) = 50 meter
Width of a garden without path = 20 - (5× 2) = 10 meter
Area of a garden without path = 50 × 10 = 500 square meter
Area of the path = 1200 - 500 = 700 square meter
Total Cost = 700 × 20 = tk 14, 000
Answer: tk 14, 000

Written Math Solution (BIBM) FB Group : BD job Info. (BCS & Bank)
Bank Job Solution (BIBM) 129

Created by Sumon K. Sarkar  FB Group : BD job Info. (BCS & Bank)  Created by Sumon K. Sarkar 
Standard Bank Ltd.
Created by Sumon K. Sarkar  FB Group : BD job Info. (BCS & Bank)  Created by Sumon K. Sarkar 

Post Name: Trainee Assistant Officer Exam Date : 2016


1. A sales person earns Tk. 250 fixed salary. He also gets 15% commission on total sales. What will
be sales volume, if he wants to earn total Tk. 1000?
Abyev`t GKRb weµqKg©xi g~j †eZb 250 UvKv| wZwb Zvi †gvU wewµi Dci 15% Kwgkb cvb| hw` wZwb 1,000 UvKv Avq
Ki‡Z Pvb Zvn‡j Zv‡K wK cwigvb wewµ Ki‡Z n‡e?
Solution:
Let, sales volume be x
According to the Question,
𝑥 × 15% + 250 = 1000
15𝑥
⇒ = 1000 − 250
100
3𝑥
⇒ = 750
20
⇒ 3x = 15, 000
⇒ x = 5000
Sales volume tk. 5000
Answer: tk. 5000

2. A trader, while selling an item, was asking for such a price that would enable him to offer a 20%
discounts and still inake a profit of 30% on cost. If the cost of the item was Tk. 50 wbat was his
asking price?
[BB AD 08, Standard Bank TAO 16, Premier Bank JO 19]
Abyev`t GKRb we‡µZv GKwU c‡Y¨i Ggb GKwU `vg e‡jb hv‡Z cY¨wU‡Z 20% wWmKvD›U w`‡jI we‡µZvi 30% jvf nq| hw`
cY¨wUi g~j¨ 50 UvKv nq Zvn‡j we‡µZv KZ `vg e‡jwQ‡jb?
Solution:
Given that, cost price = 50 Tk
30
At 30% profit, price will be = 50 + (50 × ) = 50 + 15 = Tk.65
100
Let, asking price of the item = 100 Tk.
At 20% discount price = (100-20) = 80 Tk.
So, when discounted price is Tk 80 then asking price is Tk 100
100×65
When discounted price is Tk 65 asking price is = Tk = Tk 81.25
80
Answer: tk 81.25

এ রকম গুরুত্বপূর্ণ ফাইল পপতে আমাতের গ্রুতপ প াখ রাখুন


এবং অবশ্যই ননতের বন্ধুতের ইনভাইট করুন

Written Math Solution (BIBM) FB Group : BD job Info. (BCS & Bank)
130

 Created by Sumon K. Sarkar  FB Group : BD job Info. (BCS & Bank)  Created by Sumon K. Sarkar 
Bank Job Solution (BIBM)
 Created by Sumon K. Sarkar  FB Group : BD job Info. (BCS & Bank)  Created by Sumon K. Sarkar 

3. Two banks offer interest rate of 6% and 7% respectively, on Fixed Deposit. Mr. Rahman
deposited a total amount of Tk. 4000 in the banks and in one year his interest income was Tk.
250. How much money was deposited in the bank with 7% interest?
Abyev`t ingvb mv‡ne 4,000 UvKv 6% I 7% my‡` Avjv`v fv‡e mÂq K‡ib| 1 eQi ci wZwb hw` GK‡Î 250 UvKv gybvdv cvb
Zvn‡j 7% nv‡i KZ UvKv Rgv †i‡LwQ‡jb?
Solution:
Let, Mr. Rahman deposited at 7% tk. x
& at 6% tk. (4000-x)
According to the question,
𝑥 × 7% + (4000 − 𝑥) × 6% = 250
7𝑥 (4000−𝑥)×6
+ = 250
100 100
7𝑥+24,000−6𝑥
⇒ = 250
100
⇒ x + 24,000 = 25,000
⇒ x = 25,000 – 24,000
⇒ x = 1000
Mr. Rahman deposited at 7% tk. 1000.
Answer: Tk. 1000

4. A pipe can fill a tank in 30 minutes and other can drain out the tank in 40 minutes. If both pipes
are started same time, how much time will be required to fill half of the tank?
Abyev`t GKwU bj 30 wgwb‡U GKwU †PŠev”Pv cvwbc~Y© K‡i| Aci GKwU bj 40 wgwb‡U †PŠev”PvwU‡K cvwb k~b¨ K‡i| 2wU bj
GK‡Î Pvjy Kiv n‡j, †PŠev”PvwUi A‡a©K cvwbc~Y© Ki‡Z KZ mgq jvM‡e?
Solution:
In 30 minutes, 1st pipe can fill 1 portion of the tank
1
In 1 minutes, 1st pipe can fill portion of the tank
30
st
In 40 minutes,1 pipe can empty 1portion of the tank
1
In 1 minutes , 1st pipe can fill portion of the tank
40
1 1
Both pipe can fill in 1 minutes = − part
30 40
4−3
=
120
1
= part
120
1portion of the tank will be filled in 120 minutes
1 1
So, 1portion of the tank will be filled in 120 ×
2 2
= 60 minutes
Answer: 60 minutes

এ রকম গুরুত্বপূর্ণ ফাইল পপতে আমাতের গ্রুতপ প াখ রাখুন

Written Math Solution (BIBM) FB Group : BD job Info. (BCS & Bank)
Bank Job Solution (BIBM) 131

Created by Sumon K. Sarkar  FB Group : BD job Info. (BCS & Bank)  Created by Sumon K. Sarkar 
5. The length of a rectangular field is 3 feet long than its breadth. If the perimeter of rectangular
Created by Sumon K. Sarkar  FB Group : BD job Info. (BCS & Bank)  Created by Sumon K. Sarkar 

field is 380 feet. What will be area of the field in square feet?
[SJIB TSO 16]
Abyev`t GKwU AvqZKvi gv‡Vi ˆ`N¨© Gi cÖ‡¯’i †P‡q 3 dzU j¤^v| AvqZ‡¶ÎwUi cwimxgv 380 dzU n‡j Gi †¶Îdj KZ eM©dzU?
Solution:
Let, the breadth of rectangular field be x feet
& length of rectangular field be x + 3 feet
Perimeter of rectangular field = 2(length + breadth)
= 2(x + x + 3)
= 2(2x+3)
= 4x+6
According to the question,
4x+6 = 380
⇒ 4x = 380 – 6
⇒ x = 374 /4
⇒ x = 93.5
∴ the breadth of rectangular field 93.5 feet
& length of rectangular field = 93.5+3 = 96.5 feet
Area of the field = length × breadth
= ( 93.5 × 96.5 )
= 9022.75 Square feet
Answer : 9022.75 Sq. feet

Shahjalal Islami Bank Ltd.


Post Name: Trainee Officer (Cash) Exam Date : 2016
𝟏 𝒙
1. If 𝒙 + = 𝟐 , then the value of
𝒙 𝒙𝟐 +𝒙−𝟏
Solution:
𝟏
𝒙+ =𝟐
𝒙
𝒙𝟐 +𝟏
⇒ =𝟐
𝒙

⇒ 𝑥 2 + 1 = 2x
⇒ 𝑥 2 − 2x + 1 = 0
⇒ (𝑥 − 1)2 = 0
⇒x=1
𝒙
So,
𝒙𝟐 +𝒙−𝟏
𝟏
=
𝟏𝟐 +𝟏−𝟏

=1
Answer: 1

Written Math Solution (BIBM) FB Group : BD job Info. (BCS & Bank)
132

 Created by Sumon K. Sarkar  FB Group : BD job Info. (BCS & Bank)  Created by Sumon K. Sarkar 
Bank Job Solution (BIBM)
 Created by Sumon K. Sarkar  FB Group : BD job Info. (BCS & Bank)  Created by Sumon K. Sarkar 

2. Age of three persons is now in the proportion 2: 3: 4 and in 5 years from now, the proportion
will be 5: 7: 9. What is the present age of the youngest person?
Abyev`t wZb e¨w³i eZ©gvb eq‡mi AbycvZ 2: 3: 4 Ges GLb †_‡K 5 eQi ci AbycvZ n‡e 5: 7: 9| me‡P‡q Kg eqmx e¨w³i
eZ©gvb eqm KZ?
Solution:
The age of the persons be 2x , 3x & 4x respectively
(2x + 5) : (3x + 5) = 5 : 7
(2x + 5) 5
⇒ =
(3x + 5) 7
⇒ 15x + 25 = 14x + 35
⇒ x = 35 – 25 = 10
Present age of the youngest person 2 × 10 = 20
Answer: 20

3. Mr. X can finish a work in 6 days and Mr. Y can finish the same work in 8 days. How many days
will it take to finish the work it they work together?
Abyev`t Mr. X GKwU KvR 6 w`‡b Ges Y GKB KvR 8 w`‡b Ki‡Z cv‡i| Zviv GK‡Î H KvR KZ w`‡b †kl Ki‡Z cvi‡e?
Solution:
In 6 Days, X can do 1 portion of the work
1
In 1 Days, X can do portion of the work
6
In 8 Days, Y can do 1 portion of the work
1
In 1 Days, Y can do portion of the work
8
1 1
In 1 Days, (X+Y) can do ( + ) part of the work
6 8
8+6 14 7
= = = part
48 48 24
7
portion of the work can do in 1 days
24
24
1 portion of the work can do in days
7
𝟐𝟒
Answer : days
𝟕

4. A shirt was sold for Tk 171 and the gain was as much percent as it costs in Taka amount. What
was the purchase price of the shirt?
Abyev`t GKwU kvU© 171 UvKvq weµq Kivq µqg~‡j¨i mgcwigvb kZKiv jvf Kiv nj| kvU©wUi µqg~j¨ KZ?
Solution:
Let, the purchase price of the shirt Tk. x & profit be x%
According to the question,
x + (x × x%) = 171
𝑥2
⇒x+ = 171
100
100𝑥+𝑥 2
⇒ = 171
100
2
⇒ 𝑥 + 100𝑥 = 17100
⇒ 𝑥 2 + 100𝑥 − 17,100 = 0

Written Math Solution (BIBM) FB Group : BD job Info. (BCS & Bank)
Bank Job Solution (BIBM) 133

Created by Sumon K. Sarkar  FB Group : BD job Info. (BCS & Bank)  Created by Sumon K. Sarkar 
⇒ 𝑥 2 + 190𝑥 − 90𝑥 − 17,100 = 0
Created by Sumon K. Sarkar  FB Group : BD job Info. (BCS & Bank)  Created by Sumon K. Sarkar 

⇒ 𝑥(𝑥 + 190) − 90(𝑥 + 190) = 0


⇒ (𝑥 − 90)(𝑥 + 190) = 0
⇒ 𝑥 = 90 𝑜𝑟 𝑥 = −190
∴ The purchase price of the shirt Tk.90
𝑨𝒏𝒔𝒘𝒆𝒓: 𝑻𝒌. 𝟗𝟎

Shahjalal Islami Bank Ltd.


Post Name: Trainee Senior Officer Exam Date : 2016
𝟏 𝟐 𝟏
1. If √𝒙 + = 𝒂, then 𝒙 + =?
√𝒙 𝒙𝟐
Solution:
1
Given that, √𝑥 + =𝑎
√𝑥
1 2
⇒ (√ 𝑥 + ) = 𝑎2
√𝑥
1
⇒ 𝑥 + + 2 = 𝑎2
𝑥
1
⇒ 𝑥 + = 𝑎2 – 2
𝑥
1 2
⇒ (𝑥 + ) = (𝑎2 – 2)2
𝑥
1
⇒ x2 + + 2 = a4 − 4a2 + 4
x2
1
⇒ x2 + = a4 − 4a2 + 4 − 2
x2
1
⇒ x2 + = a4 − 4a2 + 2
x2
Answer: 𝐚𝟒 − 𝟒𝐚𝟐 + 𝟐

2. A borrower pays 8.5% interest per year on the first Tk 600 he borrows and 7.25% per year on
the part of the loan in excess of Tk. 600. How much interest will the borrower pay on a loan of
Tk. 6500 for one year?
Abyev`t †Kvb FYMÖnxZv F‡Yi 1g 600 UvKvi Rb¨ 8.5% Ges 600 UvKvi AwZwi³ UvKvi 7.25% my` †`q| Zvi F‡Yi cwigvb
6,500 UvKv n‡j wZwb eQ‡i KZ UvKv my` w`‡eb?
Solution:
First Tk 600 interest rate 8.5%
( 6500 – 600 ) = Tk. 5900 interest rate 7.25%
1st Interest for tk. 600 = 600 × 8.5%= 51
Rest Interest for tk. 5900 = 5900 × 7.25%
= Tk. 427.75
So total interest for tk 6500 = (51 + 427.75)
= Tk. 478.75
Answer: Tk. 478.75

Written Math Solution (BIBM) FB Group : BD job Info. (BCS & Bank)
134

 Created by Sumon K. Sarkar  FB Group : BD job Info. (BCS & Bank)  Created by Sumon K. Sarkar 
Bank Job Solution (BIBM)
 Created by Sumon K. Sarkar  FB Group : BD job Info. (BCS & Bank)  Created by Sumon K. Sarkar 

3. A man's running speed is 3 times of his walking speed. The inan runs a distance and comes back
walking. Total time taken by him 2 hours. What is the distance in miles if he runs 9 miles per
hour?
[SJIB TSO 16. Modhumoti Bank PO 17]
Abyev`t GKRb †jv‡Ki †`Šov‡bvi MwZ Zvi nvUvi MwZi 3 ¸Y| †jvKwU GKwU wbw`©ó `~iZ¡ †`Š‡o wM‡q †n‡U †diZ Av‡mb| Zvi
†gvU 2 N›Uv mgq †j‡MwQj| hw` Zvi †`Šov‡bvi MwZ 9 gvBj/N›Uv nq Z‡e `~iZ¡ (gvBj) wbY©q Kiæb?
Solution:
Let, Distance be = D miles
Runs speed = 9 m/h
9
Walking speed = =3 m/h
3
𝐷 𝐷
According to the question, + =2
3 9
3𝐷+𝐷
Or, =2
9
4𝐷
Or, =2
9
Or, 𝐷 = 4.5
Answer: 4.5 miles

4. The length of a rectangular field is 30 feet long than its width and the perimeter is 380 feet. How
much would it cost to cover the field with grass at the rate of Tk. 20 per square?
[Standard Bank TAO 16]
Solution:
Let, the breadth of rectangular field be x feet
& length of rectangular field be x + 30 feet
Perimeter of rectangular field = 2(length + breadth)
= 2(x + x + 30)
=2(2x+30)
= 4x+60
According to the question,
4x+60 = 380
⇒ 4x = 380 – 60
320
⇒x=
4
⇒ x = 80
∴ the breadth of rectangular field 80 feet
& length of rectangular field 80+30 = 110 feet
Area of the field = (length × breadth)
= ( 80 × 110 ) Square feet
= 8800 Square feet
Total cost to cover the field with grass = Tk 8800 × 20
= Tk. 176,000
Answer : Tk. 176,000

Written Math Solution (BIBM) FB Group : BD job Info. (BCS & Bank)
Bank Job Solution (BIBM) 135

Created by Sumon K. Sarkar  FB Group : BD job Info. (BCS & Bank)  Created by Sumon K. Sarkar 
Shahjalal Islami Bank Ltd.
Created by Sumon K. Sarkar  FB Group : BD job Info. (BCS & Bank)  Created by Sumon K. Sarkar 

Post name: Management Trainee Officer Exam Date : 2016


1. Aman, Belal and Chad started a small business with a total amount tk 28,000. Aman paid tk 4500
more than Belal and Belal paid tk 7000 less than Chad. If the company made a profit of tk 5600
.How much profit should Belal receive?
[SUBB MTO 16, National Bank PO 17]
Abyev`t Avgvb, ‡ejvj I Pvu` †gvU 28000 UvKv wb‡q GKwU e¨emv ïiæ K‡i| ‡ejvj hZ UvKv †`q Avgvb Zvi †P‡q 4500 UvKv
†ewk †`q Ges Pvu` hZ UvKv †`q ‡ejvj Zvi †P‡q 7000 UvKv Kg †`q| hw` Zv‡`i †Kv¤úvwbi 5600 UvKv jvf nq Zvn‡j ‡ejvj
KZ UvKv jf¨vsk cv‡e?
Solution:
Let, The amount of Belal be x tk
The amount of Aman be (x+4500) tk
The amount of Chad be (x+7000) tk
According to the question,
x+x+4500+x+7000 = 28,000
⇒ 3x + 11,500 =28,000
⇒ 3x =28,000 – 11,500
⇒ 3x =16,500
⇒ x = 5500
The amount of Belal be tk 5,500
The amount of Aman be tk. (5500+4500) = tk 10,000
The amount of Chad be tk. ( 5500 + 7000) = tk. 12,500
Ratio of investment = Aman:Belal:Chad =10,000 : 5500 : 12500
=20 : 11 : 25
Sum of ratio = 20 + 11 + 25 = 56
Total profit =5600 tk
11
Belal should receive profit = 5600× = Tk. 1,100
56
Belal should receive profit Tk. 1100
Answer: Tk. 1100

𝟏𝟎 𝟏 𝟏𝟖
2. Solve the Equation: + =
𝟐𝒙−𝟓 𝒙+𝟓 𝟑𝒙−𝟓
(SJIB MTO 16, SJIB TSO 18, Premier Bank TJO 18, NRB Bank MTO 19]
Solution:
𝟏𝟎 𝟏 𝟏𝟖
+ =
𝟐𝒙−𝟓 𝒙+𝟓 𝟑𝒙−𝟓
𝟏𝟎 𝟏 𝟏𝟓+𝟑
⇒ + =
𝟐𝒙−𝟓 𝒙+𝟓 𝟑𝒙−𝟓
𝟏𝟎 𝟏 𝟏𝟓 𝟑
⇒ + = +
𝟐𝒙−𝟓 𝒙+𝟓 𝟑𝒙−𝟓 𝟑𝒙−𝟓
𝟏𝟎 𝟏𝟓 𝟑 𝟏
⇒ − = −
𝟐𝒙−𝟓 𝟑𝒙−𝟓 𝟑𝒙−𝟓 𝒙+𝟓
𝟑𝟎𝒙−𝟓𝟎−𝟑𝟎𝒙+𝟕𝟓 𝟑𝒙+𝟏𝟓−𝟑𝒙+𝟓
⇒ =
(𝟐𝒙−𝟓)(𝟑𝒙−𝟓) (𝟑𝒙−𝟓)(𝒙+𝟓)

Written Math Solution (BIBM) FB Group : BD job Info. (BCS & Bank)
136

 Created by Sumon K. Sarkar  FB Group : BD job Info. (BCS & Bank)  Created by Sumon K. Sarkar 
Bank Job Solution (BIBM)
 Created by Sumon K. Sarkar  FB Group : BD job Info. (BCS & Bank)  Created by Sumon K. Sarkar 

𝟐𝟓 𝟐𝟎
⇒ =
(𝟐𝒙−𝟓)(𝟑𝒙−𝟓) (𝟑𝒙−𝟓)(𝒙+𝟓)
𝟓 𝟒
⇒ =
(𝟐𝒙−𝟓) (𝒙+𝟓)

⇒ 8x – 20 = 5x +25
⇒ 8x –5x = 25 + 20
⇒ 3x = 45
⇒ x = 15
Answer: x =15

3. The length of a rectangular field is 1.5 times as large as its width. An amount of Tk. 10260 was
needed to cover the field with grass at Tk 1.9 per sq. ineter. How much would it cost to fence the
four sides of the rectangular field at the rate of Tk 2.5 per meter?
[ONE Bank SCO 17, Bank Asin MTO 17, SJIB MTO 16, Langka Bangla Finance MTO 17 Board Book]
Abyev`t GKwU AvqZKvi gv‡Vi ˆ`N¨© Gi cÖ‡¯’i †`o ¸Y| cÖwZ eM©wgUv‡i 1.9 UvKv wnmv‡e m¤ú~Y© gvVwU‡Z Nvm jvMv‡Z 10,260
UvKv cÖ‡qvRb| cÖwZ wgUvi 2.5 UvKv wnmv‡e gvVwUi Pvicv‡k †eov w`‡Z KZ UvKv LiP n‡e?
Solution:
Let, The width of the rectangular field be x meter
and length of the rectangular field = 1.5x meter
According to the question,
1.5x × x ×1.9= 10260 [ Total Cost ]
10,260
⇒ 𝑥2 =
1.5×1.9
⇒ 𝑥 2 = 3600
⇒ x = 60
The width of the rectangular field 60 meter
& length of the rectangular field = 1.5×60 = 90 meter
Perimeter of the rectangular field = 2(length + width)
= 2(60 + 90) meter
= 350 meter
So, total Cost to fence = 350×2.5 = Tk 750
Answer: Tk. 750

4. Rahim can do a piece of work in 80 days. After working 10 days he left and rest of the work is
done by Karim in 42 days. If they worked together how many days they need to complete the
work?
অনুবােঃ রনহম ৮০ নেতন একটা কাে করতে পাতর। 10 নেন কাে করার পর নেনন তল যান এবং বানক কাে 42 নেতন কনরম কতরন। োরা
একসতে কাে করতল কে নেতন কাে পশ্ে করতে হতব?
Solution:
Rahim can do a piece of work in 80 days
In 80 Days, Rahim can do 1 portion
1
In 1 Days, Rahim can do portion
80

Written Math Solution (BIBM) FB Group : BD job Info. (BCS & Bank)
Bank Job Solution (BIBM) 137

Created by Sumon K. Sarkar  FB Group : BD job Info. (BCS & Bank)  Created by Sumon K. Sarkar 
1×10 1
In 10 Days, Rahim can do portion = portion
80 8
Created by Sumon K. Sarkar  FB Group : BD job Info. (BCS & Bank)  Created by Sumon K. Sarkar 

1 8−1 7
Rest of the work = 1 − = = portion
8 8 8
7
Again, In 42 Days, Karim can do portion
8
7 1
In 1 Days, Karim can do = portion
8×42 48
1 1
In 1 Days, Rahim & Karim together can do + portion
40 48
3+5
= portion
240
8
= portion
240
1
= portion
30
1
So, they can do portion works in 1 day
30
30
They can do 1 part works in = 30 days
1
Answer: 30 Days

Al-Arafah Bank Ltd.


Post name: Management Trainee Officer Exam Date : 2016
𝟒 𝟗 𝟐𝟓
1. Solve the equation: + =
𝟐𝒙+𝟏 𝟑𝒙+𝟐 𝟓𝒙+𝟒
[Al-Arafah MTO 16, Modhumoti PO 18, SJIB TO Cash 18]
Solution:
4 9 25
+ =
2𝑥+1 3𝑥+2 5𝑥+4
4 9 10+15
⇒ + =
2x+1 3x+2 5x+4
4 9 10+15
⇒ + =
2x+1 3x+2 5x+4
4 9 10 15
⇒ + = +
2x+1 3x+2 5x+4 5x+4
4 10 15 9
⇒ − = −
2x+1 5x+4 5x+4 3x+2
20x+16−20x−10 45x+30−45x−36
⇒ =
(2x+1)(5x+4) (5x+4)(3x+2)
6 −6
⇒ =
(2x+1)(5x+4) (5x+4)(3x+2)
1 −1
⇒ =
(2x+1) (3x+2)

⇒ 3𝑥 + 2 = −2𝑥 − 1
⇒ 3𝑥 + 2𝑥 = −2 − 1
⇒ 5𝑥 = −3
⇒ 𝑥 = −3/5
Answer: 𝒙 = −𝟑/𝟓

Written Math Solution (BIBM) FB Group : BD job Info. (BCS & Bank)
138

 Created by Sumon K. Sarkar  FB Group : BD job Info. (BCS & Bank)  Created by Sumon K. Sarkar 
Bank Job Solution (BIBM)
 Created by Sumon K. Sarkar  FB Group : BD job Info. (BCS & Bank)  Created by Sumon K. Sarkar 

2. The average weight of three men A, B and C is 84 kg. Another man D joins the group and the
average now becomes 80 kg. If a fifth man E, whose weight is 3 kg more than that of D, replaces
A, then the average weight of B, C, D and E becomes 79 kg. What is the weight of A?
[AB Bank PO 02, One Bank PO 07, RAKUB SO 14, Al-Arafah MTO 16, Janata Bank EO(Cancelled) 17]
Abyev`t wZb e¨w³ A, B I C Gi Mo IRb 84Kg| Zv‡`i mv‡_ AviI GKRb D †hvM †`Iqvq bZzb Mo nq 80Kg|hw` A
Gi ¯’‡j D Gi †P‡q 3Kg †ewk IR‡bi cÂg e¨w³ E Zv‡`i mv‡_ †hvM †`q, Zvn‡j B, C, D Ges E Gi Mo IRb nq
79Kg| A Gi IRb KZ?
Solution:
Let, Total weight of A,B and C = 84×3 = 252 kg.
Total weight of A,B,C and D = 80 × 4 = 320 kg
∴Weight of D = 320 – 252 = 68 kg
∴ Weight of E= 68 + 3 = 71 kg
Total weight of B, C, D and E = 79×4 = 316 kg
∴ Weight of B and C = 316 – 68 – 71 = 177 kg
∴Weight of A = 252-177 = 75 kg.
Answer: 75 kg.

3. The perimeter of a square field is equal to the perimeter of a rectangle field. The length of the
rectangle is thrice the width of it and the area is 768 square meters. How many square sized tiles
of 80 centimeters will be required to cover the square filed?
[Al-Arafah MTO 16, National Bank PO 17, NRBC Bank MTO 18, Social Islami Bank PO 17, Modhumoti PO 18]
Abyev`t GKwU eM©vKvi gv‡Vi cwimxgv GKwU AvqZKvi gv‡Vi cwimxgvi mgvb| AvqZ‡¶ÎwUi ˆ`N©¨ Gi cÖ‡¯’i wZb¸Y Ges Gi
†¶Îdj 768 eM©wgUvi| eM©vKvi gvVwU‡K m¤ú~Y©iƒ‡c XvK‡Z 80 †m.wg KZ¸‡jv eM©vKvi UvBjm jvM‡e?
Solution:
Let, The width of the rectangle be x meter
So, The length of the rectangle be 3x meter
According to the question,
3x × x=768
or, 𝑥 2 = 256
or, x = 16
So,The perimeter of the rectangle = 2(3x + x)
=2(3×16+16)
=128
So, the perimeter of the square is 128 meters
Let, Each Side of square is y
According to the question,
4y=128
128
or, 𝑦 =
4
or, y = 32
So, area of the square be 𝑦 2 = 322
=1024 squar meter
=1024×10000 square cm
=10240000 square cm

Written Math Solution (BIBM) FB Group : BD job Info. (BCS & Bank)
Bank Job Solution (BIBM) 139

Created by Sumon K. Sarkar  FB Group : BD job Info. (BCS & Bank)  Created by Sumon K. Sarkar 
Again, area of the tiles = 802
Created by Sumon K. Sarkar  FB Group : BD job Info. (BCS & Bank)  Created by Sumon K. Sarkar 

= 6400 square cm
102,40,000
Number of tiles =
6400
=1600 unit
Answer:1600 unit

4. A, B and C started a business jointly with a total amount Tk. 28000. A paid Tk. 4500 more than
B and B paid Tk. 7000 less than C. If the company made a profit of Tk 5600, how much profit
should C receive? [Al-Arafah MTO 16]
Abyev`t A, B Ges C †gvU 28,000 UvKv wb‡q GKwU e¨emv ïiæ K‡i| C hZ UvKv †`q B Zvi †P‡q 7,000 UvKv Kg †`q Ges
B hZ UvKv †`q A Zvi †P‡q 4,500 UvKv †ewk †`q| Zv‡`i †Kv¤úvwbi 5,600 UvKv jvf n‡j, C KZ UvKv jf¨vsk cv‡e?
Solution:
Let, The amount of B be x tk
The amount of A be (x+4500) tk
The amount of C be (x+7000) tk
According to the question,
x+x+4500+x+7000 = 28,000
⇒ 3x + 11,500 =28,000
⇒ 3x =28,000 – 11,500
⇒ 3x =16,500
⇒ x = 5500
The amount of B be tk 5,500
The amount of Aman be tk. (5500+4500) = tk 10,000
The amount of C be tk. ( 5500 + 7000)= tk. 12,500
Ratio of investment = A : B : C
=10,000 : 5500 : 12500
=20 : 11 : 25
Sum of ratio = 20 + 11 + 25 = 56
Total profit =5600 tk
25
C should receive profit = 5600× = Tk. 2,500
56
C should receive profit Tk. 2,500
Answer: Tk. 2,500

এ রকম গুরুত্বপূর্ণ ফাইল পপতে আমাতের গ্রুতপ প াখ রাখুন


এবং অবশ্যই ননতের বন্ধুতের ইনভাইট করুন

Written Math Solution (BIBM) FB Group : BD job Info. (BCS & Bank)
140

 Created by Sumon K. Sarkar  FB Group : BD job Info. (BCS & Bank)  Created by Sumon K. Sarkar 
Bank Job Solution (BIBM)
 Created by Sumon K. Sarkar  FB Group : BD job Info. (BCS & Bank)  Created by Sumon K. Sarkar 

Bank Asia LTD.


Post name: Trainee Officer Exam Date : 2016
1. Average mark obtained by 300 students in a class is 80. When 10% of the students are excluded
from the group, the average mark of remaining students is 85. Find average mark of the 10% of
students excluded from the group?
অনুবােঃ একনট ক্লাতস 300 েন নশ্োর্ণীর প্রাপ্ত গড নম্বর হল 80। যখন 10% ছাত্রতক েল পর্তক বাে পেওযা হয, েখন বানক ছাত্রতের গড
নম্বর 85 হয। গ্রুপ পর্তক বাে পডা 10% ছাতত্রর গড নম্বর কে?
Solution:
Total mark of 300 student = 300 × 80 = 24,000
Excluded students = 300 × 10% = 30
Remaining Students = 300 – 30 = 270
So, Total mark of 270 student = 270 × 85 = 22,950
24,000−22,950 1050
Average mark of the 10% of excluded students = = = 35
30 30
Answer: 35

2. The ratio of gold and silver in an ornament weighing 42 gram 4:3. How much gold will need to
be added to make the ratio of gold and silver 5:3?
অনুবােঃ 42 গ্রাম ওেতনর একনট অলঙ্কাতর পসানা ও পরৌতপযর অনুপাে 4:3। স্বর্ণ ও রূপার অনুপাে 5:3 হওযার েনয কে পসানা পযাগ
করতে হতব?
Solution:
Given that, The ratio of gold & silver = 4:3
Sum of the ratio = 4+3 = 7
4
The amount of gold = 42 gm × = 24 gm
7
3
The amount of silver = 42 gm × = 18 gm
7
Let, X gm gold will need to be added.
According to the question,
(24 +X ): 18 = 5:3
(24+𝑋) 5
⇒ =
18 3
(24+𝑋) 5
⇒ =
6 1
⇒ 24 + X = 30
⇒ X = 30 - 24
⇒ X=6
Answer: 6 gm gold will need to be added

3. A cistern can be filled in 10 hours but it takes 12 hours due to a leak in its bottom. If the cistern
is full, then how much time the leak will take to empty the cistern?
অনুবােঃ একনট প ৌবাচ্চা 10 ন্টায ভরাট করা যায েতব এনটর নীত ফুতটা হওযার কারতর্ এনট 12 ন্টা সময পনয। োহতল পূর্ণ প ৌবাচ্চানট
খানল হতে কে সময লাগতব?
Given that, When there is no leakage A pipe can fill the cistern in 10 hours
1
In 1 hour without leakage A pipe can fill the cistern in = hours
10

Written Math Solution (BIBM) FB Group : BD job Info. (BCS & Bank)
Bank Job Solution (BIBM) 141

Created by Sumon K. Sarkar  FB Group : BD job Info. (BCS & Bank)  Created by Sumon K. Sarkar 
When there is leakage cistern gets filled in 12 hours
Created by Sumon K. Sarkar  FB Group : BD job Info. (BCS & Bank)  Created by Sumon K. Sarkar 

1
In 1 hour, when there is leakage cistern gets filled in = hours
12
1 1 12−10 2 1
In 1 hour, due to leakage cistern gets filled to = − = = = Part
10 12 120 120 60
1
∴ Due to leakage the cistern gets emptied in = = 60 hours.
1/60
Answer: 60 hours.

𝟖 𝟗 𝟕
4. Solve the equation: + =
𝟐𝒙−𝟏 𝟑𝒙−𝟏 𝒙+𝟏
8 9 7
+ =
2𝑥−1 3𝑥−1 𝑥+1
24𝑥−8+18𝑥−9 7
⇒ =
(2𝑥−1)(3𝑥−1) 𝑥+1
42𝑥−17 7
⇒ =
6𝑥 2 −5𝑥+1 𝑥+1
⇒ 42𝑥 + 25𝑥 − 17 = 42𝑥 2 − 35𝑥 + 7
2

⇒ 25𝑥 + 35𝑥 = 17 + 7
⇒ 60x =24
24
⇒ 𝑥=
60
2
⇒ 𝑥=
5
𝟐
𝑨𝒏𝒔𝒘𝒆𝒓: 𝒙 =
𝟓

Bangladesh Bank
Post Name: Officer (General) Exam Date : 2015
√𝟐+𝒙+√𝟐−𝒙
1. If = 𝟐 , 𝒇𝒊𝒏𝒅 𝒕𝒉𝒆 𝒗𝒂𝒍𝒖𝒆 𝒐𝒇 𝒙.
√𝟐+𝒙−√𝟐−𝒙
Solution:
√2+𝑥+√2−𝑥
Given That, =2
√2+𝑥−√2−𝑥

√2+𝑥+√2−𝑥+√2+𝑥−√2−𝑥 2+1
⇒ =
√2+𝑥+√2−𝑥−√2+𝑥+√2−𝑥 2−1

2√2+𝑥 3
⇒ =
2√2−𝑥 1

√2+𝑥
⇒ =3
√2−𝑥
2
√2+𝑥
⇒( ) =9
√2−𝑥
2+𝑥
⇒ =9
2−𝑥

⇒ 2 +x = 18 – 9x
⇒ x + 9x = 18 – 2
⇒ 10x = 16
16 8
⇒x= =
10 5

Written Math Solution (BIBM) FB Group : BD job Info. (BCS & Bank)
142

 Created by Sumon K. Sarkar  FB Group : BD job Info. (BCS & Bank)  Created by Sumon K. Sarkar 
Bank Job Solution (BIBM)
 Created by Sumon K. Sarkar  FB Group : BD job Info. (BCS & Bank)  Created by Sumon K. Sarkar 

𝟖
Answer:
𝟓

2. If the sum of five consecutive integers is S, what is the largest of those integers in terms of S?
[BB Officer 15, PKB EO Cash 18]
Abyev`t 5wU µwgK msL¨vi mgwó S n‡j, e„nËg msL¨vwU‡K S Gi gva¨‡g cÖKvk Kiæb|
Solution:
Let, 5 Consecutive integers are x – 2 , x – 1 , x , x + 1, x + 2
According to the question,
x–2+x–1+x+x+1+x+2=S
⇒ 5x = 𝑆
𝑆
⇒x=
5
𝑆
largest integers = + 2
5
𝑺
Answer: + 𝟐
𝟓

3. Of the three numbers, second is twice the first and is also thrice the third. If the average of the
three numbers is 44, then what will be the largest number?
Abyev`t wZbwU msL¨vi g‡a¨ 2q msL¨vwU 1g msL¨vi wظY Ges 3q msL¨vi wZb¸Y| hw` msL¨v wZbwUi Mo 44 nq, Zvn‡j e„nËg
msL¨vwU KZ?
Solution:
2𝑥
Let, 1st Number be x, 2nd Number be 2x and 3rd Number be
3
According to the question,
2𝑥
x + 2x + = 44 ×3
3
3𝑥+6𝑥+2𝑥
⇒ ( ) =132
3
⇒ 11x =396
⇒ x=36.
So, the largest number is = 2×36 = 72
Answer:72

4. The difference between two numbers is five and the difference of their squares is 65. What is the
larger number?
Abyev`t `ywU msL¨vi AšÍi 5 Ges Zv‡`i e‡M©i AšÍi 65| eo msL¨vwU KZ?
Solution:
Let, Larger number be x and smaller number be y
According to the question,
x – y = 5.................................................(1)
x2 – y2 = 65 …........................................(2)
⇒ (x + y) (x - y) = 65
⇒ (x + y) × 5= 65
⇒ x + y = 13..........................................(3)

Written Math Solution (BIBM) FB Group : BD job Info. (BCS & Bank)
Bank Job Solution (BIBM) 143

Created by Sumon K. Sarkar  FB Group : BD job Info. (BCS & Bank)  Created by Sumon K. Sarkar 
Adding the equation (1) & (2) we get,
Created by Sumon K. Sarkar  FB Group : BD job Info. (BCS & Bank)  Created by Sumon K. Sarkar 

x – y + x + y = 5 + 13
⇒2x = 18
⇒x=9
Put the value of x in (1) Equation,
9-y=5
⇒y=4
So, larger number = 9
Answer: 9

5. The annual incomes and expenditures of a man and his wife are in the ratios 5:3 and 3:1,
respectively. If they decide to save equally and find a balance of Tk. 4000 at the end of the year,
what was their income?
Abyev`t GKRb e¨w³ Ges Zvi ¯¿x evwl©K Avq Ges e¨‡qi AbycvZ h_vµ‡g 5 t 3 Ges 3 t 1| hw` Zviv mgvb fv‡e UvKv Rgv‡bvi
wm×všÍ †bq Ges eQi †k‡l †gvU 4,000 UvKv mÂq K‡i Zvn‡j Zv‡`i Avq KZ?
Solution:
Let, Man’s income be 5x and expenditure 3x
& his wife’s income be 3x and expenditure x.
According to the question,
(5x - 3x) + (3x - x) = 4000
⇒ 2x + 2x = 4000
⇒ 4x = 4000
⇒ x = 1000
So, Man’s income 5x = 5×1000 =Tk. 5000
his wife’s income 3x = 3×1000 =Tk. 3000
Answer: Tk. 5000 & Tk. 3000

6. A shop stocks four types of caps, there are 1/3 as many red caps as blue caps and ½ as many
green caps as red caps. There are equal number of green caps and yellow caps. If there are 42
blue caps, then what percent of the total caps in the shop are blue?
Abyev`t GKwU †`vKv‡b 4 ai‡bi K¨vc Av‡Q| hZ¸‡jv bxj K¨vc Av‡Q Zvi 1/3 fvM jvj K¨vc Av‡Q Ges hZ¸‡jv jvj K¨vc
Av‡Q Zvi 1/2 fvM meyR K¨vc Av‡Q| hZ¸‡jv meyR K¨vc Av‡Q ZZ¸‡jv njy` K¨vc Av‡Q | hw` 42wU bxj K¨vc _v‡K Zvn‡j
†`vKv‡bi †gvU K¨v‡ci kZKiv KZwU bxj K¨vc Av‡Q ?
Solution:
Given that, Blue caps = 42
42
Red caps = =14
3
14
Green caps = =7
2
Yellow caps =7
So, Total caps be 42+14+7 +7=70
42
Percentage of blue caps = × 100 = 60%
70
Answer: 60%

Written Math Solution (BIBM) FB Group : BD job Info. (BCS & Bank)
144

 Created by Sumon K. Sarkar  FB Group : BD job Info. (BCS & Bank)  Created by Sumon K. Sarkar 
Bank Job Solution (BIBM)
 Created by Sumon K. Sarkar  FB Group : BD job Info. (BCS & Bank)  Created by Sumon K. Sarkar 

7. A person sold two articles. Each for the same price of Tk. 1040. He incurs 20% loss on the first
and 10% loss on the second. Find his overall percentage of loss.
Abyev`t †Kvb e¨w³ 2 ai‡bi wRwbm weµq K‡ib| cÖ‡Z¨KwUi g~j¨ 1,040 UvKv| cÖ_gwU‡Z 20% Ges 2q wU‡Z 10% ¶wZ nq|
Zvi †gvU kZKiv ¶wZi cwigvb KZ?
Solution:
Let, Price be Tk.100
At 20% loss, selling price =100 –20 =Tk.80
& at 10% loss selling price = 100-10=Tk. 90
1040
So, cost price of 1st item = ×100 = Tk.1300
80
1040
& cost price of second item = ×100 = Tk.1155.55
90
nd
Total cost of 2 items =Tk. (1300 + 1155.55) =Tk. 2455.55
But selling price=1040 + 1040 =Tk. 2080.
So, loss =Tk. (2455.55 -2080)=Tk. 375.55
375.55
So, overall loss percentage = ×100 = 15.29%
2455.55
Answer:15.29%

8. Robi drove 100 miles to visit a friend. If he had driven 8 miles per hour faster than he did, he
would have arrived in 5/6 of the time, he actually took. How many minutes did the trip take?
Abyev`t iwe 100 gvBj Mvwo Pvwj‡q Zvi eÜzi mv‡_ †`Lv Ki‡Z hvq| hw` †m †h ‡e‡M Mvwo Pvjvq Zvi †P‡q AviI 8 gvBj/N›Uv
†ewk †e‡M Mvwo PvjvZ Z‡e †m †h mg‡q eÜzi evmvq †cŠQvq Zvi †P‡q 5/6 ¸Y Kg mg‡q †cŠQv‡Z cvi‡Zv| eÜzi evmvq †h‡Z Zvi
KZ wgwbU mgq †j‡MwQj?
Solution:
Let, Robi took x hours to cover 100 miles
100
Actual speed = mph
𝑥
100
New speed =( + 8) mph
𝑥
5 5𝑥
New time taken = x × = hours.
6 6
We know,
Speed × Time =Distance
100 5𝑥
( + 8) × = 100
𝑥 6
500 20𝑥
⇒ + = 100
6 3
500+40𝑥
⇒ = 100
6
⇒ 500 + 40𝑥 = 600
⇒ 40𝑥 = 600 − 500
⇒ 40𝑥 = 100
100 5
⇒𝑥 = = hours
40 2
5×60
⇒x= mins.
2
⇒ x =150 mins.
Answer:150 mins

Written Math Solution (BIBM) FB Group : BD job Info. (BCS & Bank)
Bank Job Solution (BIBM) 145

Created by Sumon K. Sarkar  FB Group : BD job Info. (BCS & Bank)  Created by Sumon K. Sarkar 
Bangladesh Bank
Created by Sumon K. Sarkar  FB Group : BD job Info. (BCS & Bank)  Created by Sumon K. Sarkar 

Assistant Director (FF) Exam Date : 2015


𝟏 𝟔 𝟏
1. If 𝒙 + = 𝟑, then 𝒙 + =? [BB AD(ff) 15] [Senior Officer -2018]
𝒙 𝒙𝟔
Solution:
1
Given that, 𝑥 + = 3
𝑥
1 2
⇒(𝑥 + ) = 32
𝑥
1
⇒ 𝑥2 + +2=9
𝑥2
1
⇒ 𝑥2 + =9−2
𝑥2
1
⇒ 𝑥2 + =7
𝑥2
1 3
⇒ (𝑥 2 + ) = 73
𝑥2
1 1 1
⇒ 𝑥6 + + 3. 𝑥 2 . (𝑥 2 + ) = 343
𝑥6 𝑥2 𝑥2
1
⇒ 𝑥6 + + 3.7 = 343
𝑥6
1
⇒ 𝑥6 + = 343 − 21
𝑥6
1
⇒ 𝑥6 + = 322
𝑥6
Answer: 322

2. The average age of students of a class is 15.8 years. The average age of boys in the class is 16.4
years and of the girls is 15.4 years. Find the ratio of number of boys to the number of girls in the
class.
Abyev`t GKwU †kªbx‡Z wk¶v_©x‡`i Mo eqm 15.8 eQi| H †kªYxi evjK‡`i Mo eqm 16.4 eQi Ges evwjKv‡`i Mo eqm 15.4
eQi| H †kªbx‡Z evjK Ges evwjKv‡`i msL¨vi AbycvZ wbY©q Ki|
Solution:
Let, the number of boys be x & girls be y
Total Student x + y
According to the question,
16.4x + 15.4y = 15.8(x + y)
⇒ 16.4x + 15.4y = 15.8x + 15.8y
⇒ 16.4x - 15.8x = 15.8y - 15.4y
⇒ 0.6x = 0.4 y
⇒ 6x = 4y
⇒ 3x = 2y
𝒙 𝟐
⇒ =
𝒚 𝟑
⇒x:y=2:3
Answer : 2 : 3

Written Math Solution (BIBM) FB Group : BD job Info. (BCS & Bank)
146

 Created by Sumon K. Sarkar  FB Group : BD job Info. (BCS & Bank)  Created by Sumon K. Sarkar 
Bank Job Solution (BIBM)
 Created by Sumon K. Sarkar  FB Group : BD job Info. (BCS & Bank)  Created by Sumon K. Sarkar 

3. The percentage profit earned by selling an article for Taka 1920 is equal to the percentage loss
incurred by selling the same article for Taka 1280. At what price should the article be sold to
make a 25% profit?
[South East Bank MTO 2013, BB AD(freedom fighter) 2015, SJIB MTO 2013, IBBL PO 2019, Sadharan Bima JO 2019, Sadharan Bima
AM 2019, BB AD -AME - 2023]
Abyev`t GKwU cY¨ 1,920 UvKvq weµq Ki‡j kZKiv hZ jvf nq, cY¨wU 1,280 UvKvq weµq Ki‡j kZKiv ZZ UvKv ¶wZ
nq| 25% jvf Ki‡Z n‡j cY¨wU‡K KZ `v‡g weµq Ki‡Z n‡e?
Solution:
Let, cost price an article be Tk. x
According to the question,
1920-x = x-1280 [ Selling Price – Cost price ]
⇒ 1920+1280 = x+ x
⇒ 2x = 3200
∴ x = 1600
Cost price an article is Tk. 1600
At 25% profit, new selling price an article = Tk. (1600+25% of 1600) = Tk. 2000
Answer: Tk. 2000

Alternative Way,
Let, Cost Price be Tk. x
Then, According to the question,
1920−𝑥 x−1280
×100 = ×100
𝑥 𝑥
⇒ 1920 - x = x - 1280
⇒2x = 3200
⇒ x = 1600
Cost Price be Tk. 1600
∴ Required Selling Price = 125% of Tk. 1600
125
= × 1600
100
= Tk. 2000
Answer: Tk. 2000

এ রকম গুরুত্বপূর্ণ ফাইল পপতে আমাতের গ্রুতপ প াখ রাখুন


এবং অবশ্যই ননতের বন্ধুতের ইনভাইট করুন

Written Math Solution (BIBM) FB Group : BD job Info. (BCS & Bank)
Bank Job Solution (BIBM) 147

Created by Sumon K. Sarkar  FB Group : BD job Info. (BCS & Bank)  Created by Sumon K. Sarkar 
4. A can do a work in 10 days, while B alone can do it in 15 days. They work together for 5 days
Created by Sumon K. Sarkar  FB Group : BD job Info. (BCS & Bank)  Created by Sumon K. Sarkar 

and rest of the work is done by C in 2 days. If they get Tk. 4500 for whole work, how should they
divide money?
[BASIC Bank AO Cash 14, BB AD(ft) 15]
Abyev`t A GKv GKwU KvR 10 w`‡b Ki‡Z cv‡i †hLv‡b B GKv H KvRwU 15 w`‡b K‡i| Zviv GKm‡½ 5 w`b KvR K‡i Ges
evKx KvR C GKv 2w`‡b m¤úbœ K‡i| m¤ú~Y© Kv‡Ri Rb¨ 4,500 UvKv †`qv n‡j Zviv H UvKv wb‡R‡`i g‡a¨ wKfv‡e fvM K‡i
‡b‡e?
Solution:
5 5
In 5 Days, (A+B) work ( + ) Portion
10 15
1 1
= + Portion
2 3
3+2
= Portion
6
5
= Portion
6
5
In 2 Days C does work = 1 - Portion
6
1
= Portion
6
1
A’s get = 4500 × = Tk. 2250
2
1
B’s get = 4500 × = Tk. 1500
3
1
C’s get = 4500 × = Tk. 750
6

Alternative Way,
Let, the total work be 30 units
So , Efficiency , A : B ⇒ 3 : 2
In 5 days , Done by A ⇒ 3×5 = 15 units
In 5 days , Done by B ⇒ 2×5 = 10 units
Remaining work done by C [30 – (15+10)] = 5 units ,
So , Work ratio , A : B : C = 15 : 10 : 5 = 3 : 2 : 1
Therefore , A : B : C = (3 : 2 : 1)×750
= 2250 : 1500 : 750
Answer: 2250 : 1500 : 750

5. A bus hired at the cost of Tk. 2400 and it was decided that every student would share the cost
equally. But 10 more students jointed and as a result the fare decreased by Tk. 8 per person.
How many students were travelling in the bus?
Abyev`t 2,400 UvKv w`‡q GKwU evm fvov Kiv nq Ges cÖ‡Z¨K QvÎ mgcwigvb Puv`v †`‡e wVK Kiv nq| wKš‘ 10 Rb AwZwi³
QvÎ †hvM †`qvq Puv`vi cwigvb 8 UvKv n«vm cvq| KZRb QvÎ ev‡m hv‡”Q?
Solution:
Let, The number of students be x & travelling in the bus x + 10
According to the question,
2400 2400
+ =8
𝑥 𝑥+10

Written Math Solution (BIBM) FB Group : BD job Info. (BCS & Bank)
148

 Created by Sumon K. Sarkar  FB Group : BD job Info. (BCS & Bank)  Created by Sumon K. Sarkar 
Bank Job Solution (BIBM)
 Created by Sumon K. Sarkar  FB Group : BD job Info. (BCS & Bank)  Created by Sumon K. Sarkar 

2400𝑥+24000−2400𝑥
⇒ =8
𝑥(𝑥+10)
24000
⇒ =8
𝑥(𝑥+10)
3000
⇒ =1
𝑥(𝑥+10)
2
⇒ 𝑥 + 10𝑥 = 3000
⇒ 𝑥 2 + 10𝑥 − 3000 = 0
⇒ 𝑥 2 + 60𝑥 − 50𝑥 − 3000 = 0
⇒ 𝑥(𝑥 + 60) − 50(𝑥 + 60) = 0
⇒ (𝑥 + 60)(𝑥 − 50) = 0
So, x = - 60 (not accectable), x = 50
The number of students travelling in the bus = 50 + 10 = 60
Answer: 60

Midland Bank Ltd.


Post name: Trainee Officer Exam Date : 2015
1. A person earns yearly interest of Tk. 920 by investing Tk. X at 4% and Tk. Y at 5% simple
interest rate. If he had invested Tk. X at 5% and Tk. Y at 4% simple interest rate, then his yearly
interest earning would have been reduced by Tk. 40. Find out the amount of X and Y.
[ONE Bank PO 10, NCC Bank MTO 11, Premier Bank MTO 12, Midland Bank TO 15]
Abyev`t GK e¨w³ 4% my‡` x UvKv Ges 5% my‡` y UvKv wewb‡qvM K‡i eQ‡i 920 UvKv gybvdv AR©b K‡ib| wZwb hw` 5% my‡`
x UvKv Ges 4% my‡` y UvKv wewb‡qvM K‡ib Zvn‡j gybvdvi cwigvb 40 UvKv K‡g hv| x Ges y Gi cwigvb wbY©q Ki|
Solution:
According to the question,
X×4%+Y×5% = 920
⇒ 4X+5Y = 92000 ...................................(1)
And, X×5% + Y×4%=880
⇒ 5X+4Y = 88000....................................(2)
Now, subtraction the equation (1)×5 to (2)×4
20X+25Y – 20X – 16Y = 460,000 – 352,000
⇒ 9Y =108,000
⇒Y = 12000.
Putting the value of y in equation (1)
4X+5×12000 = 92000
⇒ 4X+60,000 = 92,000
⇒ 4X = 92,000 - 60,000
⇒ 4X = 32,000
⇒ X=8000
Answer: X=8000 tk and Y= 12,000 tk

Written Math Solution (BIBM) FB Group : BD job Info. (BCS & Bank)
Bank Job Solution (BIBM) 149

Created by Sumon K. Sarkar  FB Group : BD job Info. (BCS & Bank)  Created by Sumon K. Sarkar 
2. A basketball team has won 15 games and lost 9. If these games represent 16.67% of the games to
Created by Sumon K. Sarkar  FB Group : BD job Info. (BCS & Bank)  Created by Sumon K. Sarkar 

the played, then how many more games must the team win to average 75% for the season?
[ONE Bank PO 10, NCC Bank MTO 11, Premier Bank MTO 12, Midland Bank TO 15, Bangladesh bank AP 2023]
Abyev`t GKwU ev‡¯‹Uej wUg 15wU †Ljvq Rqjvf K‡i Ges 9wU †Ljvq civwRZ nq| hw` GB †Ljv¸‡jv Øviv Zv‡`i †gvU ‡Ljvi
16.67% †Ljv‡K †evSv‡bv nq Zvn‡j H wmR‡b M‡o 75% †Ljvq †RZvi Rb¨ Zv‡`i‡K KZwU †Ljvq wRZ‡Z n‡e?
Solution:
Total game that have already been played=15+9=24.
According to the question,
2 50
16 = % = 24
3 3
24×100
Or,100% = =144.
50/3
24×100×3
Or,100% = =144.
50
To win 75% of the game the team need to win = 144×75% = 108.
So it needs to win (108-15) = 93 more games.
Answer: 93 more games

Midland Bank Ltd.


Post name: Management Trainee Officer Exam Date : 2015
1. In a country, 60% of the male citizen and 70% of the female citizen are eligible to vote. 70% of
the male and 60% of female citizen are eligible to cast their vote. What fraction of citizens voted
during their election?
[Janata Bank EO 12, South East Bank MTO 13, SJIB MTO 13, Midland Bank MTO 15]
Abyev`t GKwU †`‡ki 60% cyiæl Ges 70% gwnjv †fvU w`‡Z cvi‡eb| G‡`i g‡a¨ 70% cyiæl Ges 60% gwnjv †fvU `vb
K‡ib| KZRb bvMwiK wbe©vP‡bi mgq †fvU cÖ`vb K‡ib?
Solution:
Total number of Male citizens be 100
& Total number of Female citizens be 100
So, total citizens = 100 +100 = 200
So, Eligible Male voters = 100 × 60% = 60
Eligible Female voters = 100 × 70% =70
Casted male votes = 60 × 70% = 42
Casted Female votes = 70 × 60% = 42
42+42 84 21
∴Fraction of citizens voted = = =
100+100 200 50
𝟐𝟏
Answer :
𝟓𝟎

এ রকম গুরুত্বপূর্ণ ফাইল পপতে আমাতের গ্রুতপ প াখ রাখুন


এবং অবশ্যই ননতের বন্ধুতের ইনভাইট করুন

Written Math Solution (BIBM) FB Group : BD job Info. (BCS & Bank)
150

 Created by Sumon K. Sarkar  FB Group : BD job Info. (BCS & Bank)  Created by Sumon K. Sarkar 
Bank Job Solution (BIBM)
 Created by Sumon K. Sarkar  FB Group : BD job Info. (BCS & Bank)  Created by Sumon K. Sarkar 

2. A father has divided his property between his two sons A and B. A invests the amount at a
compound profit of 8%. B invests the amount of 10% simple profit. At the end of 2 years, the
profit received by Bis Tk. 1336 more than A. Find the amount of both. Total amount of his father
is Tk. 25000.
[Midland Bank MTO 15, SIBL PO 13, BB AME-2023]
Abyev`t †Kvb wcZv Zvi m¤úwË Zvi `yB cyÎ A I B Gi g‡a¨ fvM K‡i †`q| A Zvi UvKv 8% Pµe„w× gybvdvq Ges B Zvi UvKv
10% mij gybvdvq wewb‡qvM K‡i| 2 eQi c‡i A Gi †P‡q B 1,336 UvKv †ewk gybvdv AR©b K‡i| Df‡qi UvKvi cwigvb wbY©q
Ki| Zv‡`i wcZvi Kv‡Q 25,000 UvKv wQj|
Solution:
Let, Share of A be x Tk
& Share of B be (25000-x) Tk
A's profit = 𝑃(1 + 𝑟)2 − 𝑃
= 𝑥(1 + 8%)2 − 𝑥
8 2
= 𝑥 (1 + ) −𝑥
100
100+8 2
=𝑥( ) −𝑥
100
108 2
=𝑥( ) −𝑥
100
= 1.1664x – x
= 0.1664x
B's profit = (25000-x) ×2×10%
10
=(25000-x) ×2 ×
100
1
=(25000-x) ×
5
= 5000 – 0.2x
According to the question,
5000 – 0.2x - 0.1664x =1336
⇒ 0.2x+0.1664x = 5000 – 1336
⇒ 0.3664x = 3664
⇒ x =10000
So, Share of A=10,000 Tk and share of B =25,000 - 10,000 = 15,000 Tk
Answer: 10,000 Tk and 15,000 Tk

এ রকম গুরুত্বপূর্ণ ফাইল পপতে আমাতের গ্রুতপ প াখ রাখুন


এবং অবশ্যই ননতের বন্ধুতের ইনভাইট করুন

Written Math Solution (BIBM) FB Group : BD job Info. (BCS & Bank)
Bank Job Solution (BIBM) 151

Created by Sumon K. Sarkar  FB Group : BD job Info. (BCS & Bank)  Created by Sumon K. Sarkar 
National Bank Ltd.
Created by Sumon K. Sarkar  FB Group : BD job Info. (BCS & Bank)  Created by Sumon K. Sarkar 

Post name: Probationary Officer Exam Date : 2015


𝟐 𝟏
1. If 𝒙 = 𝟑 + √𝟖 then find the value of 𝒙 +
𝒙𝟐
Solution:
Given that, 𝑥 = 3 + √8
1 1
⇒ =
𝑥 3+√8

1 (3−√8)
⇒ =
𝑥 (3+√8)(3−√8)

1 (3−√8)
⇒ =
𝑥 (3)2 −(√8)2
1
⇒ = (3 − √8)
𝑥
1
So, 𝑥 + = 3 + √8 + 3 − √8
𝑥
1
⇒𝑥+ =6
𝑥
1 2
⇒ (𝑥 + ) = 62
𝑥
1 1
⇒ 𝑥 2 + 2𝑥 2 + = 36
𝑥2 𝑥2
1
⇒ 𝑥2 + 2 + = 36
𝑥2
1
⇒ 𝑥2 + = 36 − 2
𝑥2
1
⇒ 𝑥2 + = 34
𝑥2

Answer: 34

2. A sum of money is to be distributed equally among a group of children. If there were 25 children
less than each would get Tk. 1.50 more, and if there 50 children inore, each would get TK 1.50
less. Find the number of children and the amount of money distributed.
[One Bank SCO 18, National Bank PO 15]
Abyev`t KZ¸‡jv wkïi g‡a¨ wKQz UvKv mgvbfv‡e fvM K‡i †`qv nq| hw` 25wU wkï Kg _vKZ Zvn‡j cÖ‡Z¨‡K 1.50 UvKv †ewk
†cZ Ges hw` 50wU wkï †ewk _vKZ Zvn‡j cÖ‡Z¨‡K 1.50 UvKv Kg †cZ| KZwU wkï wQj Ges KZ UvKv †`qv n‡q‡Q, Zv wbY©q
Ki|
Solution:
Let, No. of Children be x
Each Children get y
The amount of money distributed xy
According to the 1st condition,
(x-25)(y+1.5) = xy
⇒ xy+1.5x -25y-37.5=xy
⇒ 1.5x - 25y - 37.5 = 0

Written Math Solution (BIBM) FB Group : BD job Info. (BCS & Bank)
152

 Created by Sumon K. Sarkar  FB Group : BD job Info. (BCS & Bank)  Created by Sumon K. Sarkar 
Bank Job Solution (BIBM)
 Created by Sumon K. Sarkar  FB Group : BD job Info. (BCS & Bank)  Created by Sumon K. Sarkar 

⇒ 3x - 50y – 75 = 0 ………………….(1)
According to the 2nd condition,
(x+50)(y-1.5)=xy
⇒ xy - 1.5x + 50y – 75 = xy
⇒ -3x + 100y – 150 = 0 ………………….(2)
Now, adding the equation (i) & (ii)
50y – 225 = 0
⇒ y = 225/50
⇒ y = 4.5
Put the value of y in equation (1)
3x - 50×4.5 – 75 = 0
⇒ 3x – 225 – 75 = 0
⇒ 3x – 300 = 0
⇒ 3x = 300
⇒ x = 100
No. of Children 100
The amount of money distributed = Tk. (100 × 4.5)
= Tk. 450
Answer: Tk. 450

3. A video magazine distributor made 3500 copies of the May issue of the magazine at a cost of Tk.
4,00,000. He gave 500 cassettes free to some key video libraries. He also allowed a 25% discount
on the market price of the cassette. In this manner, he was able to sell all the 3500 cassettes that
were produced. If the market price of a cassettes was Tk. 160, what is his gain or loss for the May
issue of the video magazine?
Abyev`t GKwU wfwWI g¨vMvwRb cwi‡ekK g¨vMvwRw›Ui †g msL¨vi 3500 Kwc Qvcvb 400000 UvKvq| wZwb 500wU K¨v‡mU
webvg~‡j¨ w`‡jb wKQz cÖavb cÖavb wfwWI jvB‡eªix‡Z| GQvovI wZwb g~j weµqg~‡j¨i Dci AviI 25% gyj¨ Qvo w`‡jb| Gfv‡e
wZwb me¸‡jv K¨v‡mU wewµ Ki‡Z m¶g n‡jb| hw` g~j weµqg~j¨ 160 UvKv nq, Zvn‡j H gv‡m g¨vMvwRb wewµ †_‡K Zvi KZ
jvf ev ¶wZ n‡qwQj?
Solution:
As the distributor gives 500 copy free,
The remaining number of copy =3500-500 = 3000
Market price of each cassette was Tk. 160.
So, at 25% discount, selling price = 160 -160×25% =Tk. 120.
Now selling price of 3000 cassettes = 120×3000 =Tk. 360,000
Cost of 3500 cassettes =Tk. 400000
So, loss = 4,00,000 - 3,60,000 = Tk. 40,000.
Answer: Tk. 40,000

Written Math Solution (BIBM) FB Group : BD job Info. (BCS & Bank)
Bank Job Solution (BIBM) 153

Created by Sumon K. Sarkar  FB Group : BD job Info. (BCS & Bank)  Created by Sumon K. Sarkar 
National Bank Ltd.
Created by Sumon K. Sarkar  FB Group : BD job Info. (BCS & Bank)  Created by Sumon K. Sarkar 

Post name: Probationary Officer Exam Date : 2014


1. The speed of a railway engine is 42 Km per hour when no compartment is attached, and the
reduction in speed is directly proportional to the square root of the number of compartments
attached. If the speed of the train carried by this engine is 24 km per hour when 9 compartments
are attached, the maximum number of compartments that can be carried by the engine?
Abyev`t hLb †Kvb ewM †ijI‡q Bwćbi mv‡_ jvMv‡bv Ae¯’vq _v‡Kbv ZLb Gi MwZ N›Uvq 42 wK‡jvgUvi| Bwćbi MwZ nªv‡mi
cwigvY jvMv‡bv ewMi msL¨vi eM©g~‡ji mgvbycvwZK| 9wU ewM jvMv‡bv Ae¯’vq hw` Bwćbi MwZ N›Uvq 24 wK‡jvwgUvi nq, Z‡e
m‡e©v”P KZwU ewM jvMv‡bv hv‡e?
Solution:
The number of compartments be x & constant be k
The speed of the railway engine, S = 42 - k√x
according to the question,
24 = 42 - k√9
⇒K=6
The engine can carry a maximum of that number of compartments for which its speed is greater than
zero.
Let, R is the maximum number of compartments.
so, 42 - k√R > 0
⇒ 6√R < 42
⇒ √R < 7
⇒ R < 49
Greatest integer value of R that is less than 49 is 48.
So, 48 is the maximum number of compartments that the railway engine can carry.

2. A shopkeeper lost 7.5% by selling an article. If he had bought it at 10% less and sold it for 31
taka more, he would have gained 20%. Find the cost price of the article.
[National Bank PO 14, BKB Supervisor 12]
Abyev`t GKRb †`vKvb`vi GKwU `ªe¨ 7.5% ¶wZ‡Z weµq K‡ib| hw` wZwb GwU 10% Kg `v‡g wKb‡Z cvi‡Zb Ges 31 UvKv
†ewk w`‡q wewµ Ki‡Z cvi‡Zb Zvn‡j Zvi 20% jvf n‡Zv| `ªe¨wUi µqg~j¨ wbY©q Ki|
Solution:
Let, cost price of the article be Tk. x
At 7.5% loss, selling price = Tk. (100-7.5)=Tk. 92.5
At 10% loss, cost price = Tk. (100-10) = Tk. 90
At 20% gain, Selling price = Tk. (90+20% of 90)
= Tk. (90+18)
= Tk. 108
According to the condition,
92.5% of x +31 = 108% of x
⇒ 0.925x + 31 = 1.08x
⇒ 1.08x - 0.925x =31
⇒ 0.155x = 31

Written Math Solution (BIBM) FB Group : BD job Info. (BCS & Bank)
154

 Created by Sumon K. Sarkar  FB Group : BD job Info. (BCS & Bank)  Created by Sumon K. Sarkar 
Bank Job Solution (BIBM)
 Created by Sumon K. Sarkar  FB Group : BD job Info. (BCS & Bank)  Created by Sumon K. Sarkar 

∴ x=200
The cost price of the article Tk 200
Answer: tk. 200

3. Two tanks, X and Y, are filled to capacity with jet fuel. Tank X holds 600 gallons more than tank
Y. If 100 gallons of fuel were to be pumped from each tank, tank X would then contain 3 times
as much fuel as tank Y. What is the total number of gallons of fuel in the two full tanks?
[Mercantile Band Officer 09, IFIC Bank PO 10, National Bank PO 14]
Abyev`t 2wU U¨v¼ X I Y R¡vjvbx †Zj Øviv c~Y© Kiv hvq| X U¨v¼wU Y U¨v‡¼i †P‡q 600 M¨vjb †ewk avib Ki‡Z cv‡i| Dfq
U¨v¼ †_‡K 100 M¨vjb K‡i †Zj †ei K‡i wb‡j, X U¨v†¼ Y U¨v‡¼i wZb¸Y †Zj _v‡K| U¨v¼ `ywU‡Z †Zjc~Y© Ae¯’vq †gvU KZ
M¨vjb †Zj _v‡K?
Solution:
Let, Y tank holds =p gallons
and X tank holds = (p+600) gallons.
According to the question,
3(p-100) =p+600-100
⇒ 3p-300 =p+500
⇒ 2p = 800
⇒ p = 400
Y tank holds 400 gallons
and X tank holds = 400+600 = 1000 gallons.
Total number of gallons = (1000+400) = 1400 gallons.
Answer: 1400 gallons.

Basic Bank Ltd.


Post name: Assistant Officer (Cash) Exam Date : 2014
1. A can do a work in 10 days, while B alone can do it in 15 days. They work together for 5 days
and rest of the work is done by C in 2 days. If they get Tk. 4500 for whole work, how should they
divide money?
[BASIC Bank AO Cash 14, BB AD(ff) 15]
Abyev`t A GKv GKwU KvR 10 w`‡b Ki‡Z cv‡i †hLv‡b B GKv H KvRwU 15 w`‡b K‡i| Zviv GKm‡½ 5 w`b KvR K‡i Ges
evKx KvR C GKv 2w`‡b m¤úbœ K‡i| m¤ú~Y© Kv‡Ri Rb¨ 4,500 UvKv †`qv n‡j Zviv H UvKv wb‡R‡`i g‡a¨ wKfv‡e fvM K‡i
‡b‡e?
Solution:
5 5
In 5 Days, (A+B) work ( + ) Portion
10 15
1 1
= + Portion
2 3
3+2
= Portion
6
5
= Portion
6
5
In 2 Days C does work = 1 - Portion
6
1
= Portion
6

Written Math Solution (BIBM) FB Group : BD job Info. (BCS & Bank)
Bank Job Solution (BIBM) 155

Created by Sumon K. Sarkar  FB Group : BD job Info. (BCS & Bank)  Created by Sumon K. Sarkar 
1
A’s get = 4500 × = Tk. 2250
2
Created by Sumon K. Sarkar  FB Group : BD job Info. (BCS & Bank)  Created by Sumon K. Sarkar 

1
B’s get = 4500 × = Tk. 1500
3
1
C’s get = 4500 × = Tk. 750
6
Answer: Tk. 2250, Tk. 1500, Tk. 750

2. A sum of Tk. 1260 is borrowed from a money lender at 10% p.a. compounded annually. If the
amount is to be paid in two equal annual installinents, find the annual installments.
Abyev`t hw` 10% Pµe„w× my‡` 1,260 UvKv FY †bqv nq Ges hw` 2 eQ‡i m¤ú~Y© UvKv cwi‡kva Ki‡Z nq Zvn‡j cÖ‡Z¨K eQi
KZ UvKv cwi‡kva Ki‡Z n‡e?
Solution:
We know,
Compound amount = 𝑃(1 + 𝑟)𝑛
10 2
= 1260 (1 + )
100
100+10 2
= 1260 ( )
100
110 2
= 1260 ( )
100
110 110
= 1260 × ( ) ×( )
100 100
= 1524.6
So, annual installment = 1524.6÷2 = Tk. 762.3
Answer: Tk. 762.3

3. In the figure, lines I and mare parallel. If y-z = 60°, then what is the value of x?

Solution:
From the image we get,
y + z = 180o …………………… (1)
y – z = 60o ………………………(2) [Given]
By Adding equation (1) & (2)
y + z + y – z = 180o + 60o
⇒ 2y = 240o
⇒ y = 120o
We know,

Written Math Solution (BIBM) FB Group : BD job Info. (BCS & Bank)
156

 Created by Sumon K. Sarkar  FB Group : BD job Info. (BCS & Bank)  Created by Sumon K. Sarkar 
Bank Job Solution (BIBM)
 Created by Sumon K. Sarkar  FB Group : BD job Info. (BCS & Bank)  Created by Sumon K. Sarkar 

If two parallel lines are intersected by one line then all the transversal and corresponding angles
will be equal.
So, x = y
⇒ y = 120 o
Answer: 120 o

Rajshahi Krisi Unnayan Bank Ltd.


Post name: Officer Exam Date : 2014

𝟒 √𝟏+𝒙+√𝟏−𝒙
1. If 𝒙 = , then what is the value of =?
𝟓 √𝟏+𝒙−√𝟏−𝒙
[SJIB TO 13, RAKUB Officer 14]
Solution:
Given that,
4
𝑥=
5
1 5
⇒ =
𝑥 4
1+𝑥 5+4
⇒ =
1−𝑥 5−4
1+𝑥 9
⇒ =
1−𝑥 1

√1+𝑥 3
⇒ =
√1−𝑥 1

√1+𝑥+√1−𝑥 3+1
⇒ =
√1+𝑥−√1−𝑥 3−1

√1+𝑥+√1−𝑥 4
⇒ =
√1+𝑥−√1−𝑥 2

√1+𝑥+√1−𝑥
⇒ =2
√1+𝑥−√1−𝑥

Answer: 2

2. An amount of money is invested in a savings account for two years. It increases by Tk. 420 in
two years after annual compounding at the rate of 10 % per year. What the amount in Taka,
invested initially?
অনুবােঃ দুই বছতরর েনয একনট সঞ্চযী অযাকাউতন্ট একনট পনরমার্ অর্ণ নবননতযাগ করা হয। এনট প্রনে বছর 10% হাতর বানেণক ক্রবৃনির
পর দুই বছতর 420 টাকা বৃনি পায। প্রার্নমকভাতব নবননতযাগকৃে করা টাকার পনরমার্ কে?
Solution:
Let, Initial account invested be Tk. x
According to the condition,
x + 420 = 𝑥(1 + 10%)2
10 2
⇒ x + 420 = 𝑥(1 + )
100
2
⇒ x + 420 = 𝑥(1.1)

Written Math Solution (BIBM) FB Group : BD job Info. (BCS & Bank)
Bank Job Solution (BIBM) 157

Created by Sumon K. Sarkar  FB Group : BD job Info. (BCS & Bank)  Created by Sumon K. Sarkar 
⇒ 𝑥(1.1)2 = x + 420
Created by Sumon K. Sarkar  FB Group : BD job Info. (BCS & Bank)  Created by Sumon K. Sarkar 

⇒ 1.21𝑥 = x + 420
⇒ 1.21𝑥 − x = 420
⇒ 0. 21𝑥 = 420
⇒ 𝑥 = 2000
Answer : Tk. 2000

3. A certain college has students to teacher ratio of 11 to 1. The average annual salary for teachers
is Tk. 26000. If the college pays a total of Tk. 33,80,000 in annual salaries to its teachers. How
many students does the college have?
Abyev`t †Kvb K‡j‡Ri wk¶v_©x I wk¶‡Ki AbycvZ 11 t 1| wk¶‡Ki evwl©K Mo †eZb 26,000 UvKv| hw` K‡jR †_‡K
wk¶K‡`i‡K eQ‡i †gvU 33,80,000 UvKv †eZb ‡`qv nq Zvn‡j K‡j‡R KZRb wk¶v_©x Av‡Q?
Solution:
Let, number of students be 11x and teachers be x.
According to the question,
x × 26, 000 = 338,0000
338,000
⇒x= = 130
26,000
So, number of students =(11×130) = 1430
Answer: 1430

4. Tickets to cricket game cost Tk. 40 for reserved seats and Tk. 30 for general seats. If all 500
tickets were sold for Tk. 17600. How many reserved seats were sold?
Abyev`t wµ‡KU †Ljvi wiRvf© wUwK‡Ui g~j¨ 40 UvKv Ges mvaviY wUwK‡Ui g~j¨ 30 UvKv| hw` 500wU wUwKU 17,600 UvKvq wewµ
nq Zvn‡j KZ¸‡jv wiRvf© wUwKU wewµ nq?
Solution:
Let, the number of reserved seats be x.
So, number of general seats = 500-x
According to the question,
40x+30(500-x) = 17600
⇒ 40x+15000-30x = 17600
⇒ 10x = 17600-15000 = 2600
⇒ x=260
The Number of reserved seats = 260.
Answer: 260

এ রকম গুরুত্বপূর্ণ ফাইল পপতে আমাতের গ্রুতপ প াখ রাখুন


এবং অবশ্যই ননতের বন্ধুতের ইনভাইট করুন

Written Math Solution (BIBM) FB Group : BD job Info. (BCS & Bank)
158

 Created by Sumon K. Sarkar  FB Group : BD job Info. (BCS & Bank)  Created by Sumon K. Sarkar 
Bank Job Solution (BIBM)
 Created by Sumon K. Sarkar  FB Group : BD job Info. (BCS & Bank)  Created by Sumon K. Sarkar 

5. In the ABCD is a square. If the length of the square is 10ft, then what will be the area of the
triangle OCD?
Abyev`t ABCD GKwU eM©‡¶Î hvi GK evûi •`N¨© 10 dzU| OCD wÎfz‡Ri †¶Îdj KZ?

Solution:
Given that, AB = BC = CD = DA = 10 ft
Diagonal BD = √𝐷𝐶 2 + 𝐵𝐶 2
= √102 + 102 = 10√2
1 1
OC = OD = BD = ×10√2 = 5√2
2 2
1
Area of Triangle = × OC × OD
2
1
= × 5√2 × 5√2
2
= 25 sq. ft
Answer: 25 sq. ft

Rajshahi Krisi Unnayan Bank Ltd.


Post name: Senior Officer Exam Date : 2014
𝟑
1. √𝟖𝒙𝟐 √𝟑𝟐𝒙√𝟒𝒙𝟐 = 𝟒, then the value of x?
[Premier Bank MTO 12, RAKUB SO 14, IFIC Bank Premier Bank Officer Cash 13]
Solution:
Given that,
3
√8𝑥 2 √32𝑥√4𝑥 2 = 4
3
⇒ √8𝑥 2 √32𝑥 × 2𝑥 = 4
3
⇒ √8𝑥 2 × 8 × 𝑥 = 4
3
⇒ √64 × 𝑥 3 = 4
⇒ 4𝑥 = 4
⇒x=1
Answer : x = 1

এ রকম গুরুত্বপূর্ণ ফাইল পপতে আমাতের গ্রুতপ প াখ রাখুন


এবং অবশ্যই ননতের বন্ধুতের ইনভাইট করুন

Written Math Solution (BIBM) FB Group : BD job Info. (BCS & Bank)
Bank Job Solution (BIBM) 159

Created by Sumon K. Sarkar  FB Group : BD job Info. (BCS & Bank)  Created by Sumon K. Sarkar 
2. The average weight of three men A, B and C is 84 kg. Another man D joins the group and the
Created by Sumon K. Sarkar  FB Group : BD job Info. (BCS & Bank)  Created by Sumon K. Sarkar 

average now becomes 80 kg. If a fifth man E, whose weight is 3 kg more than that of D, replaces
A, then the average weight of B, C, D and E becomes 79 kg. What is the weight of A?
[AB Bank PO 02, One Bank PO 07, RAKUB SO 14, Al-Arafah MTO 16, Janata Bank EO(Cancelled) 17]
Abyev`t wZb e¨w³ A, B I C Gi Mo IRb 84Kg| Zv‡`i mv‡_ AviI GKRb D †hvM †`Iqvq bZzb Mo nq 80Kg|hw` A
Gi ¯’‡j D Gi †P‡q 3Kg †ewk IR‡bi cÂg e¨w³ E Zv‡`i mv‡_ †hvM †`q, Zvn‡j B, C, D Ges E Gi Mo IRb nq
79Kg| A Gi IRb KZ?
Solution:
Let, Total weight of A,B and C = 84×3 = 252 kg.
Total weight of A,B,C and D = 80 × 4 = 320 kg
∴Weight of D = 320 – 252 = 68 kg
∴ Weight of E= 68 + 3 = 71 kg
Total weight of B, C, D and E = 79×4 = 316 kg
∴ Weight of B and C = 316 – 68 – 71 = 177 kg
∴Weight of A = 252-177 = 75 kg.
Answer: 75 kg.

3. A trader bought some mangoes for Tk. 150 per dozen and equal number of apples for Tk. 100
per dozen. If he sells all the fruits Tk. 140 per dozen, what will be his profit/loss in percentage?
[BASIC Bank PO 99, RAKUB SO 14, SJIB MTO 10]
Abyev`t GKRb e¨emvqx cÖwZ WRb 150 UvKv `‡i wKQz Avg Ges cÖwZ WRb 100 UvKv `‡i mgvb msL¨K Av‡cj µq
K‡ib| hw` †m me¸‡jv dj 140 UvKv WRb wn‡m‡e weµq K‡i Zvn‡j Zvi kZKiv KZ jvf ev ¶wZ n‡e?
Solution:
Let, A trader bought mangoes be x dozen
and bought apples x dozen
So, buying price = 150x + 100x= Tk. 250x
Selling price = Tk. (140x2x) = Tk. 280x
Profit = Tk. (280x-250x) = Tk. 30x
30𝑥×100
So, percentage of profit = = 12%
250𝑥
Answer: 12%

Alternative Way,
Given, cost price of per dozen mango = Tk. 150 and per dozen apple = Tk. 100
Cost price (per dozen mango + per dozen apple) = Tk. (150+100) = Tk. 250
Selling price (per dozen mango+ per dozen apple) = Tk. (140×2) = Tk. 280
Profit = Tk. (280–250) = Tk. 30
30×100
So, percentage of profit = = 12%
250
Answer: 12%

Written Math Solution (BIBM) FB Group : BD job Info. (BCS & Bank)
160

 Created by Sumon K. Sarkar  FB Group : BD job Info. (BCS & Bank)  Created by Sumon K. Sarkar 
Bank Job Solution (BIBM)
 Created by Sumon K. Sarkar  FB Group : BD job Info. (BCS & Bank)  Created by Sumon K. Sarkar 

4. Two partners A and B have 70% and 30% shares respectively in a business. After sometimes, a
third partner C joined by investing Tk. 10 lakh and thus having 20% share in the business. What
is the percentage of share of A's now in the business?
[RAKUB SO 14, SJIB MTO 10]
Abyev`t `yBRb cvU©bvi A I B GKwU e¨emvq h_vµ‡g 70% I 30% †kqv‡ii Askx`vi| wKQzw`b ci C Zv‡`i mv‡_ †hvM †`q
Ges †m 10 j¶ UvKv wewb‡qvM Kivq e¨emvq Zvi †kqv‡ii cwigvb nq 20%| eZ©gv‡b e¨emvq A Gi kZKiv †kqv‡ii cwigvb
KZ?
Solution:
Given that, A: B=70%: 30% = 7:3
Ratio of A and B, A: B= 70%: 30%= 7:3
After joining C, he got 20% share,
So, then total share of A and B = (100-20%) = 80%
7
A's share = × 80% = 56%
10
Answer: 56%

5. In the figure, rectangle PQRS inscribed in a circle and PQ = 6. If the area of the rectangular
region is 48, what is the area of the circular region?

Solution:
Given, area of PQRS = 48, PQ = 6
PQ×QR = 48
⇒ 6×QR = 48
⇒ QR = 8
Now, if we connect P to R through a line, a right triangle POR is formed.
From Pythagoras theory we know,
PR2 = PQ2 + QR
⇒ PR2 = (62+82) = 100
⇒PR = 10
PR = Diameter of the circle = 10
10 2
So, area of the circle= 𝜋 × ( ) = 25 𝜋
2
Answer: 25 𝝅

Written Math Solution (BIBM) FB Group : BD job Info. (BCS & Bank)
Bank Job Solution (BIBM) 161

Created by Sumon K. Sarkar  FB Group : BD job Info. (BCS & Bank)  Created by Sumon K. Sarkar 
United Commercial Bank Ltd.
Created by Sumon K. Sarkar  FB Group : BD job Info. (BCS & Bank)  Created by Sumon K. Sarkar 

Post name: Senior Officer Exam Date: 2014


1. A pipe is turned on to fill water into a cistern at the rate of 4 liters per minute. The cistern has a
leak which would empty it in 6 hours and the cistern is now emptied in 10 hours. Determine the
capacity of the cistern.
অনুবাে : একনট পাইপ নেতয একনট cistern প্রনে নমনটতট 4 নলটার কতর পানন পূর্ণ কতর। নকন্তু cistern (নপপা) নটতে নছদ্র র্াকায এনট
6 ণ্টায খানল হয। বেণমাতন পাইপনট পখালা র্াকায এনট 10 ণ্টায খানল হয। Cistern-নটর পানন যারর্ েমো ননর্ণয করুন।
Solution:
Let, the pipe fill the cistern in x hours
According to the question,
1 1 1
− =
6 x 10
1 1 1
⇒ = −
x 6 10
1 10−6
⇒ =
x 60
1 4
⇒ =
x 60
1 1
⇒ =
x 15
⇒ x = 15
So, the pipe fill the cistern in 15 hours = 15 × 60 = 900 minutes
In 1 minute fill water into a cistern 4 liters
In 900 minutes fill water into a cistern = 4 × 900 = 3600 liters
Answer: 3600 liters

2. If I walk at the rate of 4 kmph I miss the train by 10 minutes. If I walk at the rate of 5 kmph, I
reach 5 minutes before the arrival of the train. How far is the station from my house assuming
that I start walking from my house?
অনুবাে : আনম যনে 4 নকনম প্রনে ণ্টা পবতগ হাাঁনট োহতল আনম 10 নমননতটর মতযয পট্রন নমস করব। আনম যনে 5 নকনম প্রনে ন্টা গনেতে
হাাঁনট েতব আনম পট্রন আসার 5 নমননট আতগ পপৌাঁতছ যাই। আমার বানড পর্তক পস্টশ্ন কেেূতর যতর নননচ্ছ পয আনম আমার বানড পর্তক হাাঁটতে
শুরু কনর?
Solution:
Let, the distance of the station from my house be x km.
Here, total time (5 + 10)= 15 minutes = ¼ hours.
According to the question
𝑥 𝑥 1
− =
4 5 4
5𝑥−4𝑥 1
⇒ =
20 4
𝑥 1
⇒ =
20 4
20
⇒𝑥 =
4
⇒𝑥 =5
So, the distance of the station from my house be 5 km.
Answer: 5 km.

Written Math Solution (BIBM) FB Group : BD job Info. (BCS & Bank)
162

 Created by Sumon K. Sarkar  FB Group : BD job Info. (BCS & Bank)  Created by Sumon K. Sarkar 
Bank Job Solution (BIBM)
 Created by Sumon K. Sarkar  FB Group : BD job Info. (BCS & Bank)  Created by Sumon K. Sarkar 

3. In the figure, PS=SQ=RS and ∠P = 30°. Find x.


Solution:
Here, PS= RS
𝐼𝑓 ∠SRP = ∠SPR = 30° So, ∠PSR = 180°- (30° + 30°) = 120°
So, ∠QSR = 60°
Here, SQ = RS
So, ∠SQR =∠SRQ
so, ∠QSR = 60°
∠SQR =∠SRQ =60°
∠x = 180°-(60° +30°). = 180°-90°= 90°.
Answer: 90°

United Commercial Bank Ltd.


Post name: Management Trainee Officer Exam Date: 2013
1. A system of equation is given below: x+I= 6; x-m=5; x+p=4; x-q=3; What is the value of I+
m+p+q?
Solution:
Given that,
x+I= 6 …………………………(1)
x-m=5 ………………………….(2)
x+p=4…………………………..(3)
x-q=3 …………………………..(4)
Subtract equation (1) , (2) , (3) , (4)
⇒ x+I-x+m+x+p-x+q = 6-5+4-3
⇒ I+ m+p+q = 2
Answer: 2

2. A series has 3 numbers a, ar, ar2. In the series, the first term is twice of the second term. What is
the ratio of the sum of the first 2 terms to the sum of the last 2 terms?
[UCBL MTO 13, BB AD 12]
Abyev`t GKwU mvwi‡Z 3wU msL¨v nj a, ar, ar2| mvwiwUi 1g msL¨vwU, 2q msL¨vi wظY| 1g 2wU msL¨vi †hvMdj Ges †kl `ywU
msL¨vi †hvMd‡ji AbycvZ KZ n‡e?
Solution:
Let, 3rd term be x
ar2 = x, ar = 2x, & a = 4x
(a + ar) : (ar + ar2) = (4x+2x) : (2x+x) =6x : 3x
=2:1
Answer: 2 : 1

Written Math Solution (BIBM) FB Group : BD job Info. (BCS & Bank)
Bank Job Solution (BIBM) 163

Created by Sumon K. Sarkar  FB Group : BD job Info. (BCS & Bank)  Created by Sumon K. Sarkar 
3. A man travels from A to B at a speed x km/hr. He then rests at B for 1 hours. He then travels
Created by Sumon K. Sarkar  FB Group : BD job Info. (BCS & Bank)  Created by Sumon K. Sarkar 

from B to C at a speed 2x kin/hr and rests for 2x hours. He moves further to D at a speed twice
as that between B and C. He thus reaches D in 16 hr. If distance A-B, B-C and C-D are all equal
to 12 km, then find the time for which he rested at B.
Abyev`t GKRb e¨vw³ A †_‡K B †Z cÖwZ N›Uvq x wKwg MwZ‡Z ågb K‡ib| Zvici wZwb B †Z x N›Uvi Rb¨ wekªvg K‡ib|
Gici wZwb cÖwZ N›Uvq 2x wKwg MwZ‡Z B †_‡K C †Z ågY K‡ib Ges †mLv‡b Avevi 2x N›Uvi Rb¨ wekªvg †bb| wZwb AviI
wKQz`yi wM‡q D †Z †c․Qvb| Gmgq Zvi Mvwoi MwZ wQj B †_‡K C †Z åg‡bi MwZi wظY| wZwb Gfv‡e †gvU 16 N›Uvq D †Z
†c․Qvb| hw` A-B, B-C Ges C-D Gi me©‡gvU `~iZ¡ 12 wKwg nq Zvn‡j B †Z Zvi wekªv‡gi mgq wbY©q Kiæb|
Solution:
12
A to B travelled in hours
𝑥
12 6
B to C travelled in = hours
2𝑥 𝑥
12 3
C to D travelled in = hours
4𝑥 𝑥
According to the question,
12 6 3
+ 𝑥 + +2x+ =16
𝑥 𝑥 𝑥
12 6 3
⇒ + + +3x =16
𝑥 𝑥 𝑥
12+6+3+3𝑥 2
⇒ =16
𝑥
21+3𝑥 2
⇒ =16
𝑥
⇒ 21 + 3𝑥 2 = 16𝑥
⇒ 3𝑥 2 − 16𝑥 + 21 = 0
⇒ 3𝑥 2 − 9𝑥 − 7𝑥 + 21 = 0
⇒ 3𝑥(𝑥 − 3) − 7(𝑥 − 3) = 0
⇒ (𝑥 − 3)(3𝑥 − 7) = 0
x = 3 , x = 7/3 [ not accectable ]
A man rested at B in 3 hours
Answer: 3 hours

4. The side length of a square inscribed in a circle is 2. What is the area of the circle?
Solution:

Diameter AC = √𝐴𝐵2 + 𝐵𝐶 2
= √22 + 22

Written Math Solution (BIBM) FB Group : BD job Info. (BCS & Bank)
164

 Created by Sumon K. Sarkar  FB Group : BD job Info. (BCS & Bank)  Created by Sumon K. Sarkar 
Bank Job Solution (BIBM)
 Created by Sumon K. Sarkar  FB Group : BD job Info. (BCS & Bank)  Created by Sumon K. Sarkar 

= 2 √2
𝟐 √𝟐
Radius r = = √𝟐
𝟐
2
Area of the circle = 𝜋𝑟 2 = 𝜋√2 =2 𝜋
Answer: 2 𝛑

National Bank Ltd.


Post name: Probationary Officer Exam Date : 2013
1. In a class of 40 students, the number of students who passed the math exam is equal to half the
number of students who passed the science exam. Each student in the class passed at least one of
the two exam. If 5 students passed both exams, then how many students passed the math exam?
Abyev`t GKwU K¬v‡m 40 Rb Qv‡Îi g‡a¨ hZRb QvÎ weÁv‡b cvk K‡i Zvi A‡a©K QvÎ MwY‡Z cvk K‡i| cÖ‡Z¨K QvÎB `ywU
cix¶vi AšÍZ GKwU‡Z cvk K‡i| hw` 5 Rb QvÎ Dfq cix¶v‡ZB cvk K‡i Zvn‡j KZRb QvÎ MwY‡Z cvk K‡i?
Solution:
Let, the number of students passed in Math exam be x
and science exam be 2x.
Given that, 5 students passed in both exams.
Only math exam passed = (x-5);
Only science exam passed = (2x-5)
According to the question,
(2x-5)+(x-5)+5=40
⇒ 3x-10+5=40
⇒ 3x-5= 40
⇒ 3x=45
⇒ x = 15
So, the number of students who passed in math exam 15
Answer: 15

𝟎.𝟎𝟎𝟏𝟓×𝟏𝟎𝒎
2. If = 𝟓 × 𝟏𝟎𝟕 then find out the value of ( m – k )
𝟎.𝟎𝟑×𝟏𝟎𝒌
Solution:
Given that,
0.0015×10𝑚
= 5 × 107
0.03×10𝑘
15×100×10𝑚
⇒ = 5 × 107
3×10000×10𝑘
5×10𝑚
⇒ = 5 × 107
100×10𝑘
10𝑚
⇒ = 109
10𝑘
𝑚−𝑘
⇒ 10 = 109
⇒m–k=9
Answer: 9

Written Math Solution (BIBM) FB Group : BD job Info. (BCS & Bank)
Bank Job Solution (BIBM) 165

Created by Sumon K. Sarkar  FB Group : BD job Info. (BCS & Bank)  Created by Sumon K. Sarkar 
3. Sheela and Meela are 4 miles apart. If Sheela starts walking toward Meela at 3 miles per hour
Created by Sumon K. Sarkar  FB Group : BD job Info. (BCS & Bank)  Created by Sumon K. Sarkar 

and at the same time Meela starts walking toward Sheela at 2 miles per hours, how much time
will pass before they meet?
Abyev`t wkjv Ges wgjv 4 gvBj `~i‡Z¡ Av‡Q| hw` wkjv 3 gvBj/N›Uv †e‡M wgjvi w`‡K Ges GKB mg‡q wgjv 2 gvBj/N›Uv †e‡M
wkjvi w`‡K nuvU‡Z _v‡K| KZ mgq ci Zviv GKB hvqMvq G‡m †cŠQv‡e?
Solution:
Total distance = 4 miles
Together they walk = (3+2)=5 miles/hour
They walk 5 miles in 1 hour or 60 minutes
They walk 1 mile in 60/5 = 12 minutes
They walk 4 miles in 12×4 = 48 minutes
Answer: 48 minutes

4. In the figure, if AB = 8, BC = 6, AC = 10 and CD = 9, then AD =?

Solution:
Given that, AB = 8, BC = 6, AC = 10 and CD = 9
∆𝐴𝐵𝐶 ‘s AC2 = AB2 + BC2
⇒ 102 = 82 + BC2
⇒ BC2 = 102 - 82
⇒ BC2 = 100 – 64
⇒ BC2 = 36
⇒ BC = 6
Here BD = BC + CD = 6 + 9 =15
Again’s ∆𝐴𝐵𝐶 ‘s
AD2 = AB2 + BD2
⇒ AD2 = 82 + 152
⇒ AD2 = 64 + 225
⇒ AD2 = 64 + 225
⇒ AD2 = 289
⇒ AD = 17
Answer:17

Written Math Solution (BIBM) FB Group : BD job Info. (BCS & Bank)
166

 Created by Sumon K. Sarkar  FB Group : BD job Info. (BCS & Bank)  Created by Sumon K. Sarkar 
Bank Job Solution (BIBM)
 Created by Sumon K. Sarkar  FB Group : BD job Info. (BCS & Bank)  Created by Sumon K. Sarkar 

Shahjalal Islami Bank Ltd.


Post name: Trainee Officer (cash) Exam Date : 2013
1. Two equal amount of money are deposited in two banks, each at 15% per annum, for 3.5 years
and 5 years. If difference between their profits is Ik. 144, what is the each amount of money
deposited?
Abyev`t 15% gybvdvq mgcwigvb UvKv 3.5 eQi Ges 5 eQ‡ii Rb¨ 2wU Avjv`v e¨vs‡K ivLv n‡jv| hw` G‡`i gybvdvi cv_©K¨
144 UvKv nq Zvn‡j cÖ‡Z¨K e¨vs‡K KZ UvKv Rgv ivLv n‡qwQj?
Solution:
Intial deposit be Tk 100x
According to the question,
(100x ×5× 15%) – (100x ×3.5× 15%)=144
15 15
(100x ×5× ) – (100x ×3.5× )=144
100 100
75x – 52.5x = 144
⇒ 22.5x = 144
144×100
⇒ 100x = = 640
22.5
Answer: 640

2. An old man distributed all the gold coins he had to his two sons into two different numbers such
that the difference between the squares of the two numbers is 36 times the difference between
the two numbers. How many coins did the old man have?
Abyev`t GKRb e„× †jvK Zvi mg¯Í ¯^Y©gy`ªv Zvi `yB cy‡Îi g‡a¨ Ggb msL¨vq fvM K‡i †`b †hb msL¨v `ywUi e‡M©i AšÍi, msL¨v
`ywUi AšÍ‡ii †P†q 36 ¸Y eo nq| e„× ‡jvKwUi Kv‡Q KZwU ¯^Y©gy`ªv wQj?
Solution:
Let, the two numbers be x and y.
So, total number of coins = x+y
According to the question,
x2-y2 = 36(x-y)
(x+y)(x-y)= 36(x-y)
.:(x+y) = 36
Total number of gold coins is 36.
Answer: 36

3. If a number of two digits is divided by the product of its digits, the quotient is 3. When 18 is
added to the number, the digits of the number change their places. Find the number.
Abyev`t GKRb e„× †jvK Zvi mg¯Í ¯^Y©gy`ªv Zvi `yB cy‡Îi g‡a¨ Ggb msL¨vq fvM K‡i †`b †hb msL¨v `ywUi e‡M©i AšÍi,
msL¨v `ywUi AšÍ‡ii †P†q 36 ¸Y eo nq| e„× ‡jvKwUi Kv‡Q KZwU ¯^Y©gy`ªv wQj?
Solution:
Let, Unit’s digit be y & Ten’s digit be x
The number = 10x + y
According to the 1st Division,
10x + y
= 3 ⇒ 10x + y = 3xy ………………(1)
𝑥𝑦

Written Math Solution (BIBM) FB Group : BD job Info. (BCS & Bank)
Bank Job Solution (BIBM) 167

Created by Sumon K. Sarkar  FB Group : BD job Info. (BCS & Bank)  Created by Sumon K. Sarkar 
According to the 2nd Division,
Created by Sumon K. Sarkar  FB Group : BD job Info. (BCS & Bank)  Created by Sumon K. Sarkar 

(10x+y) + 18 = 10y + x
⇒ 10x – x +y – 10y + 18 = 0
⇒ 9x – 9y = -18
⇒y–x=2
⇒ y = 2+x ………………(2)
put the value of y in equation (1)
⇒ 10x + 2 + x = 3x (2+x)
⇒ 11x + 2 = 6x + 3x2
⇒ 6x + 3x2 - 11x -2 = 0
⇒ 3x2 - 5x -2 = 0
⇒ 3x2 - 6x+x -2 = 0
⇒ 3x (x – 2) + 1 (x – 2) =0
⇒ (x – 2) (3x + 1) = 0
x = 2 , x = - 1/3 [not accectable]
put the value of x in equation (2)
y=2+2=4
The number = 10 × 2 + 4 = 24
Answer: 24

4. In the following diagram, if BC = CD = BD =1 and angle ADC is a right angle, what is the
perimeter of triangle ABD?

Solution:
Given that, BC=CD=BD=1
So, ABCD is an equilateral triangle.
∠BCD = ∠CDB= ∠CBD = 60°
∠BDA = 90°-60° = 30°
and ∠ABD = 1800-60° = 120°
So, ∠BAD= 180°-1200-300 = 30°
∆BAD is an Isosceles triangle.
Therefore, AB = BD = 1 and AC=AB + BC = 1+1 = 2
Pythagoras theorem,
AC2 = CD2+ AD2
22 = 12 + AD2

Written Math Solution (BIBM) FB Group : BD job Info. (BCS & Bank)
168

 Created by Sumon K. Sarkar  FB Group : BD job Info. (BCS & Bank)  Created by Sumon K. Sarkar 
Bank Job Solution (BIBM)
 Created by Sumon K. Sarkar  FB Group : BD job Info. (BCS & Bank)  Created by Sumon K. Sarkar 

⇒ AD = 4-1 = √3
AD=√3
.: Perimeter of ∆ABD = 1+1+√3=2+√3
Answer: 2+√𝟑

Shahjalal Islami Bank Ltd.


Post name: Trainee Officer Exam Date : 2013
𝟒 √𝟏+𝒙+√𝟏−𝒙
1. If 𝒙 = , then what is the value of =?
𝟓 √𝟏+𝒙−√𝟏−𝒙
[SJIB TO 13, RAKUB Officer 14]
Solution:
Given that,
4
𝑥=
5
1 5
⇒ =
𝑥 4
1+𝑥 5+4
⇒ =
1−𝑥 5−4
1+𝑥 9
⇒ =
1−𝑥 1

√1+𝑥 3
⇒ =
√1−𝑥 1

√1+𝑥+√1−𝑥 3+1
⇒ =
√1+𝑥−√1−𝑥 3−1

√1+𝑥+√1−𝑥 4
⇒ =
√1+𝑥−√1−𝑥 2

√1+𝑥+√1−𝑥
⇒ =2
√1+𝑥−√1−𝑥

Answer: 2

2. In a certain Accounting class, the ratio of the number of Accounting majors to the under of
students who are not Accounting major is 2 to 5. If 2 more Accounting majors were to enter the
class, the ratio would be 1 to 2. How many students are in the class?
Abyev`t †Kvb GKwU A¨vKvDw›Us K¬v‡m hv‡`i A¨vKvDw›Us †gRi Ges hv‡`i A¨vKvDw›Us †gRi bq Zv‡`i AbycvZ 2 t 5| 2
Rb bZzb A¨vKvDw›Us †gRi K¬v‡m cÖ‡ek Kivq AbycvZ nq 1 t 2| K¬v‡m KZRb wk¶v_©x Av‡Q?
Solution:
Let, number of Accounting major be 2x
and non-Accounting major be 5x
Total Students = 2x + 5x = 7x
According to the question,
2𝑥+2 1
=
5𝑥 2
⇒ 5x = 4x + 4
⇒ 5x - 4x = 4

Written Math Solution (BIBM) FB Group : BD job Info. (BCS & Bank)
Bank Job Solution (BIBM) 169

Created by Sumon K. Sarkar  FB Group : BD job Info. (BCS & Bank)  Created by Sumon K. Sarkar 
⇒x=4
Created by Sumon K. Sarkar  FB Group : BD job Info. (BCS & Bank)  Created by Sumon K. Sarkar 

Total Students = 7 × 4 =28


Answer: 28

3. What is perimeter of AABC show in the figure?

Solution:
Given that, AC=2
∠B=x=y, ∠C=y=z , x=y=z
and ∠B = ∠C And ∠A=2z
Now, ∠A+∠B+∠C = 180°
⇒ 2z+z+z = 180°
⇒ 4z = 180°
⇒ z=45°
∠B=∠C=45° and ∠A = 2×45° = 90°
So, it is a right angled triangle.
Pythagoras theorem,
BC2 = AB2 + AC2
⇒ BC2=22+22=8 [AC=AB=2]
⇒ BC=2√2
Perimeter = AB+ AC+BC=2+2+2√2=4+2√2
Answer: 4+2√2

3. The percentage profit earned by selling an article for Tk. 1920 is equal to the percentage
loss incurred by selling the same article for Tk 1280. At what price should the article be
sold to make 25% profit?
[South East Bank MTO 2013, BB AD(freedom fighter) 2015, SJIB MTO 2013, IBBL PO 2019, Sadharan Bima JO 2019, Sadharan Bima
AM 2019, BB AD -AME - 2023]
Abyev`t GKwU cY¨ 1,920 UvKvq weµq Ki‡j kZKiv hZ jvf nq, cY¨wU 1,280 UvKvq weµq Ki‡j kZKiv ZZ UvKv ¶wZ
nq| 25% jvf Ki‡Z n‡j cY¨wU‡K KZ `v‡g weµq Ki‡Z n‡e?
Solution:
Let, profit & loss both be Tk. x
According to the question,
Selling Price – profit = Selling Price + Loss
1920 – x = 1280 + x
⇒x + x = 1920 – 1280

Written Math Solution (BIBM) FB Group : BD job Info. (BCS & Bank)
170

 Created by Sumon K. Sarkar  FB Group : BD job Info. (BCS & Bank)  Created by Sumon K. Sarkar 
Bank Job Solution (BIBM)
 Created by Sumon K. Sarkar  FB Group : BD job Info. (BCS & Bank)  Created by Sumon K. Sarkar 

⇒2x = 640
⇒x = 320
So Cost price = Selling Price – profit
= 1920 – 320 = Tk. 1600
At price 25% profit,
new selling price = 1600 + 1600×25%
= 1600 + 400 = Tk. 2000
Alternative Way,
Let, Cost Price be Tk. x
Then, According to the question,
1920−𝑥 x−1280
×100 = ×100
𝑥 𝑥
⇒ 1920 - x = x - 1280
⇒2x = 3200
⇒ x = 1600
Cost Price be Tk. 1600
∴ Required Selling Price = 125% of Tk. 1600
125
= × 1600
100
= Tk. 2000
Answer: Tk. 2000

এ রকম গুরুত্বপূর্ণ ফাইল পপতে আমাতের গ্রুতপ প াখ রাখুন


এবং অবশ্যই ননতের বন্ধুতের ইনভাইট করুন

Written Math Solution (BIBM) FB Group : BD job Info. (BCS & Bank)
Bank Job Solution (BIBM) 171

Created by Sumon K. Sarkar  FB Group : BD job Info. (BCS & Bank)  Created by Sumon K. Sarkar 
Shahjalal Islami Bank Ltd.
Created by Sumon K. Sarkar  FB Group : BD job Info. (BCS & Bank)  Created by Sumon K. Sarkar 

Post name: Trainee Senior Officer Exam Date : 2013


𝟒(√𝟔+√𝟐) 𝟐+√𝟑
1. What is the value of -
√𝟔−√𝟐 𝟐−√𝟑
Solution:
𝟒(√𝟔+√𝟐) 𝟐+√𝟑
-
√𝟔−√𝟐 𝟐−√𝟑

𝟒(√𝟔+√𝟐)(√𝟔+√𝟐) (𝟐+√𝟑)(𝟐+√𝟑)
⇒ - (𝟐−√𝟑)(𝟐+√𝟑)
(√𝟔−√𝟐)(√𝟔+√𝟐)

𝟒(√𝟔+√𝟐)𝟐 (𝟐+√𝟑)𝟐
⇒ -
𝟔−𝟐 𝟒−𝟑

4(6+2+2√6×2) 4+3+4√3
⇒ -
4 1

⇒8+ 2√12 -7 - 4√3


⇒1 + 2√12 - 4√3
⇒1 + 4√3 - 4√3
⇒1
Answer : 1

2. In a set of three numbers, the average of first two numbers is 2, the average of the last two
numbers is 3, and the average of the first and last number's is 4. What is the average of three
numbers?
Abyev`t wZbwU msL¨vi g‡a¨ 1g msL¨v `ywUi Mo 2, †kl msL¨v `ywUi Mo 3 Ges 1g I †kl msL¨vi Mo 4| msL¨v wZbwUi Mo KZ?
Solution:
Let, the three numbers be x,y,z
According to the question,
𝑥+𝑦
= 2 𝑜𝑟, 𝑥 + 𝑦 = 4 ------------------ (1)
2
𝑦+𝑧
= 3 𝑜𝑟, 𝑦 + 𝑧 = 6 ------------------- (2)
2
𝑥+𝑧
= 4 𝑜𝑟, 𝑥 + 𝑧 = 8 --------------------(3)
2
Adding the equations (1) , (2) & (3)
2(x+y+z)=18
x+y+z=9
𝑥+𝑦+𝑧 9
So, average of three numbers = = =3
3 3
Answer: 3

Written Math Solution (BIBM) FB Group : BD job Info. (BCS & Bank)
172

 Created by Sumon K. Sarkar  FB Group : BD job Info. (BCS & Bank)  Created by Sumon K. Sarkar 
Bank Job Solution (BIBM)
 Created by Sumon K. Sarkar  FB Group : BD job Info. (BCS & Bank)  Created by Sumon K. Sarkar 

3. The total income of Mr. Atig in the years 2008, 2009 and 2010 was Tk. 36,40,000. His income
increased by 20% each year. What was his income in 2010?
Abyev`t Rbve AvwZ‡Ki 2008, 2009 Ges 2010 mv‡ji †gvU Avq 36,40,000 UvKv| Zvi Avq cÖ‡Z¨K eQi 20% e„w× cvq|
2010 mv‡j Zvi Avq KZ wQj?
Solution:
Let, in 2008 income was Tk. 100x
So, in 2009 income = Tk. (100x+20% of 100 x)
= Tk. 120x
And in 2010 income = Tk. (120x+20% of 120x)
= Tk. 144x
According to the question,
100x+120x+144x = 36,40,000
⇒ 364x = 3640000
⇒ x=10000
⇒ 100x = 10,00,000
And in 2010 income = 10,000 × 144
= 1440, 000
Answer: 1440,000

4. What is the area of the equilateral triangle, if the base BC=6?


Solution:
√3
Area of the equilateral triangle = 62
4
√3
= × 36
4
= 9√3
Answer : 𝟗√𝟑

Shahjalal Islami Bank Ltd.


Post name: Management Trainee Officer Exam Date : 2013
1. In a country, 60% of the male citizen and 70% of the female citizen are eligible to vote. 70% of
the male and 60% of female citizen are eligible to cast their vote. What fraction of citizens voted
during their election?
[Janata EO 12, South East Bank MTO 13, SJIB MTO 13, Midland Bank MTO 15]
Abyev`t GKwU †`‡ki 60% cyiæl Ges 70% gwnjv †fvU w`‡Z cvi‡eb| G‡`i g‡a¨ 70% cyiæl Ges 60% gwnjv †fvU `vb
K‡ib| KZRb bvMwiK wbe©vP‡bi mgq †fvU cÖ`vb K‡ib?
Solution:
Total number of Male citizens be 100
& Total number of Female citizens be 100
So, total citizens = 100 +100 = 200
So, Eligible Male voters = 100 × 60% = 60
Eligible Female voters = 100 × 70% =70
Casted male votes = 60 × 70% = 42

Written Math Solution (BIBM) FB Group : BD job Info. (BCS & Bank)
Bank Job Solution (BIBM) 173

Created by Sumon K. Sarkar  FB Group : BD job Info. (BCS & Bank)  Created by Sumon K. Sarkar 
Casted Female votes = 70 × 60% = 42
Created by Sumon K. Sarkar  FB Group : BD job Info. (BCS & Bank)  Created by Sumon K. Sarkar 

42+42 84 21
∴Fraction of citizens voted = = =
100+100 200 50
𝟐𝟏
Answer :
𝟓𝟎

2. The percentage profit earned by selling an article for Tk. 1920 is equal to the percentage
loss incurred by selling the same article for Tk 1280. At what price should the article be
sold to make 25% profit?
[South East Bank MTO 2013, BB AD(freedom fighter) 2015, SJIB MTO 2013, IBBL PO 2019, Sadharan Bima JO 2019, Sadharan Bima
AM 2019, BB AD -AME - 2023]
Abyev`t GKwU cY¨ 1,920 UvKvq weµq Ki‡j kZKiv hZ jvf nq, cY¨wU 1,280 UvKvq weµq Ki‡j kZKiv ZZ UvKv ¶wZ
nq| 25% jvf Ki‡Z n‡j cY¨wU‡K KZ `v‡g weµq Ki‡Z n‡e?
Solution:
Let, profit & loss both be Tk. x
According to the question,
Selling Price – profit = Selling Price + Loss
1920 – x = 1280 + x
x + x = 1920 – 1280
2x = 640
x = 320
So Cost price = Selling Price – profit
= 1920 – 320 = Tk. 1600
At price 25% profit,
new selling price = 1600 + 1600×25%
= 1600 + 400 = Tk. 2000

Alternative Way,
Let, Cost Price be Tk. x
Then, According to the question,
1920−𝑥 x−1280
×100 = ×100
𝑥 𝑥
⇒ 1920 - x = x - 1280
⇒2x = 3200
⇒ x = 1600
Cost Price be Tk. 1600
∴ Required Selling Price = 125% of Tk. 1600
125
= × 1600
100
= Tk. 2000
Answer: Tk. 2000

Written Math Solution (BIBM) FB Group : BD job Info. (BCS & Bank)
174

 Created by Sumon K. Sarkar  FB Group : BD job Info. (BCS & Bank)  Created by Sumon K. Sarkar 
Bank Job Solution (BIBM)
 Created by Sumon K. Sarkar  FB Group : BD job Info. (BCS & Bank)  Created by Sumon K. Sarkar 

3. A, B and Center into a partnership in the ratio 7/2 : 4/3 : 6/5. After 4 months, A increases his
share by 50%. If the total profit at the end of one year is Tk. 21,600, then what is B's share in the
profit? [SJIB MTO 13, South East Bank MTO 13]
Abyev`t A, B I C 7/2 : 4/3 : 6/5Abycv‡Z GKwU Askx`vix Kvievi ïiæ K‡i| 4 gvm ci A Zvi †kqvi 50% e„w× K‡i| hw`
1 eQi c‡i †gvU jv‡fi cwigvb 21,600 UvKv nq Zvn‡j B Gi jf¨vsk KZ n‡e?
Solution:
Ratio of the initial investment = 27:34:56 = 105:40:36
So the actual investment of A,B,C=105x, 40x and 36x
A increases his share 50% after 4 months. Hence, the ratio of their investments =
108
= (105x×4)+(105x× ×8) : 40x × 12 : 36x×12
100
3
= 105+(105× ×2) : 40 × 3 : 36×3
2
= 105×4:40×3:36×3
= 35:10:9
Sum of the ratio =35+10+9 =54
10
B,s profit=21600 × = 4000
54
Answer: 4000

4. The ratio between the perimeter and the breadth of rectangle is 5:1. If the area of the rectangle
is 216 sq. cm, what is the length of the rectangle? [SJIB MTO 13, South East Bank MTO 13]
Abyev`t GKwU AvqZKvi †¶‡Îi cwimxgv Ges cÖ‡¯’i AbycvZ 5 t 1| AvqZ‡¶ÎwUi †¶Îdj 216 eM© †m.wg n‡j, Gi •`N©¨ KZ?
Solution:
Let, the length be x and breadth be y
So, area of the rectangle, xy= 216 ............ (1)
Now, the perimeter = 2(x + y)
According to question,
2(x+y): y= 5:1
2(𝑥+𝑦) 5
⇒ =
𝑦 1
⇒ 2x +2y=5y
⇒ 2x = 3y
3
⇒ x = y -----------------------(2)
2
Put the value of x in equation (1)
3
y × y = 216
2
2
⇒ y2 = 216×
3
⇒ y2 = 144
⇒ y = 12
Put the value of y in equation (2)
3
⇒ x= ×12 = 18
2
Length = 18 cm
Answer: 18 cm

Written Math Solution (BIBM) FB Group : BD job Info. (BCS & Bank)
Bank Job Solution (BIBM) 175

Created by Sumon K. Sarkar  FB Group : BD job Info. (BCS & Bank)  Created by Sumon K. Sarkar 
Social Islami Bank Ltd.
Created by Sumon K. Sarkar  FB Group : BD job Info. (BCS & Bank)  Created by Sumon K. Sarkar 

Post name: Probationary Officer Exam Date : 2013


1. A father has divided his property between his two sons A and B. A invests the amount at a
compound profit of 8%. B invests the amount of 10% simple profit. At the end of 2 years, the
profit received by B is Ik. 1336 more than A. Find the amount of both. Total amount of his father
is Tk. 25000.
[Midland Bank MTO 15, SIBL PO 13, BB AME-2023]
Abyev`t †Kvb wcZv Zvi m¤úwË Zvi `yB cyÎ A I B Gi g‡a¨ fvM K‡i †`q| A Zvi UvKv 8% Pµe„w× gybvdvq Ges B Zvi UvKv
10% mij gybvdvq wewb‡qvM K‡i| 2 eQi c‡i A Gi †P‡q B 1,336 UvKv †ewk gybvdv AR©b K‡i| Df‡qi UvKvi cwigvb wbY©q
Ki| Zv‡`i wcZvi Kv‡Q 25,000 UvKv wQj|
Solution:
Let, Share of A be x Tk
& Share of B be (25000-x) Tk
A's profit = 𝑃(1 + 𝑟)2 − 𝑃
= 𝑥(1 + 8%)2 − 𝑥
8 2
= 𝑥 (1 + ) −𝑥
100
100+8 2
=𝑥( ) −𝑥
100
108 2
=𝑥( ) −𝑥
100
= 1.1664x – x
= 0.1664x
B's profit = (25000-x) ×2×10%
10
=(25000-x) ×2 ×
100
1
=(25000-x) ×
5
= 5000 – 0.2x
According to the question,
5000 – 0.2x - 0.1664x =1336
⇒ 0.2x+0.1664x = 5000 – 1336
⇒ 0.3664x = 3664
⇒ x =10000
So, Share of A=10,000 Tk and share of B =25,000 - 10,000 = 15,000 Tk
Answer: 10,000 Tk and 15,000 Tk

এ রকম গুরুত্বপূর্ণ ফাইল পপতে আমাতের গ্রুতপ প াখ রাখুন


এবং অবশ্যই ননতের বন্ধুতের ইনভাইট করুন

Written Math Solution (BIBM) FB Group : BD job Info. (BCS & Bank)
176

 Created by Sumon K. Sarkar  FB Group : BD job Info. (BCS & Bank)  Created by Sumon K. Sarkar 
Bank Job Solution (BIBM)
 Created by Sumon K. Sarkar  FB Group : BD job Info. (BCS & Bank)  Created by Sumon K. Sarkar 

2. Two equal glasses are respectively 1/3 and 1/4 full of milk. They are then filled up with water
and the contents inixed in a tumbler. What is the ratio of milk and water in the tumbler?
Abyev`t `yBwU mg cwigvb Møvm 1/3 Ges 1/4 Ask `ya Øviv c~Y©| Zvici †m¸‡jv cvwb Øviv c~Y© K‡i c‡i GKUv eo cv‡Î me¸‡jv
wgkv‡bv nj| IB eo cv‡Î `ya I cvwbi AbycvZ KZ?
Solution:
Let, capacity of glasses be 12 units
1 1
Milk in glass at full of milk=12 × =12 units
3 3
1 1
Milk in glass at full of milk = 12 × =3 units
4 4
Total units of milk in 2 tumbler = 4+3=7 units
Total units of water in 2 tumbler = 8+9=17 units
The ratio of milk and water in the tumbler = 7:17
Answer: 7 : 17

3. One day, Mr. Wahid started 30 minutes late from home and reached his office 50 minutes late,
while driving 25% slower than his usual speed. How much time in minutes does Mr. Wahid
usually take to reach her office from home?
Abyev`t `yBwU mg cwigvb Møvm 1/3 Ges 1/4 Ask `ya Øviv c~Y©| Zvici †m¸‡jv cvwb Øviv c~Y© K‡i c‡i GKUv eo cv‡Î me ¸‡jv
wgkv‡bv nj| H eo cv‡Î `ya I cvwbi AbycvZ KZ?
Solution:
Let, take to reach be x minutes & speed be s
75 3𝑆
At 25% slower driving , speed =s ×75% = s × =
100 4
Time Difference = 50 – 30 = 20 minutes
According to the question,
3𝑆
× (𝑥 + 20) = 𝑥 × 𝑠
4
⇒ 3(𝑥 + 20) = 4𝑥
⇒ 4x – 3x = 60
⇒ x = 60
Answer: 60 minutes

এ রকম গুরুত্বপূর্ণ ফাইল পপতে আমাতের গ্রুতপ প াখ রাখুন


এবং অবশ্যই ননতের বন্ধুতের ইনভাইট করুন

Written Math Solution (BIBM) FB Group : BD job Info. (BCS & Bank)
Bank Job Solution (BIBM) 177

Created by Sumon K. Sarkar  FB Group : BD job Info. (BCS & Bank)  Created by Sumon K. Sarkar 
4. AD is the longest side of the right triangle ABD shown in the figure. What is the length of
Created by Sumon K. Sarkar  FB Group : BD job Info. (BCS & Bank)  Created by Sumon K. Sarkar 

longest side of △ABC?

Solution:
Given That, AD is the longest side of the right triangle ABD shown in the figure.
Here ∠𝐵 = 900
∆𝐴𝐵𝐶 ′ 𝑠 𝐴𝐶 2 = 𝐴𝐵2 + 𝐵𝐶 2
AC = √42 + 52
AC = √41
Answer: √𝟒𝟏

Al-Arafah Bank Ltd.


Post name: Management Trainee Officer Exam Date : 2013
√𝟑+𝟏 √𝟑−𝟏
1. If 𝒙 = and 𝒚 = then find the value of 𝒙𝟐 + 𝒚𝟐
√𝟑−𝟏 √𝟑+𝟏
Solution:
√3+1 √3−1
Given that, 𝑥 = and 𝑦 =
√3−1 √3+1
2 2 √3+1 2 √3−1 2 4 2 8+2 5
𝑥 +𝑦 = ( ) +( ) = + = =
√3−1 √3+1 2 4 4 2

2. According to a car dealer's sale report, 1/3 of the cars sold during a certain period were Sedans
and 1/5 of the other cars sold were station wagons. If N station wagons were sold during that
period, how many Sedans, in terms of N, were sold?
Abyev`t GKRb Mvwoi wWjv‡ii weµq ZvwjKv †_‡K †`Lv hvq †h, wbw`©ó mg‡q hZ Mvwo wewµ nq Zvi 1/3 Ask wm`vbm Ges
Ab¨vb¨ wewµZ Mvwoi 1/5 Ask †÷kb IqvMbm| hw` H mg‡q N msL¨K †÷kb IqvMb wewµ n‡q _v‡K Zvn‡j hZ¸‡jv wm`vbm
wewµ nq Zv N Øviv cÖKvk Ki?
Solution:
Let, total number of cars be 15x. [LCM of 3 & 5 = 15]
1
So, Sedan cars = of 15x =5x
3
Other cars = (15x-5x) = 10x
1
:. Station Wagons = 10x× = 2x
5
According to the question,
2x = N
𝑁
⇒x=
2

Written Math Solution (BIBM) FB Group : BD job Info. (BCS & Bank)
178

 Created by Sumon K. Sarkar  FB Group : BD job Info. (BCS & Bank)  Created by Sumon K. Sarkar 
Bank Job Solution (BIBM)
 Created by Sumon K. Sarkar  FB Group : BD job Info. (BCS & Bank)  Created by Sumon K. Sarkar 

5𝑁
⇒ 5x =
2
𝟓𝑵
Answer:
𝟐

𝑥 𝑥
3. If years ago Samad was 12, and years from now he will be 2x years old, how will he be 3x
2 2
years from now?
x x
Abyev`t hw` eQi c~‡e© mvgv‡`i eqm wQj 12 Ges eQi c‡i Zvi eqm 2x nq, Z‡e 3x eQi c‡i Zvi eqm KZ n‡e?
2 2
Solution:
𝑥
years ago Samad was 12
2
𝑥
Adam's present age is 12 +
2
𝑥 𝑥 𝑥
After, from now, his age will be 12 + + = 2x
2 2 2
According to the question ,
12 + x = 2x
⇒ x = 12.
12
So, his present age is 12 + = 18 years.
2
3x years from now is 3 ×12 = 36 years from now, he will be 18 + 36 = 54 years.
Answer: 54 years

4. The ratio between the length and breadth of a rectangular park is 3:2. If a man cycling along the
boundary of the park at the speed of 12 km per hour completes one round in 8 minutes then what
is the area of the park?
[Al-Arafah MTO 13, City Bank MTO 17]
Abyev`t GKwU AvqZKvi cv‡K©i ˆ`N©¨ I cÖ‡¯’i AbycvZ 3 t 2| †Kvb e¨w³ mvB‡Kj wb‡q N›Uvq 12 wK.wg †e‡M 8 wgwb‡U HcvK©wUi
Pvicv‡k GKevi Ny‡i Av‡mb| cvK©wUi †¶Îdj KZ?
Solution:
Perimeter = Distance travelled in 8 minutes,
8
⇒ perimeter =12000× =1600m
60
Let, length is 3x and width is 2x
We know perimeter of rectangle is 2(length + width)
According to the question ,
2(3x+2x)=1600
⇒ 10x =1600
⇒ x=160
So Length of the park =160×3=480m
and Width of the park =160×2=320m
Finally, Area of the park =length×breadth
=480×320
=153600 m2
2
Answer: 153600 m

Written Math Solution (BIBM) FB Group : BD job Info. (BCS & Bank)
Bank Job Solution (BIBM) 179

Created by Sumon K. Sarkar  FB Group : BD job Info. (BCS & Bank)  Created by Sumon K. Sarkar 
Premier Bank Ltd.
Created by Sumon K. Sarkar  FB Group : BD job Info. (BCS & Bank)  Created by Sumon K. Sarkar 

Post name: Trainee Junior Officer Exam Date : 2013


1. A man borrows Tk. 2500 at 4% p.a. and Tk. 1800 at 5% simple interest for the same period. If
he pays Tk. 570 as total interest, find the time for which the sums were borrowed.
Abyev`t †Kvb e¨w³ GKwU wbw`©ó mg‡qi Rb¨ 4% my‡` 2,500 UvKv Ges 5% my‡` 1,800 UvKv FY MÖnY K‡ib| hw` wZwb GK‡Î
570 UvKv my` cÖ`vb K‡ib, Zvn‡j mgq wbY©q Ki|
Solution:
Let, the time be x
2500 × 𝑥 × 4% + 1800 × 𝑥 × 5% = 570
4 5
⇒2500 × 𝑥 × + 1800 × 𝑥 × = 570
100 100
⇒100x + 90x =570
⇒190x = 570
⇒x=3
Answer: 3

2. The price of sugar and rice are in the ratio 4:5. If the price of sugar is increased by 10% and rice
by 20% find the ratio between increased prices of sugar and rice.
Abyev`t wPwb Ges Pv‡ji `v‡gi AbycvZ 4 t 5| hw` wPwbi `vg 10% Ges Pv‡ji `vg 20% e„w× cvq Zvn‡j e„w× cvIqvi ci wPwb
I Pv‡ji `v‡gi AbycvZ wbY©q Kiæb|
Solution:
Let, the price of sugar and rice are Tk. 400 and Tk. 500.
Now, increased price of sugar-110% of 400 Tk. 440
and increased price of rice =120% of 500 = Tk. 600.
So, ratio of increased price of sugar and rice
= 440: 600 =11:15
Answer: 11 : 15

3. The length of each side of a triangle is 12 cm. Find the height of the triangle.
অনুবােঃ একনট নত্রভুতের প্রনেনট বাহুর দে ণয 12 পসনম। নত্রভুতের উচ্চো ননর্ণয কর
Solution:

Let, The height of Triangle x


√3
Area of the equilateral triangle= 122 = 36√3
4
1
Again , Area of the equilateral triangle = × 12 × 𝑥 = 6𝑥
2

Written Math Solution (BIBM) FB Group : BD job Info. (BCS & Bank)
180

 Created by Sumon K. Sarkar  FB Group : BD job Info. (BCS & Bank)  Created by Sumon K. Sarkar 
Bank Job Solution (BIBM)
 Created by Sumon K. Sarkar  FB Group : BD job Info. (BCS & Bank)  Created by Sumon K. Sarkar 

According to the question,


6x = 36√3
x = 6√3
Answer: 6√𝟑

𝒂 𝟏 𝟑𝒂+𝟐𝒃
4. If = then the value of
𝒃 𝟑 𝟑𝒂−𝟐𝒃
Solution:
𝑎 1
Given that, =
𝑏 3
3𝑎 3
=
2𝑏 6
𝟑𝒂+𝟐𝒃 𝟑+𝟔 𝟗
= = = −𝟑
𝟑𝒂−𝟐𝒃 𝟑−𝟔 −𝟑
Answer: - 3

Premier Bank Ltd.


Post name: Management Trainee Officer Exam Date : 2012
𝟑
1. √𝟖𝒙𝟐 √𝟑𝟐𝒙√𝟒𝒙𝟐 = 𝟒, then the value of x?
[Premier Bank MTO 12, RAKUB SO 14, IFIC Bank Premier Bank Officer Cash 13]
Solution:
Given that,
3
√8𝑥 2 √32𝑥√4𝑥 2 = 4
3
⇒ √8𝑥 2 √32𝑥 × 2𝑥 = 4
3
⇒ √8𝑥 2 × 8 × 𝑥 = 4
3
⇒ √64 × 𝑥 3 = 4
⇒ 4𝑥 = 4
⇒x=1
Answer : x = 1

2. A person earns yearly interest of Tk. 920 by investing Tk. X at 4% and Tk. Y at 5% simple
interest rate. If he had invested Tk. X at 5% and Tk. Y at 4% simple interest rate, then his yearly
interest earning would have been reduced by Tk. 40. Find out the amount of X and Y.
[ONE Bank PO 10, NCC Bank MTO 11, Premier Bank MTO 12, Midland Bank TO 15]
Abyev`t GK e¨w³ 4% my‡` x UvKv Ges 5% my‡` y UvKv wewb‡qvM K‡i eQ‡i 920 UvKv gybvdv AR©b K‡ib| wZwb hw` 5% my‡`
x UvKv Ges 4% my‡` y UvKv wewb‡qvM K‡ib Zvn‡j gybvdvi cwigvb 40 UvKv K‡g hv| x Ges y Gi cwigvb wbY©q Ki|
Solution:
According to the question,
X×4%+Y×5% = 920
⇒ 4X+5Y = 92000 ...................................(1)
And, X×5% + Y×4%=880

Written Math Solution (BIBM) FB Group : BD job Info. (BCS & Bank)
Bank Job Solution (BIBM) 181

Created by Sumon K. Sarkar  FB Group : BD job Info. (BCS & Bank)  Created by Sumon K. Sarkar 
⇒ 5X+4Y = 88000....................................(2)
Created by Sumon K. Sarkar  FB Group : BD job Info. (BCS & Bank)  Created by Sumon K. Sarkar 

Now, subtraction the equation (1)×5 to (2)×4


20X+25Y – 20X – 16Y = 460,000 – 352,000
⇒ 9Y =108,000
⇒Y = 12000.
Putting the value of y in equation (1)
4X+5×12000 = 92000
⇒ 4X+60,000 = 92,000
⇒ 4X = 92,000 - 60,000
⇒ 4X = 32,000
⇒ X=8000
Answer: X=Tk 8000 and Y= Tk12,000

3. A basketball team has won 15 games and lost 9. If these games represent 16.67% of the games to
the played, then how many more gaines must the team win to average 75% for the season?
[ONE Bank PO 10, NCC Bank MTO 11, Premier Bank MTO 12, Midland Bank TO 15, Bangladesh bank AP 2023]
Abyev`t GKwU ev‡¯‹Uej wUg 15wU †Ljvq Rqjvf K‡i Ges 9wU †Ljvq civwRZ nq| hw` GB †Ljv¸‡jv Øviv Zv‡`i †gvU ‡Ljvi
16.67% †Ljv‡K †evSv‡bv nq Zvn‡j H wmR‡b M‡o 75% †Ljvq †RZvi Rb¨ Zv‡`i‡K KZwU †Ljvq wRZ‡Z n‡e?
Solution:
Total game that have already been played=15+9=24.
According to the question,
2 50
16 = % = 24
3 3
24×100
Or,100% = =144.
50/3
24×100×3
Or,100% = =144.
50
To win 75% of the game the team need to win = 144×75% = 108.
So it needs to win (108-15) = 93 more games.
Answer: 93 more games

One Bank Ltd.


Post name: Special Cadre Officer Exam Date : 2012

1. Suppose 81p + 62q=138 and 62p + 819 = 5, find out the value of p & q.
Solution:
81p +62q = 138..................... (1)
62p+812 = 5.......................... (2)
Subtract equation (1) × 62 from equation (2) × 81
62 ×81p+62× 62q = 138×62
81×62p + 81×81q= 5×81
----------------------------------------
-2717q = 8151
⇒ q= -3

Written Math Solution (BIBM) FB Group : BD job Info. (BCS & Bank)
182

 Created by Sumon K. Sarkar  FB Group : BD job Info. (BCS & Bank)  Created by Sumon K. Sarkar 
Bank Job Solution (BIBM)
 Created by Sumon K. Sarkar  FB Group : BD job Info. (BCS & Bank)  Created by Sumon K. Sarkar 

Put the value of q' in equation (1)


81p+62(-3)=138
⇒ 81p-186= 138
⇒ 81p= 138+186
⇒81p = 324
⇒ p=4
Answer: p=4 and q=-3

2. A person spends 1/3rd of the money with him on food, 1/5th of the remaining on education, 1/4th
of the remaining on treatment. Now he is left with Tk 200. How much did he have with him in
the beginning?
Abyev`t GK e¨w³ Zvi UvKvi 1/3 Ask Lvev‡i e¨q K‡ib| Aewkó UvKvi 1/5 Ask covïbvq e¨q K‡ib| Gici hv Aewkó _v‡K
Zvi 1/4 Ask wPwKrmvLv‡Z e¨q Kivi ci Zvi nv‡Z 200 UvKv Aewkó _v‡K| Zvi Kv‡Q KZ UvKv wQj?
Solution:
Let, In the beginning the person had Tk. x
𝑥
1/3rd of the money is spent for food, So remaining money =Tk. (𝑥 − )
3
3𝑥−𝑥
= Tk.
3
2𝑥
= Tk.
3
2𝑥 1 2𝑥
1/5th of the remaining is spent for education = × =
3 5 15
2𝑥 2𝑥 10𝑥−2𝑥 8𝑥
Remaining money = − = =
3 15 15 15
th 8𝑥 1 2𝑥
1/4 of the remaining money spent on treatment = × =
15 4 15
8𝑥 2𝑥 6𝑥 2𝑥
Remaining money = − = tk = tk
15 15 15 5
According to the condition,
2𝑥
= 200
5
⇒ 𝑥 = 500
In the beginning the person had Tk. 500
Answer: Tk 500

এ রকম গুরুত্বপূর্ণ ফাইল পপতে আমাতের গ্রুতপ প াখ রাখুন


এবং অবশ্যই ননতের বন্ধুতের ইনভাইট করুন

Written Math Solution (BIBM) FB Group : BD job Info. (BCS & Bank)
Bank Job Solution (BIBM) 183

Created by Sumon K. Sarkar  FB Group : BD job Info. (BCS & Bank)  Created by Sumon K. Sarkar 
Shahjalal Islami Bank Ltd.
Created by Sumon K. Sarkar  FB Group : BD job Info. (BCS & Bank)  Created by Sumon K. Sarkar 

Post: Trainee Officer Exam Date : 2011

1. The length of each side of a triangle is 12 cm. Find the height of the triangle.
অনুবােঃ একনট নত্রভুতের প্রনেনট বাহুর দে ণয 12 পসনম। নত্রভুতের উচ্চো ননর্ণয কর
Solution:
Let, height of triangle be x cm. A
Given that, length of each side of a triangle AB = BC = AC = 12 cm
We know,
1 1
Area of triangle = ×BC×AD = ×12×x = 6x x
2 2
√3
Again, Area of Equilateral Triangle = ×(12)2 = 36√3
4
According to the condition, B M C
6x = 36√3
⇒ x = 6√3
∴ Height of the triangle 6√3 cm.
Answer: 6√𝟑 cm.

2. The average income of A for 15 days is Tk 70. The average for first five days is Tk. 60 and that
for the last nine days is Tk. 80. What is the income for the sixth day?
Abyev`t A Gi 15 w`‡bi Mo Avq 70 UvKv| cÖ_g 5 w`‡bi Mo Avq 60 UvKv Ges †kl 9 w`‡bi Mo Avq 80 UvKv| 6ô w`‡b Zvi
Avq KZ wQj?
Solution:
Total income of A in 15 days = Tk. (70×15) = Tk. 1050
1st 5 days A’s Income = Tk. (60×5) = Tk. 300
Last 9 days A’s Income = Tk. (80×9) = Tk. 720
∴ Income of sixth day = Tk. (1050-300-720) = Tk. 30
Answer: Tk. 30

3. A man sells articles at 5% profit. If had bought it at 5% less and sold it for Tk. 1 less, he would
have gained 10%. What is the cost price of the article?
Abyev`t †Kvb e¨w³ 5% jvf cY¨ weµq K‡ib| hw` wZwb 5% Kg `v‡g wK‡b 1 UvKv K‡g weµq Ki‡Zb Zvn‡j Zvi 10% jvf
n‡Zv| cY¨wUi µqg~j¨ KZ?
Solution:
Let, cost Price of the article = Tk. 100x.
5
At 5% profit, selling price of the article = (100x+5% of 100x) = (100𝑥 + 100𝑥 × 100) =Tk. 105x
When cost price 5% reduced,
5
New cost of the article = (100x - 5% of 100x) =(100𝑥 − 100𝑥 × 100) = (100x – 5x) = Tk. 95x
When gained 10%,

Written Math Solution (BIBM) FB Group : BD job Info. (BCS & Bank)
184

 Created by Sumon K. Sarkar  FB Group : BD job Info. (BCS & Bank)  Created by Sumon K. Sarkar 
Bank Job Solution (BIBM)
 Created by Sumon K. Sarkar  FB Group : BD job Info. (BCS & Bank)  Created by Sumon K. Sarkar 

10
New selling price of the article = (95x+10% of 95x) = (95𝑥 − 95𝑥 × 100) = Tk. 104.5x
According to the question,
105x - 104.5x = 1
⇒ 0.5x = 1
5𝑥
⇒ =1
10
100𝑥
⇒ = 100
2
⇒ 100𝑥 = 200
So, cost Price of the article Tk. 200
Answer: Tk. 200

4. Nipu, Nila and Dipa formed a partnership with investments of Tk. 75000, Tk. 60000 and Tk.
40000 respectively. After 3 years of operation, the partnership had a net profit of Tk 26250. What
was the share of Dipa in the profit?
Abyev`t wbcy , bxjv Ges w`cy GKwU Askx`vwi‡Z¡ h_vµ‡g 75000, 60000 Ges 40000 UvKv wewb‡qvM K‡i| 3 eQi ci†gvU
jv‡fi cwigvY wQj 26250 UvKv | GB gybvdvq `xcvi Ask KZ UvKv wQj?
Solution:
Ratio of the investments of Nipu, Nila and Dipa
Nipu: Nila: Dipa = 75000: 60000 : 40000 = 75: 60 : 40 = 15: 12 : 8
Sum of the ratio = 15+12+8 = 35
8
∴ Share of Dipa in the profit = Tk. (26,250 × ) = 𝑇𝑘. 6,000
35
Answer: Tk. 6000

Shahjalal Islami Bank Ltd.


Post: Trainee Senior Officer Exam Date : 2011

𝒂 𝟏 𝟑𝒂+𝟐𝒃
1. If = then the value of
𝒃 𝟑 𝟑𝒂−𝟐𝒃
Solution:
𝑎 1
Given that , =
𝑏 3
3𝑎 1×3
⇒ =
2𝑏 3×2
3𝑎 1
⇒ =
2𝑏 2
3𝑎+2𝑏 1+2
⇒ =
3𝑎−2𝑏 1−2
3𝑎+2𝑏
⇒ = −3
3𝑎−2𝑏
Answer: -3

Written Math Solution (BIBM) FB Group : BD job Info. (BCS & Bank)
Bank Job Solution (BIBM) 185

Created by Sumon K. Sarkar  FB Group : BD job Info. (BCS & Bank)  Created by Sumon K. Sarkar 
2. A club has 20 members. They are electing a principal and a vice president. How inany different
Created by Sumon K. Sarkar  FB Group : BD job Info. (BCS & Bank)  Created by Sumon K. Sarkar 

outcome of the election are possible? (Assume the president and the vice president must be
different members of the club.)
Solution:4
A President can be elected from 20 members by 20C1 = 20 ways.
A vice President can be elected from 19 members by 19C1 = 19 ways.
∴ Possible outcome of the election = 20×19 = 380
Answer: 380

3. Tk. 1500 is invested at a rate of 10% simple interest and interest is added to the principal after
every 5 years in how many years will it amount to Tk. 2500?
Abyev`t hw` 10% mij gybvdvq 1,500 UvKv wewb‡qvM Kiv nq Ges cÖ‡Z¨K 5 eQi ci hw` gybvdv Avm‡ji mv‡_ †hvM Kiv nq
Zvn‡j KZ eQ‡i UvKvi cwigvb 2,500 n‡e?
Solution:
At 10 % simple interest, Interest of 5 years = 1500×5×10% = Tk. 750 [ I = Pnr]
After 5 years Total amount (P) = Tk. (1500+750) = Tk. 2250.
Interest (I) = Tk. (2500-2250) = Tk. 250
10 1
Rate of Interest (r) = 10% = =
100 10
No. of year (n)
We know, I = Pnr
1
⇒ 250 = 2250× n ×
10
⇒ 250 = 225× n
250
⇒n=
225
10
⇒n=
9
1
⇒n=1
9
12
⇒ n = 1 𝑦𝑒𝑎𝑟𝑠 months
9
4
⇒ n = 1 𝑦𝑒𝑎𝑟𝑠 months
3
1
⇒ n = 1 𝑦𝑒𝑎𝑟𝑠 1 months
3
30
⇒ n = 1 𝑦𝑒𝑎𝑟𝑠 1month days
3
⇒ n = 1 𝑦𝑒𝑎𝑟𝑠 1month 10 days
So, Total time = 5 years +1 𝑦𝑒𝑎𝑟𝑠 1months 10 days = 6 𝑦𝑒𝑎𝑟𝑠 1month 10 days
Answer: 𝟔 𝒚𝒆𝒂𝒓𝒔 𝟏months 𝟏𝟎 days

এ রকম গুরুত্বপূর্ণ ফাইল পপতে আমাতের গ্রুতপ প াখ রাখুন


এবং অবশ্যই ননতের বন্ধুতের ইনভাইট করুন

Written Math Solution (BIBM) FB Group : BD job Info. (BCS & Bank)
186

 Created by Sumon K. Sarkar  FB Group : BD job Info. (BCS & Bank)  Created by Sumon K. Sarkar 
Bank Job Solution (BIBM)
 Created by Sumon K. Sarkar  FB Group : BD job Info. (BCS & Bank)  Created by Sumon K. Sarkar 

4. The base of a rectangle exceeds three times the height by 8. Find the dimensions of the rectangle
if- (a) the semi-perimeter is 32 (b) the perimeter is 100
Solution:
Let, the height of a rectangle be x.
So, the base of a rectangle = 3x+8
According to the question (a)
2(Base + Height )
Semi- perimeter = = Base + Height = 3x+8+ x = 4x +8
2
According to the question (a),
4x +8 = 32
⇒ 4x = 32-8
⇒ 4x = 24
⇒x=6
So, height of a rectangle = 6 and base = 3×6+8 = 26
Answer: height 6 & base 26
According to the question (b)
Perimeter = 2(Base+ Height) = 2(3x+8+ x) = 8x+16
According to the question (b),
8x +16 = 100
⇒8x = 100-16
⇒ 8x = 84
⇒ x = 10.5
height of a rectangle = 10.5 and base = 3×10.5+8 = 39.5
Answer: Height 10.5 and base 39.5

Shahjalal Islami Bank Ltd.


Post: Management Trainee Officer Exam Date : 2011

1. Two Cans have the same height equal to 21 m. One Can is cylindrical; the diameter of whose
base is 10 cmn. The other can has square base of side 10 cm. What is the difference in their
capacities.
Abyev`t `ywU cv‡Îi cÖwZwUi D”PZv 21 wgUvi| GKwU cvÎ wmwjÛvi AvK…wZi hvi f~wgi e¨vm 10 †m.wg| Ab¨wU eM©vK…wZi hvi GK
av‡ii ˆ`N¨© 10 †m.wg| cvÎ `ywUi avib¶gZvi cv_©K¨ KZ?
Solution:
Height of a cylindrical Can, (h) = 21m = 2100cm [1 meter = 100 cm]
10
Radius of cylindrical Can, (r) = cm = 5cm [Radius = Diameter]
2
We know,
22
Volume of cylindrical Can = πr2h = ×52×2100 =165,000 cm3
7
Volume of square box Can = side2×height = 102×2100 = 2,10,000 cm3
Difference of Can capacities = (210,000-165,000) cm3 = 45,000 cm3
Answer: 45,000 cm3

Written Math Solution (BIBM) FB Group : BD job Info. (BCS & Bank)
Bank Job Solution (BIBM) 187

Created by Sumon K. Sarkar  FB Group : BD job Info. (BCS & Bank)  Created by Sumon K. Sarkar 
2. The cost price of two watches taken together is Tk. 840. If by selling one at a profit of 16% and
Created by Sumon K. Sarkar  FB Group : BD job Info. (BCS & Bank)  Created by Sumon K. Sarkar 

the other at a loss of 12%, there is no loss or gain in the whole transaction, find the cost price of
the two watches.
[SJIB MTO 11, SIBL PO 17, NRBC Bank MTO 18]
Abyev`t GKm‡½ †Kbv 2wU Nwoi µqg~j¨ GK‡Î 840 UvKv| hw` GwU‡K 16% jv‡f Ges Ab¨wU‡K 12% ¶wZ‡Z weµq Ki‡j
†gv‡Ui Dci †Kvb jvf ev ¶wZ nq bv Zvn‡j Nwo 2wUi cÖwZwUi µqg~j¨ wbY©q Ki|
Solution:
Let, the cost of one watch = Tk. x
Then cost of second watch = Tk. (840-x)
According to the question,
16% of x =12% of (840-x)
16 12
⇒x× = (840−x) ×
100 100
⇒ 16x = 5760 −12x
⇒ 16x + 12x = 12×840
⇒ 28x=12×840
𝟏𝟐×𝟖𝟒𝟎
⇒x= = 360
𝟐𝟖
Hence ,cost of one watch Tk. 360
Cost of another watch = Tk. (840 – 360)
= Tk. 480
Answer: Tk. 360 & Tk. 480

3. A train travelling at 48 km/h completely crosses another train having half its length and
travelling in opposite direction at 42 km/h, in 12 second. It also passes a railway platform in 45
second. What is the length of the platform?
Abyev`t 48 wK.wg/N›Uv †e‡Mi GKwU †Uªb wecixZ w`K †_‡K 42 wK.wg/N›Uv †e‡M Avmv Gi A‡a©K ˆ`‡N©¨i GKwU †Uªb‡K 12
†m‡K‡Û m¤ú~Y© iƒ‡c AwZµg K‡i| GwU †ijI‡q cøvUdg©wU‡K 45 †m‡K‡Û AwZµg K‡i| cøvUd‡g©i ˆ`N©¨ KZ?
Solution:
Let, length of the smaller train be ‘x’ meters
So, length of the longer train be ‘2x’ meters
Total length of two trains = x+2x = 3x meters
5
Actual speed of two trains = (48+42) km/hr = 90 km/hr = 90× m/s = 25 m/s
18
We know,
3x = 25×12 [ Distance = Speed ×Time ]
⇒ x = 100
So, length of the smaller train = 100 meters
length of the longer train = 2×100 = 200 meters
Let, length of the platform be ‘y’ meters.
According to the question,
5
200 + y = 45 × (48× ) [ Distance = Time ×Speed ]
18
⇒ 200 + y = 600

Written Math Solution (BIBM) FB Group : BD job Info. (BCS & Bank)
188

 Created by Sumon K. Sarkar  FB Group : BD job Info. (BCS & Bank)  Created by Sumon K. Sarkar 
Bank Job Solution (BIBM)
 Created by Sumon K. Sarkar  FB Group : BD job Info. (BCS & Bank)  Created by Sumon K. Sarkar 

⇒ y = 400
So, the length of the platform = 400 meters.
Answer: 400 meters

4. A, B and C can do a piece of work in 16, 32 and 48 days respectively. They started working
together but C left after working 4 days and B left 2 days before the completion of work. How
many days it took to complete the work?
[SJIB MTO 11, Social Islami Bank PO 17, NRBC Bank MTO 18, National Bank PO 17, Langka Bangla Finance MTO 17]
Abyev`t A, B I C GKwU KvR h_vµ‡g 16, 32 Ges 48 w`‡b Ki‡Z cv‡i| Zviv GKmv‡_ KvR Ki‡Z ïiæ K‡i wKš‘ KvR ïiæ
nIqvi 4 w`‡b ci C KvR †Q‡o †`q Ges KvR †kl nIqvi 2 w`b Av‡M B KvR †Q‡o †`q| m¤ú~Y© KvRwU †kln‡Z KZw`b mgq
jv‡M?
Solution:
Let, the work be done in x days.
A worked for x days, B for (x-2) and C for 4 days.
𝑥 𝑥−2 4
+ + =1
16 32 48
𝑥 𝑥−2 1
⇒ + + =1
16 32 12
6𝑥+3𝑥−6+8
⇒ =1
96

⇒ 9x +2 = 96
⇒ 9x = 96 – 2
⇒ 9x = 94
94
⇒x=
9
4
⇒ x = 10 𝑑𝑎𝑦𝑠
9
𝟒
Answer: 𝟏𝟎 𝒅𝒂𝒚𝒔
𝟗

Standard Bank Ltd.


Post name: Trainee Assistant Officer(Cash) Exam Date : 2011
𝒂𝟏/𝟐 +𝒂−𝟏/𝟐 𝟏−𝒂−𝟏/𝟐
1. Simplify: +
𝟏−𝒂 𝟏+√𝒂
Solution:
𝑎1/2 +𝑎 −1/2 1−𝑎 −1/2
Given that, +
1−𝑎 1+√𝑎
1 1
𝑎1/2 + 1− 1/2
𝑎1/2 𝑎
⇒ +
1−𝑎 1+√𝑎
1 1
√𝑎+ 1−
⇒ √𝑎
+ √𝑎
[ 𝑎1/2 = √𝑎 ]
1−𝑎 1+√𝑎
𝑎+1 √𝑎−1
√𝑎 √𝑎
⇒ +
1−𝑎 1+√𝑎

Written Math Solution (BIBM) FB Group : BD job Info. (BCS & Bank)
Bank Job Solution (BIBM) 189

Created by Sumon K. Sarkar  FB Group : BD job Info. (BCS & Bank)  Created by Sumon K. Sarkar 
𝑎+1 √𝑎−1
⇒ ÷ (1 − 𝑎) + ÷ (1 + √𝑎)
Created by Sumon K. Sarkar  FB Group : BD job Info. (BCS & Bank)  Created by Sumon K. Sarkar 

√𝑎 √𝑎

𝑎+1 1 √𝑎−1 1
⇒ × + ×
√𝑎 1−𝑎 √𝑎 1+√𝑎

𝑎+1 1 √𝑎−1 1
⇒ × + ×
√𝑎 (1)2 −(√𝑎)2 √𝑎 1+√𝑎

𝑎+1 1 √𝑎−1 1
⇒ × + ×
√𝑎 (1+√𝑎)(1−√𝑎) √𝑎 1+√𝑎

𝑎+1 √𝑎−1
⇒ +
√𝑎(1+√𝑎)(1−√𝑎) √𝑎(1+√𝑎)

𝑎+1+(√𝑎−1)(1−√𝑎)

√𝑎(1+√𝑎)(1−√𝑎)

𝑎+1+√𝑎−1−𝑎+√𝑎

√𝑎(1+√𝑎)(1−√𝑎)

2√𝑎

√𝑎(1+√𝑎)(1−√𝑎)
2
⇒ [ 1 − a = (1 + √a)(1 − √a) ]
(1−𝑎)
𝟐
Answer:
(𝟏−𝒂)

2. In a survey, 60% of those surveyed owned a car and 80% of those surveyed owned a TV. If 55%
owned both a car and a TV, what percent of those surveyed owned a car or a TV or both?
Abyev`t GKwU cwimsL¨v‡b †`Lv hvq 60% †jv‡Ki GKwU Mvwo Av‡Q Ges 80% †jv‡Ki GKwU wUwf Av‡Q| hw` 55% †jv‡Ki
GKwU Mvwo Ges GKwU wUwf _v‡K Zvn‡j kZKiv KZRb †jv‡Ki GKwU Mvwo A_ev GKwU wUwf DfqB Av‡Q?
Solution:
Given that, percent of surveyed owned a car, n(a) = 60%,
percent of surveyed owned a TV, n(b) = 80%
percent of surveyed owned both a car and a TV, n(𝑎 ∩ 𝑏)= 55%
So, percent of surveyed owned only car, n(𝑎 ∩ 𝑏 ′ )= n(a) - n(𝑎 ∩ 𝑏) = (60-55)% = 5%
and percent of surveyed owned only TV, n(𝑎′ ∩ 𝑏)= n(b) - n(𝑎 ∩ 𝑏) = (80-55)% = 25%
So, total percent of surveyed owned a Car or a TV or Both = 5%+25%+55% = 85%
Answer: 85%

এ রকম গুরুত্বপূর্ণ ফাইল পপতে আমাতের গ্রুতপ প াখ রাখুন


এবং অবশ্যই ননতের বন্ধুতের ইনভাইট করুন

Written Math Solution (BIBM) FB Group : BD job Info. (BCS & Bank)
190

 Created by Sumon K. Sarkar  FB Group : BD job Info. (BCS & Bank)  Created by Sumon K. Sarkar 
Bank Job Solution (BIBM)
 Created by Sumon K. Sarkar  FB Group : BD job Info. (BCS & Bank)  Created by Sumon K. Sarkar 

3. Water has been poured into an empty rectangular tank at the rate of 5 cubic feet per minute for
6 minutes. The length of the tank is 4 feet and the width is one half of the length. How deep is the
water in the tank?
Abyev`t cvwbk~Y¨ GKwU AvqZKvi †PŠev”Pv cÖwZ wgwb‡U 5 Nb dzU nv‡i cvwbc~Y© Ki‡Z 6 wgwbU jv‡M| †PŠev”PvwUi ˆ`N©¨ 4 dzU Ges
cÖ¯’ ˆ`‡N¨©i A‡a©K| †PŠev”PvwUi MfxiZv KZ?
Solution:
Water's volume in rectangular tank for 6 minutes = 5×6 = 30 cubic feet.
1
Given that, The length of the tank = 4 feet and The width of the tank = 4× = 2 feet.
2
Let, the deep is the water in the tank be x feet.
So, Volume in rectangular tank = 4×2×x =8x
According to the question,
8x = 30
30
⇒𝑥 =
8
15
⇒𝑥 =
4
3
⇒𝑥 =3
4
3
⇒ x = 3 Feet ( × 12) inches [ 1feet = 12 inches]
4
⇒ x = 3 Feet 9 inches
Answer: 3 Feet 9 inches

NCC Bank Ltd.


Post : Management Trainee Officer Exam Date : 2011

1. A person earns yearly interest of Tk. 920 by investing Tk. X at 4% and Tk. Y at 5% simple
interest rate. If he had invested Tk. X at 5% and Tk. Y at 4% simple interest rate, then his yearly
interest earning would have been reduced by Tk. 40. Find out the amount of X and Y.
[ONE Bank PO 10, NCC Bank MTO 11, Premier Bank MTO 12, Midland Bank TO 15 ]
Abyev`t GK e¨w³ 4% my‡` x UvKv Ges 5% my‡` y UvKv wewb‡qvM K‡i eQ‡i 920 UvKv gybvdv AR©b K‡ib| wZwb hw` 5% my‡`
x UvKv Ges 4% my‡` y UvKv wewb‡qvM K‡ib Zvn‡j gybvdvi cwigvb 40 UvKv K‡g hv| x Ges y Gi cwigvb wbY©q Ki|
Solution:
According to the question,
X×4%+Y×5% = 920
⇒ 4X+5Y = 92000 ...................................(1)
And, X×5% + Y×4%=880
⇒ 5X+4Y = 88000....................................(2)
Now, subtraction the equation (1)×5 to (2)×4
20X+25Y – 20X – 16Y = 460,000 – 352,000
⇒ 9Y =108,000
⇒Y = 12000.
Putting the value of y in equation (1)
4X+5×12000 = 92000

Written Math Solution (BIBM) FB Group : BD job Info. (BCS & Bank)
Bank Job Solution (BIBM) 191

Created by Sumon K. Sarkar  FB Group : BD job Info. (BCS & Bank)  Created by Sumon K. Sarkar 
⇒ 4X+60,000 = 92,000
Created by Sumon K. Sarkar  FB Group : BD job Info. (BCS & Bank)  Created by Sumon K. Sarkar 

⇒ 4X = 92,000 - 60,000
⇒ 4X = 32,000
⇒ X=8000
Answer: X=8000 tk and Y= 12,000 tk

2. A basketball team has won 15 games and lost 9. If these games represent 16.67% of the games to
the played, then how many more games must the team win to average 75% for the season?
[ONE Bank PO 10, NCC Bank MTO 11. Premier Bank MTO 12, Midland Bank TO 15, Bangladesh bank AP 2023]
Abyev`t GKwU ev‡¯‹Uej wUg 15wU †Ljvq Rqjvf K‡i Ges 9wU †Ljvq civwRZ nq| hw` GB †Ljv¸‡jv Øviv Zv‡`i †gvU ‡Ljvi
16.67% †Ljv‡K †evSv‡bv nq Zvn‡j H wmR‡b M‡o 75% †Ljvq †RZvi Rb¨ Zv‡`i‡K KZwU †Ljvq wRZ‡Z n‡e?
Solution:
Total game that have already been played=15+9=24.
According to the question,
2 50
16 = % = 24
3 3
24×100
Or,100% = =144.
50/3
24×100×3
Or,100% = =144.
50
To win 75% of the game the team need to win = 144×75% = 108.
So it needs to win (108-15) = 93 more games.
Answer: 93 more games

3. A book sells for Tk. 65. This price gives the seller a profit of 30% on his cost. What will be the
new selling price if he cuts his profit to 10% of its cost?
Abyev`t GKwU eB‡qi weµqg~j¨ 65 UvKv| G‡Z we‡µZvi 30% jvf nq| hw` wZwb 10% jvf K‡ib Zvn‡j Zvi bZzb weµqg~j¨
KZ n‡e?
Solution:
Let, Cost of a book = Tk. 100.
At 30% profit, selling price of a book = Tk. 130.
100×65
So, cost of the book = ( ) = Tk. 50
130
Now, profit at 10% of a book = 50×10% = Tk. 5
So, new price of a book = Tk. (50+5) = Tk. 55
Answer: Tk. 55

এ রকম গুরুত্বপূর্ণ ফাইল পপতে আমাতের গ্রুতপ প াখ রাখুন


এবং অবশ্যই ননতের বন্ধুতের ইনভাইট করুন

Written Math Solution (BIBM) FB Group : BD job Info. (BCS & Bank)
192

 Created by Sumon K. Sarkar  FB Group : BD job Info. (BCS & Bank)  Created by Sumon K. Sarkar 
Bank Job Solution (BIBM)
 Created by Sumon K. Sarkar  FB Group : BD job Info. (BCS & Bank)  Created by Sumon K. Sarkar 

UCB Bank Ltd.


Post: Management Trainee Officer Exam Date : 2011

1. A pension fund has a total of Tk. 1 million invested in Bangladesh Biman bond and DEF
Corporation's debenture. The Bangladesh Biman bond yields 12% in cash each year, and the
DEF debenture pays 10% in cash each year. The pension fund received a total of Tk 115,000 in
cash from Bangladesh Biman bond and DEF debenture last year. How much money was invested
in Bangladesh Biman bond?
Abyev`t GKwU †cbkb dv‡Ûi 1 wgwjqb (10 j¶) UvKv evsjv‡`k wegv‡bi Pzw³c‡Î Ges wWBGd K‡c©v‡ik‡bi FYc‡Î wewb‡qvM
Kiv n‡q‡Q| evsjv‡`k wegv‡bi Pzw³cÎ Ges wWBGd K‡c©v‡ik‡bi FYcÎ †_‡K cÖwZ eQi h_vµ‡g 12% I 10% nv‡i bM` gybvdv
Avq nq| MZ eQi Gfv‡e †cbkb dvÛwU bM` 115,000 UvKv Avq K‡i| evsjv‡`k wegv‡bi Pzw³c‡Î KZ UvKv wewb‡qvM Kiv
n‡qwQ‡jv?
Solution:
Let, Invested in Bangladesh Biman bond be Tk. x
and DEF Corporation’s debenture be Tk. (10,00,000-x)
According to the question,
12% of x + 10% of (10,00,000-x) = 115,000
12𝑥 10(10,00,000−x)
+ = 115,000
100 100
⇒ 12x + 100,00,000 − 10x = 115,00,000
⇒ 2x = 115,00,000 − 100,00,000
⇒ 2x = 15,00,000
⇒ x = 7,50,000
So, Invested in Bangladesh Biman bond Tk. 7,50,000
Answer: Tk. 7,50,000

2. Company A owns 40% of the stock in ABC Company. Company B owns 15,000 shares. Company
C owns all the share not owned by Companies A or B. How many shares of stock does Company
A own if Company C has 25% more share than Company A?
[Premier Bank Officer 03, UCBL MTO 11]
Abyev`t XYZ Kc©‡ik‡bi 40% ÷K †Kv¤úvbx A Gi gvwjKvbvaxb| Company B Gi †kqv‡i cwigvb 15,000| A Ges B
Gi ev‡` evwK mg¯Í †kqv‡qi gvwjK C| C Gi hw` A Gi †P‡q 25% †ewk †kqvi _v‡K Zvn‡j A Gi †kqv‡ii cwigvb KZ?
Solution:
Let, total share be 100x
So, A’s Share = 40x and C’s Share = (40x + 25% of 40x) = 50x
According to the question,
100x - (40x + 50x) = 15,000
⇒ 100x – 90x = 15000
⇒ 10x = 15,000
⇒ 40x = 15,000 × 4
⇒ 40x = 60,000
‘A’ owns 60,000 shares of the stock.
Answer: 60,000 shares

Written Math Solution (BIBM) FB Group : BD job Info. (BCS & Bank)
Bank Job Solution (BIBM) 193

Created by Sumon K. Sarkar  FB Group : BD job Info. (BCS & Bank)  Created by Sumon K. Sarkar 
3. A manufacturer of boxes wants to make a profit of x taka. When he sells 5,000 boxes it costs Ik.
Created by Sumon K. Sarkar  FB Group : BD job Info. (BCS & Bank)  Created by Sumon K. Sarkar 

5 a box to make the first 1000 boxes and then it costs Tk. y a box to make the remaining 4,000
boxes. What price in taka should be charge for the 5000 boxes?
Abyev`t GKRb ev· cÖ¯‘ZKvix x UvKv jvf Ki‡Z Pvb| hLb wZwb 5000 ev· weµq K‡ib ZLb wZwb cÖ_g 1000 ev‡·i cÖwZwU‡Z
LiP K‡ib 5 UvKv, Gici evwK 4000 ev· ˆZix‡Z LiP K‡ib y UvKv| 5000 ev‡·i g~j¨ KZ avh© nIqv DwPZ?
Solution:
Cost of first 1,000 boxes = Tk. (5×1000) = Tk. 5000
Cost of remaining 4000 boxes = Tk. 4000y
∴ Total cost of 5,000 boxes = Tk. (4000y + 5000)
To make a profit of x taka, Selling price = Cost price + profit
= Tk. (4000y + 5000 +x)
Total price for 5000 boxes = Tk. (4000y + 5000 + x)
Answer: Tk. (4000y + 5000 +x)

4. In an increasing sequence of 10 consecutive integers, the sum of the first 5 integers is 560. What
is the sum of the last 5 integers in the sequence?
Abyev`t 10wU µgea©gvb c~Y©msL¨vi, cÖ_g 5wUi †hvMdj 560| †kl 5wU msL¨vi †hvMdj KZ?
Solution:
Let, the first 5 consecutive integers are x-2, x-1, x, x+1, x+2
According to the question,
x-2 + x-1+ x + x+1 + x+2 = 560
⇒ 5x = 560 ⇒ x = 112
The sum of last 5 integers = x+3 + x+4 + x+ 5 + x+ 6 + x + 7 = 5x +25
Put the value of x = 5x +25 = 5 ×112+25 = 585
Answer: 585

UCB Bank Ltd.


Post name: Officer Exam Date : 2011
2
1. If x + xy +1 = 16 and x-y=1, then x+y=?
Solution:
Given that,
x2 + xy +1 = 16 ------------- (1)
x-y=1
⇒ x=1+y ------------------------ (2)
Put the value of x in Equation (1)
(1+y)2 + (1+y)y +1 = 16
⇒ 1+2y+ y2+y+ y2 +1=16
⇒ 2y2+3y+2-16=0
⇒ 2y2+3y+14=0
⇒ 2y2+7y-4y+14=0
⇒ y(2y+7)-2(2y+7)=0
⇒ (2y+7)(y-2)=0

Written Math Solution (BIBM) FB Group : BD job Info. (BCS & Bank)
194

 Created by Sumon K. Sarkar  FB Group : BD job Info. (BCS & Bank)  Created by Sumon K. Sarkar 
Bank Job Solution (BIBM)
 Created by Sumon K. Sarkar  FB Group : BD job Info. (BCS & Bank)  Created by Sumon K. Sarkar 

∴ (2y+7)= 0 or (y-2)=0
7
𝑦=− or y = 2
2
Put the value of y in Equation (2)
7 7 2−7 −5
If 𝑦 = − , x = 1 − = =
2 2 2 2
5 7 −5−7 −12
So, x + y = − − = = = −6
2 2 2 2
If 𝑦 = 2 , x = 1 +2 = 3
So, x + y = 3+2 = 5
Answer: -6, and 5

2. Mr. B invests Ik. 2400 at 5% interest annually. How much additional money needs to invest at
8% interest to earn overall interest at 6% on entire amount?
[UCBL Officer 11, Modhumoti PO 18, SJIB TSO 18]
Abyev`t B mv‡ne evwl©K kZKiv 5 UvKv mij gybvdvq 2400 UvKv wewb‡qvM K‡ib| evwl©K kZKiv 8 UvKv mij gybvdvqAvi KZ
UvKv wewb‡qvM Ki‡j wZwb †gvU wewb‡qv‡Mi Dci GK‡Î evwl©K kZKiv 6 UvKv mij gybvdv cv‡eb?
Solution:
Let, Mr B Additional money invested be tk. x
According to the question,
8% of x + 5% of 2400 = 6%(x+2400)
8𝑥 5×2400 6(𝑥+2400)
⇒ + =
100 100 100
⇒ 8x+12000 = 6x+14400
⇒ 8x - 6x = 14400 – 12000
⇒ 2x = 2400
⇒ x = 1200
Mr B Additional money invest is tk. 1200
Answer: tk. 1200

3. A boy covers a distance of 6 km partly by walking and partly by cycling. If he cycles at 18 km


per hour and walks at 6 km per hour and takes 35 minutes in all, find the distance he covers by
walking.
Abyev`t GKRb evjK 6 wKwg `~i‡Z¡i wKQz Ask †nu‡U Ges wKQz Ask mvB‡Kj Pvwj‡q †M‡jv| hw` mvB‡K‡j N›Uvq 18 wKwg †e‡M
Ges N›Uvq 6 wKwg †e‡M †nu‡U hvq Ges †gvU 35 wgwbU mgq †bq, Zvn‡j †nu‡U AwZµvšÍ `~iZ¡ wbY©q Kiæb|
[Combined Cash 08, UCBL Officer 11]
Solution:
Let, the boy covers by walking in x km
the boy covers by cycling in 6 – x km
According to the question,
𝑥 6−𝑥 35
+ =
6 18 60
3𝑥+6−𝑥 7
⇒ =
18 12
2𝑥+6 7
⇒ =
18 12
2(𝑥+3) 7
⇒ =
18 12

Written Math Solution (BIBM) FB Group : BD job Info. (BCS & Bank)
Bank Job Solution (BIBM) 195

Created by Sumon K. Sarkar  FB Group : BD job Info. (BCS & Bank)  Created by Sumon K. Sarkar 
𝑥+3 7
⇒ =
9 12
Created by Sumon K. Sarkar  FB Group : BD job Info. (BCS & Bank)  Created by Sumon K. Sarkar 

𝑥+3 7
⇒ =
3 4
⇒ 4𝑥 + 12 = 21
⇒ 4𝑥 = 21 − 12
⇒ 4𝑥 = 9
9
⇒𝑥 = = 2.25
4
the boy covers by walking in 2.25 km
Answer: 2.25 km

4. A square office, 1000 feet by 1000 feet, is to be partitioned into two offices by a single interior
wall. The difference between the perimeters of the resulting two officers be 400 feet. What are
their dimensions? [BB AD 06, UCBL Officer 11]
অনুবােঃ ১০০নফট বাই ১০০০ নফট এর বগণাকার অনফস দুই ভাতগ নবভি করতে হতব পযন দুই অংতশ্র পনরসীমার পার্ণকয ৪০০ নফট হে।
তরর দে ণয ও প্রস্থ কে?
Solution:
Let, width of first portion after partition = x feet
So, width of second portion other side = 1000 - x feet
According to the question,
2(1000 + x) - 2(1000 + 1000 - x) = 400
⇒ 2000 + 2x – 4000 + 2x = 400
⇒ 4x = 2000 + 400
⇒ x = 600
So, width of first portion = 600 feet
Width of other side = 1000 – 600 = 400 feet
So, dimensions of 1st office = 1000 feet by 600 feet
And, dimensions of 2nd office = 1000 feet by 400 feet
Answer: 1000 feet by 600 feet & 1000 feet by 400 feet

এ রকম গুরুত্বপূর্ণ ফাইল পপতে আমাতের গ্রুতপ প াখ রাখুন


এবং অবশ্যই ননতের বন্ধুতের ইনভাইট করুন

Written Math Solution (BIBM) FB Group : BD job Info. (BCS & Bank)
196

 Created by Sumon K. Sarkar  FB Group : BD job Info. (BCS & Bank)  Created by Sumon K. Sarkar 
Bank Job Solution (BIBM)
 Created by Sumon K. Sarkar  FB Group : BD job Info. (BCS & Bank)  Created by Sumon K. Sarkar 

IFIC Bank Ltd.


Post: Officer Exam Date : 2011
1. A trader bought 65 meter cloth and sold 20 meter at a profit of 15%, 30 meter at a profit of 19%
and the rest at a profit of 25%. IN total he made a profit of tk. 498.What was the purchase price
of pa meter of cloth?
অনুবােঃ একেন নবতক্রো ৬৫ নমটার কাপড ক্রয কতরন এবং ১৫% লাতভ ২০ নমটার, ১৯% লাতভ ৩০ নমটার এবং অবনশ্ষ্ট কাপড ২৫%
লাতভ নবক্রয করন! নেনন পমাট ৪৯৮ টাকা লাভ কতরন। প্রনে নমটার কাপতডর ক্রযমূলয কে টাকা ?
Solution:
Let, purchase price of per meter cloth be Tk. x.
Selling price of 20 meter cloth= Tk. (20x+20x x15%) =Tk.23x.
Selling price of 30 meter cloth = Tk. (30x+30x x 19%) = Tk. 35.7x
Rest cloth = 65- (20+30) = 15meter.
And selling price of rest 15 meter = Tk. (15x+15x x 25%) = Tk. 18.75x
According to the question,
(23x +35.7x+18.75x) - 65x =498
⇒ 77.45x- 65x= 498
⇒ 12.45x = 498
⇒ x= 40
So, purchase price of per meter of cloth is Tk.40.
Answer: Tk.40.

2. A boat sailing against the current takes 8 hours to travel 32 kms, while sailing with the current
it takes only 4 hours to travels the same distance. What is the speed of the current?
অনুবােঃ পরাতের নবপরীতে একনট পনৌকা 32 নকতলানমটার পযতে 8 ন্টা সময পনয, যখন পরাতের সাতর্ যাত্রা কতর একই েূরত্ব পযতে 4
ন্টা লাতগ। পরাতের গনে কে?
Solution:
Let, Speed of current = y & Speed of boat = x
According to the question,
x - y = 32/8 = 4— (i)
x + y = 32/4 = 8 ——– (ii).
From, (ii) – (i),
x+y-x+y=8-4
⇒ 2y = 4
⇒y=2
∴ Speed of current = 2 km/hr
Answer: 2 km/hr
𝒂 𝒙 𝒙 𝒃
3. If + = + then x=?
𝒙 𝒂 𝒃 𝒙
Solution:
𝒂 𝒙 𝒙 𝒃
+ = +
𝒙 𝒂 𝒃 𝒙
𝒂 𝒃 𝒙 𝒙
⇒ − = −
𝒙 𝒙 𝒃 𝒂

Written Math Solution (BIBM) FB Group : BD job Info. (BCS & Bank)
Bank Job Solution (BIBM) 197

Created by Sumon K. Sarkar  FB Group : BD job Info. (BCS & Bank)  Created by Sumon K. Sarkar 
𝒂−𝒃 𝒂𝒙−𝒃𝒙
⇒ =
𝒙 𝒂𝒃
Created by Sumon K. Sarkar  FB Group : BD job Info. (BCS & Bank)  Created by Sumon K. Sarkar 

𝒂−𝒃 𝒙(𝒂−𝒃)
⇒ =
𝒙 𝒂𝒃
2
⇒ x = ab
⇒ x=±√𝑎𝑏
Answer: x=±√𝒂𝒃

UCB Bank Ltd.


Post: Officer Exam Date : 2011
𝟏 𝟏 𝟏
1. If 2x = 4y = 8z and + + = 𝟒 then find the value of x
𝟐𝒙 𝟒𝒚 𝟒𝒛
Solution:
Given that, 2x = 4y = 8z
So, 2x = 4y & 2x = 8z ⇒ x = 4z
Again Given That,
1 1 1
+ + =4
2x 4y 4z
1 1 1
⇒ + + =4
2x 2x x
1+1+2
⇒ =4
2x
4
⇒ =4
2x
⇒ 8x = 4
4 1
⇒ x= =
8 2
𝟏
Answer: 𝐱 =
𝟐

2. Fifteen years hence A will be twice as old as his son, but five years ago he was 4 times as old as
his son, Find their present age.
অনুবােঃ ১৫ বছর পর A এর বেস হতব োর পুতত্রর বেতসর নিগুন। নকন্তু ৫ বছর পূতবণ পস োর পুতত্রর বেতসর ৪ গুন নছল। োতের বেণমান
বেস কে?
Solution:
Let, five years ago son’s age = x years and A’s age= 4x years.
Present age of son= (x+5) years and A’s age= (4x+5) years.
According to the question,
2(x+5+15) = 4x+5+15
⇒ 2(x+20) = 4x+ 20
⇒ 2x+ 40 = 4x+20
⇒ 4x-2x= 40-20
⇒ 2x= 20
∴ x= 10
Present age of son= (10+5) = 15 years and A’s age = (4×10+5) = 45 years.
Answer: 15 years and 45 years.

Written Math Solution (BIBM) FB Group : BD job Info. (BCS & Bank)
198

 Created by Sumon K. Sarkar  FB Group : BD job Info. (BCS & Bank)  Created by Sumon K. Sarkar 
Bank Job Solution (BIBM)
 Created by Sumon K. Sarkar  FB Group : BD job Info. (BCS & Bank)  Created by Sumon K. Sarkar 

3. In a stream running at 2 kmph, a motor boat goes 10 km upstream and back again to the starting
point in 55 minutes. Find the speed of the motorboat in still water.
Abyev`t GKwU †gvUi PvwjZ †bŠKv 2 wK.wg / N›Uv †e‡Mi †mªv‡Zi wecix‡Z 10 wK.wg wM‡q Avevi wd‡i Avm‡Z †gvU 55 wgwbU mgq
†bq| w¯’i cvwb‡Z †bŠKvwUi †eM wbY©q Ki|
Solution:
Let, the speed of the motorboat in still water be x kmph.
So, speed of downstream = (x+2) kmph and speed of upstream = (x-2) kmph.
According to the question,
10 10 55
+ =
𝑥+2 𝑥−2 60
10𝑥−20+10𝑥+20 11
⇒ =
(𝑥+2)(𝑥−2) 12
20𝑥 11
⇒ =
(𝑥 2 −4) 12
2
⇒ 11𝑥 − 44 = 240𝑥
⇒ 11𝑥 2 − 240𝑥 − 44 = 0
⇒ 11𝑥 2 –242x+2x – 44 = 0
⇒ 11𝑥(𝑥−22)+2(x-22)=0
⇒ (x-22) (11x+2) = 0
∴ x = 22 or, x = - 2/11 [Neglecting the negative value]
∴ The speed of the motorboat in still water 22 kmph
Answer: 22 kmph

4. Janaki spends 20% of her monthly salary for her son's education. When she will get her
increment of Tk. 170 next month, she plans to spend half of that also for her son's education. The
total money she would spend then, would be Tk. 645. What is her present salary?
Abyev`t †RvbwK Zvi mšÍv‡bi cov‡jLvi †cQ‡b Zvi †eZ‡bi 20% e¨q K‡ib| wZwb wm×všÍ wb‡q‡Qb †h, AvMvgx gv‡m hLb Zvi
†eZb 170 UvKv e„w× cv‡e, ZLb wZwb GB UvKvi A‡a©KI Zvi mšÍv‡bi cov‡kvbvi †cQ‡b e¨q Ki‡eb | ZLb wZwb †gvU 645
UvKv e¨vq Ki‡eb | Zvi eZ©gvb †eZb KZ?
Solution:
Let, Janaki’s present monthly salary be Tk. x
According to the question,
1
20% of x + of 170 = 645
2
20𝑥
⇒ + 85 = 645
100
20𝑥
⇒ = 645- 85
100
20𝑥
⇒ = 560
100
560×100
⇒𝑥=
20
⇒ 𝑥 = 2800
Janaki’s present monthly salary be Tk. 2800
Answer: Tk. 2800

Written Math Solution (BIBM) FB Group : BD job Info. (BCS & Bank)
Bank Job Solution (BIBM) 199

Created by Sumon K. Sarkar  FB Group : BD job Info. (BCS & Bank)  Created by Sumon K. Sarkar 
5. A wheel makes 1000 revolution in covering a distance of 88 km. Determine the diameter of the
Created by Sumon K. Sarkar  FB Group : BD job Info. (BCS & Bank)  Created by Sumon K. Sarkar 

wheel.
Abyev`t GKwU PvKvi 80 wKwg `~iZ¡ cvwo w`‡Z 1000 N~Y©‡bi cÖ‡qvRb nq| PvKvwUi e¨vm wbY©q Kiæb|
Solution:
Let, Radius of the wheel ‘r’
We know, perimeter of the wheel = 2πr;
According to the question,
2πr×1000 = 88,000 [88 km= 88,000 meters]
22
⇒ 2× ×r×1000 = 88,000
7
88,000×7
⇒𝑟=
2×22×1000
⇒ 𝑟 = 14
⇒ 2𝑟 = 28
∴ Diameter 2𝑟 = 28 meters
Answer: 28 meters

6. In an examination, a student was asked to find 3/4 of a certain number. By mistaken he found
3/14 of it. His answer was 150 more than the correct answer. Find the number.
Abyev`t GKwU cix¶vq GKRb Qv·K GKwU msL¨vi 3/4 Ask †ei Ki‡Z ejv nj| fzjekZ †m AsKwUi 3/14 Ask †ei K‡i
†djj| cÖvß djvdj mwVK djvdj †_‡K 150 †ewk wQj| msL¨vwU wbY©q Kiæb|
Solution:
Let, the certain number be x.
According to the question,
3 3
of x - of x = 150
14 4
3𝑥 3𝑥
⇒ − = 150
14 4
6𝑥−21𝑥
⇒ = 150
28
−15𝑥
⇒ = 150
28
150×28
⇒x=
−15
∴ x = - 280
Answer: - 280

এ রকম গুরুত্বপূর্ণ ফাইল পপতে আমাতের গ্রুতপ প াখ রাখুন


এবং অবশ্যই ননতের বন্ধুতের ইনভাইট করুন

Written Math Solution (BIBM) FB Group : BD job Info. (BCS & Bank)
200

 Created by Sumon K. Sarkar  FB Group : BD job Info. (BCS & Bank)  Created by Sumon K. Sarkar 
Bank Job Solution (BIBM)
 Created by Sumon K. Sarkar  FB Group : BD job Info. (BCS & Bank)  Created by Sumon K. Sarkar 

Shahjalal Islami Bank Ltd.


Post: Management Trainee Officer Exam Date : 2010

1. A man is standing on a railway bridge which is 180 m long. He finds that a train crosses the
bridge in 20 seconds but him in 8 seconds. Find the length of the train.
[Sonali Officer IT 16, SJIB MTO 10]
Abyev`t †Kvb e¨w³ 180 wg. j¤^v GKwU †ijI‡q eªx‡Ri Dci `vwo‡q Av‡Q| wZwb †`L‡jb †h GKwU †Uªb 20 †m‡K‡Û eªxRwU
AwZµg K‡i wKš‘ Zv‡K 8 †m‡K‡Û AwZµg K‡i hvq| †Uª‡bi ˆ`N¨© wbY©q Ki|
Solution:
Let, the length of the train be x metres.
Then, the train covers x metres in 8 seconds and (x + 180) metres in 20 seconds.
According to the question,
𝑥 𝑥+180 𝐷𝑖𝑠𝑡𝑎𝑛𝑐𝑒
∴ = [ 𝑠𝑝𝑒𝑒𝑑 = ]
8 20 time
𝑥 𝑥+180
⇒ =
2 5
⇒ 5x = 2(x + 180)
⇒ 5x = 2x + 360
⇒ 5x - 2x = 360
⇒ 3x = 360
∴ x = 120.
So, Length of the train = 120 m.
120
Speed of the train = m/sec
8
120 18
=( × ) kmph
8 5
= 54 kmph.
Answer: 54 kmph.

2. A trader bought some mangoes for Tk. 150 per dozen and equal number of apples for Tk. 100
per dozen. If he sells all the fruits Tk. 140 per dozen, what will be his profit/loss in percentage?
[BASIC Bank PO 99, RAKUB SO 14, SJIB MTO 10]
Abyev`t GKRb e¨emvqx cÖwZ WRb 150 UvKv `‡i wKQz Avg Ges cÖwZ WRb 100 UvKv `‡i mgvb msL¨K Av‡cj µq K‡ib| hw`
†m me¸‡jv dj 140 UvKv WRb wn‡m‡e weµq K‡i Zvn‡j Zvi kZKiv KZ jvf ev ¶wZ n‡e?
Solution:
Let, A trader bought mangoes be x dozen and bought apples x dozen
So, Cost price = 150x + 100x= Tk. 250x
Selling price = Tk. (140x2x) = Tk. 280x
Profit = Tk. (280x-250x) = Tk. 30x
30𝑥×100
So, percentage of profit = = 12%
250𝑥
Answer: 12%
Alternative Way:
Given, cost price of per dozen mango = Tk. 150 and per dozen apple = Tk. 100
Cost price (per dozen mango + per dozen apple) = Tk. (150+100) = Tk. 250
Selling price (per dozen mango+ per dozen apple) = Tk. (140×2) = Tk. 280

Written Math Solution (BIBM) FB Group : BD job Info. (BCS & Bank)
Bank Job Solution (BIBM) 201

Created by Sumon K. Sarkar  FB Group : BD job Info. (BCS & Bank)  Created by Sumon K. Sarkar 
Profit = Tk. (280–250) = Tk. 30
Created by Sumon K. Sarkar  FB Group : BD job Info. (BCS & Bank)  Created by Sumon K. Sarkar 

30×100
So, Percentage of profit = % = 12%
250
Answer: 12%

𝟏 𝟓
3. In a school, there are equal number of boys and girls. Among the students of the girls and th
𝟖 𝟔
of the boys are residing in the hostel. What percent of the Students consists of boys who do not
reside in the hostel among all students?
[Modhumoti Bank MTO 16, SJIB MTO 10]
Abyev`t GKwU we`¨vj‡q †Q‡j Ges †g‡qi msL¨v mgvb| wk¶v_©x‡`i g‡a¨ 1/8 Ask †g‡q Ges 5/6 Ask †Q‡j †nv‡÷‡j evm K‡i|
†gvU wk¶v_©xi g‡a¨ kZKiv KZRb †Q‡j †nv‡÷‡j evm K‡i bv?
Solution:
Let, the number of boys be ‘x’
So, the number of girls be ‘x’.
So, Total students = x + x = 2x
5 5x 6x−5x x
Non residing boys, = x − of x = x − = =
6 6 6 6
x
6
∴Required percentage = × 100% = 8.33%
2x
Answer: 8.33%
Alternative Way,
Let, total student be 48
So, boys= 24 and girls= 24
5
Boys residing in the hostel =24× = 20
6
Boys do not reside in the hostel =24-20= 4
4
Required percentage = ×100 % = 8.33 %
48
Answer: 8.33 %

4. Two partner's A and B have 70% and 30% shares respectively in a business. After sometimes, a
third partner C joined by investing Tk. 10 lakh and thus having 20% share in the business. What
is the percentage of share of A's now in the business?
[RAKUB SO 14, SJIB MTO 10]
Abyev`t `yBRb cvU©bvi A I B GKwU e¨emvq h_vµ‡g 70% I 30% †kqv‡ii Askx`vi| wKQzw`b ci C Zv‡`i mv‡_ †hvM †`q
Ges †m 10 j¶ UvKv wewb‡qvM Kivq e¨emvq Zvi †kqv‡ii cwigvb nq 20%| eZ©gv‡b e¨emvq A Gi kZKiv †kqv‡ii cwigvb
KZ?
Solution:
Given that, A: B=70%: 30% = 7:3
Ratio of A and B, A: B= 70%: 30%= 7:3
After joining C, he got 20% share, So, then total share of A and B = (100-20%) = 80%
7
A's share = × 80% = 56%
10
Answer: 56%

Written Math Solution (BIBM) FB Group : BD job Info. (BCS & Bank)
202

 Created by Sumon K. Sarkar  FB Group : BD job Info. (BCS & Bank)  Created by Sumon K. Sarkar 
Bank Job Solution (BIBM)
 Created by Sumon K. Sarkar  FB Group : BD job Info. (BCS & Bank)  Created by Sumon K. Sarkar 

5. Taxi fare is described by the following relationship:


Total taxi Fare = Fixed Charge: Tk. A up to 2 km + Tk. B per km run exceeding 2 kın + Tk 60 for per hour's waiting.
A person paid Tk. 432 for running 52 km and 2 hours of waiting charge. The same person paid
Tk. 732 for running 102 km and 2 hours of waiting charge. Find the value of A and B
[ Bangladesh bank officer (electrical) 2023]
Abyev`t wb‡Pi m¤c‡K©i mvnvh¨ †Uw· fvov †`Lv‡bv hvq:
†gvU †Uw· fvov = ¯’vqx PvR© A UvKv 2wKwg ch©šÍ + B UvKv 2 wKwg c‡i +60 UvKv cÖwZ N›Uv A‡c¶v PvR©|
GKRb e¨w³ 52 wKwg Ges 2 N›Uv A‡c¶vi Rb¨ 432 UvKv w`j| GKB e¨w³ 102 wKwg I 2 N›Uv A‡c¶vi Rb¨ 732 UvKv cÖ`vb
K‡i| A Ges B wbb©q Ki|
Solution:
Given that,
Total taxi Fare = Tk. A upto 2 km+Tk. B per km (exceeding 2 km)+Tk.60 for per hour waiting
According to the 1st condition,
A + (52 – 2 )B + 2× 60 = 432 Let, Total distance d km & time t hours
⇒ A + 50B + 120 = 432 Total fare = A + (d – 2 )B + t × 60
⇒ A + 50B = 432 – 120
⇒ A + 50B = 312 --------------------------(1)
According to the 2nd condition,
A + (102 – 2 )B + 2× 60 = 732
⇒ A + 100B + 120 = 732
⇒ A + 100B = 732 – 120
⇒ A + 100B = 612 ------------------------(2)
Subtract equation (1) from (2)
A + 100B – A – 50B = 612 – 312
⇒ 50B = 300
⇒B=6
Put the value of B in Equation (1)
⇒ A + 50 × 6 = 312
⇒ A + 300 = 312
⇒ A = 312 – 300
⇒ A = 12
Answer: A = 12 & B = 6

6. In the figure below, AB is perpendicular to BC and BD = DC. If AD = √𝟏𝟎 cm and AC = 4 cm,


then what is the value of BC?

Given That, AB is perpendicular to BC and BD = DC. If AD = √10 cm and AC = 4 cm

Written Math Solution (BIBM) FB Group : BD job Info. (BCS & Bank)
Bank Job Solution (BIBM) 203

Created by Sumon K. Sarkar  FB Group : BD job Info. (BCS & Bank)  Created by Sumon K. Sarkar 
Let, BD = DC = x
Created by Sumon K. Sarkar  FB Group : BD job Info. (BCS & Bank)  Created by Sumon K. Sarkar 

So, BC = x + x = 2x
∆𝐴𝐵𝐷′ 𝑠 𝐴𝐷2 = 𝐴𝐵2 + 𝐵𝐷 2
2
⇒ √10 = 𝐴𝐵2 + 𝑥 2
⇒ 10 = 𝐴𝐵2 + 𝑥 2
⇒ 𝐴𝐵2 = 10 − 𝑥 2
∆𝐴𝐵𝐶 𝑠 𝐴𝐶 = 𝐴𝐵2 + 𝐵𝐶 2
′ 2

⇒ 42 = 10 − 𝑥 2 + (2𝑥)2
⇒ 16 = 10 − 𝑥 2 + 4𝑥 2
⇒ 3𝑥 2 = 16 − 10
⇒ 3𝑥 2 = 6
⇒ 𝑥2 = 2
∴ x = √2
The value of BC = 2x = 2√2
Answer: 2√𝟐

Social Islami Bank Ltd.


Post name: Trainee Officer Exam Date : 2010
1. Solve the following equation:
𝟐 𝟑
+ =𝟏
𝒙−𝟐 𝒙+𝟑
Solution:
𝟐 𝟑
+ =𝟏
𝒙−𝟐 𝒙+𝟑
𝟐 𝟑
⇒ =𝟏−
𝒙−𝟐 𝒙+𝟑
𝟐 𝒙+𝟑−𝟑
⇒ =
𝒙−𝟐 𝒙+𝟑
𝟐 𝒙
⇒ =
𝒙−𝟐 𝒙+𝟑
⇒ 𝑥 2 − 2𝑥 = 2𝑥 + 6
⇒ 𝑥 2 − 2𝑥 − 2𝑥 − 6 = 0
⇒ 𝑥 2 − 4𝑥 − 6 = 0
⇒ 𝑥 2 − 2. 𝑥. 2 + 22 − 22 − 6 = 0
⇒ (𝑥 − 2)2 − 4 − 6 = 0
⇒ (𝑥 − 2)2 − 10 = 0
⇒ (𝑥 − 2)2 = 10
⇒ 𝑥 − 2 = ±√10
⇒ 𝑥 = 2 ± √10
Answer: 𝒙 = 𝟐 ± √𝟏𝟎

Written Math Solution (BIBM) FB Group : BD job Info. (BCS & Bank)
204

 Created by Sumon K. Sarkar  FB Group : BD job Info. (BCS & Bank)  Created by Sumon K. Sarkar 
Bank Job Solution (BIBM)
 Created by Sumon K. Sarkar  FB Group : BD job Info. (BCS & Bank)  Created by Sumon K. Sarkar 

2. An article is sold for Tk. 190, hence gaining a certain amount. Had the article been sold for Tk.
175, he would have suffered loss equal to 50% of the gain in the first case. Find cost price of the
article.
Abyev`t GKUv cY¨ 190 UvKvq weµq K‡i wbw`©ô cwigvb jvf nj| hw` cY¨wU 175 UvKvq weµq Kiv n‡Zv Z‡e Zvi Av‡Mi jv‡fi
50% Gi mgvb ¶wZ n‡Zv| cY¨wUi µqg~j¨ KZ?
Solution:
Let, Cost price of an article be Tk. x
At tk 190 selling, Gain (190 – x)
At tk 175 selling, loss (x – 175)
According to the condition,
(190 – x)×50% = x – 175
(190 – 𝑥)×50
⇒ = x – 175
100
(190 – 𝑥)
⇒ = x – 175
2
⇒ 2x – 350 = 190 – x
⇒ 2x + x = 190 + 350
⇒ 3x = 540
⇒ x = 180
Cost price of an article be Tk. 180
Answer: Tk. 180
Alternative way,
Let, Profit of an article be Tk. 2x
So,Loss of an article be Tk. 2x ×50% = x
According to the condition,
190 – 2x = 175 + x
⇒ 3x = 190 – 175
⇒ 3x = 15
⇒x=5
Profit of an article be 2x =Tk. 2×5 = 10
Cost price of an article = Selling price – Loss
= 190 – 10 = Tk.180
Answer: Tk.180

3. In an examination, a student gets 38% of the maximum marks and fails by 6 marks. Another
student who gets 45% of the maximum marks, gets 15 marks more than the required passing
percentage. Find passing percentage of the marks.
Abyev`t †Kvb cix¶vq GKRb wk¶v_©x H cix¶vq cÖvß m‡e©v”P b¤^‡ii 38% †c‡q 6 b¤^‡ii e¨veav‡b †dj K‡i| Ab¨ GKRb
wk¶v_©x m‡e©v”P bv¤^v‡ii 45% †c‡q cvk b¤^‡ii †P‡q 15 b¤^i †ewk cvq| H cix¶vi kZKiv cvk b¤^i KZ wQj?
Solution:
Let, the maximum marks be x
Then Pass mark for the 1st student = x × 38% + 6
38
=x× +6
100
38𝑥
= +6
100
Pass mark for the 2nd student = x × 45% - 15

Written Math Solution (BIBM) FB Group : BD job Info. (BCS & Bank)
Bank Job Solution (BIBM) 205

Created by Sumon K. Sarkar  FB Group : BD job Info. (BCS & Bank)  Created by Sumon K. Sarkar 
45
=x× - 15
100
Created by Sumon K. Sarkar  FB Group : BD job Info. (BCS & Bank)  Created by Sumon K. Sarkar 

45𝑥
= - 15
100
Since pass marks are same for both the students
45𝑥 38𝑥
So, According to the question, - 15 = +6
100 100
45𝑥 38𝑥
⇒ − = 6 + 15
100 100
45𝑥−38𝑥
⇒ = 21
100
⇒ 7x = 2100
⇒ x = 300
∴ Pass mark = 300× 38% + 6 = 114 + 6 =120
120
So, Pass percentage = ×100 = 40%
300
Answer: 40%

4. A sum of Tk. 5500 is divided into two parts and invested at 4% p.a. for 3 years and at 5% p.a.
for 6 years respectively. If interest earned on the second part is three times of that on the first
part, find the amount invested in second part.
Abyev`t 5500 UvKv `yB fv‡M fvM K‡i, GK fvM evwl©K 4% gybvdvq 3 eQ‡ii Rb¨ Ges Aci fvM evwl©K 5% gybvdvq 6
eQ‡ii Rb¨| hw` wØZxq fv‡M cÖvß gybvdv cÖ_g fv‡Mi Zzjbvq 3 ¸Y †ewk nq, Zvn‡j wØZxq fv‡M wewb‡qvMK…Z
A‡_©i cwigvY wbY©q Kiæb|
Solution:
Let, the amount invested in second part at 5% p.a. for 6 years be Tk. x
the amount invested in first part at 4% p.a. for 3 years be Tk. (5500 – x)
𝑥×6×5 3𝑥
Interest Tk. x at 5% p.a. for 6 years = x × 6 × 5% = =
100 10
(5500 – x) ×3×4 3(5500 – x)
Interest Tk. x at 4% p.a. for 3 years = (5500 – x) × 3 × 4% = =
100 25
According to the question,
3𝑥 3(5500 – x)
= 3×
10 25
𝑥 3(5500 – x)
⇒2= 5
⇒ 5x = 33000 – 6x
⇒ 5x + 6x = 33000
⇒ 11x = 33000
⇒ x = 3000
The amount invested in second part at 5% p.a. for 6 years be Tk. 3000
Answer: Tk. 3000

এ রকম গুরুত্বপূর্ণ ফাইল পপতে আমাতের গ্রুতপ প াখ রাখুন


এবং অবশ্যই ননতের বন্ধুতের ইনভাইট করুন

Written Math Solution (BIBM) FB Group : BD job Info. (BCS & Bank)
206

 Created by Sumon K. Sarkar  FB Group : BD job Info. (BCS & Bank)  Created by Sumon K. Sarkar 
Bank Job Solution (BIBM)
 Created by Sumon K. Sarkar  FB Group : BD job Info. (BCS & Bank)  Created by Sumon K. Sarkar 

5. The length of a rectangle is 7 more than its width. If the perimeter of the rectangle is the same
as the perimeter of a square of side 8.5, what is the length of a diagonal of the rectangle?
Abyev`t GKwU AvqZ‡¶‡Îi ˆ`N¨©, cÖ‡¯’i Zzjbvq 7 GKK †ewk| hw` AvqZ‡¶ÎwUi cwimxgv, 8.5 GKK avi wewkó eM©‡¶‡Îi
cwimxgvi mgvb nq; Zvn‡j AvqZ‡¶‡Îi K‡Y©i ˆ`N¨© KZ?
Solution:
Let, the width of a rectangle be x unit
the length of a rectangle be x+7 unit
Perimeter of a square = 8.5 × 4 = 34 Unit
Perimeter of a rectangle = 2(length + width) = 2(x+7+x) = 2(2x+7)
According to the question,
2(2x+7) = 34
⇒ 2x+7 = 17
⇒ 2x = 17 – 7
⇒ 2x = 10
⇒x=5
Width of a rectangle 5 unit & the length of a rectangle = 5+7 = 12
The length of a diagonal of the rectangle = √length𝟐 + width𝟐
=√122 + 52
=√144 + 25
=√169
=13
Answer: 13 unit

One Bank Ltd.


Post: Probationary Officer Exam Date : 2010

5 15
1. A snapshot measures inches by inches. It is to be enlarged so that the longer dimension will
2 8
be 4 inches. What will be the length of the enlarged shorter dimension?
5 15
Abyev`t GKwU mœ¨vc k‡Ui cwigvc Bw evB BwÂ| GUv‡K Ggbfv‡e evov‡bv nq †hb e„nËi cÖvšÍwU 4 Bw nq| ¶z`ªZi
2 8
cÖvšÍwUi ˆ`N¨© KZ n‡e?
Solution:
5 15
Old length inches & old width inches Alternative way,
2 8
5 5×4 20
New length 4 inches Ratio, = =
2 2×4 8
Let, new width be x inches Ratio of longer & shorter dimension
According to the condition, 20 15
5 15 = ∶
8 8
∶ =4∶𝑥
2 8 = 20 : 15
5 15
⇒ ÷ =4 ÷𝑥 =4:3
2 8
5 8 4 length of shorter dimension 3 inches.
⇒ × =
2 15 𝑥
Answer: 3 inches.

Written Math Solution (BIBM) FB Group : BD job Info. (BCS & Bank)
Bank Job Solution (BIBM) 207

Created by Sumon K. Sarkar  FB Group : BD job Info. (BCS & Bank)  Created by Sumon K. Sarkar 
4 4
⇒ =
3 𝑥
Created by Sumon K. Sarkar  FB Group : BD job Info. (BCS & Bank)  Created by Sumon K. Sarkar 

⇒ 𝑥=3
length of shorter dimension 3 inches.
Answer: 3 inches.

2. A basketball team has won 15 games and lost 9. If these games represent 16.67% of the games to
the played, then how many more games must the team win to average 75% for the season?
[ONE Bank PO 10, NCC Bank MTO 11, Premier Bank MTO 12, Midland Bank TO 15 , Bangladesh bank AP 2023]
Abyev`t GKwU ev‡¯‹Uej wUg 15wU †Ljvq Rqjvf K‡i Ges 9wU †Ljvq civwRZ nq| hw` GB †Ljv¸‡jv Øviv Zv‡`i †gvU
‡Ljvi 16.67% †Ljv‡K †evSv‡bv nq Zvn‡j H wmR‡b M‡o 75% †Ljvq †RZvi Rb¨ Zv‡`i‡K KZwU †Ljvq
wRZ‡Z n‡e?
Solution:
Total game that have already been played=15+9=24.
According to the question,
16.67% = 24
24×100
Or,100% = =144.
16.67
To win 75% of the game the team need to win = 144×75% = 108.
So it needs to win (108-15) = 93 more games.
Answer: 93 more games

𝟒√𝒑 ×𝟏𝟔𝒑𝒒 ×𝟐−𝟐√𝒑


3. × 𝟒−𝒑𝒒 = ?
𝟖√𝒒 ×𝟒𝒑𝒒 ×𝟐−𝟑√𝒒
Solution:
4 √p ×16pq ×2−2√p
× 4−pq
8√q ×4 pq ×2−3√q
22√p ×24pq ×2−2√p
= × 2−2pq
23√q ×22pq ×2−3√q
22√p+4pq−2√p−2pq
=
23√q+2pq−3√q
22pq
= =1
22pq
Answer: 1

এ রকম গুরুত্বপূর্ণ ফাইল পপতে আমাতের গ্রুতপ প াখ রাখুন


এবং অবশ্যই ননতের বন্ধুতের ইনভাইট করুন

Written Math Solution (BIBM) FB Group : BD job Info. (BCS & Bank)
208

 Created by Sumon K. Sarkar  FB Group : BD job Info. (BCS & Bank)  Created by Sumon K. Sarkar 
Bank Job Solution (BIBM)
 Created by Sumon K. Sarkar  FB Group : BD job Info. (BCS & Bank)  Created by Sumon K. Sarkar 

4. A person earns yearly interest of Tk. 920 by investing Tk. X at 4% and Tk. Y at 5% simple
interest rate. If he had invested Tk. X at 5% and Tk. Y at 4% simple interest rate, then his yearly
interest earning would have been reduced by Tk. 40. Find out the amount of X and Y.
[ONE Bank PO 10, NCC Bank MTO 11, Premier Bank MTO 12, Midland Bank TO 15]
Abyev`t GK e¨w³ 4% my‡` x UvKv Ges 5% my‡` y UvKv wewb‡qvM K‡i eQ‡i 920 UvKv gybvdv AR©b K‡ib| wZwb hw`
5% my‡` x UvKv Ges 4% my‡` y UvKv wewb‡qvM K‡ib Zvn‡j gybvdvi cwigvb 40 UvKv K‡g hv| x Ges y Gi
cwigvb wbY©q Ki|
Solution:
According to the question,
X×4%+Y×5% = 920
⇒ 4X+5Y = 92000 ...................................(1)
And, X×5% + Y×4%=880
⇒ 5X+4Y = 88000....................................(2)
Now, subtraction the equation (1)×5 to (2)×4
20X+25Y – 20X – 16Y = 460,000 – 352,000
⇒ 9Y =108,000
⇒Y = 12000.
Putting the value of y in equation (1)
4X+5×12000 = 92000
⇒ 4X+60,000 = 92,000
⇒ 4X = 92,000 - 60,000
⇒ 4X = 32,000
⇒ X=8000
Answer: X= 8000 tk and Y= 12,000 tk

5. 10% fruit of a seller was damaged during transportation, another 15% was rotten. At what profit
in percentage should he sell rest fruit so that he can make an overall profit of 20%?
Abyev`t GKRb dj we‡µZvi 10% dj cwien‡bi mgq bó nq Ges 15% dj cu‡P hvq| kZKiv KZ jv‡f Aewkó
dj¸‡jv weµq Ki‡j †gv‡Ui Dci Zvi 20% jvf n‡e?
Solution:
Let, seller bought the mangoes for Tk. 100
Fress Mangoes = 100 – (100 ×10%) - (100 ×15%) = 100 – 10 – 15 = 100 – 25 = 75
If he can make an overall profit of 20%, selling price be Tk. (100+20)= Tk.120
120−75
Now, profit in percentage = × 100% = 60%
75
Answer: 60%

এ রকম গুরুত্বপূর্ণ ফাইল পপতে আমাতের গ্রুতপ প াখ রাখুন


এবং অবশ্যই ননতের বন্ধুতের ইনভাইট করুন

Written Math Solution (BIBM) FB Group : BD job Info. (BCS & Bank)
Bank Job Solution (BIBM) 209

Created by Sumon K. Sarkar  FB Group : BD job Info. (BCS & Bank)  Created by Sumon K. Sarkar 
Bangladesh Bank
Created by Sumon K. Sarkar  FB Group : BD job Info. (BCS & Bank)  Created by Sumon K. Sarkar 

Post Name: Assistant Director Exam Date : 2010

1. Three partners A, B, and C start a business. Twice the capital of A is equal to thrice the capital
of B, and the capital of B is 4 times the capital of C. They share the profit in the ratio of their
capital. In a particular year, the gross profit is Tk. 2,50,000 and the administrative expenses are
20% of the gross profit. Find the share of profit of each partner.
অনুবােঃ নেন অংশ্ীোর A, B, এবং C একনট বযবসা শুরু কতর। A এর নিগুর্ মূলযন B এর নেনগুর্ মূলযতনর সমান এবং B এর মূলযন
C এর মূলযতনর 4 গুর্। োরা োতের মূলযতনর অনুপাতে মুনাফা ভাগ কতর পনয। একনট নননেণষ্ট বছতর পমাট মুনাফা হয ২৫০,০০০ টাকা।
এবং প্রশ্াসননক বযয পমাট লাতভর 20%। প্রনেনট অংশ্ীোতরর লাতভর অংশ্ কে?
2. Solution:
Let, Capital of C Tk x
So, Capital of B Tk 4x
4𝑥×3
∴ Capital of A Tk = Tk 6𝑥
2
Ratio of the Capital = 6x : 4x : x
=6:4:1
Sum of the ratio = 6+4+1=11
Profit excluding administrative expenses = 2,50,000 - 2,50,000 × 20%
= 2,50,000 – 50,000
= 2,00,000
6
Share profit of A = 2,00,000 × = 1,09,090
11
4
Share profit of B = 2,00,000 × = 72,727
11
1
Share profit of C = 2,00,000 × = 18,183
11
Answer: Tk 𝟏, 𝟎𝟗, 𝟎𝟗𝟎 , Tk 𝟕𝟐, 𝟕𝟐𝟕, Tk 𝟏𝟖, 𝟏𝟖𝟑

3. Tanim bought some oranges. He gave ½ of them to his sister, 1/4th of the remainder to his
neighbor, 3/5th of those left to his children and had 6 left in the end. How many oranges did
Tanim buy?
Abyev`t Zvwbg wKQz Kgjv‡jey wKbj, hvi A‡a©K †m Zvi †evb‡K w`j| Aewkó Kgjvi 1/4 fvM cÖwZ‡ewk‡`i w`j| Gici hv
Aewkó wQj Zvi 3/5 fvM Zvi mšÍvb‡`i †`Iqvi ci Zvi Kv‡Q 6wU Kgjv iBj| Zvwbg KZwU Kgjv wK‡bwQj?
[BB AD 10, MTB MTO 19]
Solution:
Let, Tanim bought x oranges.
𝑥 𝑥
After giving ½ to her sister, she has x - =
2 2
he give 1/4th of the remainder to her neighbour,
𝑥 𝑥 𝑥 𝑥 4𝑥−𝑥 3𝑥
so he now has = - = - = =
2 2×4 2 8 8 8
3𝑥 3 9𝑥
children got = × =
8 5 40
3𝑥 9𝑥 15𝑥−9𝑥 6𝑥 3𝑥
Now, she has = − = = =
8 40 40 40 20

Written Math Solution (BIBM) FB Group : BD job Info. (BCS & Bank)
210

 Created by Sumon K. Sarkar  FB Group : BD job Info. (BCS & Bank)  Created by Sumon K. Sarkar 
Bank Job Solution (BIBM)
 Created by Sumon K. Sarkar  FB Group : BD job Info. (BCS & Bank)  Created by Sumon K. Sarkar 

According to the Question,


3𝑥
⇒ =6
20
6×20
⇒𝑥 = = 40
3
⇒x = 40
So he bought 40 oranges.
Answer: 40 oranges.

4. The length of rectangular plot is greater than its breadth by 20 meters. If the perimeter of the
plot is 160 meters, what is the area of the plot in square meters?
অনুবােঃ আযেতেত্রাকার প্লতটর দে ণয োর প্রতস্থর প তয 20 নমটার পবনশ্। প্লতটর পনরসীমা 160 নমটার হতল, বগণ নমটাতর প্লতটর পেত্রফল কে?
Solution:
Let, breadth be x meters and length be (x + 20) meters
So, perimeters =2(x + x +20) = (4x + 40) meters.
According to the question,
4x + 40 = 160
Or, 4x = 160 - 40
Or, x = 30
So, breadth = 30 meters and length = 30 + 20 = 50 meters.
Area of the pot = (50 × 30) = 1500 square meters
Answer: 1500 square meters

Bangladesh Bank
Post Name: Assistant Director Exam Date : 2009

1. A boy purchased some chocolates from a shop for Tk. 120. If the price of chocolate was reduced
by 20%, he could have purchased 2 more chocolates. How many chocolates did he buy?
অনুবােঃ একনট পছতল একনট পোকান পর্তক 120 টাকায নকছু কতলট নকতনতছ। যনে কতলতটর োম 20% কমাতনা পযে, োহতল নেনন
আরও 2নট কতলট নকনতে পারতেন। পস কযনট কতলট নকতনতছ?
Solution:
Let, The numbers of total chocolates be x
120
So, price of each chocolate =
𝑥
120
On the next shop price for each chocolate = ( – 3)
𝑥
According to the question,
120 120
– 3=
𝑥 𝑥+2
120−3𝑥 120
⇒ =
𝑥 𝑥+2
⇒ 120x=(x+2)(120-3x)
⇒ 120x= 120x-3x2+240-6x
⇒ 3x2+6x-240=0
⇒ 3(x2+2x-80)=0

Written Math Solution (BIBM) FB Group : BD job Info. (BCS & Bank)
Bank Job Solution (BIBM) 211

Created by Sumon K. Sarkar  FB Group : BD job Info. (BCS & Bank)  Created by Sumon K. Sarkar 
⇒ x2+10x-8x-80=0
Created by Sumon K. Sarkar  FB Group : BD job Info. (BCS & Bank)  Created by Sumon K. Sarkar 

⇒ x(x+10)-8(x+10)=0
 (x+10)(x-8)=0
So, x=8 or, x = -10 ( Numbers of chocolates could not be negative)
 He bought 8 chocolate.
Answer: 8 chocolate.

2. Recently Kamal's hourly wage has been increased by 10%. Before this increase, Kamal's total
weekly wage was Tk. 137. If his weekly working hours were to decrease by 10% from last week's
total working hours, what would be the change, if any, in Kamal's total weekly wage?
অনুবােঃ সম্প্রনে কামাতলর ণ্টাপ্রনে মেুনর ১০ শ্োংশ্ বৃনি করা হতযতছ। এই বৃনির আতগ কামাতলর পমাট সাপ্তানহক মেুনর নছল ১৩৭ টাকা।
যনে োর সাপ্তানহক কমণ ন্টা গে সপ্তাতহর পমাট কমণ ণ্টার পর্তক 10% কতম যায, োহতল কামাতলর পমাট সাপ্তানহক মেুনরতে নক পনরবেণন
হতব?
Solution:
Let, weekly working time= x hr
137
Weekly hourly wage = Tk.
𝑥
137 137
After 10% increase, New hourly wage = Tk. + Tk. × 10%
𝑥 𝑥
137 137 10
= Tk. + Tk. ×
𝑥 𝑥 100
137 137 1
= Tk. + Tk. ×
𝑥 𝑥 10
1370+137
= Tk.
10𝑥
1507
= Tk.
10𝑥
10𝑥 100𝑥−10𝑥 90𝑥 9𝑥
After 10% decrease, New working hour = x – (10% of x) = x - = = =
100 100 100 10
9𝑥 1507
New weekly total wage = × Tk. = Tk.135.63
10 10𝑥
 Weekly wage reduced = 137 – 135.63 = Tk. 1.37
Answer: Tk. 1.37

3. A loss of 15% is incurred by selling a watch for Tk. 612. How much is the sum of money by which
it is sold to make a profit of 10%?
অনুবােঃ একনট নড টাকায নবনক্র করতল 15% পলাকসান হয। 612. 10% মুনাফা করতে নবনক্র করা টাকার পযাগফল কে?
Solution:
Let, cost of a watch be Tk.100
At 15% loss selling price be = Tk.(100 – 15) = Tk. 85
100
Therefore, cost of watch = Tk. 612 × = Tk.720
85
Now, at 10% profit Selling price becomes = Tk.(100+10) = Tk.110
110
So, the selling price= Tk.720 × 110% = Tk. 720 × = Tk. 792
100
Answer: Tk. 792

Written Math Solution (BIBM) FB Group : BD job Info. (BCS & Bank)
212

 Created by Sumon K. Sarkar  FB Group : BD job Info. (BCS & Bank)  Created by Sumon K. Sarkar 
Bank Job Solution (BIBM)
 Created by Sumon K. Sarkar  FB Group : BD job Info. (BCS & Bank)  Created by Sumon K. Sarkar 

Basic Bank Ltd.


Post: Assistant Officer Exam Date : 2009

𝟏 𝟏
1. If a - = m, then show 𝒂𝟒 + = 𝒎𝟒 + 𝟒𝒎𝟐 + 𝟐 [BASIC AO 09, Board Book 8 & 9/10]
𝒂 𝒂𝟒
Solution:
1
Given that, 𝑎 − =m
𝑎
1
⇒ (a − )2 = 𝑚2
𝑎
1
⇒ 𝑎2 + − 2 = 𝑚2
𝑎2
1
⇒ 𝑎2 + = 𝑚2 +2
𝑎2
1
⇒ (𝑎2 + 2 )2 = (𝑚2 + 2)2
𝑎
1
⇒𝑎4 + 4 + 2 = 𝑚4 + 4𝑚2 +4
𝑎
1
⇒𝑎4 + 4 = 𝑚4 + 4𝑚2 + 4 −2
𝑎
1
⇒𝑎4 + 4 = 𝑚4 + 4𝑚2 + 2 (Showed)
𝑎

2. On a certain X-Y committee, 2/3rd of the members are men, and 3/8th of the men from country
Y. If 3/5th of the committee members are from country X, what fraction of the members are
women from country Y?
Abyev`t GKwU x-y KwgwU‡Z, m`m¨‡`i 2/3 Ask cyiæl Ges cyiæl‡`i 3/8 Ask y †`‡ki| hw` KwgwUi m`m¨‡`i 3/5
Ask x †`‡ki nq Zvn‡j KZRb gwnjv m`m¨ y †`‡ki?
Solution:
Let, Number of total member x
2 2𝑥 2𝑥 3𝑥−2𝑥 𝑥
Number of Men = x × = & Number of women = 𝑥 − = =
3 3 3 3 3
2𝑥 3 𝑥
Number of Men from country Y = × =
3 8 4
2𝑥 𝑥 8𝑥−3𝑥 5𝑥
Number of Men from country X = – = =
3 4 12 12
3 3𝑥
Number of Members from country X = x × =
5 5
3𝑥 5𝑥 36𝑥−25𝑥 11𝑥
So, Number of Women from country X = − = =
5 12 60 60
𝑥 11𝑥 20𝑥−11𝑥 9𝑥 3𝑥
Number of Women from country Y = − = = =
3 60 60 60 20
3𝑥
20 3
Fraction of women from country Y = =
𝑥 20
𝟑
Answer:
𝟐𝟎

Alternative Way,
Let, Number of total member 120 [L.C.M of 3,8,5=120]
2
Number of Men = 120 × = 80 & Number of women = 120 – 80 =40
3
3
Number of Men from country Y = 80 × = 30
8

Written Math Solution (BIBM) FB Group : BD job Info. (BCS & Bank)
Bank Job Solution (BIBM) 213

Created by Sumon K. Sarkar  FB Group : BD job Info. (BCS & Bank)  Created by Sumon K. Sarkar 
Number of Men from country X = 80 – 30 = 50
Created by Sumon K. Sarkar  FB Group : BD job Info. (BCS & Bank)  Created by Sumon K. Sarkar 

3
Number of Members from country X = 120 × = 72
5
So, Number of Women from country X = 72 – 50 = 22
Number of Women from country Y = 40 – 22 = 18
18 3
Fraction of women from country Y = =
120 20
𝟑
Answer:
𝟐𝟎

3. Malek spends 75% of his income. His income is increased by 20% and be increases his
expenditure by 10%. Calculate the percentage of his increased amount of savings?
Abyev`t gv‡jK Zvi Av‡qi 75% LiP K‡i| Zvi Avq 20% e„w× cvq Ges LiP 10% e„w× cvq| Zvi mÂq †h cwigvb e„w×
cvq Zvi kZKiv cwigvb wbY©q Ki|
Solution:
Let, Malek’s income be Tk. 100
Malek’s expenditure = Tk. 100 × 75% = Tk. 75
Malek’s savings = Tk. (100 – 75) = Tk. 25
If income is increased by 20% then Malek’s income = Tk. (100 + 100 × 20%) = Tk.120
If expenditure is increases by 10%. then Malek’s expenditure = Tk. (75 + 75 × 10%) = Tk.82.5
So, After increases income & expenditure, Malek’s savings = Tk. (120 – 82.5) = Tk. 37.5
37.5−25
The percentage of his increased amount of savings = × 100 = 50%
25
Answer: 𝟓𝟎%

4. Three partners A, B and C start a business. Twice the investment of A is equal to thrice the
capital of B and the capital of B is 4 times the capital of C. Find the share of each out of a profit
of Tk. 297,000.
Abyev`t wZbRb Askx`vi A, B Ges C GKwU e¨emv ïiæ K‡i| A Gi wewb‡qv‡Mi wظY B Gi wewb‡qv‡Mi wZb¸‡bi mgvb
Ges B Gi wewb‡qvM C Gi wewb‡qv‡Mi Pvi¸‡Yi mgvb| 2,97,000 UvKv jvf n‡j Zv‡`i cÖ‡Z¨‡Ki jf¨vsk wbY©q
Ki| [BASIC Bank AO 09. Rupali Officer 13]
Solution:
Let, Capital of C be Tk x ,
Capital of B be Tk. 4x
According to the question, 2A = 3B
⇒ 2A = 3×4x
⇒ A = 6x
Ratio of A, B, C‘s Capital = 6x : 4x : x = 6 : 4 : 1
Sum of the ratio = 6 + 4 + 1 = 11
6
So, A’s share of profit =2,97,000 × = 𝑇𝑘. 162,000
11
4
B’s share of profit = 2,97,000 × = 𝑇𝑘. 108,000
11
1
C’s share of profit =2,97,000 × = 𝑇𝑘. 27,000
11
Answer: Tk. 162,000; Tk. 108,000; Tk. 27,000

Written Math Solution (BIBM) FB Group : BD job Info. (BCS & Bank)
214

 Created by Sumon K. Sarkar  FB Group : BD job Info. (BCS & Bank)  Created by Sumon K. Sarkar 
Bank Job Solution (BIBM)
 Created by Sumon K. Sarkar  FB Group : BD job Info. (BCS & Bank)  Created by Sumon K. Sarkar 

5. A tank can be filled by a tap in 20 minutes and by another tap in 60 minutes. Both the taps are
kept open for 10 minutes and then the first tap is shut off. After this, the tank will be completely
filled in what tiine?
Abyev`t GKwU †PŠev”Pv X bj Øviv cvwbc~Y© n‡Z 20 wgwbU Ges Y bj Øviv cvwb c~Y© n‡Z 60 wgwbU jv‡M| 2wU bj GKmv‡_
10 wgwbU Pvjy ivLvi ci X bjwU eÜ K‡i ‡`qv nq| Zvici, †PŠev”PvwU cvwbc~Y© Ki‡Z KZ mgq jvM‡e?
[PKB SEO 18, BASIC Bank AO 09]
Solution:
In 20 minutes, 1st pipe can fill 1 portion of the tank
1
In 1 minutes, 1st pipe can fill portion of the tank
20
st
In 60 minutes,1 pipe can empty 1portion of the tank
1
In 1 minutes , 1st pipe can fill portion of the tank
60
1 1
Both pipe can fill in 1 minutes = + part
20 60
3+1 4 1
= = = part
60 60 15
10 2
Both pipe can fill in 10 minutes = part = part
15 3
2 3−2 1
Remaining part = 1 − = =
3 3 3
1
portion of the tank will be filled in 1 minutes
60
1 60
So, portion of the tank will be filled in minutes = 20 minutes
3 3
Answer: 20 minutes

6. On a certain X-Y committee, 2/3 of the members are men, and 3/8 of the men are from country
Y, If 3/5 of the committee members are from country X. What fraction of the members are
Women from country Y?
অনুবােঃ একনট নননেণষ্ট X-Y কনমনটতে, সেসযতের 2/3 পুরুে, এবং পুরুেতের 3/8 েন Y পেশ্ পর্তক, যনে কনমনটর সেসযতের 3/5 সেসয
পেশ্ X পর্তক হয। সেসযতের কে অংশ্ Y পেতশ্র মনহলা?
Solution:
Let, Total member be x
2 2𝑥 2𝑥 3𝑥−2𝑥 𝑥
So men = x × = and Women = 𝑥 − = =
3 3 3 3 3
2𝑥 3 𝑥
Men from country Y = × =
3 8 4
3 3𝑥
Men from country X = x × =
5 5
3𝑥 2𝑥
∴ Member from county Y = 𝑥 − =
5 5
2𝑥 𝑥 8𝑥−5𝑥 3𝑥
∴ Women from county Y = − = =
5 4 20 20
3𝑥
20 3
The fraction of the members are Women from country Y = =
𝑥 20
𝟑
Answer:
𝟐𝟎

Written Math Solution (BIBM) FB Group : BD job Info. (BCS & Bank)
Bank Job Solution (BIBM) 215

Created by Sumon K. Sarkar  FB Group : BD job Info. (BCS & Bank)  Created by Sumon K. Sarkar 
7. In the square ABCD, the four un-shaded parts are quarter circles each having the same radius.
Created by Sumon K. Sarkar  FB Group : BD job Info. (BCS & Bank)  Created by Sumon K. Sarkar 

If the side of the square is 16. What is the area of the shaded part? Express the answer in term
of 𝝅 .
Solution:
Side of the square ABCD = 16
∴ ½ Side of the square ABCD = 16/2 = 8
Radius of each circle = 8
1
∴ Area of the shaded part = 162 - 4 × π.82 ×
4
= 256 - 64 π
Answer: 256 - 64 π

ICB
Post name: Assistant Programmer Exam Date : 2008

1. In a certain conference room, 2/3 of the male participants & 1/2 of the female participants speak
Bangla. If the ratio of male & female participants is 6:4, then what fraction of the participants
of conference room speak Bangla?
Abyev`t GKwU wbw`©ó Kbdv‡iÝ K‡¶, cyiæl AskMÖnYKvix‡`i 2/3 Ask Ges gwnjv AskMÖnYKvix‡`i 1/2 Ask evsjvq K_v
e‡j| hw` cyiæl I gwnjv AskMÖnYKvix‡`i AbycvZ 6t4 nq, Zvn‡j Kbdv‡iÝ K‡¶ AskMÖnYKvix‡`i KZ Ask
evsjvq K_v e‡j?
Solution:
Let, male participants be 3x
Female participants be 2x
So, Total participants = 3x + 2x = 5x
2
Male participants who speaks Bangla = 3x × = 2x
3
1
Female participants who speaks Bangla = 2x × = x
2
Total participants who speaks Bangla = 2x + x = 3
3𝑥 3
∴ fraction of the participants = =
5𝑥 5
𝟑
Answer :
𝟓
2. If 𝟑𝒂 − 𝟓𝒃 = 𝟒 𝒂𝒏𝒅 𝟑𝒂+𝟐 − 𝟓𝒃+𝟏 = 𝟓𝟔 then determine the values of a & b?
Solution:
Given that, 3𝑎 − 5𝑏 = 4
⟹ 3𝑎 = 4 + 5𝑏 -------- (1)
3𝑎+2 − 5𝑏+1 = 56
⟹ 3𝑎 . 32 − 5𝑏 . 51 = 56
⟹ 9(4 + 5𝑏 ) − 5. 5𝑏 = 56
⟹ 36 + 9. 5𝑏 − 5. 5𝑏 = 56
⟹ 4 . 5𝑏 = 56 − 36
⟹ 4 . 5𝑏 = 20

Written Math Solution (BIBM) FB Group : BD job Info. (BCS & Bank)
216

 Created by Sumon K. Sarkar  FB Group : BD job Info. (BCS & Bank)  Created by Sumon K. Sarkar 
Bank Job Solution (BIBM)
 Created by Sumon K. Sarkar  FB Group : BD job Info. (BCS & Bank)  Created by Sumon K. Sarkar 

⟹ 5𝑏 = 5
⟹b=1
Put the value of b in equation (1)
3𝑎 = 4 + 51
⟹ 3𝑎 = 9
⟹ 3𝑎 = 32
⟹ 𝑎=2
Answer: a = 2, b = 1

3. A water tank has two taps (Tap-1 and tap-2). Tap-l can fill a tank in 8 hours and Tap-2 can
empty the tank in 16 hours. How long will they to fill the tank if both taps are opened
simultaneously but tap-2 is closed after 8 hours?
Abyev`t GKwU cvwbi U¨v‡¼i `yBwU U¨vc (U¨vc-1 I U¨vc -2)| U¨vc-1 GKwU U¨v¼ AvU NÈvq c~Y© K‡i Ges U¨vc-2 16
NÈvq U¨v¼wU Lvwj K‡i| hw` Dfq U¨vc GK‡Î †Lvjv _v‡K wKš‘ AvU NÈv ci U¨vc-2 eÜ K‡i †`Iqv nq, Z‡e
U¨v¼wU c~Y© n‡Z KZ mgq jvM‡e?
Solution:
(Tap-1 and tap-2) pipe can fill in 1 hours
1 1
= − part
8 16
2−1 1
= = part
16 16
(Tap-1 and tap-2) pipe can fill in 8 hours
8 1
= part = part
16 2
Tap-1 can fill 1 part of the tank in 8 hours
1
Tap-1 can fill part part of the tank in 4 hours
2
So, Total Time = 8+4 = 12 hours
Answer: 12 hours

4. A certain principal amount including interest in 3 years is Tk. 390 and it will be Tk. 450 in 5
years. Determine the simple interest rate & principle amount.
Abyev`t †Kvb Avmj my` Avm‡j 3 eQ‡i 390 Ges 5 eQ‡i 450 UvKv nq| kZKiv my‡`i nvi I Avmj wbY©q Kiæb|
[ Class 8 Board Book]
Solution:
In 5 years, Principal + Interest = Tk. 450
In 3 years, Principal + Interest = Tk. 390
( - ) In 2 years, Interest = Tk. 60
60×5
In 5 years, Interest = Tk. = Tk. 150
2
Principal P = Tk. (450 – 150 ) = Tk. 300
Interest I = Tk. 150
Time n = 5
Interest rate r%
We Know, I = Pnr

Written Math Solution (BIBM) FB Group : BD job Info. (BCS & Bank)
Bank Job Solution (BIBM) 217

Created by Sumon K. Sarkar  FB Group : BD job Info. (BCS & Bank)  Created by Sumon K. Sarkar 
𝐼 150×100
r= = % =10%
𝑃𝑛 300×5
Created by Sumon K. Sarkar  FB Group : BD job Info. (BCS & Bank)  Created by Sumon K. Sarkar 

Interest rate 10%


Answer: 10%

Bangladesh Bank
Post Name: Assistant Director Exam Date : 2008

1. Mr. Rahim pays 10% tax on all income earned over Tk 60000 but he does not pay any tax on
interest on saving certificate. In 2006 he paid 7500 as tax & he earned 12000 as interest on savings
certificates. What is his total income in 2006.
অনুবােঃ েনাব রনহম 60000 টাকার উপতর অনেণে সমস্ত আতযর উপর 10% টযাক্স পেন েতব পসনভং সানটণনফতকতটর সুতের উপর নেনন
পকান টযাক্স পেন না। 2006 সাতল নেনন টযাক্স নহসাতব 7500 টাকা পেন এবং সঞ্চতযর সুে নহসাতব 12000 লাভ কতরন । 2006 সাতল
োর পমাট আয কে?
Solution:
Here, Mr. Rahim gives 10% taxes upon his income.
So, when tax is Tk.10 then income is Tk. 100
100
When tax is Tk. 1 then income is Tk.
10
100×7500
When tax is Tk. 7500 then income is Tk = Tk. 75,000
10
So, his total income = Tk. (75000 + 60000 + 12000) = Tk. 1,47,000
Answer: Tk. 1,47,000

2. A trader, while selling an item, was asking for such a price that would enable him to offer a 20%
discounts and still make a profit of 30% on cost. If the cost of the item was Tk. 50 what was his
asking price?
[BB AD 08, Standard Bank TAO 16, Premier Bank JO 19]
Abyev`t GKRb we‡µZv GKwU c‡Y¨i Ggb GKwU `vg e‡jb hv‡Z cY¨wU‡Z 20% wWmKvD›U w`‡jI we‡µZvi 30%
jvf nq| hw` cY¨wUi g~j¨ 50 UvKv nq Zvn‡j we‡µZv KZ `vg e‡jwQ‡jb?
Solution:
Given that, cost price = 50 Tk
30
At 30% profit, price will be = 50 + (50 × ) = 50 + 15 = Tk.65
100
Let, asking price of the item = 100 Tk.
At 20% discount price = (100-20) = 80 Tk.
So, when discounted price is 80 Tk then asking price is 100 Tk
100×65
When discounted price is 65 Tk asking price is = = tk81.25
80
Answer: tk 81.25

3. A man deposits Tk. 1000 in a bank at 8% interest rate compounded annually. At the end of
the 3rd year, what will be the total amount including interest?
অনুবােঃ একেন মানুে 1000 টাকা একনট বযাতঙ্ক 8% বানেণক ক্রবৃনি সুতের হাতর েমা পেন । ৩য বছর পশ্তে সুেসহ পমাট মুনাফার
পনরমার্ কে হতব?

Written Math Solution (BIBM) FB Group : BD job Info. (BCS & Bank)
218

 Created by Sumon K. Sarkar  FB Group : BD job Info. (BCS & Bank)  Created by Sumon K. Sarkar 
Bank Job Solution (BIBM)
 Created by Sumon K. Sarkar  FB Group : BD job Info. (BCS & Bank)  Created by Sumon K. Sarkar 

Here, Principal value, p = 1000 tk


Rate of interest, r = 8%
Number of years, n = 3
We know,
Compound amount = 𝑃(1 + 𝑟)𝑛
8 3
= 1000 (1 + )
100
100+8 3
= 1000 ( )
100
108 3
= 1000 ( )
100
108 108 108
= 1000 × ( )×( )×( )
100 100 100
= 1259.712
Answer: 1259.712

Bangladesh Commerce Bank


Post name: Junior officer Exam Date : 2008

1. A can do a piece of work in 40 days. He works at it for 8 days and then B finishes it in 16 days.
How long will they take to complete the work if they do it together?
Abyev`t A GKwU KvR 40 w`‡b Ki‡Z cv‡i| †m 8 w`‡b KvR K‡i Ges Gici B KvRwU 16 w`‡b †kl K‡i| Zviv GK‡Î
KvRwU KZw`‡b †kl Ki‡Z cvi‡e?
2. Solution:
1
Work done by A in 1 day = Part
40
8 1
Work done by A in 8 day = = Part
40 5
1 5−1 4
Remaining work=(1− ) = =
5 5 5
5
Full Work done by B in 1 day = 16 × days =20 days
4
1
Work done by B in 1 day = Part
20
1 1
A & B can do the work in 1 day = + part
40 20
1+2 3
= = part
40 40
40 1
A & B can do the work in days = 13 days
3 3
𝟏
Answer: 13 days
𝟑

2. A car gets 20 kms per gallon of gas when it travels at 50 kms per hour. The car gets 12% fewer
kms to the gallon at 60 kms per hour. How far can the car travel at 60 kms per hour on 11 gallons
of gas?
Abyev`t GKwU Mvwoi N›Uvq 50 wK.wg †e‡M 20 wK.wg †h‡Z 1 M¨vjb M¨vm `iKvi nq| MvwowU N›Uvq 60 wK.wg †e‡M 1
M¨vjb M¨v‡m Av‡Mi †P‡q 12% Kg `~iZ¡ AwZµg K‡i| N›Uvq 60 wK.wg †e‡M 11 M¨vjb M¨v‡m MvwowU KZ`~i
†h‡Z cvi‡e?

Written Math Solution (BIBM) FB Group : BD job Info. (BCS & Bank)
Bank Job Solution (BIBM) 219

Created by Sumon K. Sarkar  FB Group : BD job Info. (BCS & Bank)  Created by Sumon K. Sarkar 
Solution:
Created by Sumon K. Sarkar  FB Group : BD job Info. (BCS & Bank)  Created by Sumon K. Sarkar 

Given that, car gets 20 kms per gallon of gas when it travels at 50 kms per hour
Car can go = 20 × (100 – 12)%
= 20 × 88%
= 17.6 kms per gallon
So, it can get using 11 gallon = 17.6 × 11
= 196.6 kms
Answer: 196.6 kms

3. A man sells articles at a profit of 20%. If had bought it at 20% less and sold it for Tk. 5 less, he
would have gained 25%. Find the cost price of the article?
Abyev`t GKRb e¨w³ 20% jv‡f GKwU AvwU©‡Kj weµq K‡ib| hw` wZwb 20% K‡g wK‡b Ges 5 UvKv K‡g weµq Ki‡Z
cvi‡Zb Zvn‡j 25% jvf n‡Zv| AvwU©‡KjwUi µqg~j¨ wbY©q Ki|
Solution:
Let, cost Price = Tk. x
So, selling price = Tk. (x + x×20%) = Tk. 1.2x
when cost price 20% reduced, new cost price = Tk. (x - x×20%) = Tk. 0.80x
then selling price =Tk. (0.80x+25% of 0.80x) = Tk. x
According to the question,
1.2x – x = 5
⟹ 0.2x =5
⟹ x= 25
Cost price = Tk. 25.
Answer: Tk. 25

𝟐𝒎 (𝟐𝒎−𝟏 )𝒎 𝟐𝟐𝒎
4. Simplify the following algebraic expression:
𝟐𝒎+𝟏 𝟐𝒎−𝟏 ((𝟐𝒎)𝒎
Solution:
2𝑚(2𝑚−1 )𝑚 22𝑚
2𝑚+1 2𝑚−1 (2𝑚 )𝑚
2
2𝑚 2𝑚 −𝑚 22𝑚
= 2
2𝑚+1 2𝑚−1 2𝑚
2
2𝑚+𝑚 −𝑚+2𝑚
= 2
2𝑚+1+𝑚−1+𝑚
2
2𝑚 +2𝑚
= 2
2𝑚 +2𝑚
=1
Answer: 1

এ রকম গুরুত্বপূর্ণ ফাইল পপতে আমাতের গ্রুতপ প াখ রাখুন


এবং অবশ্যই ননতের বন্ধুতের ইনভাইট করুন

Written Math Solution (BIBM) FB Group : BD job Info. (BCS & Bank)
220

 Created by Sumon K. Sarkar  FB Group : BD job Info. (BCS & Bank)  Created by Sumon K. Sarkar 
Bank Job Solution (BIBM)
 Created by Sumon K. Sarkar  FB Group : BD job Info. (BCS & Bank)  Created by Sumon K. Sarkar 

5. In the accompanying diagram ABCD is a rectangle. The area of isosceles right triangle ABE = 7,
and EC = 2BE. What would be the area of ABCD?

Solution:
Given that, EC = 2BE
BC = BE + EC = BE + 2BE = 3BE
𝐵𝐶
BE =
3
According to the question,
The area of isosceles right triangle ABE = 7
1
× 𝐴𝐵 × 𝐵𝐸 = 7
2
1 𝐵𝐶
× 𝐴𝐵 × =7
2 3
𝐴𝐵 × BC = 42
the area of ABCD = 42
Answer: 42

6. Arif and Mukul start running at the same time and from the same point around a circle of 216
meters circumference. If Arif can complete one round in 40 seconds and Mukul in 50 seconds,
how many seconds will it be before they both reach the starting point simultaneously and how
many rounds they will complete by this time?
Abyev`t Avwid Ges gyKzj GKmv‡_ GKB ¯’vb †_‡K, GKB mg‡q 216 wgUvi cwiwa wewkó GKwU e„ËvKvi †¶‡Îi Pvwiw`‡K †`Šo
ïiæ K‡ib| Avwid hw` GK ivDÛ 40‡m‡K‡Û †kl K‡ib Zvn‡j gyKzj Zv K‡ib 50 †m‡K‡Û| GKmv‡_ Avevi †`Šo ïiæi ¯’v‡b
G‡m wgwjZ n‡Z Zv‡`i KZ †m‡KÛ mgq jvM‡e? ZZ¶‡Y Zviv cÖ‡Z¨‡K KZ ivDÛ mgvß Ki‡e?
Solution:
LCM of 40 and 50 is 200.
So, after 200s they will be in same point
200
In 200s, Arif will complete = = 5 round
40
200
In 200s, Arif will complete = = 4 round
50
Answer: 5 round, 4 round

এ রকম গুরুত্বপূর্ণ ফাইল পপতে আমাতের গ্রুতপ প াখ রাখুন


এবং অবশ্যই ননতের বন্ধুতের ইনভাইট করুন

Written Math Solution (BIBM) FB Group : BD job Info. (BCS & Bank)
Bank Job Solution (BIBM) 221

Created by Sumon K. Sarkar  FB Group : BD job Info. (BCS & Bank)  Created by Sumon K. Sarkar 
One Bank Ltd.
Created by Sumon K. Sarkar  FB Group : BD job Info. (BCS & Bank)  Created by Sumon K. Sarkar 

Post: Probationary Officer Exam Date : 2008


1. Three workers can do a job in 20 days. Two of the workers work twice as fast as the third. How
long would it take one of the faster workers to do the job himself?
Abyev`t 3 Rb kªwgK GKwU KvR 20 w`‡b Ki‡Z cv‡i| kªwgK‡`i 2 RbB 3q Rb A‡c¶v wØMyY MwZ‡Z KvR Ki‡Z cv‡i| `ªæZ
MwZ m¤úbœ‡`i 1 R‡bi H KvRwU GKv Ki‡Z KZ mgq jvM‡e?
Solution:
Let, Faster’s two workers can do in x days
Slower’s workers can do in 2x days
According to the question,
1 1 1 1
+ + =
𝑥 𝑥 2𝑥 20
2+2+1 1
⟹ =
2𝑥 20
5 1
⟹ =
2𝑥 20
⟹ 2𝑥 = 100
⟹ 𝑥 = 50
one of the faster workers can do in 50 days
Answer: 50 days

এ রকম গুরুত্বপূর্ণ ফাইল পপতে আমাতের গ্রুতপ প াখ রাখুন


এবং অবশ্যই ননতের বন্ধুতের ইনভাইট করুন

2. A labor is paid Tk. 8 per hour for an 8 hour day and 1.5 times that rate for each hour in excess
of 8 hours in a single day. If the labor received Tk. 80 for a single day's work, how long did he
work on that day?
Abyev`t GKRb kªwgK w`‡b 8 NÈv KvR Kivi Rb¨ N›Uv cÖwZ 8 UvKv K‡i cvb Ges 8 N›Uvi AwZwi³ KvR Kivi Rb¨ N›Uv cÖwZ
1.5 ¸Y †ewk UvKv cvb| hw` †m GK w`‡b 80 UvKv cvq Zvn‡j H w`‡b †m KZ¶Y KvR K‡i?
Solution:
In 8 hours A labor received = Tk.(8 × 8) =Tk. 64
Per time overtime payment rate = Tk.8 × 1.5 = Tk. 12
Remaining Amount = Tk.(80 × 64) =Tk. 16
16 4
For Tk. 16, A labor worked = = hours
12 3
4×60
= minutes
3
= 80 minutes
= 1 hour 20 minutes
Total Working time = 8 hour + 1 hour 20 minutes
= 9 hour 20 minutes
Answer: 9 hour 20 minutes

Written Math Solution (BIBM) FB Group : BD job Info. (BCS & Bank)
222

 Created by Sumon K. Sarkar  FB Group : BD job Info. (BCS & Bank)  Created by Sumon K. Sarkar 
Bank Job Solution (BIBM)
 Created by Sumon K. Sarkar  FB Group : BD job Info. (BCS & Bank)  Created by Sumon K. Sarkar 

3. In the figure below, ABC is a right angle and AB = BD = AD = 20 feet. Find out the length of BC.

Solution:
Given that , AB = BD = AD = 20 feet
So, ∠𝐴𝐵𝐷 = ∠𝐵𝐷𝐴 = ∠𝐷𝐴𝐵 = 600
∠𝐵𝐷𝐶 = 1800 − 600 = 1200
∠𝐷𝐵𝐶 = 900 − 600 = 300
∠𝐷𝐶𝐵 = 1800 − (1200 + 300 ) = 300
∠𝐷𝐵𝐶 = ∠𝐷𝐶𝐵
So, BD = CD = 20 feet
So, AC = AD + CD = ( 20 + 20 ) = 40 feet
∆𝐴𝐵𝐶 ′𝑠 𝐴𝐵2 + 𝐵𝐶 2 = 𝐴𝐶 2
𝑜𝑟, 202 + 𝐵𝐶 2 = 402
𝑜𝑟, 𝐵𝐶 2 = 402 − 202
𝑜𝑟, 𝐵𝐶 2 = 1600 − 400
𝑜𝑟, 𝐵𝐶 2 = 1200
𝑜𝑟, 𝐵𝐶 = √1200
𝑜𝑟, 𝐵𝐶 = 20√3
Answer : 𝟐𝟎√𝟑 feet

4. Mr. X sold two properties Pl and P2 for Tk. 100,000 each. He sold property Pl for 20% loss than
what he paid for it. What is the percentage of profit of property P2 so that he is not in gain or
loss on the sale of the two properties?
Abyev`t Rbve X Zvi m¤úwË P1 Ges P2 Gi cÖwZwU 1,00,000 UvKvq weµq K‡ib| wZwb P1 †K †h `v‡g µq
K‡iwQ‡jb Zvi †P‡q 20% Kg `v‡g weµq K‡ib| P2 †K kZKiv KZ jv‡f weµq Ki‡j †gvU c‡Y¨i Dci jvf
ev ¶wZ †KvbwUB n‡e bv?
Solution:
Let, the cost price of property P1 be A
⇒ Selling Price of P1=Tk.1,00,000
He get 20% loss on selling price P1
20𝐴
⇒A− =1,00,000
100
𝐴
⇒A− =1,00,000
5
5𝐴−𝐴
⇒ =1,00,000
5
4𝐴
⇒ =1,00,000
5
⇒A=1,25,000
∴ The amount he losses on selling price P1=Tk. 25,000

Written Math Solution (BIBM) FB Group : BD job Info. (BCS & Bank)
Bank Job Solution (BIBM) 223

Created by Sumon K. Sarkar  FB Group : BD job Info. (BCS & Bank)  Created by Sumon K. Sarkar 
As he got no loss/ gain on sale of P1 and P2, the gain on selling P2=Tk.25,000
Created by Sumon K. Sarkar  FB Group : BD job Info. (BCS & Bank)  Created by Sumon K. Sarkar 

But the selling price of P2= Tk.1,00,000


⇒Cost price of P2=Tk.(100,000 – 25,000) = Tk. 75,000
𝑮𝒂𝒊𝒏 25,000
Hence profit % of P2= ×100= ×100 = 33.33%
Cost price 75,000
Answer: 33.33%

Alternative Way,
Given that, the sell price of property P1 = Tk. 100,000
100000×100
At 20% loss, Cost price of property P1 = Tk. = Tk. 125000
80
Loss = (125000 × 20%) = Tk. 25000
Let, the percentage of profit of property P2 = x%
Cost price of property P2 = Tk. (100,000-25000)
= Tk. 75000
So, Profit=75,000 × x%
According to the question,
Profit of P2 = Loss of P1
⟹75000 × x% = 25000
𝑥 25000
⟹ =
100 75000
25000×100
⟹ 𝑥=
75000
100
⟹ 𝑥=
3
⟹ 𝑥 = 33.33
the percentage of profit of property P2 = 33.33%
Answer: 33.33%
𝟗(𝟒𝒙 )𝟐
5. Simplify the following algebraic expression.
𝟏𝟔𝒙+𝟐 −𝟐𝒙+𝟏 (𝟖𝒙 )
Solution:
9(4𝑥 )2
16𝑥+2 −2𝑥+1 (8𝑥 )
32 (22𝑥 )2
=
(24 )𝑥+2 −2𝑥+1 (23𝑥 )
32 (24𝑥 )
=
24𝑥+8 −2𝑥+1+3𝑥
32 (24𝑥 )
=
24𝑥+8 −24𝑥+1
32 (24𝑥 )
=
24𝑥 . 28 −24𝑥 .21
32 (24𝑥 )
=
24𝑥 ( 28 −21 )
32 (24𝑥 )
=
24𝑥 ( 256−2)
9 (24𝑥 ) 9
= =
24𝑥 . 254 254
𝟗
Answer :
𝟐𝟓𝟒

Written Math Solution (BIBM) FB Group : BD job Info. (BCS & Bank)
224

 Created by Sumon K. Sarkar  FB Group : BD job Info. (BCS & Bank)  Created by Sumon K. Sarkar 
Bank Job Solution (BIBM)
 Created by Sumon K. Sarkar  FB Group : BD job Info. (BCS & Bank)  Created by Sumon K. Sarkar 

𝟐 𝟑
6. Solve the following equation: + = 𝟏 [One Bank PO 2007, SIBL TO 10]
𝒙−𝟐 𝒙+𝟑
Solution:
𝟐 𝟑
+ =𝟏
𝒙−𝟐 𝒙+𝟑
2 3
or = 1−
x−2 x+3
2 x+3−3
or, =
x−2 x+3
2 x
or, =
x−2 x+3
or, x 2 − 2x = 2x + 6
or, x 2 − 2x − 2x − 6 = 0
or, x 2 − 4x − 6 = 0
or, x 2 − 4x − 6 = 0
or, x 2 − 2.2x + 4 − 10 = 0
or, x 2 − 2.2x + 22 = 10
or, (x − 2)2 = 10
or, x − 2 = ±√10
or, x = 2 ± √10
Answer: 𝐱 = 𝟐 ± √𝟏𝟎

7. 200 children came to the park last Sunday. All of the older children rode bicycles into the park
and all of the younger children came on tricycles. 480 wheels rode into the park that day, all of
them functioning on the children's bicycles or tricycle. How many young children came to the
park last Sunday?
Abyev`t MZ iweevi GKwU cv‡K© 200wU wkï ågY Ki‡Z Av‡m| hviv eq‡m eo Zviv mevB evB mvB‡K‡j Ges hviv †QvU Zviv UªvB
mvB‡Kj wb‡q Av‡m| H w`b wkï‡`i evB mvB‡Kj Ges UªvB mvB‡Kj wgwj‡q cv‡K© †gvU 480 wU PvKv wQj|MZ iweevi KZRb
Kgeqmx wkï G‡mwQj?
8. Solution:
Let, the number of younger children to the park last Sunday who rode tricycle be x
So, the number of older children to the park last Sunday who rode bicycle = 200-x
According to the question,
3x+2(200-x)=480
⇒ 3x+400-2x = 480
⇒ 3x-2x =480-400
⇒x=80
So, the number of younger children who rode tricycle = 80
Answer: 80

এ রকম গুরুত্বপূর্ণ ফাইল পপতে আমাতের গ্রুতপ প াখ রাখুন


এবং অবশ্যই ননতের বন্ধুতের ইনভাইট করুন

Written Math Solution (BIBM) FB Group : BD job Info. (BCS & Bank)
Bank Job Solution (BIBM) 225

Created by Sumon K. Sarkar  FB Group : BD job Info. (BCS & Bank)  Created by Sumon K. Sarkar 
8. Mr. Karim deposited certain amount of money for a fixed period of time. On maturity he
Created by Sumon K. Sarkar  FB Group : BD job Info. (BCS & Bank)  Created by Sumon K. Sarkar 

received a total of Tk. 45000 when the ratio of interest and investinent became 3:6. If the interest
rate was 5% (Simple) per annum, calculate the time period for which the money was invested.
[BASIC Bank PO 1999, ONE Bank PO 07]
Abyev`t Kwig mv‡ne GKwU wbw`©ó mg‡qi Rb¨ wKQz UvKv e¨vs‡K Rgv iv‡Lb| H mgq ci wZwb e¨vsK †_‡K †gvU 45,000 UvKv
jvf K‡ib G‡Z gybvdv Ges Avm‡ji AbycvZ nq 3 t 6| hw` gybvdvi nvi 5% nq Zvn‡j mgq wbY©q Ki|
Solution:
Given that, interest : investiment = 3:6
Sum of the ratio = 3 + 6 = 9
3
So, interest, I = Tk. 45,000 × = 15,000
9
6
Investiment, P = Tk. 45,000 × = 30,000
9
Rate of the interest, r = 5%
Let, the time period n
We know, I = Pnr
15,000 = 30,000× n × 5%
5
⇒15,000 = 30,000× n ×
100
⇒1500n = 15,000
⇒ n = 10
The time period 10 years
Answer: 10 years

9. A is two years older than B, who is twice as old as C. If the average age of the three is 9 years,
how old is B? [ One Bank (po) 2008 ]
Abyev`t Kwig mv‡ne GKwU wbw`©ó mg‡qi Rb¨ wKQz UvKv e¨vs‡K Rgv iv‡Lb| H mgq ci wZwb e¨vsK †_‡K †gvU 45,000
UvKv jvf K‡ib G‡Z gybvdv Ges Avm‡ji AbycvZ nq 3 t 6| hw` gybvdvi nvi 5% nq Zvn‡j mgq wbY©q Ki|
Solution:
Let, the age of C be x years
Age of B = 2x years
Age of A = 2x + 2
According to the question,
𝑥+2𝑥+2𝑥+2
=9
3
5𝑥+2
⇒ =9
3
⇒ 5𝑥 + 2 = 27
⇒ 5𝑥 = 27 − 2
⇒ 5𝑥 = 25
⇒x=5
So, Age of B = 2 × 5 years = 10 years
Answer: 10 years

Written Math Solution (BIBM) FB Group : BD job Info. (BCS & Bank)
226

 Created by Sumon K. Sarkar  FB Group : BD job Info. (BCS & Bank)  Created by Sumon K. Sarkar 
Bank Job Solution (BIBM)
 Created by Sumon K. Sarkar  FB Group : BD job Info. (BCS & Bank)  Created by Sumon K. Sarkar 

10. The average weight of three men A, B and C is 84 kg. Another man D joins the group and the
average now becomes 80 kg. If a fifth man E, whose weight is 3 kg more than that of D, replaces
A, then the average weight of B, C, D and E becomes 79 kg. What is the weight of A?
[AB Bank PO 02, One Bank PO 07, RAKUB SO 14, Al-Arafah MTO 16, Janata Bank EO(Cancelled) 17]
Abyev`t wZb e¨w³ A, B I C Gi Mo IRb 84Kg| Zv‡`i mv‡_ AviI GKRb D †hvM †`Iqvq bZzb Mo nq 80Kg|hw` A
Gi ¯’‡j D Gi †P‡q 3Kg †ewk IR‡bi cÂg e¨w³ E Zv‡`i mv‡_ †hvM †`q, Zvn‡j B, C, D Ges E Gi Mo IRb nq
79Kg| A Gi IRb KZ?
Solution:
Let, Total weight of A,B and C = 84×3 = 252 kg.
Total weight of A,B,C and D = 80 × 4 = 320 kg
∴Weight of D = 320 – 252 = 68 kg
∴ Weight of E= 68 + 3 = 71 kg
Total weight of B, C, D and E = 79×4 = 316 kg
∴ Weight of B and C = 316 – 68 – 71 = 177 kg
∴Weight of A = 252-177 = 75 kg.
Answer: 75 kg.

Sonali, Janata, Agrani & Rupali Bank (BRC)


Post name: Senior Officer Exam Date : 2008

1. A manager has a budget of Tk. 60,000 for giving salary increments to 4 full-time and 2 part time
employees. Each of the full-time employees receives the same increment, which is twice the
increment that each of the part-time employees received. What is the amount of the increment
that each of the full time employees received?
অনুবােঃ একেন মযাতনোতরর 4 েন ফুলটাইম এবং 2 েন পাটণটাইম কমণ ারীতের পবেন পেোর েনয বাতেট 60,000 টাকা। ফুল-টাইম
কমণ ারীতের প্রতেযতক একই ইননক্রতমন্ট পায, যা প্রনেনট খণ্ডকালীন কমণ ারীর প্রাপ্ত ইননক্রতমতন্টর নিগুর্। পূর্ণকালীন কমণ ারীতের প্রতেযতক
পয ইননক্রতমন্ট প্রাপ্ত হয োর পনরমার্ কে?
Solution:
Let each part time employee receive a raise of Tk. y
Then each full time employee receives a raise of Tk. 2y
There are 4 full- time employees and 2 part- time employees
The total budget is Tk. 60000
According to questions.
4(2y) + 2y = 60000
⇒ 10y = 60000
⇒ y = 6000
Raise for each full-time employee , 2y = 2× 6000 =12,000
Answer: Tk. 12,000

Written Math Solution (BIBM) FB Group : BD job Info. (BCS & Bank)
Bank Job Solution (BIBM) 227

Created by Sumon K. Sarkar  FB Group : BD job Info. (BCS & Bank)  Created by Sumon K. Sarkar 
2. A shopkeeper, who sells, on an average, 18 pencils and 12 markers per day. The profit from a
Created by Sumon K. Sarkar  FB Group : BD job Info. (BCS & Bank)  Created by Sumon K. Sarkar 

pencil is 1/3 times the profit made by selling a marker. If he makes a profit of Tk. 900 in a month (a month
consists of 30 days) by selling pencils, how much profit does he make per month by selling markers?
অনুবােঃ একেন নবতক্রো নেতন গতড ১৮নট পপনিল ও ১৫নট মারকার নবক্রয কতর। পস মারকাতর পয লাভ কতর পপনিতল োর গুর্ লাভ
কতর। পস পপনিল নবনক্র কতর প্রনেমাতস ৯০০ টাকা লাভ কতর (? ৩০ নেতন ১ মাস যরতে হতব। )। প্রনেমাতস মারকার নবনক্রয কতর কে টাকা
লাভ কতর?
Solution:
Let, Profit from 1 marker = x tk
So, in a month he gets profit = Tk 12× 30 × x = Tk 360x
𝑥
Again, profit from 1 pencil = Tk
3
𝑥
Then monthly profit = Tk 18×30× = Tk 180x
3
According to questions.
180x = 900
900
⇒ x=
180
⇒ x = 5 tk
So, profit from markers = Tk 360 ×5 = Tk 1800
Answer: Tk 1800

Alternative Way:
The total no. of sold pencil in a month=30×18=540
The total no. of sold markers in a month=30×12= 360
Total profit Tk 900 by selling pencil in a month.
540 pencil make profit Tk 900
900 5
1 ” ” ” = Tk = Tk
540 3
5
As a pen is 3 times profitable than pencil, the profit from a marker = Tk ×3= Tk 5
3
So, the total profit by selling markers per month = Tk 360×5 = Tk 1800
Answer: Tk 1800

3. The ratio of gold and silver in an ornament weighing 42 gm is 4:3. How much gold will need to
be added for the ratio of gold and silver to be 5: 3?
অনুবােঃ 42 গ্রাম ওেতনর একনট অলঙ্কাতর পসানা ও পরৌতপযর অনুপাে 4:3। স্বর্ণ ও রূপার অনুপাে 5:3 হওযার েনয কে পসানা পযাগ
করতে হতব?
Solution:
Given that, The ratio of gold & silver = 4:3
Sum of the ratio = 4+3 = 7
4
The amount of gold = 42 gm × = 24 gm
7
3
The amount of silver = 42 gm × = 18 gm
7
Let, X gm gold will need to be added.
According to the question,

Written Math Solution (BIBM) FB Group : BD job Info. (BCS & Bank)
228

 Created by Sumon K. Sarkar  FB Group : BD job Info. (BCS & Bank)  Created by Sumon K. Sarkar 
Bank Job Solution (BIBM)
 Created by Sumon K. Sarkar  FB Group : BD job Info. (BCS & Bank)  Created by Sumon K. Sarkar 

(24 +X ): 18 = 5:3
(24+𝑋) 5
⇒ =
18 3
(24+𝑋) 5
⇒ =
6 1
⇒ 24 + X = 30
⇒ X = 30 - 24
⇒ X=6
Answer: 6 gm gold will need to be added

Sonali, Janata, Agrani & Rupali Bank (BRC)


Post name: Officer Exam Date : 2008
1. A goods has been sold at a loss of 12%. If it could be sold by Tk. 1000 more, there would have
been profit of 8%. What is the cost price of that goods?
অনুবােঃ একনট পর্য 12% েনেতে নবনক্র করা হতযতছ। যনে আরও ১০০০ টাকা পবনশ্তে নবনক্র করা পযে েতব ৮% লাভ হে। পসই পতর্যর
ক্রেমূলয কে?
Solution:
Let, the cost price of goods Tk. 100x
At 12% loss Selling price be = 100x – (100x×12%) = 100x – 12x = 88x
At 8% gain Selling price be = 100x + (100x×8%) = 100x + 8x = 108x
According to the question,
108x - 88x = 1000
⇒ 20x = 1000
1000
⇒x=
20
⇒ x = 50
⇒ 100x = 100 × 50 = 5000
∴ The cost price of article is Tk. 5000
Answer: Tk. 5000

2. Two trains of equal length of 500 meters starts moving on two parallel railway tracks at a speed
of 104 kilometers per hour and 80 kilometers per hour respectively. How much time the faster
train will take to cross the slower one.
অনুবােঃ 500 নমটার সমান দেত ণযর দুনট পট্রন দুনট সমান্তরাল পরলপতর্ যর্াক্রতম 104 নকতলানমটার প্রনে ন্টা এবং 80 নকতলানমটার প্রনে
ন্টা গনেতে লতে শুরু কতর। দ্রুেগামী পট্রননট যীরগনের পট্রননট অনেক্রম করতে কে সময লাগতব?
Solution:
5 20
Difference of speeds = 104 - 80 = 24 km/hr = 24× meter/second = meter/second
18 3
Distance = 500 meters
𝐷𝑖𝑠𝑡𝑎𝑛𝑐𝑒 500
.: Required time = = = 75 second
𝑠𝑝𝑒𝑒𝑑 20/3
Answer: 75 second

Written Math Solution (BIBM) FB Group : BD job Info. (BCS & Bank)
Bank Job Solution (BIBM) 229

Created by Sumon K. Sarkar  FB Group : BD job Info. (BCS & Bank)  Created by Sumon K. Sarkar 
3. For making a cupboard it requires human labor three times the labor required to make a bench.
Created by Sumon K. Sarkar  FB Group : BD job Info. (BCS & Bank)  Created by Sumon K. Sarkar 

Six carpenters can make 36 benches and 5 cupboards in 12 days. How many days will 10
carpenters require to make 61 beanches and 8 cupboards?
অনুবােঃ একনট পবঞ্চ দেনরর েনয পয মানুতের শ্রতমর প্রতোেন হে, একনট আলমানর দেনরর েনয োর নেনগুর্ মানুতের শ্রতমর প্রতযােন হয।
ছযেন নমস্ত্রী 12 নেতন 36নট পবঞ্চ এবং 5নট আলমানর দেনর করতে পাতরন। 61নট পবঞ্চ এবং 8নট আলমানর দেনর করতে 10 েন নমস্ত্রীর
কে নেন লাগতব?
Solution:
1 cupboard = 3 beanches
5 cupboard = 3 × 5 = 15 beanches
8 cupboard = 3 × 8 = 24 beanches
36 benches and 5 cupboards = (36 +15) = 51 beanches
61 beanches and 8 cupboards = 61 + 24 = 85 beanches
6 carpenters can make 51 beanches in 12 days
12×6
∴1 ,, ,, 1 ,, days
51
12×6×85
∴ 10 ,, ,, 85 ,, = 12 days
51×10
Answer: 12 days

Sonali, Janata, Agrani & Rupali Bank (BRC)


Post name: Officer (Cash) Exam Date : 2008

1. A boy covers a distance of 6 km partly by walking and partly by cycling. If he cycles at 18 km


per hour and walks at 6 km per hour and takes 35 minutes in all, find the distance he covers by
walking.
Abyev`t GKRb evjK 6 wKwg `~i‡Z¡i wKQz Ask †nu‡U Ges wKQz Ask mvB‡Kj Pvwj‡q †M‡jv| hw` mvB‡K‡j N›Uvq 18 wKwg †e‡M
Ges N›Uvq 6 wKwg †e‡M †nu‡U hvq Ges †gvU 35 wgwbU mgq †bq, Zvn‡j †nu‡U AwZµvšÍ `~iZ¡ wbY©q Kiæb|
[Combined Cash 08, UCBL Officer 11]
Solution:
Let, the boy covers by walking in x km
the boy covers by cycling in 6 – x km
According to the question,
𝑥 6−𝑥 35
+ =
6 18 60
3𝑥+6−𝑥 7
⇒ =
18 12
2𝑥+6 7
⇒ =
18 12
2(𝑥+3) 7
⇒ =
18 12
𝑥+3 7
⇒ =
9 12
𝑥+3 7
⇒ =
3 4
⇒ 4𝑥 + 12 = 21
⇒ 4𝑥 = 21 − 12
⇒ 4𝑥 = 9

Written Math Solution (BIBM) FB Group : BD job Info. (BCS & Bank)
230

 Created by Sumon K. Sarkar  FB Group : BD job Info. (BCS & Bank)  Created by Sumon K. Sarkar 
Bank Job Solution (BIBM)
 Created by Sumon K. Sarkar  FB Group : BD job Info. (BCS & Bank)  Created by Sumon K. Sarkar 

9
⇒𝑥 = = 2.25
4
the boy covers by walking in 2.25 km
Answer: 2.25 km

2. In an MBA class the ratio of number of commerce graduates to the number of science graduates
is 2 to 5. If 2 more commerce graduates enter the class the ratio becomes 1 to 2. How many
commerce graduates are in the class?
অনুবােঃ একনট এমনবএ ক্লাতস নবজ্ঞান স্নােকতের সংখযার সাতর্ বানর্েয স্নােতকর সংখযার অনুপাে 2 পর্তক 5। যনে আরও 2 েন বানর্েয
স্নােক ক্লাতস প্রতবশ্ কতর েতব অনুপাে 1 পর্তক 2 হতব। কেেন বানর্েয স্নােক পশ্রনর্তে রতযতছ?
Solution:
Let, there are 2X commerce graduates in the class and 5X science graduates.
According to the question,
(2X +2): 5X = 1:2
⇒ (2X+2)/5X = ½
⇒ 4X + 4 = 5X
⇒ 5X - 4X = 4
⇒X=4
No. of commerce graduates is 2 × 4 = 8
Answer: 8

3. A can dig a pond in 30 days and B can dig the same pond in 20 days. In how many days A and B
can dig the pond if they work together?
[Combined Cash 08, Standard Bank TAO Cash 16]
Abyev`t A GKwU cyKzi 30 w`‡b Ges B GKB cyKzi 20 w`‡b Lbb Ki‡Z cv‡i| Zviv GK‡Î H cyKziwU KZ w`‡b Lbb Ki‡Z
cvi‡e?
Solution:
In 30 Days, A can dig 1 portion of the pond
1
In 1 Days, A can dig portion of the pond
30
In 20 Days, B can dig 1 portion of the pond
1
In 1 Days, B can dig portion of the pond
20
1 1
In 1 Days, (A+B) can dig + portion of the pond
30 20
3+2
= portion of the pond
60
5
= portion of the pond
60
1
= portion of the pond
12
1
portion of the pond can dig in 1 day.
12
1 portion of the pond can dig in 12 day.
Answer: 12 day

Written Math Solution (BIBM) FB Group : BD job Info. (BCS & Bank)
Bank Job Solution (BIBM) 231

Created by Sumon K. Sarkar  FB Group : BD job Info. (BCS & Bank)  Created by Sumon K. Sarkar 
Shahjalal Islami Bank Ltd.
Created by Sumon K. Sarkar  FB Group : BD job Info. (BCS & Bank)  Created by Sumon K. Sarkar 

Post name: Trainee Senior Officer Exam Date : 2007

1. The sum of square of two numbers is 80 and the square of their difference is 16. Determine the
product of the two numbers.
অনুবােঃ দুনট সংখযার বগণতেতত্রর পযাগফল 80 এবং োতের বতগণর পার্ণকয হল 16। দুনট সংখযার গুর্ফল ননর্ণয কর।
Solution:
Let , the number be x & y
According to the 1st condition,
𝑥 2 + 𝑦 2 = 80
According to the 1st condition,
(𝑥 − 𝑦)2 = 16
⟹ 𝑥 2 + 𝑦 2 − 2𝑥𝑦 = 16
⟹ 80 − 2𝑥𝑦 = 16
⟹ 80 − 16 = 2𝑥𝑦
⟹ 2𝑥𝑦 = 64
⟹ 𝑥𝑦 = 34
So, the product of the two numbers is 34
Answer: 34

2. A number consists of two digits. The sum of the digits is 15. If 27 is subtracted from the number,
its digits are interchanged. Find the number.
Abyev`t GKwU msL¨v `ywU A¼ wb‡q MwVZ| A¼Ø‡qi †hvMdj 15| hw` msL¨vwU †_‡K 27 we‡qvM †`qv nq Zvn‡j A¼Øq ¯’vb
wewbgq K‡i| msL¨vwU wbY©q Kiæb|
Solution:
Let, the ten's digit be x.
So, unit's digit (15 - x)
Number = 10x+(15-x)=9x+15
Number obtained by reversing the digits
=10 (15-x)+x = 150-9x.
According to the question,
(9x+15) – 27 = 150 - 9x
⟹ 9x+15 – 27 = 150 - 9x
⟹ 9x+9x = 150-15+27
⟹ 18x = 162
⟹x=9
The ten's digit = 9
and unit's digit = (15-9) = 6
So, Number = 9×9+15= 96.
Answer : 96

Written Math Solution (BIBM) FB Group : BD job Info. (BCS & Bank)
232

 Created by Sumon K. Sarkar  FB Group : BD job Info. (BCS & Bank)  Created by Sumon K. Sarkar 
Bank Job Solution (BIBM)
 Created by Sumon K. Sarkar  FB Group : BD job Info. (BCS & Bank)  Created by Sumon K. Sarkar 

3. Masum has twice as much money as Selim and Selim has 50% more money than what Badal has.
The average money with thein is Tk. 110, then determine the amount of Masum's money?
Abyev`t gvmy‡gi Kv‡Q †mwj‡gi Zzjbvq wظY UvKv Av‡Q Ges †mwj‡gi Kv‡Q ev`‡ji Zzjbvq 50% †ewk UvKv Av‡Q| Zv‡`i
cÖ‡Z¨‡Ki M‡o 110 UvKv Av‡Q| gvmy‡gi Kv‡Q KZ UvKv Av‡Q?
Solution:
Let, Badal has Tk. 2x
So, Selim has =Tk. ( 2x + 2x × 50%)
= Tk.( 2x + x) = Tk. 3x
Masum's has = Tk. 3x × 2 = Tk. 6x
According to the question,
2𝑥+3𝑥+6𝑥
= 110
3
⟹ 11𝑥 = 110 × 3
⟹ 𝑥 = 10 × 3
⟹ 𝑥 = 30
Masum's has = Tk. 6 × 30 = Tk. 180
Answer: Tk. 180

4. If 𝟐𝒂 + 𝟑𝒃 = 𝟏𝟕 𝒂𝒏𝒅 𝟐𝒂+𝟐 − 𝟑𝒃+𝟏 = 𝟓 then determine the values of a and b.


Solution:
Given that, 2𝑎 + 3𝑏 = 17
⟹ 2𝑎 = 17 − 3𝑏 -------- (1)
𝑎+2
Again, 2 − 3𝑏+1 = 5
⟹ 2𝑎 . 22 − 3𝑏 . 31 = 5
⟹ 4(17 − 3𝑏 ) − 3.3𝑏 = 5
⟹ 68 − 4.3𝑏 − 3.3𝑏 = 5
⟹ − 7.3𝑏 = 5 − 68
⟹ − 7.3𝑏 = −63
⟹ 3𝑏 = 9
⟹ 3𝑏 = 32
⟹b=2
Put the value of b in equation (1)
⟹ 2𝑎 = 17 − 32
⟹ 2𝑎 = 17 − 9
⟹ 2𝑎 = 8
⟹ 2𝑎 = 23
⟹ 𝑎=3
Answer: a = 3, b = 2

Written Math Solution (BIBM) FB Group : BD job Info. (BCS & Bank)
Bank Job Solution (BIBM) 233

Created by Sumon K. Sarkar  FB Group : BD job Info. (BCS & Bank)  Created by Sumon K. Sarkar 
5. A and B are two stations 195 km. apart. A train starts from A at 8 a.in, and travels towards B at
Created by Sumon K. Sarkar  FB Group : BD job Info. (BCS & Bank)  Created by Sumon K. Sarkar 

60 km/hr. Another train starts from B at 9 a.m. and travels towards A at 75 km/hr. At what time
do they meet?
Abyev`t 2wU †÷kb A I B Gi `~iZ¡ 195 wK.wg | GKwU †Uªb †ejv 8 Uvi mgq A †_‡K hvÎv ïiæ K‡i Ges B Gi w`‡K 60
wK.wg / N›Uv †e‡M hvq| Aci GKwU †Uªb B †_‡K †ejv 9 Uvi mgq hvÎv ïiæ K‡i A Gi w`‡K 75 wK.wg/ N›Uv †e‡M †h‡Z _v‡K|
KLb Zviv gy‡LvgyLx wgwjZ n‡e?
Solution:
Let, they meet x hours after 8 a.m.
So, first train travelled = x hours
and second train travelled = (x-1) hours
According to the question,
60x+75(x-1)= 195
⟹ 60x+75x-75 = 195
⟹ 135x=195 +75
⟹ 135x=270
⟹ x = 270/135 = 2 hours
So, they meet = (8 a.m. +2 hrs) = 10a.m.
Answer: 10 a.m

এ রকম গুরুত্বপূর্ণ ফাইল পপতে আমাতের গ্রুতপ প াখ রাখুন


এবং অবশ্যই ননতের বন্ধুতের ইনভাইট করুন

Bangladesh Bank
Post Name: Assistant Director Exam Date : 2001
1. A total of 50 employees work in a bank branch. Of these 22 have taken the accounting course, 15
have taken finance, 14 have taken marketing c, 9 of them taken exactly 2 of the courses, 1 of
them has taken all. How many of the 50 employees have taken none of the course?
Abyev`t GKwU e¨vsK Gi kvLvq 50 Rb PvKyix K‡i| Zv‡`i g‡a¨ 22 Rb accounting, 15 Rb finance Ges 14 Rb
marketing ‡Kvm© wb‡qwQj| 9 Rb gvÎ 2 wU K‡i ‡Kvm© wb‡qwQj Ges 1 Rb me KqwU ‡Kvm© wb‡qwQj| 50 Rb Kg©Pvixi g‡a¨
KZRb ‡Kvb ‡Kvm©B ‡bqwb|
Solution:
Here, one of the employees has taken all of the courses.
Nine of the employees have taken exactly 2 of the courses.
Number of employee have taken only Accounting = 22 – (9 + 1) = 12.
Number of employee have taken only Finance = 15 – (9 + 1) = 5
Numbers of employee have taken only Marketing = 14-1=13
Numbers of total employee have taken at least one course = 12 + 5 + 13 + 9 + 1 = 40
So, employees who have not taken any course = 50 - 40 = 10
Answer: 10

Written Math Solution (BIBM) FB Group : BD job Info. (BCS & Bank)
234

 Created by Sumon K. Sarkar  FB Group : BD job Info. (BCS & Bank)  Created by Sumon K. Sarkar 
Bank Job Solution (BIBM)
 Created by Sumon K. Sarkar  FB Group : BD job Info. (BCS & Bank)  Created by Sumon K. Sarkar 

2. Mr. X, a sales person earns 5% commission on all sales between Tk.20000 and 40000 and 8%
on all sales exceeding Tk.40000 in a month. He does not earn any commission if sales in a month
amount to less than Tk. 20000. His monthly salary is Tk.60000, he has to pay tax 20% tax on his
basic salary, but no tax on commission, in April 2001, the total net income (salary + commission)
of the sales person was Tk 65000. How much were the sales in April?
Abyev`t নমঃ X একেন নবতক্রো ২০,০০০ টাকা পর্তক ৪০,০০০ টাকা পযণন্ত নবক্রতযর েনয ৫% এবং ৪০,০০০ টাকার অনযক নবক্রতযর
েনয ৮% কনমশ্ন প্রনে মাতস পায । পয যনে ২০,০০০ টাকার ননত নবক্রয কতর োহতল পকান কনমশ্ন পায না । োর মানসক পবেন ৬০,০০০
টাকা । োর টযাক্স নেতে হয োর পবেতনর উপর ২০% নকন্তু কনমশ্তনর উপর পকান টযাক্স নেতে হয না । এনপ্রল ২০০১ সাতল োর মূল আয
(পবেন + কনমশ্ন) নেল ৬৫,০০০ টাকা । এনপ্রল মাতস পস কে নবক্রয কতর নেল?
Solution:
Tax on salary = 60000 × 20% = 12000
Net salary = 60000 – 12000 = 48000
So, commission found in April = 65000 – 48000 = 17000
Let the sales in April = x
According to the question,
40000 × 5% + (x - 40000) × 8% = 17000
Or, 2000 + 0.08x – 3200 = 17000
Or, 0.08x – 1200 = 17000
Or, 0.08x = 18200
Or, x = 227500
So, the sales in April = Tk. 227500
Answer: Tk. 227500

3. A trader sells on an average 18 pencils and 12 pens per day. The profit comes from pencil is 1/3rd
of the profit made from selling a pen. If he makes profits of tk 900 in a month by selling pencils,
how much profit does he make per month by selling pens? The trader sells 30days in a month.
অনুবােঃ একেন নবতক্রো নেতন গতড ১৮নট পপনিল ও ১৫নট কলম নবক্রয কতর। পস কলতম পয লাভ কতর পপনিতল োর গুর্ লাভ কতর। পস
পপনিল নবনক্র কতর প্রনেমাতস ৯০০ টাকা লাভ কতর। প্রনেমাতস কলম নবনক্রয কতর কে টাকা লাভ কতর? ৩০ নেতন ১ মাস যরতে হতব।
Solution:
Let, Profit from 1 pen = x tk
So, in a month he gets profit = Tk 12× 30 × x = Tk 360x
𝑥
Again, profit from 1 pencil = Tk
3
𝑥
Then monthly profit = Tk 18×30× = Tk 180x
3
According to questions.
180x = 900
900
or, x =
180
or, x = 5 tk
So, profit from pen = Tk 360 ×5 = Tk 1800
Answer: Tk 1800

Alternative Way:
The total no. of sold pencil in a month=30×18=540

Written Math Solution (BIBM) FB Group : BD job Info. (BCS & Bank)
Bank Job Solution (BIBM) 235

Created by Sumon K. Sarkar  FB Group : BD job Info. (BCS & Bank)  Created by Sumon K. Sarkar 
The total no. of sold pen in a month=30×12= 360
Created by Sumon K. Sarkar  FB Group : BD job Info. (BCS & Bank)  Created by Sumon K. Sarkar 

Total profit Tk 900 by selling pencil in a month.


540 pencil make profit Tk 900
900 5
1 ” ” ” = Tk = Tk
540 3
5
As a pen is 3 times profitable than pencil, the profit from a pen = Tk ×3= Tk 5
3
So, the total profit by selling pens per month = Tk 360×5 = Tk 1800
Answer: Tk 1800

4. Mr. A purchased a house for tk 1000000 tk in 1995, he spent 100000 tk for routine maintenance
& upkeep of the house. In 1999 he sold the house for 25% of more then what he paid for it. He
paid 5% of the proceeds as gain tax & he has to pay 50% of his net profit to the broker, what is
his net income?
অনুবােঃ নমঃ A 1995 সাতল 1000000 টাকা নেতয একনট বানড ক্রয করতলন। বানড রের্াতবেতর্র েনয নেনন 100000 টাকা খর
করতলন । 1999 সাতল নেনন োর খরত র 25% লাতভ বানডনট নবক্রে করতলন । পস 5% sales tax নেতলন এবং লাতভর 50% broker
(োলাল) পক নেতলন । োর কাতছ (লাতভর) কে রইল?
Solution:
Here,
Purchasing cost + routine maintenance & upkeep cost = Tk.(10,00,000 + 1,00,000) = Tk.11,00,000
25
Profit gained @ 25% of total cost = 11,00,000×25% = 11,00,000× = Tk. 2,75,000
100
5
5% gain tax = Tk. 2,75,000×5% = Tk. 2,75,000 × = Tk. 13,750
100
So, net profit after deducting gain tax = Tk. (2,75,000 –13,750)
= Tk. 2,61,250
50
∴ Net income after deducting brokers commission = Tk. 2,61,250×50% = Tk. 2,61,250×
100
= Tk. 1,30,625
Answer: Tk. 1,30,625

5. A simple interest rate of a bank was reduced to 5% from 7%. As a consequences Mr. B’s income
was reduced by tk 2100 in 5 yrs. How much is Mr. B’s initial deposit in the bank?
Abyev`t e¨vs‡Ki mijmy‡` 7% ‡_‡K 5% K‡g ‡Mj| d‡j wgt B Gi Avq e¨vsK ‡_‡K my‡`i Avq 5 eQ‡i 2100 UvKv K‡g ‡Mj|
ïiz‡Z wgt B KZ UvKv e¨vs‡K Rgv ‡i‡LwQj?
Solution:
The rate of interest rate reduced= 7% - 5% = 2%
In 5 years interest reduced = Tk. 2100
2100
In 1 year interest reduced = = Tk. 420
5
Tk.2 reduce when deposit Tk. 100
100
Tk. 1 ” ” ” Tk.
2
100×420
∴ Tk.420 ” ” ” Tk. = 21,000
2
Answer: Tk. 21,000

Written Math Solution (BIBM) FB Group : BD job Info. (BCS & Bank)
236

 Created by Sumon K. Sarkar  FB Group : BD job Info. (BCS & Bank)  Created by Sumon K. Sarkar 
Bank Job Solution (BIBM)
 Created by Sumon K. Sarkar  FB Group : BD job Info. (BCS & Bank)  Created by Sumon K. Sarkar 

6. One fifth of the products made by a company are defective. Four-fifth of the defectives was
rejected and one-twentieth of the products were rejected by mistake. What percent of the
products sold by the company is defective?
অনুবােঃ একনট পকাম্পানীর উৎপানেে পতর্যর এক পঞ্চমাংশ্ ত্রুনটপূর্ণ । ঐ ত্রুনটপূর্ণ পতর্যর ার-পঞ্চমাংশ্ বাে যায এবং ভাতলা পতনযর অংশ্
ভুলক্রতম বাে যায। যনে পয সকল পর্য বাে যাযনন ো সবটাই নবনক্র হয েতব ত্রুনটপূর্ণ পর্য কে শ্োংশ্ নবনক্র হয?
Solution:
Let, total product = 100.
100
Total defective products = = 20
5
So, non defective products = 100 – 20 = 80
4
Defective product rejected = 20 × = 16
5
Defective but not rejected product = 20 – 16 = 4
1
∴ Rejected by mistake = 80 × =4
20
Total products sold = 100 – (16 + 4) = 80
Defective in 80 products is 4
4×100
∴ Defective in 100 products is = 5%
80
Answer: 5%

Bangladesh Bank
Post Name: Officer (general) Exam Date : 2001

1. A trader sells on an average 18 pencils and 12 pens per day. The profit comes from pencil is 1/3
times the profit made by selling a pen. If he makes a profit of Tk 900 in a month by selling pencils,
how much profit does he make per month by selling pens? The trader sells 30days in a month.
অনুবােঃ একেন নবতক্রো নেতন গতড ১৮নট পপনিল ও ১৫নট কলম নবক্রয কতর। পস কলতম পয লাভ কতর পপনিতল োর গুর্ লাভ কতর। পস
পপনিল নবনক্র কতর প্রনেমাতস ৯০০ টাকা লাভ কতর। প্রনেমাতস কলম নবনক্রয কতর কে টাকা লাভ কতর? ৩০ নেতন ১ মাস যরতে হতব।
Solution:
Let, Profit from 1 pen = Tk. x
So, in a month he gets profit = Tk 12× 30 × x = Tk. 360x
𝑥
Again, profit from 1 pencil = Tk.
3
𝑥
Then monthly profit = Tk. 18×30× = Tk. 180x
3
According to questions.
180x = 900
900
or, x =
180
or, x = 5
So, profit from pen = Tk. 360 ×5 = Tk. 1800
Answer: Tk. 1800

Alternative Way:
The total no. of sold pencil in a month=30×18=540
The total no. of sold pen in a month=30×12= 360

Written Math Solution (BIBM) FB Group : BD job Info. (BCS & Bank)
Bank Job Solution (BIBM) 237

Created by Sumon K. Sarkar  FB Group : BD job Info. (BCS & Bank)  Created by Sumon K. Sarkar 
Total profit Tk 900 by selling pencil in a month.
Created by Sumon K. Sarkar  FB Group : BD job Info. (BCS & Bank)  Created by Sumon K. Sarkar 

540 pencil make profit Tk 900


900 5
 1 pencil make profit = Tk. = Tk.
540 3
5
As a pen is 3 times profitable than pencil, the profit from a pen = Tk. ×3= Tk. 5
3
So, the total profit by selling pens per month = Tk 360×5 = Tk. 1800
Answer: Tk. 1800

2. Mr. X pays 10% tax on all income over 60000Tk but he does not pay any tax on interest on postal
saving certificate. In 2000 he paid 7500 as tax & he earned 12000 as interest on postal savings
account. What is his net income in 2000?
অনুবােঃ নমঃ X 60,000 টাকার উপর আতযর 10% টযাক্স পেন নকন্তু নেনন Saving certificates এর উপর পকাতনা টযাক্স পেন না ।
2,000 সাতল নেনন 7,500 টাকা টযাক্স নেতলন এবং 12,000 টাকা Saving certificate হতে সুে বাবে পপতলন । 2,000 সাতল
োর নীট আে কে ?
Solution:
Mr. X gives 10% taxes upon his income.
so, If tax is Tk 10 then income is Tk 100
100
If tax is Tk 1 then income is Tk
10
7500×100
 If tax is Tk 7500 then income is = Tk 75000
10
His total income = Tk (75000+60000+12000) = Tk 147000
 Net income= Tk (147000–7500) = Tk 1,39,500
Answer: Tk 1,39,500

3. A candidate answered all 22 questions in a test & received 63.5 marks. If the total marks were
derived by adding 3.5 marks each correct answer and deducting 1 mark for each incorrect
answer, how many questions did the student answer incorrectly?
অনুবােঃ একেন পনরোর্ণী 22নট প্রশ্নই উত্তর করল এবং 63.5 নম্বর পপল । প্রনেনট শুি উত্ততরর েনয যনে 3.5 ও ভুল উত্ততরর েনয 1
নম্বর কাটা যায েতব পয কেগুতলা প্রতশ্নর উত্তর ভুল কতরনছল?
Solution:
If all the answers were right he might got = 22×3.5=77
So, his deducted number is = 77 – 63.5 = 13.5
Number deducted for each wrong answer = 1+3.5 = 4.5
13.5
Total no. of given wrong answer = =3
4.5
Answer: 3

Alternative Way:
Let, Total no. of given wrong answer be x,
So, correct answer = 22 - x
According to the question,
3.5×(22-x)-1×x=63.5
Or, 77-3.5x-x= 63.5

Written Math Solution (BIBM) FB Group : BD job Info. (BCS & Bank)
238

 Created by Sumon K. Sarkar  FB Group : BD job Info. (BCS & Bank)  Created by Sumon K. Sarkar 
Bank Job Solution (BIBM)
 Created by Sumon K. Sarkar  FB Group : BD job Info. (BCS & Bank)  Created by Sumon K. Sarkar 

Or, -4.5x = 63.5-77


 x =3
So, Total no. of given wrong answer 3
Answer: 3

4. In an organization 30% of all employees live over 10 miles away from the place of work & 60%
of worker who live who live over 10miles use company transport. If 40% of employees of the
company use company transport, what percent of the employees live 10miles or less from work
and use company transport?
অনুবােঃ পকান একনট প্রনেষ্ঠাতনর 30% কমণেীবী 10 মাইতলর পবনশ্ েূতর র্াতক । 10 মাইতলর পবনশ্ েূতর র্াতক এমন কমণেীবীর 60%
পকাম্পাননর পনরবহর্ বযবহার কতর । যনে পকাম্পানীর পমাট কমণেীবীর 40% পকাম্পানন পনরবহর্ বযবহার কতর োহতল কমণেীবীতের শ্েকরা
কে ভাগ কমণস্থল পর্তক েশ্মাইল বা এর কম েূরতত্ব র্াতক এবং পকাম্পানীর পনরবহর্ বযবহার কতর?
Solution:
Let, total employees = 100,
employees live over 10 miles = 30
So, employees live in 10 mile or less = 100 – 30 = 70
Total 40% or 40 employees use transport.
60×30
Now, employees live over 10miles using transport = 60% of 30 = = 18
100
So, Company employees who use transport live in 10miles or less ten miles = 40 –18 = 22.
22×100 3
So, percentage of using transport of employee who live in 10miles or less = = 31 %
70 7
𝟑
Answer: 31 %
𝟕

5. Mr. Rich sold two properties P1 & P2 for Tk 50000 each. He sold property P1 for 20% more
then what he paid for it & sold P2 less than 20% what he paid for it. What was his total gain or
loss, if any, on the scale of two properties?
অনুবােঃ নমঃ নর P1, ও P2 দুনট সম্পনত্ত 50,000 টাকা কতর নবক্রে করতলন । P1 নেনন োর ক্রযমূতলযর প তয 20% পবনশ্ P2 োর
ক্রযমূতলযর প তয 20% কতম নবক্রে করতলন । োতে োর পমাট কে লাভ/েনে হতলা?
Solution:
At 20% Profit of P1,
If the cost price Tk. 100 then selling price Tk. (100+20) = Tk. 120
When selling price Tk. 120 the cost price Tk. 100
100
When selling price Tk. 1 the cost price Tk.
120
100×50000 125,000
When selling price Tk. 50,000 the cost price Tk. =
120 3
Tk At 20% less of P2,
If the cost price Tk. 100 then selling price Tk. (100 - 20) = Tk. 80
When selling price Tk. 80 the cost price Tk. 100
100
When selling price Tk. 1 the cost price Tk.
80
100×50000
When selling price Tk. 50,000 the cost price Tk. = Tk. 62,500
80
125,000
Total cost price = + 62,500 = Tk (4166.67 + 62,500) = Tk. 1,04,166.67
3

Written Math Solution (BIBM) FB Group : BD job Info. (BCS & Bank)
Bank Job Solution (BIBM) 239

Created by Sumon K. Sarkar  FB Group : BD job Info. (BCS & Bank)  Created by Sumon K. Sarkar 
Total selling price = 50,000×2 = Tk. 1,00,000
Created by Sumon K. Sarkar  FB Group : BD job Info. (BCS & Bank)  Created by Sumon K. Sarkar 

 Total loss =1,04,166.67 - 1,00,000 = Tk. 4,166.67


Answer: Tk. 4,166.67

6. Mr. X has a investable amount of Tk 100000, he will invest the amount for two years. He has two
options. He can invest at simple interest rate of 12% per annum; alternatively he can invest at
compound rate of 10% (compounded semi annually). Calculate the earnings at two option and
advice him accordingly.
অনুবােঃ নমঃ X দুই বছতরর েনয 100000 টাকা 12% হাতর সরল সুতে নকংবা 10% হাতর ক্রবৃনি হাতর খাটাতে পাতরন । দুনট নবকতল্প
আে গর্না করুন এবং পসই অনুযাযী োতক পরামশ্ণ নেন।
Solution:
At 12% simple interest,
Mr. X can get, IS = Pnr
= (1,00,000×2×12%)
12
= 1,00,000×2× = Tk 24,000
100
At 10% compound interest,
Mr. X can get, IC = 𝑃(1 + 𝑟)𝑛 − 𝑃
10 4
= 100,000 (1 + ) − 100,000 (n=4 because of semi annually)
100
= 121550.6 −100,000
= Tk 21550.6
So, simple interest is better for Mr. X
Answer: Simple interest

যনযবাে
এ রকম গুরুত্বপূর্ণ ফাইল পপতে আমাতের গ্রুতপ প াখ রাখুন
এবং অবশ্যই ননতের বন্ধুতের ইনভাইট করুন

Written Math Solution (BIBM) FB Group : BD job Info. (BCS & Bank)
240

 Created by Sumon K. Sarkar  FB Group : BD job Info. (BCS & Bank)  Created by Sumon K. Sarkar 
Bank Job Solution (BIBM)
 Created by Sumon K. Sarkar  FB Group : BD job Info. (BCS & Bank)  Created by Sumon K. Sarkar 

সবাই গ্রুতপ েতেন করুনঃ

এ রকম গুরুত্বপূর্ণ ফাইল পপতে আমাতের গ্রুতপ প াখ রাখুন


এবং অবশ্যই ননতের বন্ধুতের ইনভাইট করুন

Written Math Solution (BIBM) FB Group : BD job Info. (BCS & Bank)

You might also like